You are on page 1of 479

“Mimicry”

“ You are afraid of death because you erected your kingdom in this life and you
wrecked your palace in the hereafter.”

- Ust. Munir Muluk

“There are always two afflictions you experience while you live. One is a
blessing in which the affliction is not apparent. The other one is the real affliction
which many people could say that it is an affliction.”
- Ust. Munir Muluk

Just master the laws, their nuances and application; never mind the suggested
answer; sometimes, they are not accurate.
Bravery is in the attitude. Attitude comes from belief. Lions see the elephants as
lunch; elephants see the lions as eaters; lions win; elephants die;

“It is the character which allows a person to speak; it is the character which
allows a person to write; it is the character which allows a person to excel and it is the
character which allows a person to do everything he wants, not the necessary logistics
he needs.
During the bar exams, do not entertain doubts; you know the law 100%.

In the facts, examiners sometime intend not to use the proper word, like for
example, the word expropriation in cases involving an expropriation of private
property to allow you to think out of the box.
“Always imagine the interplay of the law; this is the core of reading the law and
the reason why reading cases is important.”
If you can cite the name of RAs, citation of cases, legal terms, topics like Bill of
Rights, Custodial Investigation, Warrantless Arrest, among others, it is appreciated.

For as long as you know the law, that is enough for the bar exams.
Just show to the examiner that you know the topic he asked such as warrantless
arrest, custodial investigation, non-establishment clause, free-exercise clause, among
others.
Dean Willard Riano had focused on the PBE in his books and discussions. He
called the provisions which were not asked in the PBE as codal.

Master: 1) Previous BE; 2) Syllabus; and 3) Legal terms- res gestate, privilege
communication, deadman statute, non-impairment clause; 4) Epigraph in a fertile
provision- filial privilege, best evidence rule; 5) Principles of law; 6) Citations of
cases; and 7) Title of the statutes- Human Security Act.

When Justice Leonen said you go beyond a perfect score, it means that you cite legal
basis in you answer and articulate it; a yes or a no answer does not matter for as long
as you have the legal basis and you are able to logically frame your answer. Argue
like a lawyer; give a lawyer-like answer.

Always determine the characters and the problems in the facts; also, determine
the solution to that problem.

During the bar exams, there must be a leap from being safe to be bold; it is the
character that makes you articulate; write like you play a computer game.
Force yourself to give the law; tell the examiner that you know the law by using
legal terms, citing the law or case citations.

Memorize definitions and legal terms; use key words; just supply the connectors.

“Personalize the context of your knowledge.”

“In taking notes, do not just capture. The most important thing is to make it your own.
Make your brain explore with the context. Remember these: Information, inspiration
and implementation. Always ask: How can I use it? Why must I use it? When can I
use it?”

“Sometimes, the question is also the answer.”

“ You do not need to get a perfect score in the Bar Examinations. Go beyond a perfect
score.”

-Marivic Leonen, J.

“The best teacher is the one who uses analogy.”


“In answering the Bar Exams, make the facts simple by reengineering it to a simple
sentence; simplify the complex sentence.”

“Even if your answer is wrong, if you get the issues, you will be entitled to a certain
credit.”

-Judge Bernardo

“you got the issues if you are within the topic. Meaning, you are not out of topic.”
“ Do not write a kilometric answer in the Bar Exams; 3 sentences suffice provided
they are complete.”

“Control the oil; you will control the nations. Control the food; you will control the
people.”

“Use the call question to answer the bar exams to get the sympathy of the examiners.”

“Just answer the bar exams using legal basis. The Exactness of the application of the
law is not necessary.”

TABLE OF CONTENTS

GENERAL
PRINCIPLES ...................................................................................................................
................ 10

Civil law vs. Common Law (1997)


................................................................................................................ 10 Effect of
Obiter & Dissenting Opinion; SC Decisions
(1994)........................................................................ 10 Effectivity of Laws (1990)
................................................................................................................................ 10
Equity follows the Law
(2003)................................................................................................................ 10
Ignorance of the Law vs. Mistake of Fact
(1996)............................................................................................ 11 Inferior Courts
Decisions (1994)
....................................................................................................................... 11
Prejudicial Questions
(1997)..............................................................................................................................
11

PERSONS........................................................................................................................
........................................ 11

Change of Name; Under RA 9048 (2006)


........................................................................................................... 11 Death; Effects;
Simultaneous Death
(1998)......................................................................................................... 12 Death;
Effects; Simultaneous Death
(1999)....................................................................................................... 12 Death;
Effects; Simultaneous Death
(2000)........................................................................................................ 12 Juridical
Capacity vs. Capacity to Act (1996)
...................................................................................................... 12 Juridical
Capacity; Natural Persons (1999)
........................................................................................................ 13 Waiver of
Rights (2004)
.........................................................................................................................................
13

CONFLICT OF
LAWS...............................................................................................................................
. 13

Appilicable Laws; laws governing contracts (1992)


.................................................................................... 13 Applicable Laws; Arts 15,
16 & 17 (1998) ...................................................................................................... 13
Applicable Laws; Arts 15, 16, 17
(2002).......................................................................................................... 14
Applicable Laws; Capacity to Act
(1998)........................................................................................................ 14
Applicable Laws; Capacity to Buy Land (1995)
............................................................................................. 15 Applicable Laws;
Capacity to Contract
(1995)................................................................................................ 15 Applicable
Laws; capacity to succeed
(1991)................................................................................................. 15 Applicable
Laws; contracts contrary to public policy (1996)
....................................................................... 15 Applicable Laws; Contracts of
Carriage (1995) ............................................................................................. 16
Applicable Laws; Labor Contracts (1991)
...................................................................................................... 16 Applicable Laws;
laws governing marriages (1992)
...................................................................................... 17 Applicable Laws; laws
governing marriages (2003) ......................................................................................
17 Applicable Laws; Sale of Real Property
(1995)............................................................................................... 17 Applicable
Laws; Succession; Intestate & Testamentary
(2001).................................................................. 18 Applicable Laws; Sucession of
Aliens (1995)................................................................................................. 18
Applicable Laws; Wills executed abroad (1993)
............................................................................................ 18 Definition; Cognovit;
Borrowing Statute; Characterization (1994) ..............................................................
18 Definition; forum non-conveniens; long-arm statute (1994)
........................................................................ 19 Divorce; effect of divorce granted
to former Filipinos; Renvoi Doctrine (1997)......................................... 19
Domiciliary theory vs. Nationality Theory
(2004)........................................................................................... 19 Forum Non
Conveniens & Lex Loci Contractus
(2002)................................................................................. 19 Nationality Theory
(2004)................................................................................................................................
.. 20 Naturalization (2003)
........................................................................................................................................
20 Theory; significant relationships theory
(1994)............................................................................................. 20 Torts;
Prescriptive Period (2004)
.................................................................................................................... 21

ADOPTION.....................................................................................................................
....................... 21
Adoption; Use of Surname of her Natural Mother (2006)
............................................................................. 21 Inter-Country Adoption;
Formalities (2005).................................................................................................... 21
Parental Authority; Rescission of Adoption (1994)
...................................................................................... 21 Qualification of Adopter
(2005)........................................................................................................................ 22
Qualification of Adopter; Applicable Law (2001)
............................................................................................ 22 Qualifications of
Adopter (2000)
....................................................................................................................... 22
Qualifications of Adopter (2003)
........................................................................................................................ 23
Successional Rights of Adopted Child (2004)
................................................................................................. 23

FAMILY
CODE...............................................................................................................................
... 23

Emancipation
(1993)................................................................................................................................
....... 23 Family Code; Retroactive Application; Vested Rights
(2000)..................................................................... 24

Family Home; Dwelling House (1994)


............................................................................................................. 24 Family;
Constitutional Mandates; Divorce (1991)
.......................................................................................... 24 Marriage; Annulment;
Effects; Requisites Before Remarriage (1990) .........................................................
24 Marriage; Annulment; Grounds
(1991)............................................................................................................. 25
Marriage; Annulment; Judicial Declaration
(1993).......................................................................................... 25 Marriage;
Annulment; Legal Separation; Prescription of Actions (1996)
.................................................... 25 Marriage; Annulment; Proper Party
(1990)...................................................................................................... 26 Marriage;
Annulment; Proper Party
(1995)...................................................................................................... 26 Marriage;
Divorce Decree; Void Marriages (1992)
......................................................................................... 26 Marriage; Divorce
Decrees; Filiation of Children (2005)
............................................................................... 26 Marriage; Divorce Decrees;
Filipino Spouses becoming Alien (1996)......................................................... 27
Marriage; Divorce Decrees; Filipino Spouses becoming Alien
(1999)......................................................... 27 Marriage; Donations by Reason of
Marriage; Effect of Declaration of Nullity (1996) ................................ 28
Marriage; Grounds; Declaration of Nullity: Annulment: Legal Separation:
Separation of Property (2003) …………………….28 Marriage; Grounds;
Nullity; Annulment; Legal Separation
(1997)................................................................ 29 Marriage; Legal Separation;
Declaration of Nullity (2002)............................................................................. 29
Marriage; Legal Separation; Grounds; Prescriptive Period (1994)
.............................................................. 29 Marriage; Legal Separation; Mutual
guilt (2006) ............................................................................................ 29 Marriage;
Non-Bigamous Marriages (2006)
.................................................................................................... 30 Marriage;
Property Relations; Void Marriages (1991)
................................................................................... 30 Marriage; Psychological
Incapacity (1996)..................................................................................................... 30
Marriage; Psychological Incapacity
(2006)..................................................................................................... 31 Marriage;
Psychological Incapacity
(2006)..................................................................................................... 31 Marriage;
Requisites (1995)
............................................................................................................................. 31
Marriage; Requisites (1999)
............................................................................................................................. 32
Marriage; Requisites; Marriage License (1996)
............................................................................................. 32 Marriage; Requisites;
Marriage License (2002) .............................................................................................
33 Marriage; Requisites; Solemnizing Officers
(1994)........................................................................................ 33 Marriage;
Requisites; Void Marriage (1993)
................................................................................................... 33 Marriage; Void
Marriages (2004)
..................................................................................................................... 34
Marriage; Void Marriages (2006)
..................................................................................................................... 34
Marriage; Void Marriages; Psychological Incapacity (2002)
........................................................................ 35 Parental Authority; Child under 7
years of age (2006) .................................................................................. 35 Parental
Authority; Special Parental Authority; Liability of Teachers
(2003).............................................. 35 Parental Authority; Substitute vs. Special
(2004) .......................................................................................... 35 Paternity &
Filiation (1999)
.............................................................................................................................. 36
Paternity & Filiation; Artificial Insemination; Formalities (2006)
................................................................. 36 Paternity & Filiation; Common-Law
Union (2004).......................................................................................... 36 Paternity &
Filiation; Proofs; Limitations; Adopted Child
(1995).................................................................. 36 Paternity & Filiation;
Recognition of illegitimate Child (2005).......................................................................
37 Paternity & Filiation; Rights of Legitimate Children (1990)
.......................................................................... 37 Presumptive Legitime (1999)
........................................................................................................................... 38
Property Relations; Absolute Community (1994)
.......................................................................................... 38 Property Relations;
Ante Nuptial Agreement (1995)
..................................................................................... 39 Property Relations;
Conjugal Partnership of Gains (1998)
........................................................................... 39 Property Relations; Marriage
Settlement; Conjugal Partnership of Gains (2005) ...................................... 39
Property Relations; Marriage Settlements (1991)
.......................................................................................... 40 Property Relations;
Marriage Settlements (1995)
.......................................................................................... 40 Property Relations;
Obligations; Benefit of the Family (2000)
...................................................................... 41 Property Relations; Unions without
Marriage (1992) ..................................................................................... 41 Property
Relations; Unions without Marriage (1997)
..................................................................................... 41 Property Relations; Unions
without Marriage (2000) ....................................................................................... 42
SUCCESSION.................................................................................................................
..................... 42

Amount of Successional Rights (2004)


.......................................................................................................... 42 Barrier
between illegitimate & legitimate relatives (1993)
............................................................................ 42 Barrier between illegitimate &
legitimate relatives (1996) ............................................................................ 43
Collation (1993)
..........................................................................................................................................
....... 43 Disinheritance vs. Preterition (1993)
............................................................................................................... 43
Disinheritance; Ineffective (1999)
.................................................................................................................... 43
Disinheritance; Ineffective; Preterition (2000)
................................................................................................ 44

Heirs; Intestate Heirs; Reserva Troncal (1995)


.............................................................................................. 44 Heirs; Intestate Heirs;
Shares (2003)...............................................................................................................
45 Intestate Succession
(1992).............................................................................................................................
45 Intestate Succession
(1997).............................................................................................................................
45 Intestate Succession
(1998).............................................................................................................................
46 Intestate Succession
(1998).............................................................................................................................
46 Intestate Succession
(1999).............................................................................................................................
46 Intestate Succession
(2000).............................................................................................................................
46 Intestate Succession; Reserva Troncal (1999)
.............................................................................................. 47 Legitime
(1997)................................................................................................................................
.................. 47 Legitime; Compulsory Heirs
(2003)................................................................................................................. 47
Legitime; Compulsory Heirs vs. Secondary Compulsory Heirs
(2005)........................................................ 48 Preterition
(2001)................................................................................................................................
............... 48 Preterition; Compulsory Heir (1999)
............................................................................................................... 48
Proceedings; Intestate Proceedings; Jurisdiction (2004)
............................................................................. 48 Succession; Death; Presumptive
Legitime (1991) ......................................................................................... 49 Wills;
Codicil; Institution of Heirs; Substitution of Heirs
(2002).................................................................... 49 Wills; Formalities (1990)
................................................................................................................................... 50
Wills; Holographic Wills; Insertions & Cancellations (1996)
........................................................................ 50 Wills; Holographic Wills;
Witnesses (1994)..................................................................................................... 50
Wills; Joint Wills (2000)
.................................................................................................................................... 50
Wills; Probate; Intrinsic Validity (1990)
........................................................................................................... 51 Wills;
Probate; Notarial and Holographic Wills (1997)
................................................................................... 51 Wills; Revocation of Wills;
Dependent Relative Revocation (2003)............................................................... 51
Wills; Testamentary Disposition
(2006)............................................................................................................ 52 Wills;
Testamentary Intent (1996)
.................................................................................................................... 52

DONATION ...................................................................................................................
........................ 52

Donation vs. Sale


(2003)................................................................................................................................
... 52 Donations; Condition; Capacity to Sue (1996)
.............................................................................................. 52 Donations;
Conditions; Revocation
(1991)..................................................................................................... 53 Donations;
Effect; illegal & immoral conditions
(1997)................................................................................. 53 Donations; Formalities;
Mortis Causa (1990) ................................................................................................ 54
Donations; Formalities; Mortis Causa (1998)
................................................................................................ 54 Donations; Inter
Vivos; Acceptance (1993)
..................................................................................................... 54 Donations;
Perfection (1998)
............................................................................................................................ 54
Donations; Requisites; Immovable
Property................................................................................................... 55
Donations; Unregistered; Effects; Non-Compliance; Resolutory Condition (2006)
.................................... 55 Donations; Validity; Effectivity; for Unborn Child
(1999) ............................................................................... 55 Donations; with
Resolutory Condition
(2003).................................................................................................. 56

PROPERTY....................................................................................................................
...................... 56

Accretion; Alluvion (2001)


............................................................................................................................. 56
Accretion; Avulsion (2003)
............................................................................................................................ 56
Builder; Good Faith
(1992)..............................................................................................................................
57 Builder; Good Faith vs. Bad Faith (1999)
..................................................................................................... 57 Builder; Good
Faith vs. Bad Faith (2000)
..................................................................................................... 57 Builder; Good
Faith vs. Bad Faith; Accession (2000)
................................................................................. 58 Builder; Good Faith vs. Bad
Faith; Presumption (2001).............................................................................. 58
Chattel Mortgage vs. Pledge (1999)
.............................................................................................................. 58 Chattel
Mortgage; Immovables
(1994)........................................................................................................... 59 Chattel
Mortgage; Immovables
(2003)........................................................................................................... 59 Chattel
Mortgage; Possession (1993)
........................................................................................................... 60 Chattel
Mortgage; Preference of Creditors (1995)
....................................................................................... 60 Easement vs. Usufruct
(1995)......................................................................................................................... 60
Easement; Effects; Discontinuous Easements; Permissive Use (2005)
.................................................... 61 Easement; Nuisance; Abatement (2002)
....................................................................................................... 61 Easements;
Classification
(1998)................................................................................................................... 62
Easements; Right of Way
(1993)..................................................................................................................... 62
Easements; Right of Way
(2000)..................................................................................................................... 62
Easements; Right of Way; Inseparability (2001)
.......................................................................................... 62

Easements; Right of Way; Requisites (1996)


................................................................................................. 63 Ejectment Suit vs.
Cancellation of Title (2005)
.............................................................................................. 63 Ejectment Suit;
Commodatum (2006)
............................................................................................................. 63 Extra-
Judicial Partition; Fraud
(1990).............................................................................................................. 63
Hidden Treasure (1995)
.................................................................................................................................... 64
Hidden Treasures (1997)
.................................................................................................................................. 64
Mortgage; Pactum Commissorium (1999)
...................................................................................................... 64 Mortgage;
Pactum Commissorium (2001)
...................................................................................................... 65 Mortgage; Right
of Redemption vs. Equity of Redemption (1999)
............................................................... 65 Nuisance; Family House; Not Nuisance
per se (2006) ................................................................................... 65 Nuisance;
Public Nuisance vs. Private Nuisance
(2005)................................................................................. 65 Ownership; Co-
Ownership (1992)
.................................................................................................................... 66
Ownership; Co-Ownership; Prescription (2000)
............................................................................................. 66 Ownership; Co-
Ownership; Prescription (2002)
............................................................................................. 67 Ownership; Co-
Ownership; Redemption (1993)
............................................................................................. 67 Ownership; Co-
Ownership; Redemption (2000)
............................................................................................. 67 Ownership; Co-
Ownership; Redemption (2002)
............................................................................................ 67 Possession
(1998)................................................................................................................................
.............. 68 Property; Real vs. Personal Property (1995)
................................................................................................... 68 Property; Real vs.
Personal Property (1997)
................................................................................................... 68 Sower; Good Faith/
Bad Faith (2000)
............................................................................................................... 69 Usufruct
(1997)................................................................................................................................
.................... 69

LAND TRANSFER &


DEEDS................................................................................................ 69

Acquisition of Lands; Citizenship Requirement


(2003)................................................................................... 69 Adverse Claims;
Notice of Levy (1998)
............................................................................................................ 69 Annotation of
Lis Pendens; When Proper
(2001)............................................................................................. 70 Foreshore Lands
(2000)................................................................................................................................
...... 70 Forgery; Innocent Purchaser; Holder in Bad Faith
(2005)............................................................................... 70 Forgery; Innocent
Purchaser; Mirror Principle (1991)
.................................................................................... 71 Fraud; Procurement of
Patent; Effect (2000) ...................................................................................................
71 Homestead Patents; Void Sale (1999)
.............................................................................................................. 71 Innocent
Purchaser for Value
(2001)................................................................................................................. 72
Mirror Principle (1990)
.......................................................................................................................................
72 Mirror Principle; Forgery; Innocent Purchaser (1999)
.................................................................................... 73 Notice of Lis Pendens (1995)
............................................................................................................................ 73
Notice of Lis Pendens; Transferee Pendente Lite (2002)
............................................................................... 73 Prescription & Laches;
Elements of Laches (2000) ........................................................................................ 74
Prescription & Laches; Indefeasibility Rule of Torrens Title (2002)
.............................................................. 74 Prescription (1990)
..........................................................................................................................................
... 75 Prescription; Real Rights (1992)
....................................................................................................................... 75
Primary Entry Book; Acquisitive Prescription; Laches (1998)
...................................................................... 76 Reclamation of Foreshore Lands;
Limitations (2000) .................................................................................... 76
Registration; Deed of Mortgage
(1994)............................................................................................................. 77
Remedies; Judicial Confirmation; Imperfect Title (1993)
............................................................................... 77 Remedies; Judicial
Reconstitution of Title (1996)
.......................................................................................... 77 Remedies; Procedure;
Consulta (1994)............................................................................................................
77 Remedies; Reconveyance vs. Reopening of a Decree; Prescriptive Period
(2003)..................................... 78 Remedies; Reconveyance; Elements (1995)
................................................................................................... 78 Remedies;
Reconveyance; Prescriptive Period (1997)
.................................................................................. 79 Remedies; Reopening of a
Decree; Elements (1992)...................................................................................... 79
Torrens System vs. Recording of Evidence of Title (1994)
............................................................................ 80 Unregistered Land (1991)
.... ............................................................................................................................. 80
CONTRACTS ................................................................................................................
....................... 80

Consensual vs. Real Contracts; Kinds of Real Contracts (1998)


.................................................................. 80 Consideration; Validity
(2000)........................................................................................................................... 80
Contract of Option; Elements
(2005)................................................................................................................ 81
Inexistent Contracts vs. Annullable Contracts
(2004).................................................................................... 81 Nature of Contracts;
Obligatoriness (1991).....................................................................................................
81 Nature of Contracts; Privity of Contract (1996)
.............................................................................................. 82

Nature of Contracts; Relativity of Contracts (2002)


....................................................................................... 82 Rescission of Contracts;
Proper Party (1996) ................................................................................................ 82

OBLIGATIONS
………..............................................................................................................................
...... 83

Aleatory Contracts; Gambling


(2004)............................................................................................................ 83
Conditional Obligations (2000)
......................................................................................................................... 83
Conditional Obligations (2003)
......................................................................................................................... 83
Conditional Obligations; Promise (1997)
........................................................................................................... 84 Conditional
Obligations; Resolutory Condition (1999)
................................................................................... 84 Extinguishment; Assignment
of Rights (2001) ................................................................................................. 84
Extinguishment; Cause of Action
(2004).......................................................................................................... 85
Extinguishment; Compensation (2002)
.............................................................................................................. 85
Extinguishment; Compensation vs. Payment (1998)
........................................................................................ 85 Extinguishment;
Compensation/Set-Off; Banks (1998)
.................................................................................... 85 Extinguishment;
Condonation (2000)
................................................................................................................ 85
Extinguishment; Extraordinary Inflation or Deflation
(2001)............................................................................ 86 Extinguishment; Loss
(1994) .............................................................................................................................
86 Extinguishment; Loss; Impossible Service (1993)
.......................................................................................... 86 Extinguishment;
Novation
(1994)......................................................................................................................... 87
Extinguishment; Payment (1995)
........................................................................................................................ 87
Liability; Lease; Joint Liability (2001)
................................................................................................................ 87 Liability;
Solidary Liability
(1998)........................................................................................................................ 87
Liability; Solidary Obligation
(1992)................................................................................................................. 88
Liability; Solidary Obligation; Mutual Guaranty (2003)
.................................................................................... 88 Loss of the thing due; Force
Majeure (2000) ....................................................................................................... 88
Non-Payment of Amortizations; Subdivision Buyer; When justified
(2005)..................................................... 89 Period; Suspensive Period
(1991)........................................................................................................................ 89

TRUST.............................................................................................................................
.................................... 89

Express Trust; Prescription (1997)


..................................................................................................................... 89 Implied
Trust
(1998)................................................................................................................................
.............. 90 Trust; Implied Resulting Trust
(1995).................................................................................................................. 91

SALES..............................................................................................................................
................................... 91

Assignment of Credit vs. Subrogation


(1993).................................................................................................. 91 Conditional
Sale vs. Absolute Sale
(1997)........................................................................................................... 91
Contract of Sale vs. Agency to Sell (1999)
........................................................................................................ 91 Contract of
Sale; Marital Community Property; Formalities (2006)
................................................................ 91 Contract to Sell
(2001)................................................................................................................................
........... 92 Contract to Sell vs. Contract of Sale
(1997)....................................................................................................... 92 Contract
to Sell; Acceptance; Right of First Refusal
(1991)............................................................................ 92 Double Sales
(2001)................................................................................................................................
.............. 92 Double Sales
(2004)................................................................................................................................
............. 93 Equitable Mortgage
(1991)................................................................................................................................
93 Equitable Mortgage vs. Sale
(2005)..................................................................................................................... 93
Immovable Property; Rescission of Contract (2003)
........................................................................................ 94 Maceda Law
(2000)................................................................................................................................
................ 94 Maceda Law; Recto Law
(1999).......................................................................................................................... 95
Option Contract (2002)
.................................................................................................................................... 95
Option Contract; Earnest Money (1993)
........................................................................................................... 95 Perfected Sale;
Acceptance of Earnest Money (2002)
...................................................................................... 95 Redemption; Legal;
Formalities (2001)
.............................................................................................................. 96 Redemption;
Legal; Formalities (2002)
............................................................................................................... 96 Right of
First Refusal; Lessee; Effect (1996)
....................................................................................................... 96 Right of First
Refusal; Lessee; Effect (1998)
...................................................................................................... 97 Right of
Repurchase (1993)
................................................................................................................................ 97
Transfer of Ownership; Non-Payment of the Price
(1991).................................................................................. 97 Transfer of
Ownership; Risk of Loss (1990)
....................................................................................................... 97

LEASE.............................................................................................................................
................................ 97

Extinguishment; Total Distruction; Leased Property (1993)


.......................................................................... 97 Implied New Lease
(1999)...............................................................................................................................
98 Lease of Rural Lands (2000)
................................................................................................................................. 98

Leasee & Lessor; Rights and Obligations (1990)


.............................................................................................. 98 Leasee; Death
Thereof; Effects (1997)
............................................................................................................... 98 Option to
Buy; Expired
(2001)..............................................................................................................................
98 Sublease vs. Assignment of Lease; Rescission of Contract (2005)
............................................................... 99 Sublease; Delay in Payment of Rentals
(1994) .................................................................................................. 99 Sublease;
Sublessee; Liability (1999)
.............................................................................................................. 100 Sublease;
Sublessee; Liability (2000)
.............................................................................................................. 100 Sublease;
Validity; Assignment of Sublease (1990)
....................................................................................... 100

COMMON
CARRIERS.....................................................................................................................
........... 100
Extraordinary Diligence (2000)
......................................................................................................................... 100

AGENCY ........................................................................................................................
.................................. 101

Agency (2003)
..........................................................................................................................................
........... 101 Agency vs. Sale (2000)
......................................................................................................................................
101 Agency; coupled with an interest (2001)
......................................................................................................... 101 Agency;
Guarantee Commission
(2004)........................................................................................................... 101
Agency; Real Estate Mortgage (2004)
.............................................................................................................. 101
Appointment of Sub-Agent
(1999)..................................................................................................................... 102
General Agency vs. Special Agency
(1992)...................................................................................................... 102 Powers of
the Agent
(1994)................................................................................................................................
102 Termination; Effect of Death of Agent (1997)
.................................................................................................. 103

PARTNERSHIP .............................................................................................................
...................... 103

Composition of Partnerships; Spouses; Corporations (1994)


...................................................................... 103 Conveyance of a Partner’s Share
Dissolution (1998)...................................................................................... 103
Dissolution of Partnership (1995)
..................................................................................................................... 103
Dissolution of Partnership; Termination
(1993)............................................................................................... 104 Effect of
Death of Partner
(1997)....................................................................................................................... 104
Obligations of a Partner (1992)
......................................................................................................................... 104
Obligations of a Partner; Industrial Partner (2001)
......................................................................................... 104

COMMODATUM &
MUTUUM ....................................................................................................... 104

Commodatum (1993)
................................................................................................................................... 104
Commodatum (2005)
........................................................................................................................................
105 Commodatum vs. Usufruct (1998)
.................................................................................................................... 105
Mutuum vs. Commodatum (2004)
...................................................................................................................... 106
Mutuum; Interests
(2001)................................................................................................................................
.... 106 Mutuum; Interests
(2002)................................................................................................................................
.... 106 Mutuum; Interests
(2004)................................................................................................................................
.... 106

DEPOSIT.........................................................................................................................
.............. 107

Compensation; Bank Loan


(1997)..................................................................................................................... 107
Deposit; Exchange
(1992)................................................................................................................................
.. 107

SURETY..........................................................................................................................
............................ 107
Recovery of Deficiency (1997)
.......................................................................................................................... 107

ANTICHRESIS...............................................................................................................
....................... 107
Antichresis (1995)
..........................................................................................................................................
.... 107

PLEDGE .........................................................................................................................
............................ 108

Pledge (1994)
..........................................................................................................................................
............ 108 Pledge (2004)
..........................................................................................................................................
............ 108 Pledge; Mortgage; Antichresis (1996)
.............................................................................................................. 108

QUASI-
CONTRACT...................................................................................................................
...... 108

Quasi-Contracts; Negotiorium Gestio (1992)


................................................................................................... 109 Quasi-Contracts;
Negotiorium Gestio (1993)
................................................................................................... 109 Quasi-Contracts;
Negotiorium Gestio (1995)
................................................................................................... 109 Quasi-Contracts;
Solutio Indebiti (2004)
.......................................................................................................... 110

TORTS &
DAMAGES ....................................................................................................................
110

Collapse of Structures; Last Clear Chance


(1990)........................................................................................... 110 Damages
(1994)................................................................................................................................
................... 111 Damages arising from Death of Unborn Child (1991)
..................................................................................... 111 Damages arising from
Death of Unborn Child (2003) .....................................................................................
111 Death Indemnity
(1994)................................................................................................................................
....... 111

Defense; Due Diligence in Selection


(2003)...................................................................................................... 112 Filing of
Separate Civil Action; Need for Reservation (2003)
........................................................................ 112 Fortuitous Event; Mechanical
Defects (2002) .................................................................................................. 112
Liability; Airline Company; Non-Performance of an Obligation
(2004).......................................................... 112 Liability; Airline Company; Non-
Performance of an Obligation (2005).......................................................... 113
Liability; Employer; Damage caused by Employees (1997)
........................................................................... 113 Liability; owner who was in the
vehicle (1996) ................................................................................................ 114
Liability; owner who was in the vehicle (1998)
................................................................................................ 114 Liability; owner
who was in the vehicle (2002)
................................................................................................ 114 Moral Damages &
Atty Fees (2002)
.................................................................................................................. 114 Moral
Damages; Non-Recovery Thereof (2006)
.............................................................................................. 115 Quasi-Delict (1992)
..........................................................................................................................................
... 115 Quasi-Delict (2005)
..........................................................................................................................................
... 115 Quasi-Delict; Acts contrary to morals (1996)
.................................................................................................. 115 Quasi-Delict;
Mismanagement of Depositor’s Account
(2006)....................................................................... 116 Vicarious Liability (1991)
................................................................................................................................... 116
Vicarious Liability (2001)
................................................................................................................................... 117
Vicarious Liability (2002)
................................................................................................................................... 117
Vicarious Liability (2004)
................................................................................................................................... 117
Vicarious Liability (2006)
................................................................................................................................... 117
Vicarious Liability; Public Utility (2000)
........................................................................................................... 118

INTELLECTUAL
PROPERTY ...................................................................................................... 118

Intellectual Creation
(2004)................................................................................................................................
118

GENERAL PRINCIPLES
Civil law vs. Common Law (1997)
How would you compare the Civil Law system in its governance and trend
with that of the Common Law system?

SUGGESTED ANSWER:
As regards "governance": Governance in Civil Law is codal, statutory and
written law. It is additionally derived from case law. Common law is basically derived
from case law.

As regards "trend": Civil law is now tending to rely more and more on decisions
of the courts explaining the laws. Common law is now codifying laws more and more.
So they are now merging towards similar systems.

Notes:
With regards to governance, governance in Civil Law is codal, statutory and
written law. On the other hand, governance in Common law is basically derived from
case law.

With regards to trend, the trend in Civil law is now tending to rely more and
more on decisions of the courts explaining the laws. On the other hand, the trend in
Common law is now codifying laws more and more, so they are now merging towards
similar systems.

Additional Answers:
COMMON LAW refers to the traditional part of the law as distinct from
legislation; it refers to the universal part of law as distinct from particular local
customs (Encyclopedia Americana, Vol. 7). On the other hand, CIVIL LAW is
understood to be that branch of law governing the relationship of persons in respect of
their personal and private interests as distinguished from both public and international
laws.

In common law countries, the traditional responsibility has for the most part
been with the judges; in civil law countries, the task is primarily reposed on the
lawmakers. Contemporary practices, however, so indicate a trend towards centralizing
that function to professional groups that may indeed, see the gradual assimilation in
time of both systems. [Vitug, Civil. Law and Jurisprudence, p. XX)

In Civil Law, the statutes theoretically take precedence over court decisions
interpreting them; while in Common Law, the court decisions resolving specific cases
are regarded as law rather than the statutes themselves which are, at the start, merely
embodiments of case law. Civil Law is code law or written law, while Common Law
is case law. Civil Law adopts the deductive method - from the general to the
particular, while the Common Law uses the inductive approach from the particular to
the general. Common Law relies on equity. Civil Law anchors itself on the letter of
the law. The civilists are for the judge-proof law even as the Common Law Is judge-
made law. Civil Law judges are merely supposed to apply laws and not interpret them.

Notes:
Civil Law is code law or written law, while Common Law is case law.

Effect of Obiter & Dissenting Opinion; SC Decisions (1994)


2) What are the binding effects of an obiter dictum and a dissenting opinion? 3)
How can a decision of the Supreme Court be set aside?

ALTERNATIVE ANSWERS:
2) None. Obiter dictum and opinions are not necessary to the determination of a
case. They are not binding and cannot have the force of official precedents. It is as if
the Court were turning aside from the main topic of the case to collateral subjects: a
dissenting opinion affirms or overrules a claim, right or obligation. It neither disposes
nor awards anything it merely expresses the view of the dissenter. (Civil Code, Paras]

3) A decision of a division of the Supreme Court maybe set aside by the


Supreme Court sitting en banc, a Supreme Court decision may be set aside by a
contrary ruling of the Supreme Court itself or by a corrective legislative act of
Congress, although said laws cannot adversely affect those favored prior to the
Supreme Court decision. [Civil Code, Paras).

Notes:
Obiter dictum and opinions are not necessary to the determination of a case.
They are not binding and cannot have the force of official precedents. It is as if the
Court were turning aside from the main topic of the case to collateral subjects

The Supreme Court decision in division may be set aside by the decision of the
Supreme Court en banc. It can also be set aside by a contrary ruling of the Supreme
Court itself.

Effectivity of Laws (1990)


After a devastating storm causing widespread destruction in four Central Luzon
provinces, the executive and legislative branches of the government agreed to enact a
special law appropriating P1 billion for purposes of relief and rehabilitation for the
provinces. In view of the urgent nature of the legislative enactment, it is provided in
its effectivity clause that it shall take effect upon approval and after completion of
publication in the Official Gazette and a newspaper of general circulation in the
Philippines. The law was passed by the Congress on July 1, 1990. signed into law by
the President on July 3, 1990, and published in such newspaper of general circulation
on July 7, 1990 and in the Official Gazette on July 10, 1990.

a. As to the publication of said legislative enactment, is there sufficient


observance or compliance with the requirements for a valid publication? Explain
your answer.

b. When did the law take effect? Explain your answer.

c. Can the executive branch start releasing and disbursing funds


appropriated by the said law the day following its approval? Explain your
answer.
SUGGESTED ANSWER:
a. Yes, there is sufficient compliance. The law itself prescribes the requisites of
publication for its effectivity, and all requisites have been complied with. (Article 2,
Civil Code)
b. The law takes effect upon compliance with all the conditions for effectivity,
and the last condition was complied with on July 10, 1990. Hence, the" law became
effective on that date.
c. No. It was not yet effective when it was approved by Congress on July 1,
1990 and approved by the President on July 3, 1990. The other requisites for its
effectivity were not yet complete at the time.

Notes:
a) Yes, there is sufficient observance or compliance with the requirements for a
valid publication because it is to be published in the Official Gazzette or in a
newspaper of general circulation in the Philippines.
b. The law will take effect after its complete publication.
c. No, the executive branch cannot start releasing and disbursing funds
appropriated by the said law the day following its approval because there is no
publication yet. The law will become effective only after a complete publication.
Article 2. Laws shall take effect fifteen days after the completion of their
publication in the official Gazzette, or in a newspaper of general circulation in the
Philippines, unless the effectivity of the law is provided.(Re-engineered)

Article 2. Laws shall take effect fifteen days following the completion of their
publication either in the official Gazzette, or in a newspaper of general circulation in
the Philippines, unless it is otherwise provided.

Equity follows the Law (2003)


It is said that “equity follows the law” What do you understand by this phrase,
and what are its basic implications? 5%
SUGGESTED ANSWER:
“Equity Follows the law” means that courts exercising equity jurisdiction are
bound by rules of law and have no arbitrary discretion to disregard them. (Arsenal v
IAC, 143 SCRA 40 [1986]). Equity is applied only in the absence of but never
against statutory law. (Toyota Motor Phil. V CA, 216 SCRA 236 [1992]).

Notes:
In Arsenal v IAC, the Supreme Court held that the phrase equity Follows the
law” means that courts exercising equity jurisdiction are bound by rules of law and
have no discretion to disregard them; court will apply equity only in the absence of the
law;

Ignorance of the Law vs. Mistake of Fact (1996)


Is there any difference in their legal effect between ignorance of the law and
ignorance or mistake of fact?

SUGGESTED ANSWER:
Yes, there is a difference. While ignorance of the law is not an excuse for not
complying with it, ignorance of fact eliminates criminal intent as long as there is no
negligence (Art, NCC). In addition, mistake on a doubtful or difficult question of law
may be the basis of good faith (Art. 526. NCC). Mistake of fact may, furthermore,
vitiate consent in a contract and make it voidable (Art. 1390. NCC).

Notes:
Under the Civil code, ignorance of the law is not a defense, while ignorance of
the fact is a defense so long as there is no negligence. (Atty. Ryan Jumaani)

ALTERNATIVE ANSWER:
Yes. ignorance of the law differs in legal effect from Ignorance or mistake of
fact. The former does not excuse a party from the legal consequences of his conduct
while the latter does constitute an excuse and is a legal defense.

Inferior Courts Decisions (1994)


Are decisions of the Court of Appeals considered laws?

ALTERNATIVE ANSWERS:
a) No, but decisions of the Court of Appeals may serve as precedents for inferior
courts on points of law not covered by any Supreme Court decision, and a ruling of
the Court of Appeals may become a doctrine. (Miranda vs.. Imperial 77 Phil. 1066).
b) No. Decisions of the Court of Appeals merely have persuasive, and therefore
no mandatory effect. However, a conclusion or pronouncement which covers a point
of law still undecided may still serve as judicial guide and it is possible that the same
maybe raised to the status of doctrine. If after it has been subjected to test in the
crucible of analysis, the Supreme Court should find that it has merits and qualities
sufficient for its consideration as a rule of jurisprudence (Civil Code, Paras).
Notes:
In Miranda v. Imperial, it was held that the decisions of the Court of appeals
have persuasive effect but they do not have mandatory effect.

(Under the Civil Code, Only the decisions of the Supreme Court interpreting the
law will form part the law of the land. Thus, decisions of Court of Appeals are not
considered laws.

Prejudicial Questions (1997)


In the context that the term is used in Civil Law, state the (a) concept, (b)
requisites and (c) consequences of a prejudicial question.

SUGGESTED ANSWER:
(a) Concept

A prejudicial question is one which must be decided first before a criminal


action may be instituted or may proceed because a decision therein is vital to the
judgment in the criminal case. In the case of People vs. Adelo Aragon (L5930, Feb.
17, 1954), the Supreme Court defined it as one which arises in a case, the resolution of
which question is a logical antecedent of the issues involved in said case and the
cognizance of which pertains to another tribunal (Paras, Vol. 1, Civil. Code
Annotation, 1989 ed. p, 194).

(b) Requisites

The prejudicial question must be determinative of the case before the court.
Jurisdiction to try said question must be lodged in another tribunal.

Occurs when a civil action is instituted prior to a criminal action including issues
which are intimately related with one another, and the resolution of which determines
whether or not criminal action may proceed
ADDITIONAL ANSWER:
1. The civil action involves an issue similar or intimately related to the issue
raised in the criminal action, and
2. the resolution of such issue determines whether or not the criminal action may
proceed.
(c) Consequences The criminal case must be suspended. Thus, in a criminal case for
damages to one's property, a civil action that involves the ownership of said property
should first be resolved (De Leon vs. Mabanag. 38 Phil. 202)

Notes:
(a) In the case of People vs. Adelo Aragon, the Supreme court held that a
prejudicial question is one which must be decided first before a criminal action may
be instituted or may proceed because a decision therein is vital to the judgment in the
criminal case.
(b) The requisites of the prejudicial questions are as follows:

1. The civil action involves an issue that is similar or related to the issue in the
criminal action; and
2. the resolution of such issue determines whether or not the criminal action may
proceed.
(c) The consequence is that the criminal case must be suspended. Thus, in a
criminal case for damages to property, a civil action to determine the ownership of
said property should first be resolved.

ARTICLE 36. Pre-judicial questions must be decided before any criminal


prosecution may be instituted or may proceed. (Re-engineered)

ARTICLE 36. Pre-judicial questions, which must be decided before any criminal
prosecution may be instituted or may proceed, shall be governed by rules of court
which the Supreme Court shall promulgate and which shall not be in conflict with the
provisions of this Code.

PERSONS

Change of Name; Under RA 9048 (2006)


Zirxthoussous delos Santos filed a petition for change of name with the Office of
the Civil Registrar of Mandaluyong City under the administrative proceeding
provided in Republic Act No. 9048. He alleged that his first name sounds ridiculous
and is extremely difficult to spell and pronounce. After complying with the
requirements of the law, the Civil Registrar granted his petition and changed his first
name Zirxthoussous to "Jesus." His full name now reads "Jesus delos Santos."
Jesus delos Santos moved to General Santos City to work in a multi-national
company. There, he fell in love and married Mary Grace delos Santos. She requested
him to have his first name changed because his new name "Jesus delos Santos" is the
same name as that of her father who abandoned her family and became a notorious
drug lord. She wanted to forget him. Hence, Jesus filed another petition with the
Office of the Local Civil Registrar to change his first name to "Roberto." He claimed
that the change is warranted because it will eradicate all vestiges of the infamy of
Mary Grace's father.
Will the petition for change of name of Jesus delos Santos to Roberto delos
Santos under Republic Act No. 9048 prosper? Explain. (10%)

SUGGESTED ANSWER:
No, under the law, Jesus may only change his name once. In addition, the
petition for change of name may be denied on the following grounds:

1. Jesus is neither ridiculous, nor tainted with dishonor nor extremely difficult to
write or pronounce.
2. There is no confusion to be avoided or created with the use of the registered
first name or nickname of the petitioner.

3.The petition involves the same entry in the same document, which was
previously corrected or changed under this Order [Rules and Regulations
Implementing RA 9048].
Notes:
Under the law, a person may petition the court to change his name only once.

The petition for change of name may be denied on the following grounds:
1. That the name is not ridiculous, tainted with dishonour or extremely difficult
to write or pronounce.
2. There is no confusion to be avoided or created with the use of the registered
first name or nickname of the petitioner.

What entries in the Civil Registry may be changed or corrected without a


judicial order? (2.5%)
SUGGESTED ANSWER:
Only clerical or typographical errors and first or nick names may be
changed or corrected without a judicial order under RA 9048. Clerical or
typographical errors refer to mistakes committed in the performance of clerical work
in writing, copying, transcribing or typing an entry in the civil register. The mistake is
harmless and innocuous, such as errors in spelling, visible to the eyes or obvious to
the understanding, and can be corrected or changed only by reference to other existing
records. Provided, however, that no correction must involve the change of nationality,
age, status or sex of the petitioner.

Notes:
Under RA 9048, only clerical or typographical errors, and first or nicknames
may be changed without court order.

The typographical errors are, among others, mistakes in the performance of


clerical work like writing, typing, copying, and first or nick names may be changed
without judicial order.

Further, change of nationality, age, status, or sex is allowed to be changed if


there is a judicial order.

SIMULTANEOUS DEATH

Death; Effects; Simultaneous Death (1998)


Jaime, who is 65, and his son, Willy, who is 25, died in a plane crash. There is
no proof as to who died first. Jaime's only surviving heir is his wife, Julia, who is also
Willy's mother. Willy's surviving heirs are his mother, Julia and his wife, Wilma.

1. In the settlement of Jaime's estate, can Wilma successfully claim that her
late husband, Willy had a hereditary share since he was much younger than his
father and, therefore, should be presumed to have survived longer? [3%]no,
issues involving succession, in the absence of proof, both are presumed to have died
simultaneously, hence no transmission of rights

2. Suppose Jaime had a life insurance policy with his wife, Julia, and his
son, Willy, as the beneficiaries. Can Wilma successfully claim that one-half of the
proceeds should belong to Willy's estate? |2%yes life insurance is not a matter of
succession hence rules of court on presumption of survivorship shall govern

SUGGESTED ANSWER:
1. No, Wilma cannot successfully claim that Willy had a hereditary share in his
father's estate. Under Art. 43, Civil Code, two persons "who are called to succeed each
other" are presumed to have died at the same time, in the absence of proof as to which
of them died first. This presumption of simultaneous death applies in cases involving
the question of succession as between the two who died, who in this case are mutual
heirs, being father and son.

Notes:
Under the Civil Code, persons who died in the same calamity are presumed to
have died at the same time and there shall be no transmission of inheritance. Whoever
alleges that one died ahead of the other must prove the same.

However, this presumption applies only in a case involving succession.

ARTICLE 43. If there is a doubt, as between two or more persons who are
called to succeed each other, as to which of them died first, whoever alleges the death
of one prior to the other, shall prove the same; in the absence of proof, it is presumed
that they died at the same time and there shall be no transmission of rights from one to
the other.

SUGGESTED ANSWER:
2. Yes, Wilma can invoke the presumption of survivorship and claim that one-
half of the proceeds should belong to Willy's estate, under Sec. 3 (jj) par. 5 Rule 131,
Rules of Court, as the dispute does not involve succession. Under this presumption,
the person between the ages of 15 and 60 years is deemed to have survived one whose
age was over 60 at the time of their deaths. The estate of Willy endowed with juridical
personality stands in place and stead of Willy, as beneficiary.

Notes:

Under the Rules of Court, if two persons perish in the same calamities and if one
is under 15 and the other above 60, the former is deemed to have survived.

Sec. 3 (jj) par. 5 Rule 131. Disputable presumption.- The following


presumptions are satisfactory if uncontradicted, but may be contradicted and
overcome by other evidence: That except for purpose of succession, when two person
perish in the same calamity, such as wreck, battle, or conflagration, and it is not
shown who died first, and there are no particular circumstances from which it can be
inferred , the survivorship is determined from the probabilities resulting from the
strength and age of the sexes, according to the following rules:
1. If both were under the age of 15 years, the older is deemed to have survived;
2. If both were above the age of 60, the younger is deemed to have survived;
3. If one is under 15 and the other above 60, the former is deemed to have
survived;
4. If both be over 15 and under 60, and the sex be different, the male is deemed
to have survived; if the sex be the same, the older;
5. If one be under 15 or over 60, and the other between those ages, the latter is
deemed to have survived.

The presumption under the Rules of Court applies only if the issue does not
include succession.

Death; Effects; Simultaneous Death (1999)


Mr. and Mrs. Cruz, who are childless, met with a serious motor vehicle
accident with Mr. Cruz at the wheel and Mrs. Cruz seated beside him, resulting in the
instant death of Mr. Cruz. Mrs. Cruz was still alive when help came but she also
died on the way to the hospital. The couple acquired properties worth One Million (P1
,000,000.00) Pesos during their marriage, which are being claimed by the parents of
both spouses in equal shares.

(a) Is the claim of both sets of parents valid and why? (3%) no, not equal
share of inheritance

(b) Suppose in the preceding question, both Mr. and Mrs. Cruz were
already dead when help came, so that nobody could say who died ahead of the
other, would your answer be the same to the question as to who are entitled to
the properties of the deceased couple? (2%) no, this time, they are presumed to
have died simultaneously absent any proof of who between them died first, hence no
transmission

SUGGESTED ANSWER:
(a) No, the claim of both parents is not valid. When Mr. Cruz died, he was
succeeded by his wife and his parents as his intestate heirs who will share his estate
equally. His estate was 0.5 Million pesos which is his half share in the absolute
community amounting to 1 Million Pesos. His wife, will, therefore, inherit O.25
Million Pesos and his parents will inherit 0.25 Million Pesos. When Mrs. Cruz died,
she was succeeded by her parents as her intestate heirs. They will inherit all of her
estate consisting of her 0.5 Million half share in the absolute community and her 0.25
Million inheritance from her husband, or a total of 0.750 Million Pesos.

In sum, the parents of Mr. Cruz will inherit 250,000 Pesos while the parents of
Mrs. Cruz will inherit 750,000 Pesos.

(b) This being a case of succession, in the absence of proof as to the time of
death of each of the spouses, it is presumed they died at the same time and no
transmission of rights from one to the other is deemed to have taken place. Therefore,
each of them is deemed to have an estate valued at P500,000,00, or one-half of their
conjugal property of P1 million. Their respective parents will thus inherit the entire P1
Million in equal shares, of P500,000.00 per set of parents.

Death; Effects; Simultaneous Death (2000)


b) Cristy and her late husband Luis had two children, Rose and Patrick. One
summer, her mother-in-law, aged 70, took the two children, then aged 10 and 12,
with her on a boat trip to Cebu. Unfortunately, the vessel sank en route, and the bodies
of the three were never found. None of the survivors ever saw them on the water. On
the settlement of her mother-in-law's estate, Cristy files a claim for a share of her
estate on the ground that the same was inherited by her children from their
grandmother in representation of their father, and she inherited the same from them.
Will her action prosper? (2%)
SUGGESTED ANSWER: no, presumed to have died simultaneously
No, her action will not prosper. Since there was no proof as to who died first, all
the three are deemed to have died at the same time and there was no transmission of
rights from one to another, applying Article 43 of the New Civil Code.

ALTERNATIVE ANSWER:
No, her action will not prosper. Under Article 43 of the New Civil Code,
inasmuch as there is no proof as to who died first, all the three are presumed to have
died at the same time and there could be no transmission of rights among them. Her
children not having inherited from their grandmother. Cristy has no right to share in
her mother-in-law's estate. She cannot share in her own right as she is not a legal heir
of her mother-in-law. The survivorship provision of Rule 131 of the Rules of Court
does not apply to the problem. It applies only to those cases where the issue involved
is not succession.

Notes:
Under the Civil Code, when persons die in the same calamity, they are presumed
to have died at the same time.
JURIDICAL CAPACITY

Juridical Capacity vs. Capacity to Act (1996)


Distinguish juridical capacity from capacity to act.

SUGGESTED ANSWER:
JURIDICAL CAPACITY is the fitness to be the subject of legal relations while
CAPACITY TO ACT is the power to do acts with legal effect. The former is inherent
in every natural person and is lost only through death while the latter is merely
acquired and may be lost even before death (Art. 37, NCC).

Notes:
Under the Civil Code, juridical capacity is the fitness to be the subject of legal
relations, while capacity to act is the power to do acts with legal effect.

The Juridical capacity is inherent in every natural person and is lost only through
death while the capacity to act is merely acquired and may be lost even before death

Juridical capacity is passive while the capacity to act is active; juridical capacity
can exist without capacity to act, while capacity to act cannot exist without juridical
capacity.

ALTERNATIVE ANSWER;
Juridical capacity, as distinguished from capacity to act: (a) the former is passive
while the latter is active, (b) the former is inherent in a person while the latter is
merely acquired, (c) the former is lost only through death while the latter may be lost
through death or restricted by causes other than death, and Id) the former can exist
without capacity to act while the latter cannot exist without juridical capacity.

ARTICLE 37. Juridical capacity, which is the fitness to be the subject of legal
relations, is inherent in every natural person and is lost only through death. Capacity
to act, which is the power to do acts with legal effect, is acquired and may be lost. (n)

Juridical Capacity; Natural Persons (1999)


Elated that her sister who had been married for five years was pregnant for the
first time, Alma donated P100,000.00 to the unborn child. Unfortunately, the baby
died one hour after delivery. May Alma recover the P100.000.00 that she had donated
to said baby before it was born considering that the baby died? Stated otherwise, is the
donation valid and binding? Explain. (5%) valid and binding provided that the baby
was born under the conditions provided in Art. 41
SUGGESTED ANSWER:
The donation is valid and binding, being an act favorable to the unborn child, but
only if the baby had an intra-uterine life of not less than seven months and provided
there was due acceptance of the donation by the proper person representing said child.
If the child had less than seven months of intra-uterine life, it is not deemed born
since it died less than 24 hours following its delivery, in which case the donation
never became effective since the donee never became a person, birth being
determinative of personality.

ALTERNATIVE ANSWER:
Even if the baby had an intra-uterine life of more than seven months and the
donation was properly accepted, it would be void for not having conformed with the
proper form. In order to be valid, the donation and acceptance of personal property
exceeding five thousand pesos should be in writing. (Article 748, par. 3)

Notes:
Requisites for the donation to the unborn child:
1) The donation must be accepted by the mother;
2) The child must be born alive; if he dies after the delivery from the maternal
womb, he must have an intra-uterine life of not less than 7 months; if he dies after
delivery, he must survive for 24 hours if he has an intra-uterine life of less than 7
months.
3. The donation and acceptance must be in writing if it is a personal property and
the amount exceeds P5,000.
4. The donation must, likewise, be in writing if it is a real property regardless of
the amount.

WAIVER OF RIGHTS

Waiver of Rights (2004)


B. DON, an American businessman, secured parental consent for the
employment of five minors to play certain roles in two movies he was producing at
home in Makati. They worked at odd hours of the day and night, but always
accompanied by parents or other adults. The producer paid the children talent fees at
rates better than adult wages.

But a social worker, DEB, reported to OSWD that these children often missed
going to school. They sometimes drank wine, aside from being exposed to drugs.
In some scenes, they were filmed naked or in revealing costumes. In his defense,
DON contended all these were part of artistic freedom and cultural creativity. None
of the parents complained, said DON. He also said they signed a contract
containing a waiver of their right to file any complaint in any office or tribunal
concerning the working conditions of their children acting in the movies.

Is the waiver valid and binding? Why or why not? Explain. (5%) no,
waiver of rights which are contrary to law, morals, good customs, public order,
public policy are void

SUGGESTED ANSWER:
The waiver is not valid. Although the contracting parties may establish such
stipulations, clauses, terms and conditions as they may deem convenient, they may not
do so if such are contrary to law, morals, good customs, public order, or public
policy (Article 1306, Civil Code). The parents' waiver to file a complaint concerning
the working conditions detrimental to the moral well-being of their children acting in
the movies is in violation of the Family Code and Labor laws. Thus, the waiver is
invalid and not binding.

The Child Labor Law is a mandatory and prohibitory law and the rights of the
child cannot be waived as it is contrary to law and public policy.

Notes:
Under the Labor Code, minors below 15 years of age may be employed but their
employment must not be detrimental to their health. Thus, the waiver executed by
their parents allowing their children to work in hazardous industry is void because it is
against the law.

ARTICLE 1306. The contracting parties may establish stipulations, clauses,


terms and conditions as they may deem convenient, provided they are not contrary to
law, morals, good customs, public order, or public policy. (Re-engineered)
ARTICLE 1306. The contracting parties may establish such stipulations,
clauses, terms and conditions as they may deem convenient, provided they are not
contrary to law, morals, good customs, public order, or public policy. (1255a)

CONFLICT OF LAWS

LAWS APPLICABLE TO CONTRACTS

Appilicable Laws; laws governing contracts (1992)


X and Y entered into a contract in Australia, whereby it was agreed that X
would build a commercial building for Y in the Philippines, and in payment for the
construction, Y will transfer and convey his cattle ranch located in the United States
in favor of X. What law would govern: a) The validity of the contract? b) The
performance of the contract? c) The consideration of the contract?

SUGGESTED ANSWER:
(a) The validity of the contract will be governed by Australian law, because the
validity refers to the element of the making of the contract in this case.
(Optional Addendum:"... unless the parties agreed to be bound by another law".}
(b) The performance will be governed by the law of the Philippines where the
contract is to be performed.
(c) The consideration will be governed by the law of United States where the
ranch is located. (Optional Addendum: In the foregoing cases, when the foreign law
would apply, the absence of proof of that foreign law would render Philippine law
applicable under the "eclectic theory".)

Notes:
As to the validity of a contract, it is governed by the law of the place where it is
executed. With respect to the performance of the contract, it is governed by the law
of the place where it is executed or performed; with regards to the payment under the
contract, it is governed by the law of the place where the payment is to be made.

Applicable Laws; Arts 15, 16 & 17 (1998)


Juan is a Filipino citizen residing in Tokyo, Japan. State what laws govern:
His capacity to contract marriage in Japan, [ 1%] national law
His successional rights as regards his deceased Filipino father's property in
Texas, U.S.A. [1%] national law od the deceased
The extrinsic validity of the last will and testament which Juan executed while
sojourning in Switzerland. [2%] Lex loci celeb

The intrinsic validity of said will. (1%)

SUGGESTED ANSWER:
Juan's capacity to contract marriage is governed by Philippine law -i.e.,
the Family Code -pursuant to Art. 15, Civil Code, which provides that our laws
relating to, among others, legal capacity of persons are binding upon citizens of the
Philippines even though living abroad.

Notes:
The Civil Code provides that the legal capacity of a person to a contract
including the contract to marriage is governed by his national law even though he is
living abroad.

SUGGESTED ANSWER:
2. By way of exception to the general rule of lex rei sitae prescribed by the first
paragraph of Art. 16. Civil Code, a person's successional rights are governed by the
national law of the decedent (2nd par. Art. 16). Since Juan's deceased father was a
Filipino citizen, Philippine law governs Juan's successional rights.

Notes:
Under the Civil Code, the national law of the decedent shall govern the
successional right of a person to his estate.

ANOTHER ANSWER:
2. Juan's successional rights are governed by Philippine law, pursuant to Article
1039 and the second paragraph of Article 16, both of the Civil Code. Article 1039,
Civil Code, provides that capacity to succeed shall be governed by the "law of the
nation" of the decedent, i.e.. his national law. Article 16 provides in paragraph two
that the amount of successional rights, order of succession, and intrinsic validity of
testamentary succession shall be governed by the "national law" of the decedent who
is identified as a Filipino in the present problem.
SUGGESTED ANSWER:
3. The extrinsic validity of Juan's will is governed by (a) Swiss law, it being the
law where the will was made (Art. 17. 1st par. Civil Code), or (b) Philippine law, by
implication from the provisions of Art. 816, Civil Code, which allows even an alien
who is abroad to make a will in conformity with our Civil Code.

SUGGESTED ANSWER:
4. The intrinsic validity of his will is governed by Philippine law, it being his
national law. (Art. 16, Civil Code)

Notes:

Under the Civil Code, the intrinsic validity of a will of a testator is governed by
his national law.

ARTICLE 15. Laws regarding family rights and duties, status, condition and
legal capacity of persons are binding upon citizens of the Philippines, even though
living abroad. (Re-engineered)

(National law of a person shall govern his family rights and duties, status,
condition and legal capacity)

ARTICLE 16. Real property or personal property is subject to the law of the
country where it is situated.

However, intestate and testamentary successions, both with respect to the order
of succession and to the amount of successional rights, capacity to succeed, and to the
intrinsic validity of testamentary provisions, shall be regulated by the national law of
the person whose succession is under consideration, whatever may be the nature of the
property and regardless of the country wherein said property may be found. (10a) (Re-
engineered)

(National law of the decedent shall govern)

ARTICLE 17. The forms and solemnities of contracts, wills, and other public
instruments shall be governed by the laws of the country where they are executed.

When contracts, wills, and other public instruments are executed before the
diplomatic or consular officials of the Republic of the Philippines in a foreign
country, the solemnities established by Philippine laws shall be observed in their
execution because office of . diplomatic or consular officials of the Republic of the
Philippines in a foreign country are considered extension of jurisdiction of
Philippines.
Prohibitive laws concerning persons and their acts or property shall not be
rendered ineffective by laws or judgments promulgated in a foreign country or
conventions agreed upon in a foreign country

Those that pertains to public order, public policy and good customs shall not be
rendered ineffective by laws or judgments promulgated, or by determinations or
conventions agreed upon in a foreign country. (Re-engineered)

ARTICLE 15. Laws relating to family rights and duties, or to the status,
condition and legal capacity of persons are binding upon citizens of the Philippines,
even though living abroad. (9a)

ARTICLE 16. Real property as well as personal property is subject to the law of
the country where it is situated.

However, intestate and testamentary successions, both with respect to the order
of succession and to the amount of successional rights and to the intrinsic validity of
testamentary provisions, shall be regulated by the national law of the person whose
succession is under consideration, whatever may be the nature of the property and
regardless of the country wherein said property may be found. (10a)

ARTICLE 17. The forms and solemnities of contracts, wills, and other public
instruments shall be governed by the laws of the country in which they are executed.

When the acts referred to are executed before the diplomatic or consular officials
of the Republic of the Philippines in a foreign country, the solemnities established by
Philippine laws shall be observed in their execution.

Prohibitive laws concerning persons, their acts or property, and those which
have for their object public order, public policy and good customs shall not be
rendered ineffective by laws or judgments promulgated, or by determinations or
conventions agreed upon in a foreign country.

The capacity to contract marriage is governed by the national law of the


contracting parties; successional rights are governed by the national law of the
decedent; The extrinsic validity of the last will and testament is governed by the
national law of the testator;

Applicable Laws; Arts 15, 16, 17 (2002)


Felipe and Felisa, both Filipino citizens, were married in Malolos, Bulacan on
June 1, 1950. In 1960 Felipe went to the United States, becoming a U.S. citizen in
1975. In 1980 they obtained a divorce from Felisa, who was duly notified of the
proceedings. The divorce decree became final under California Law. Coming back to
the Philippines in 1982, Felipe married Sagundina, a Filipino Citizen. In 2001, Filipe,
then domiciled in Los Angeles, California, died, leaving one child by Felisa, and
another one by Sagundina. He left a will which he left his estate to Sagundina and his
two children and nothing to Felisa. Sagundina files a petition for the probate of
Felipe’s will. Felisa questions the intrinsic validity of the will, arguing that her
marriage to Felipe subsisted despite the divorce obtained by Felipe because said
divorce is not recognized in the Philippines. For this reason, she claims that the
properties and that Sagundina has no successional rights.

A. Is the divorce secured by Felipe in California recognizable and


valid in the Philippines? How does it affect Felipe’s marriage to Felisa? Explain.
(2%). Yes, art 26

B. What law governs the formalities of the will? Explain. (1%) where it was
executed, national law, philippines

C. Will Philippine law govern the intrinsic validity of the will? Explain.
(2%) no, national law of felipe

SUGGESTED ANSWER:
A. (1.) The divorce secured by Felipe in California is recognizable and valid in
the Philippines because he was no longer a Filipino at that time he secured it. Aliens
may obtain divorces abroad which may be recognized in the Philippines provided that
they are valid according to their national law (Van Dorn V. Romillo, Jr., 139 SCRA
139 [1985]; Quita v. Court of Appeals, 300 SCRA 406 [1998]; Llorente v. Court of
Appeals, 345 SCRA 595 [2000] ).

(2). With respect to Felipe the divorce is valid, but with respect to Felisa it is
not. The divorce will not capacitate Felisa to remarry because she and Felipe were
both Filipinos at the time of their marriage. However, in DOJ Opinion No. 134 series
of 1993, Felisa is allowed to remarry because the injustice sought to be corrected by
Article 26 also obtains in her case.

Notes:
In the case of Quita v. Court of Appeals, the Supreme court held that aliens may
obtain divorces abroad which may be recognized in the Philippines provided that they
are valid according to their national law.

Filipinos who became aliens are covered by this rule.

SUGGESTED ANSWER:
B. The foreigner who executes his will in the Philippines may observed the
formalities described in:
1. The Law of the country of which he is a citizen under Article 817 of the New
Civil Code, or the law of the Philippines being the law of the place of execution under
Article 17 of the New Civil Code.

Notes:
Under the Civil Code, the law which governs the formalities of a will of a
foreigner is his national law or the law of the country where it is executed.

This rule applies to Filipinos.

SUGGESTED ANSWER:
C. Philippine law will not govern the intrinsic validity of the will. Article 16 of
the New Civil Code provides that intrinsic validity of testamentary provisions shall be
governed by the National Law of the person whose succession is under consideration.
California law will govern the intrinsic validity of the will.

Applicable Laws; Capacity to Act (1998)


Francis Albert, a citizen and resident of New Jersey, U.S.A., under whose law
he was still a minor, being only 20 years of age, was hired by ABC Corporation of
Manila to serve for two years as its chief computer programmer. But after serving for
only four months, he resigned to join XYZ Corporation, which enticed him by
offering more advantageous terms. His first employer sues him in Manila for damages
arising from the breach of his contract of employment. He sets up his minority as a
defense and asks for annulment of the contract on that ground. The plaintiff
disputes this by alleging that since the contract was executed in the Philippines
under whose law the age of majority is 18 years, he was no longer a minor at the
time of perfection of the contract.

Will the suit prosper? [3%] no, nationality determines capacity, hence he is
minor under USA law
Suppose XYZ Corporation is impleaded as a codefendant, what would be
the basis of its liability, if any? [2%] liable as a tortfeasor

SUGGESTED ANSWER:
1. The suit will not prosper under Article 15, Civil Code, New Jersey law
governs Francis Albert's capacity to act, being his personal law from the standpoint of
both his nationality and his domicile. He was, therefore, a minor at the time he entered
into the contract.

Notes:
Under Civil Code, the legal capacity to contract is governed by the national law
of the contracting parties. Thus, the capacity to contract of Francis Albert is governed
by US law.

ARTICLE 15. Laws relating legal capacity of persons are binding upon citizens
of the Philippines, even though living abroad. (Re-engineered)

ALTERNATIVE ANSWER:
1. The suit will not prosper. Being a U.S. national, Albert's capacity to enter into
a contract is determined by the law of the State of which he is a national, under which
he is still a minor. This is in connection with Article 15 of the Civil Code which
embodies the said nationality principle of lex patriae. While this principle intended to
apply to Filipino citizens under that provision, the Supreme Court in Recto v. Harden
is of the view that the status or capacity of foreigners is to be determined on the basis
of the same provision or principle, i.e., by U.S. law in the present problem. Plaintiffs
argument does not hold true, because status or capacity is not determined by lex loci
contractus but by lex patriae.
ANOTHER ANSWER:
1. Article 17 of the Civil Code provides that the forms and solemnities of
contracts, wills and other public instruments shall be governed by the laws of the
country in which they are executed.

Since the contract of employment was executed in Manila, Philippine law should
govern. Being over 18 years old and no longer a minor according to Philippine Law,
Francis Albert can be sued. Thus, the suit of ABC Corporation against him for
damages will prosper.

Notes:
(Art. 17 speaks of forms and solemnities of contract; it does not speak of
capacity to enter into a contract. Art. 15 applies instead of Art.17).

SUGGESTED ANSWER:
2. XYZ Corporation, having enticed Francis Albert to break his contract
with the plaintiff, may be held liable for damages under Art. 1314, Civil Code.

Notes:

Under the Civil Code, a third person who induces another to violate his contract
shall be liable for damages to the other contracting party.

ALTERNATIVE ANSWER:
2. The basis of liability of XYZ Corporation would be Article 28 of the Civil
Code which states that: "Unfair competition in agricultural, commercial, or industrial
enterprises or in labor through the use of force, intimidation, deceit, machination or
any other unjust, oppressive or highhanded method shall give rise to a right of action
by the person who thereby suffers damage."
Notes:

ARTICLE 28. Unfair competition in industrial enterprises through the use of


unjust method shall give a right of action to person who thereby suffers damage.(Re-
engineered)

ARTICLE 28. Unfair competition in agricultural, commercial or industrial


enterprises or in labor through the use of force, intimidation, deceit, machination or
any other unjust, oppressive or highhanded method shall give rise to a right of action
by the person who thereby suffers damage.

ANOTHER ANSWER:
2. No liability arises. The statement of the problem does not in any way suggest
intent, malice, or even knowledge, on the part of XYZ Corporation as to the
contractual relations between Albert and ABC Corporation.

Applicable Laws; Capacity to Buy Land (1995)


3. What law governs the capacity of the Filipino to buy the land? Explain
your answer and give its legal basis. Lex rei sitae, under the law, the capacity of a
person to acquire personal or real property is governed by the place where such
properties are situated.

SUGGESTED ANSWER:
Philippine law governs the capacity of the Filipino to buy the land. In addition to
the principle of lex rei sitae given above. Article 15 of the NCC specifically provides
that Philippine laws relating to legal capacity of persons are binding upon citizens
of the Philippines no matter where they are.

Notes:
Under the Civil Code, the Philippine law shall govern the capacity of Filipinos
to buy land.

Applicable Laws; Capacity to Contract (1995)


2. What law governs the capacity of the Japanese to sell the land? Explain
your answer and give its legal basis. Japan- national law

SUGGESTED ANSWER:
Japanese law governs the capacity of the Japanese to sell the land being his
personal law on the basis of an interpretation of Art. 15, NCC.

Notes:
Under the Civil Code, the Japanese law shall govern the capacity of Japanese to
buy land.

ALTERNATIVE ANSWERS;
a) Since capacity to contract is governed by the personal law of an individual,
the Japanese seller's capacity should be governed either by his national law (Japanese
law) or by the law of his domicile, depending upon whether Japan follows the
nationality or domiciliary theory of personal law for its citizens.
b) Philippine law governs the capacity of the Japanese owner in selling the land.
While as a general rule capacity of persons is governed by the law of his nationality,
capacity concerning transactions involving property is an exception. Under Article 16
of the NCC the capacity of persons in transactions involving title to property is
governed by the law of the country where the property is situated. Since the
property is in the Philippines, Philippine law governs the capacity of the seller.
Notes:
Under the Civil Code, the national law of the Japanese shall govern his capacity
to enter into contract.
Applicable Laws; capacity to succeed (1991)
Jacob, a Swiss national, married Lourdes, a Filipina, in Berne, Switzerland.
Three years later, the couple decided to reside in the Philippines. Jacob subsequently
acquired several properties in the Philippines with the money he inherited from his
parents. Forty years later. Jacob died intestate, and is survived by several legitimate
children and duly recognized illegitimate daughter Jane, all residing in the Philippines.

(a) Suppose that Swiss law does not allow illegitimate children to inherit,
can Jane, who is a recognized illegitimate child, inherit part of the properties of
Jacob under Philippine law? No, national law of decedent

(b) Assuming that Jacob executed a will leaving certain properties to Jane
as her legitime in accordance with the law of succession in the Philippines, will
such testamentary disposition be valid?

SUGGESTED ANSWER:
A. No. As stated in the problem. Swiss law does not allow illegitimate children
to inherit Hence, Jane cannot inherit the property of Jacob under Philippine law.

Notes:
Under the Civil Code, the capacity to succeed shall be governed by the national
law of the decedent.

SUGGESTED ANSWER:
B. The testamentary disposition will not be valid if it would contravene Swiss
law; otherwise, the disposition would be valid. Unless the Swiss law is proved, it
would be presumed to be the same as that of Philippine law under the Doctrine of
Processual Presumption.

Notes:
Under the Civil Code, the capacity to succeed shall be governed by the national
law of the decedent. Therefore, the disposition is not valid.
ARTICLE 16. Real property or personal property is subject to the law of the
country where it is situated.

However, with respect to the order of succession, the amount of successional


rights and the intrinsic validity of testamentary provisions whether intestate or
testamentary, they shall be regulated by the national law of the person whose
succession is under consideration regardless of the country where said property may
be found. (Re-engineered)
(The enumerated items above are regulated by the law of the decedent)

ARTICLE 16. Real property as well as personal property is subject to the law of
the country where it is situated.

However, intestate and testamentary successions, both with respect to the order
of succession and to the amount of successional rights and to the intrinsic validity of
testamentary provisions, shall be regulated by the national law of the person whose
succession is under consideration, whatever may be the nature of the property and
regardless of the country wherein said property may be found. (10a)

Notes:
Order of succession, amount of successional rights, capacity to succeed and
the intrinsic validity of testamentary provisions are governed by the national law of
the decedent, regardless of the country where said property may be found.

Applicable Laws; contracts contrary to public policy (1996)


Alma was hired as a domestic helper in Hongkong by the Dragon Services,
Ltd., through its local agent. She executed a standard employment contract designed
by the Philippine Overseas Workers Administration (POEA) for overseas Filipino
workers. It provided for her employment for one year at a salary of US$1,000.00 a
month. It was submitted to and approved by the POEA. However, when she arrived in
Hongkong, she was asked to sign another contract by Dragon Services, Ltd. which
reduced her salary to only US$600.00 a month. Having no other choice, Alma signed
the contract but when she returned to the Philippines, she demanded payment of the
salary differential of US$400.00 a month. Both Dragon Services, Ltd. and its local
agent claimed that the second contract is valid under the laws of Hongkong, and
therefore binding on Alma. Is their claim correct? Explain.

SUGGESTED ANSWER:
Their claim is not correct. A contract is the law between the parties but the law
can disregard the contract if it is contrary to public policy. The provisions of the 1987
Constitution on the protection of labor and on social justice (Sec. 10. Art II) embody
a public policy of the Philippines. Since the application of Hongkong law in this case
is in violation of that public policy, the application shall be disregarded by our Courts.
(Cadalin v. POEA, 238 SCRA 762)

Notes:
In the case of Cadalin v. POEA, the Supreme court held that contracts may be
voided if they are contrary to public policy like constitutional guarantee on full
protection to labor.

ALTERNATIVE ANSWERS;
a) Their claim is not correct. Assuming that the second contract is binding under
Hongkong law, such second contract is invalid under Philippine law which recognizes
as valid only the first contract. Since the case is being litigated in the Philippines, the
Philippine Court as the forum will not enforce any foreign claim obnoxious to the
forum's public policy. There is a strong public policy enshrined in our Constitution on
the protection of labor. Therefore, the second contract shall be disregarded and the
first contract will be enforced. (Cadalin v. POEA, 238 SCRA 762).

SECTION 18. The State affirms labor as a primary social economic force. It
shall protect the rights of workers and promote their welfare.

Alternative Answer:

b) No, their claim is not correct. The second contract executed in Hongkong,
partakes of the nature of a waiver that is contrary to Philippine law and the public
policy governing Filipino overseas workers. Art. 17, provides that our prohibitive
laws concerning persons, their acts, or their property or which have for their object
public order, public policy and good customs shall not be rendered ineffective by laws
or conventions agreed upon in a foreign country. Besides, Alma's consent to the
second contract was vitiated by undue influence, being virtually helpless and under
financial distress in a foreign country, as indicated by the given fact that she signed
because she had no choice. Therefore, the defendants claim that the contract is valid
under Hongkong law should be rejected since under the DOCTRINE OF
PROCESSUAL PRESUMPTION a foreign law is deemed similar or identical to
Philippine law in the absence of proof to the contrary, and such is not mentioned in
the problem as having been adduced.

Notes:
ARTICLE 1390. The following contracts are voidable or annullable, even
though there may have been no damage to the contracting parties:

(1) Those where one of the parties is incapable of giving consent to a contract;

(2) Those where the consent is vitiated (ineffective) by mistake, violence,


intimidation, undue influence or fraud.

These contracts are binding, unless they are annulled by a proper action in court.
They are susceptible of ratification. (Re-engineered)

ARTICLE 1390. The following contracts are voidable or annullable, even


though there may have been no damage to the contracting parties:

(1) Those where one of the parties is incapable of giving consent to a contract;

(2) Those where the consent is vitiated by mistake, violence, intimidation, undue
influence or fraud.

These contracts are binding, unless they are annulled by a proper action in court.
They are susceptible of ratification. (n)

Applicable Laws; Contracts of Carriage (1995)


On 8 December 1991 Vanessa purchased from the Manila office of Euro-Aire
an airline ticket for its Flight No. 710 from Dallas to Chicago on 16 January 1992. Her
flight reservation was confirmed. On her scheduled departure Vanessa checked in on
time at the Dallas airport. However, at the check-in counter she discovered that she
was waitlisted with some other passengers because of intentional overbooking, a
Euro-Aire policy and practice. Euro-Alre admitted that Vanessa was not advised of
such policy when she purchased her plane ticket. Vanessa was only able to fly two
days later by taking another airline.

Vanessa sued Euro-Aire in Manila for breach of contract and damages.


Euro-Aire claimed that it cannot be held liable for damages because its practice
of overbooking passengers was allowed by the U.S. Code of Federal Regulations.
Vanessa on the other hand contended that assuming that the U.S. Code of
Federal Regulations allowed Intentional overbooking, the airline company
cannot invoke the U.S. Code on the ground that the ticket was purchased in
Manila, hence, Philippine law should apply, under which Vanessa can recover
damages for breach of contract of carriage. Decide. Discuss fully.
SUGGESTED ANSWER:
Vanessa can recover damages under Philippine law for breach of contract of
carriage, Philippine law should govern as the law of the place where the plane tickets
were bought and the contract of carriage was executed. In Zalamea v. Court of
Appeals (G.R No. 104235, Nov. 10, 1993)
Supreme Court applied Philippine law in recovery of damages for breach of
contract of carriage for the reason that it is the law of the place where the contract was
executed. Lex loci contractus

Notes:
In Zalamea v. Court of Appeals, the Supreme Court held that contracts entered
into in the Philippines shall be governed by Philippine law. Therefore, Vanessa can
recover damages for breach of contract because it is the law of the Philippines which
governs the parties, and not the law of US.

ALTERNATIVE ANSWER:
If the violation of the contract was attended with bad faith, there is a ground to
recover moral damages. But since there was a federal regulation which was the basis
of the act complained of, the airline cannot be in bad faith. Hence, only actual
damages can be recovered. The same is true with regards to exemplary damages.

Notes:
In one case, the Supreme Court held that since the plane ticket was bought in
Manila, the Philippine laws shall govern the contracting parties which are the common
carriers and the passenger in this case. Thus, Vanessa can recover damages from the
Euro-Aire.

Applicable Laws; Labor Contracts (1991)


A. The Japan Air Lines (JAL), a foreign corporation licensed to do business in
the Philippines, executed in Manila a contract of employment with Maritess Guapa
under which the latter was hired as a stewardess on the aircraft flying the Manila-
Japan-Manila route. The contract specifically provides that (1) the duration of the
contract shall be two (2) years, (2) notwithstanding the above duration, JAL may
terminate the agreement at any time by giving her notice in writing ten (10) days in
advance, and (3) the contract shall be construed as governed under and by the laws of
Japan and only the court in Tokyo, Japan shall have the jurisdiction to consider any
matter arising from or relating to the contract.
JAL dismissed Maritess on the fourth month of her employment without giving
her due notice. Maritess then filed a complaint with the Labor Arbiter for
reinstatement, backwages and damages. The lawyer of JAL contends that neither
the Labor Arbiter nor any other agency or court in the Philippines has
jurisdiction over the case in view of the above provision (3) of the contract which
Maritess voluntarily signed. The contract is the law between her and JAL.
Decide the issue.
Such contract is invalid because it violates public order which is labor
legislation of the [hilippines

B. Where under a State's own conflicts rule that domestic law of another
State should apply, may the courts of the former nevertheless refuse to apply the
latter? If so, under what circumstance?

SUGGESTED ANSWER:
A. Labor Legislations are generally intended as expressions of public policy on
employer-employee relations. The contract therefore, between Japan Air Lines (JAL)
and Maritess may apply only to the extent that its provisions are not inconsistent with
Philippine labor laws intended particularly to protect employees.

Under the circumstances, the dismissal of Maritess without complying with


Philippine Labor law would be invalid and any stipulation in the contract to the
contrary is considered void. Since the law of the forum in this case is the Philippine
law the issues should-be resolved in accordance with Philippine law.
B. The third paragraph of Art. 17 of the Civil Code provides that:

"Prohibitive laws concerning persons, their acts or property, and those which
have for their object public order, public policy and good customs shall not be
rendered ineffective by laws or judgments promulgated, or by determinations or
conventions agreed upon in a foreign country."

Accordingly, a state's own conflict of laws rule may, exceptionally be


inapplicable, given public policy considerations by the law of the forum.

Going into the specific provisions of the contract in question, I would rule as
follows:
1. The duration of the contract is not opposed to Philippine law and it can
therefore be valid as stipulated;
2. The second provision to the effect that notwithstanding duration, Japan Air
Lines (JAL) may terminate her employment is invalid, being inconsistent with our
Labor laws;

3. That the contract shall be construed as governed under and by the laws of
Japan and only the courts of Tokyo, Japan shall have jurisdiction, is invalid as clearly
opposed to the aforecited third paragraph of Arts. 17 and 1700 of the Civil Code,
which provides: Art. 17 states that contract executed here is governed by Philippine
law
Art. 1700. The relations between capital and labor are not merely contractual. They
are so impressed with public interest that labor contracts must yield to the common
good. Therefore, such contracts are subject to the special laws on labor unions,
collective bargaining, strikes and lockouts, closed shop, wages, working conditions,
hours of labor and similar subjects."

ALTERNATIVE ANSWER;
A. When a contract has a foreign element such as in the factual setting stated in
the problem where one of the parties is a foreign corporation, the contract can be
sustained as valid particularly the stipulation expressing that the contract is governed
by the laws of the foreign country. Given this generally accepted principle of
international law, the contract between Maritess and JAL is valid and it should
therefore be enforced.

Notes:
Under the Constitution, the State shall afford full protection to labor, local or
overseas. Therefore, the stipulation that it should be the law of Japan which governs
the contract between Japan Airlines and Marites Guapa is not valid because Japanese
law is against public policy.

ARTICLE 1306. The contracting parties may establish such stipulations,


clauses, terms and conditions as they may deem convenient, provided they are not
contrary to law, morals, good customs, public order, or public policy.

Applicable Laws; laws governing marriages (1992)


In 1989, Maris, a Filipino citizen, married her boss Johnson, an American
citizen, in Tokyo in a wedding ceremony celebrated according to Japanese laws. One
year later, Johnson returned to his native Nevada, and he validly obtained in that state
an absolute divorce from his wife Maris.

After Maris received the final judgment of divorce, she married her childhood
sweetheart Pedro, also a Filipino citizen, in a religious ceremony in Cebu City,
celebrated according to the formalities of Philippine law. Pedro later left for the
United States and became naturalized as an American citizen. Maris followed Pedro
to the United States, and after a serious quarrel, Maris filed a suit and obtained a
divorce decree issued by the court in the state of Maryland.

Maris then returned to the Philippines and in a civil ceremony celebrated in


Cebu City according to the formalities of Philippine law, she married her former
classmate Vincent likewise a Filipino citizen. a) Was the marriage of Maris and
Johnson valid when celebrated? Is their marriage still validly existing now? Reasons.
SUGGESTED ANSWER: yes, because it was in accordance to the place where it
was celebrated
(a) The marriage of Mans and Johnson was valid when celebrated because all
marriages solemnized outside the Philippines (Tokyo) in accordance with the laws in
force in the country where they are solemnized (Japan), and valid there as such, are
also valid in the Philippines.

Their marriage no longer validly subsists, because it has been dissolved by the
absolute divorce validly obtained by Johnson which capacitated Maris to remarry (Art.
26. Family Code).

Notes:
Under the Civil Code, all marriages solemnized outside of the Philippines are
valid if valid in the country where they were celebrated provided they are not contrary
to Philippine law.

Art. 26. All marriages solemnized outside the Philippines which are valid in that
country in accordance with the laws in force thereat shall also be valid in this country,
except those prohibited under Articles 35 (1), (4), (5) and (6), 3637 and 38. (17a)
{Reengineered}

Where a marriage between a Filipino citizen and a foreigner is validly celebrated


and a divorce is thereafter validly obtained abroad by the alien spouse capacitating
him or her to remarry, the Filipino spouse shall have capacity to remarry under
Philippine law. (As amended by Executive Order 227)
Art. 26. All marriages solemnized outside the Philippines, in accordance with
the laws in force in the country where they were solemnized, and valid there as such,
shall also be valid in this country, except those prohibited under Articles 35 (1), (4),
(5) and (6), 3637 and 38. (17a)

Where a marriage between a Filipino citizen and a foreigner is validly celebrated


and a divorce is thereafter validly obtained abroad by the alien spouse capacitating
him or her to remarry, the Filipino spouse shall have capacity to remarry under
Philippine law. (As amended by Executive Order 227)

Notes:
Under the Civil Code, if a foreigner spouse obtained a decree of divorce in a
foreign country capacitating him or her to marry, the Filipino spouse shall also have
the capacity to marry.

Applicable Laws; laws governing marriages (2003)


Gene and Jane, Filipino, met and got married in England while both were
taking up post-graduate courses there. A few years after their graduation, they
decided to annul their marriage. Jane filed an action to annul her marriage to Gene in
England on the ground of latter’s sterility, a ground for annulment of marriage in
England. The English court decreed the marriage annulled. Returning to the
Philippines, Gene asked you whether or not he would be free to marry his former
girlfriend. What would your legal advice be? 5%

SUGGESTED ANSWER:
No, Gene is not free to marry his former girlfriend. His marriage to Jane is valid
according to the forms and solemnities of British law, is valid here (Article 17, 1st
par., NCC). However, since Gene and Jane are still Filipinos although living in
England, the dissolution of their marriage is still governed by Philippine law (Article
15, NCC). Since, sterility is not one of the grounds for the annulment of a marriage
under Article 45 of the Family Code, the annulment of Gene’s marriage to Jane on
that ground is not valid in the Philippines (Article 17, NCC)
ALTERNATIVE ANSWER:
Yes, Gene is free to marry his girlfriend because his marriage was validly
annulled in England. The issue of whether or not a marriage is voidable, including the
grounds therefore, is governed by the law of the place where the marriage was
solemnized (lex loci celebrationis). Hence, even if sterility is not a ground to annul
the marriage under the Philippine law, the marriage is nevertheless voidable because
sterility makes the marriage voidable under English law. Therefore, annulment of the
marriage in England is valid in the Philippines.

Notes:

Under the Civil Code, sterility is not a ground for annulment of marriage.
Therefore, the marriage of Gene and Jane is still subsisting.

Art. 26. All marriages solemnized outside the Philippines, in accordance with
the laws in force in the country where they were solemnized, and valid there as such,
shall also be valid in this country, except those prohibited under Articles 35 (1), (4),
(5) and (6), 3637 and 38. (17a)

Where a marriage between a Filipino citizen and a foreigner is validly


celebrated and a divorce is thereafter validly obtained abroad by the alien spouse
capacitating him or her to remarry, the Filipino spouse shall have capacity to remarry
under Philippine law. (As amended by Executive Order 227)

Applicable Laws; Sale of Real Property (1995)


While in Afghanistan, a Japanese by the name of Sato sold to Ramoncito, a
Filipino, a parcel of land situated in the Philippines which Sato inherited from his
Filipino mother.

1. What law governs the formality in the execution of the contract of sale ?
Explain your answer and give its legal basis. Afghanistan, lex loci celebrationis

SUGGESTED ANSWER:
Under Art. 16 par. 1, NCC, real property is subject to the law of the country
where it is situated. Since the property is situated in the Philippines, Philippine law
applies. The rule of lex rei sitae in Article 16 prevails over lex loci contractu in Article
17 of the NCC.

Notes:
Real property is subject to the law of the country where it is situated. Therefore,
the law of the Philippines shall govern the formality of the contract of sale because the
parcel of land is situated in the Philippines.

ALTERNATIVE ANSWER:
Afghanistan law governs the formal requirements of the contract since the
execution is in Afghanistan. Art. 17 of the Civil Code provides that the forms and
solemnities of contracts, wills, and other public instruments shall be governed by the
laws of the country in which they are executed. However, if the contract was executed
before the diplomatic or consular officials of the Republic of the Philippines in
Afghanistan, Philippine law shall apply.

Notes:
Under the Section 17 of the Civil Code, forms and solemnities of contract shall
be governed by the law of the place where the contract is executed.

ARTICLE 17. The forms and solemnities of contracts, wills, and other public
instruments shall be governed by the laws of the country in which they are executed.

When the acts referred to are executed before the diplomatic or consular officials
of the Republic of the Philippines in a foreign country, the solemnities established by
Philippine laws shall be observed in their execution.

Prohibitive laws concerning persons, their acts or property, and those which
have for their object public order, public policy and good customs shall not be
rendered ineffective by laws or judgments promulgated, or by determinations or
conventions agreed upon in a foreign country. (11a)

Applicable Laws; Succession; Intestate & Testamentary (2001)


Alex was born a Filipino but was a naturalized Canadian citizen at the time of
his death on December 25, 1998. He left behind a last will and testament in which he
bequeathed all his properties, real and personal, in the Philippines to his
acknowledged illegitimate Fillpina daughter and nothing to his two legitimate
Filipino sons. The sons sought the annulment of the last will and testament on the
ground that it deprived them of their legitimes but the daughter was able to
prove that there were no compulsory heirs or legitimes under Canadian law.
Who should prevail? Why? (5%) the illegitimate pursuant to the provision that the
amount, order, intrinsic validity, and capacity to succeed is governed by the law of
the testatot

SUGGESTED ANSWER:
The daughter should prevail because Article 16 of the New Civil Code provides
that intestate and testamentary succession shall be governed by the national law of the
person whose succession is under consideration.
Notes:
Succession whether testate or intestate shall be governed by the national law of
the decedent. Alex was already a Canadian when he died. Therefore, Canadian law
should govern the succession to the deceased.

Applicable Laws; Sucession of Aliens (1995)


Michelle, the French daughter of Penreich, a German national, died in Spain
leaving real properties in the Philippines as well as valuable personal properties in
Germany.

1. What law determines who shall succeed the deceased? Explain your
answer and give its legal basis. Law on France pursuant to art. 16

2. What law regulates the distribution of the real properties in the


Philippines? Explain your answer and give its legal basis. Law on France pursuant
to art. 16

3. What law governs the distribution of the personal properties in


Germany? Explain your answer and give its legal basis. Law on France Art. 16

SUGGESTED ANSWER:
Assuming that the estate of the decedent is being settled in the Philippines)
1. The national law of the decedent (French law) shall govern in determining
who will succeed to his estate. The legal basis is Art. 16 par. 2, NCC.

Notes:
Under Article 16 of the Civil Code, succession to the deceased shall be governed
by his national law.

ALTERNATIVE ANSWER:
French law shall govern the distribution of his real properties in the Philippines
except when the real property is land which may be transmitted to a foreigner only by
hereditary succession.

SUGGESTED ANSWER:
2. The distribution of the real properties in the Philippines shall be governed by
French law. The legal basis is Art. 16, NCC).
SUGGESTED ANSWER:
3. The distribution of the personal properties in Germany shall be governed by
French law. The legal basis is Art. 16, NCC).
Notes:

ARTICLE 16. Real property as well as personal property is subject to the law of
the country where it is situated.

However, intestate and testamentary successions, both with respect to the order
of succession and to the amount of successional rights and to the intrinsic validity of
testamentary provisions, shall be regulated by the national law of the person whose
succession is under consideration, whatever may be the nature of the property and
regardless of the country wherein said property may be found. (10a)

Applicable Laws; Wills executed abroad (1993)


A, a Filipino, executed a will in Kuwait while there as a contract worker.
Assume that under the laws of Kuwait, it is enough that the testator affix his signature
to the presence of two witnesses and that the will need not be acknowledged before a
notary public. May the will be probated in the Philippines?
SUGGESTED ANSWER:
Yes. Under Articles 815 and 17 of the Civil Code, the formality of the
execution of a will is governed by the law of the place of execution. If the will was
executed with the formalities prescribed by the laws of Kuwait and valid there as
such, the will is valid and may be probated in the Philippines.

Notes:
Under the Civil Code, formality of the execution of a will is governed by the law
of the place of execution. Therefore, the will of A can be probated in the Philippines
because it was executed according to the law of Kuwait, the place of execution.

ARTICLE 815. When a Filipino is in a foreign country, he is authorized to make


a will in any of the forms established by the law of that country. Such will may be
probated in the Philippines.(Reengineered)

ARTICLE 815. When a Filipino is in a foreign country, he is authorized to make


a will in any of the forms established by the law of the country in which he may be.
Such will may be probated in the Philippines. (n)
Definition; Cognovit; Borrowing Statute; Characterization(1994)
In Private International Law (Conflict of Laws) what is: 1} Cognovit? 2) A
borrowing statute? 3) Characterization?

SUGGESTED ANSWER:
a) COGNOVIT is a confession of judgment whereby a portion of the complaint
is confessed by the defendant who denies the rest thereof (Philippine law Dictionary,
3rd Ed.) (Ocampo v. Florenciano, L-M 13553, 2/23/50).

b) COGNOVIT is a "statement of confession" Oftentimes, it is referred to as a


"power of attorney" or simply as a "power", it is the written authority of the debtor
and his direction to the clerk of the district court, or justice of the peace to enter
judgment against the debtor as stated therein. (Words and Phrases, vol. 7, pp. 115-
166).

c) COGNOVIT is a plea in an action which acknowledges that the defendant did


undertake and promise as the plaintiff in its declaration has alleged, and that it cannot
deny that it owes and unjustly detains from the plaintiff the sum claimed by him in his
declaration, and consents that judgment be entered against the defendant for a certain
sum. [Words and Phrases, vol. 7, pp. 115-166).

d) COGNOVIT is a note authorizing a lawyer for confession of judgment by


defendant.
2) "BORROWING STATUTE" -Laws of the state or jurisdiction used by
another state in deciding conflicts questioned involved in the choice of law (Black's
Law Dictionary, 5th ed. 1979).
3) a) "CHARACTERIZATION" is otherwise called "classification" or
"qualification." It is the process of assigning a disputed question to its correct legal
category (Private International Law, Salonga).
b) "CHARACTERIZATION" is a process in determining under what category a
certain set of facts or rules fall. (Paras, Conflict of Laws, p. 94. 1984 ed.)

Notes:
1. Cognovit is a note which authorizes a lawyer to confess a judgment; the
confession is actually made by the defendant.
2. Borrowing statute means that laws of one state is used by another to
decide conflicts.
3. Characterization is a process to classify under what category a certain set
of facts or rules fall.

Definition; forum non-conveniens; long-arm statute (1994)


1) What is the doctrine of Forum non conveniens? 2) What is a "long arm
statute"?
Where the ends of justice strongly indicates that a controversy is more suitable to be
tried elsewhere, the forum shall resist upon its jurisdiction
SUGGESTED ANSWER:
1) a) FORUM NON CONVENIENS is a principle in Private International Law
that where the ends of justice strongly indicate that the controversy may be more
suitably tried elsewhere, then jurisdiction should be declined and the parties relegated
to relief to be sought in another forum. (Moreno. Philippine Law Dictionary, p. 254,
1982 ed.).

b) Where in a broad sense the ends of justice strongly indicate that the
controversy may be more suitably tried elsewhere, then jurisdiction should be declined
and the parties relegated to relief to be sought in another forum. (Handbook on Private
International Law, Aruego).

c) FORUM NON CONVENIENS means simply that a court may resist


imposition upon its jurisdiction even when jurisdiction is authorized by the letter of a
general venue statute. (Salonga. Private International Law. p, 51. 1967 ed.)

d) Forum non conveniens is a doctrine whereby a court of law having full


Jurisdiction over a case brought in a proper venue or district declines to determine the
case on its merits because Justice would be better served by the trial over the case in
another jurisdiction. (Webster's Dictionary)
SUGGESTED ANSWER:
(2} a) LONG ARM STATUTE is a legislative act which provides for personal
jurisdiction, via substituted service or process, over persons or corporations which
are nonresidents of the state and which voluntarily go into the state, directly or by
agent or communicate with persons in the state for limited purposes, inactions which
concern claims relating to performance or execution of those purposes (Black's Law
Dictionary, 5th Ed. 1979).
b) Long arm statute refers simply to authorized substituted service.

Notes:
a) Forum non conveniens is a doctrine which a court of law that has Jurisdiction
over a case declines to try the case because another jurisdiction is chosen so that
justice may be served better.
b) Long arm statute refers simply to authorized substituted service.

Divorce; effect of divorce granted to former Filipinos; Renvoi Doctrine (1997)


In 1977, Mario and Clara, both Filipino citizens, were married in the
Philippines. Three years later, they went to the United States of America and
established their residence in San Francisco, California. In 1987, the couple applied
for, and were granted, U.S. citizenship. In 1989, Mario, claiming to have been
abandoned by Clara, was able to secure a decree of divorce in Reno, Nevada, U.S.A.
In 1990, Mario returned to the Philippines and married Juana who knew
well Mario's past life. Is the marriage between Mario and Juana valid? Would
the renvoi doctrine have any relevance to the case?

SUGGESTED ANSWER:
Yes, because Phil law recognizes the divorce between Mario and Clara as valid.

SUGGESTED ANSWER:
(b) No. The renvoi doctrine is relevant in cases where one country applies the
domiciliary theory and the other the nationality theory, and the issue involved is
which of the laws of the two countries should apply to determine the order of
succession, the amount of successional rights, or, the intrinsic validity of testamentary
provisions. Such issue is not involved in this case.

Notes:
The couple were no longer Filipinos when they divorced, so their divorce is
valid.

(b) No. Renvoi doctrine has no relevance to the case because it does not involve
a conflict of the application of nationality theory and domiciliary theory.

Renvoi doctrine is a doctrine applicable in succession which two countries apply


two different laws; one country applies the domiciliary domiciliary theory and the
other applies the nationality theory.

ALTERNATIVE ANSWER:
Yes. "Renvoi" - which means "referring back" is relevant because here, we are
applying U.S. law to Mario, being already its citizen, although the formalities of the
second marriage will be governed by Philippine law under the principle of lex loci
celebrationis.

Domiciliary theory vs. Nationality Theory (2004)


Distinguish briefly but clearly between: Domiciliary theory and nationality
theory of personal law. (5%)

SUGGESTED ANSWER:
DOMICILIARY THEORY posits that the personal status and rights of a
person are governed by the law of his domicile or the place of his habitual residence.
The NATIONALITY THEORY, on the other hand, postulates that it is the law of the
person's nationality that governs such status and rights

Notes:
Domiciliary theory means that the law of his residence is applicable to the
personal status and rights of a person, while nationality theory means that the
national law of a person governs his rights and personal status.

Forum Non Conveniens & Lex Loci Contractus (2002)


Felipe is a Filipino citizen. When he went to Sydney for vacation, he met a
former business associate, who proposed to him a transaction which took him to
Moscow. Felipe brokered a contract between Sydney Coals Corp. (Coals), an
Australian firm, and Moscow Energy Corp. (Energy), a Russian firm, for Coals to
supply coal to Energy on a monthly basis for three years. Both these firms were not
doing, and still do not do, business in the Philippines. Felipe shuttled between Sydney
and Moscow to close the contract. He also executed in Sydney a commission
contract with Coals and in Moscow with Energy, under which contracts he was
guaranteed commissions by both firms based on a percentage of deliveries for the
three-year period, payable in Sydney and in Moscow, respectively, through deposits in
accounts that he opened in the two cities. Both firms paid Felipe his commission for
four months, after which they stopped paying him. Felipe learned from his contacts,
who are residents of Sydney and Moscow, that the two firms talked to each other and
decided to cut him off. He now files suit in Manila against both Coals and Energy for
specific performance.

A. Define or explain the principle of “lex loci contractus”. (2%)


B. Define or explain the rule of “forum non conveniens” (3%)

C. Should the Philippine court assume jurisdiction over the case? Explain.
(5%)

SUGGESTED ANSWER:
A. LEX LOCI CONTRACTUS may be understood in two senses, as follows:

(1) It is the law of the place where contracts, wills, and other public instruments
are executed and governs their “forms and solemnities”, pursuant to the first
paragraph, Article 17 of the New Civil Code; or

(2) It is the proper law of the contract; e.i., the system of law intended to govern
the entire contract, including its essential requisites, indicating the law of the place
with which the contract has its closest connection or where the main elements of the
contract converge. As illustrated by Zalamea v. Court of Appeals (228 SCRA 23
[1993]), it is the law of the place where the airline ticket was issued, where the
passengers are nationals and residents of, and where the defendant airline company
maintained its office.

ALTERNATIVE ANSWER:
A. Under the doctrine of lex loci contractus, as a general rule, the law of the
place where a contract is made or entered into governs with respect to its nature and
validity, obligation and interpretation. This has been said to be the rule even though
the place where the contract was made is different from the place where it is to be
performed, and particularly so, if the place of the making and the place of
performance are the same (United Airline v. CA, G.R. No. 124110, April 20, 2001).

Notes:
A. Under the doctrine of lex loci contractus, the law of the place where the
contract was executed shall govern its formality and solemnity.
B. Under the forum of non conveniens, a court that has jurisdiction over a
certain issue may refuse to try it if it is in a view that justice may better be served in
another forum.
C. No. The Philippine court should not assume jurisdiction over the case
because under the principle of lex loci contractus, the law of the place where the
contract is executed shall apply.
SUGGESTED ANSWER:
B. FORUM NON CONVENIENS means that a court has discretionary
authority to decline jurisdiction over a cause of action when it is of the view that the
action may be justly and effectively adjudicated elsewhere.

Notes:
Under the forum of non conveniens, courts may refuse to assume jurisdiction
over a case if the same could be justly and effectively adjudicated elsewhere.

SUGGESTED ANSWER:
C. No, the Philippine courts cannot acquire jurisdiction over the case of Felipe.
Firstly, under the rule of forum non conveniens, the Philippine court is not the
convenient forum as all of the incidents of the case happened outside the Philippines.
Neither are both Coals and Energy doing business inside the Philippines. Secondly,
the contracts were not perfected in the Philippines. Under the principle of lex loci
contractus, the law of the place where the contract is made shall apply. Lastly, the
Philippine court has no power to determine the facts surrounding the execution of said
contracts. And even if a proper decision could be reached, such would have no biding
effect on Coals and Energy as the court was not able to acquire jurisdiction over the
said corporations. (Manila Hotel Corp. v. NLRC. 343 SCRA 1, 1314[2000])

Notes:
In Manila Hotel Corp. v. NLRC, it was held that the Philippine court is not a
convenient forum since all the incidents of the case occurred outside the Philippines.
This is the rule applicable under the doctrine of forum non conveniens. This doctrine
is also applicable when jurisdiction over the parties cannot be obtained.

Nationality Theory (2004)


PH and LV are HK Chinese. Their parents are now Filipino citizens who live in
Manila. While still students in MNS State, they got married although they are first
cousins. It appears that both in HK and in MNS State first cousins could marry
legally. They plan to reside and set up business in the Philippines. But they have been
informed, however, that the marriage of first cousins here is considered void from the
beginning by reason of public policy. They are in a dilemma. They don’t want to
break Philippine law, much less their marriage vow. They seek your advice on
whether their civil status will be adversely affected by Philippine domestic law?
What is your advice? (5%)
SUGGESTED ANSWER:
My advise is as follows: The civil status of' PH and LV will not be adversely
affected by Philippine law because they are nationals of Hong Kong and not Filipino
citizens. Being foreigners, their status, conditions and legal capacity in the Philippines
are governed by the law of Hong Kong, the country of which they are citizens. Since
their marriage is valid under Hong Kong law, it shall be valid and respected in the
Philippines.

Notes:
Under the Civil Code, legal capacity of a person is governed by his national law.
Naturalization (2003)
Miss Universe, from Finland, came to the Philippines on a tourist visa. While
in this country, she fell in love with and married a Filipino doctor. Her tourist visa
having been expired and after the maximum extension allowed therefore, the Bureau
of Immigration and Deportation (BID) is presently demanding that she immediately
leave the country but she refuses to do so, claiming that she is already a Filipino
Citizen by her marriage to a Filipino citizen. Can the BID still order the deportation
of Miss Universe? Explain. 5%
SUGGESTED ANSWER:
Yes, the BID can order the deportation of Miss Universe. The marriage of an
alien woman to a Filipino does not automatically make her a Filipino Citizen. She
must first prove in an appropriate proceeding that she does not have any
disqualification for Philippine citizenship. (Yung Uan Chu v. Republic of the
Philippines, 158 SCRA 593 [1988]). Since Miss Universe is still a foreigner, despite
her marriage to a Filipino doctor, she can be deported upon expiry of her allowable
stay in the Philippines.
ANOTHER SUGGESTED ANSWER:
No, the Bureau of Immigration cannot order her deportation. An alien woman
marrying a Filipino, native-born or naturalized, becomes ipso facto a Filipino if she is
not disqualified to be a citizen of the Philippines (Mo Ya Lim v Commission of
Immigration, 41 SCRA 292 [1971]), (Sec 4, Naturalization Law). All that she has to
do is prove in the deportation proceeding the fact of her marriage and that she is not
disqualified to become a Filipino Citizen.

Notes:
In Mo Ya Lim v Commission of Immigration, it was held that an alien woman
who married a Filipino becomes ipso facto a Filipino if she satisfactorily proves that
she is not disqualified to be a citizen of the Philippines.
ANOTHER SUGGESTED ANSWER:
It depends. If she is disqualified to be a Filipino citizen, she may be deported. If
she is not disqualified to be a Filipino citizen, she may not be deported. An alien
woman who marries a Filipino citizen becomes one. The marriage of Miss Universe
to the Filipino doctor did not automatically make her a Filipino citizen. She still has to
prove that she is not disqualified to become a citizen.

Theory; significant relationships theory (1994)


Able, a corporation domiciled in State A, but, doing business in the
Philippines, hired Eric, a Filipino engineer, for its project in State B. In the contract
of employment executed by the parties in State B, it was stipulated that the contract
could be terminated at the company's will, which stipulation is allowed in State B.
When Eric was summarily dismissed by Able, he sued Able for damages in the
Philippines. Will the Philippine court apply the contractual stipulation?

SUGGESTED ANSWER:
a) Using the "SIGNIFICANT RELATIONSHIPS THEORY", there are contracts
significant to the Philippines. Among these are that the place of business is the
Philippines, the employee concerned is a Filipino and the suit was filed in the
Philippines, thereby justifying the application of Philippine law. In the American
Airlines case the Court held that when what is involved is PARAMOUNT STATE
INTEREST such as the protection of the rights of Filipino laborers, the court can
disregard choice of forum and choice of law. Therefore the Philippine Court should
not apply the stipulation in question.

ALTERNATIVE ANSWER:
b) No, lex fori should be applied because the suit is filed in Philippine courts and
Eric was hired in the Philippines. The Philippine Constitution affords full protection to
labor and the stipulation as to summary dismissal runs counter to our fundamental and
statutory laws.

Notes:
In the American Airlines case, it was held that when what is involved is
paramount state interest such as the protection of the rights of Filipino laborers, the
court can disregard choice of forum and choice of law. Thus, he Philippine Court
should not apply the stipulation in question.
Torts; Prescriptive Period (2004)
In a class suit for damages, plaintiffs claimed they suffered injuries from torture
during martial law. The suit was filed upon President EM’s arrival on exile in HI, a
U.S. state. The court in HI awarded plaintiffs the equivalent of P100 billion under the
U.S. law on alien tort claims. On appeal, EM’s Estate raised the issue of prescription.
It argued that since said U.S. law is silent on the matter, the court should apply: (1)
HI’s law setting a two-year limitation on tort claims; or (2) the Philippine law which
appears to require that claims for personal injury arising from martial law be brought
within one year.

Plaintiffs countered that provisions of the most analogous federal statute, the
Torture Victims Protection Act, should be applied. It sets ten years as the period for
prescription. Moreover, they argued that equity could toll the statute of limitations for
it appeared that EM had procured Constitutional amendments granting himself and
those acting under his direction immunity from suit during his tenure.

In this case, has prescription set in or not? Considering the differences in


the cited laws, which prescriptive period should be applied: one year under
Philippine law, two years under HI’s law, ten years under U.S. federal law, or
none of the above? Explain. (5%)

SUGGESTED ANSWER:
The US Court will apply US law, the law of the forum, in determining the
applicable prescriptive period. While US law is silent on this matter, the US Court
will not apply Philippine law in determining the prescriptive period. It is generally
affirmed as a principle in private international law that procedural law is one of the
exceptions to the application of foreign law by the forum. Since prescription is a
matter of procedural law even in Philippine jurisprudence, (Codaltn v. POEA/
JVLRC/Broum and Root International, 238 SCRA 721 [1994]), the US Court will
apply either HI or Federal law in determining the applicable prescriptive period and
not Philippine law. The Restatement of American law affirms this principle.

Notes:
In Codaltn v. POEA/ JVLRC/Broum and Root International, the Supreme Court
held that the law of the forum which is the US law in this case shall apply when the
cause of action arosed from torts. Therefore, the prescriptive period is 10 years under
the US law.
ADOPTION

Adoption; Use of Surname of her Natural Mother (2006)


May an illegitimate child, upon adoption by her natural father, use the
surname of her natural mother as the middle name? (2.5%) y

SUGGESTED ANSWER:
Yes, an illegitimate child, upon adoption by her natural father, can use the
surname of her natural mother as her middle name. The Court has ruled that there is
no law prohibiting an illegitimate child adopted by her natural father to use, as middle
name, her mother's surname. What is not prohibited is allowed. After all, the use of
the maternal name as the middle name is in accord with Filipino culture and customs
and adoption is intended for the benefit of the adopted [In re: Adoption of Stephanie
Nathy Astorga Garcia, G.R. No. 148311, March 31, 2005; Rabuya, The Law on
Persons and Family Relations, p. 613].

Notes:
In re: Adoption of Stephanie Nathy Astorga Garcia, it was held that an
illegitimate child, upon adoption by her natural father, can use the surname of her
natural mother as her middle name. The Court has ruled that there is no law
prohibiting an illegitimate child adopted by her natural father to use, as middle name,
her mother's surname. What is not prohibited is allowed. After all, the use of the
maternal name as the middle name is in accord with Filipino culture and customs.

Inter-Country Adoption; Formalities (2005)


Hans Berber, a German national, and his Filipino wife, Rhoda, are permanent
residents of Canada. They desire so much to adopt Magno, an 8-year old orphaned
boy and a baptismal godson of Rhoda. Since the accidental death of Magno's parents
in 2004, he has been staying with his aunt who, however, could hardly afford to feed
her own family. Unfortunately, Hans and Rhoda cannot come to the Philippines to
adopt Magno although they possess all the qualifications as adoptive parents.

Is there a possibility for them to adopt Magno? How should they go about
it? (5%)
SUGGESTED ANSWER:
Yes, it is possible for Hans and Rhoda to adopt Magno. Republic Act No. 8043
or the Inter-Country Adoption Act, allows aliens or Filipinos permanently residing
abroad to apply for inter-country adoption of a Filipino child. The law however
requires that only legally free child, or one who has been voluntarily or involuntarily
committed to the DSWD or any of its accredited agencies, may be subject of inter-
country adoption. The law further requires that aside from possessing all the
qualifications, the adoptive parents must come from a country where the Philippines
has diplomatic relations and that the government maintains a similarly accredited
agency and that adoption is allowed under the national law of the alien. Moreover, it
must be further shown that all possibilities for a domestic adoption have been
exhausted and the inter-country adoption is best for the interest of the child.

Hans and Rhoda have to file an application to adopt Magno, either with the
Regional Trial Court having jurisdiction over Magno or with the Inter-Country
Adoption Board in Canada. Hans and Rhoda will then undergo a trial custody for six
(6) months from the time of placement. It is only after the lapse of the trial custody
that the decree of adoption can be issued.

Notes:
Under Republic Act No. 8043 or the Inter-Country Adoption Act, aliens or
Filipinos permanently residing abroad may apply for inter-country adoption of a
Filipino child provided that the child to be adopted is a legally free child or one who
is committed to the DSWD and the adoptive parents must come from the country
which has diplomatic relation with the Philippines.
The applicants may file an application to adopt Magno, either with the Regional
Trial Court having jurisdiction over Magno or with the Inter-Country Adoption
Board in Canada. The applicants will then undergo a trial custody for six (6) months
from the time of placement. It is only after the lapse of the trial custody that the
decree of adoption can be issued.

Parental Authority; Rescission of Adoption (1994)


In 1975, Carol begot a daughter Bing, out of wedlock. When Bing was ten years
old, Carol gave her consent for Bing's legal adoption by Norma and Manuel, which
was granted by the court in 1990. In 1991, Carol learned that Norma and Manuel were
engaged in a call-girl-ring that catered to tourists. Some of the girls lived with Norma
and Manuel. Carol got Bing back, who in the first place wanted to return to her natural
mother. 1) Who has a better right to the custody of Bing, Carol or Norma? 2) Aside
from taking physical custody of Bing, what legal actions can Carol take to protect
Bing?

SUGGESTED ANSWER:
a) It depends on whether or not Bing was at least 18 years old at the time Carol
asserts the prerogative to take custody of Bing. If she was at least 18 years old, then
she is no longer under parental authority and neither Carol nor Norma can assert the
prerogative to take custody. However, if she was less than 18 years old, then Norma
has a better right since the adoption by Norma of Bing terminates the parental
authority of Carol over Bing.

b) The natural mother, Carol, should have the better right in light of the principle
that the child's welfare is the paramount consideration in custody rights. Obviously,
Bing's continued stay in her adopting parents' house, where interaction with the call
girls is inevitable, would be detrimental to her moral and spiritual development. This
could be the reason for Bing's expressed desire to return to her natural mother. It
should be noted, however, that Bing is no longer a minor, being 19 years of age now.
It is doubtful that a court can still resolve the question of custody over one who is sui
juris and not otherwise incapacitated.

SUGGESTED ANSWER:
a) On the assumption that Bing is still a minor or otherwise incapacitated, Carol
may petition the proper court for resolution or rescission of the decree of adoption on
the ground that the adopting parents have exposed, or are exposing, the child to
corrupt influence, tantamount to giving her corrupting orders or examples. She can
also ask for the revesting in her of parental authority over Bing. If However, Bing is
already 19 years of age and therefore no longer a minor, it is not Carol but Bing
herself who can petition the court for judicial rescission of the adoption, provided she
can show a ground for disinheritance of an ascendant.
b) Carol may file an action to deprive Norma of parental authority under Article
231 of the Family Code or file an action for the rescission of the adoption under
Article 191 in relation to Article 231 (2) of the Family Code.

Notes:
Under Article 231 of the Family Code, the natural parents may file an action for
the rescission of the decree of adoption on the ground that the adopting parents are
exposing the child to corrupt influence or they are giving corrupting orders.
Art. 231. The court in an action filed for the purpose in a related case may also
suspend parental authority if the parent or the person exercising the same:

(1) Treats the child with excessive harshness or cruelty;

(2) Gives the child corrupting orders, counsel or example;

(3) Compels the child to beg; or

(4) Subjects the child or allows him to be subjected to acts of lasciviousness.

The grounds enumerated above are deemed to include cases which have resulted
from culpable negligence of the parent or the person exercising parental authority.

If the degree of seriousness so warrants, or the welfare of the child so demands,


the court shall deprive the guilty party of parental authority or adopt such other
measures as may be proper under the circumstances.

The suspension or deprivation may be revoked and the parental authority revived
in a case filed for the purpose or in the same proceeding if the court finds that the
cause therefor has ceased and will not be repeated. (33a)

QUALIFICATIONS OF ADOPTER

Qualification of Adopter (2005)


In 1984, Eva, a Filipina, went to work as a nurse in the USA. There, she met
and fell in love with Paul, an American citizen, and they got married in 1985. Eva
acquired American citizenship in 1987. During their sojourn in the Philippines in
1990, they filed a joint petition for the adoption of Vicky, a 7-year old daughter of
Eva's sister. The government, through the Office of the Solicitor General, opposed
the petition on the ground that the petitioners, being both foreigners, are disqualified
to adopt Vicky.

a) Is the government's opposition tenable? Explain. (2%)

SUGGESTED ANSWER:
The government's position is untenable. Under paragraph 3, Article 184 of the
Family Code, an alien, as a general rule, cannot adopt. However, an alien who is a
former Filipino citizen and who seeks to adopt a relative by consanguinity is qualified
to adopt, (par. 3[a], Art. 184, Family Code)
In the given problem, Eva, a naturalized American citizen would like to adopt
Vicky, a 7-year old daughter of her sister. Thus, under the above-cited provision, Eva
is qualified to adopt Vicky.

Notes:
Under the New Domestic Adoption Act, an alien is already qualified to adopt a
Filipino provided that he possesses all the qualifications and none of the
disqualification.

b) Would your answer be the same if they sought to adopt Eva's illegitimate
daughter? Explain. (2%)

SUGGESTED ANSWER:
My answer will still be the same. Paragraph 3(a) of Article 184 of the Family
Code does not make any distinction. The provision states that an alien who is a former
Filipino citizen is qualified to adopt a relative by consanguinity.
Notes:
Under the Family Code, an alien who is a former Filipino citizen is qualified to
adopt a relative by consanguinity.

c) Supposing that they filed the petition to adopt Vicky in the year 2000, will
your answer be the same? Explain. (2%)

SUGGESTED ANSWER:
Yes, my answer will still be the same. Under Sec. 7(b), Art. III of the New
Domestic Adoption Act, an alien who possesses all the qualifications of a Filipino
national who is qualified to adopt may already adopt provided that his country
has diplomatic relations with the Philippines, that he has been living in the
Philippines for at least three (3) continuous years prior to the filing of the
application for adoption and maintains such residence until the adoption decree is
entered, that he has been certified by his diplomatic or consular office or any
appropriate government agency that he has the legal capacity to adopt in his country,
and that his government allows the adoptee to enter his country as his adopted child.

Notes:

Under the New Domestic Adoption Act, an alien who possesses all the
qualifications of a Filipino national who is qualified to adopt may already adopt
provided that his country has diplomatic relations with the Philippines, that he has
been living in the Philippines for at least three (3) continuous years prior to the filing
of the application for adoption and that his government allows the adoptee to enter
his country.

Art. 184. The following persons may not adopt:

(1) The guardian with respect to the ward prior to the approval of the final
accounts rendered upon the termination of their guardianship relation;

(2) Any person who has been convicted of a crime involving moral turpitude;

(3) An alien, except:

(a) A former Filipino citizen who seeks to adopt a relative by consanguinity;

(b) One who seeks to adopt the legitimate child of his or her Filipino spouse; or

(c) One who is married to a Filipino citizen and seeks to adopt jointly with his or
her spouse a relative by consanguinity of the latter.

Aliens not included in the foregoing exceptions may adopt Filipino children in
accordance with the rules on inter-country adoptions as may be provided by law. (28a,
E. O. 91 and PD 603)

Art. 185. Husband and wife must jointly adopt, except in the following cases:

(1) When one spouse seeks to adopt his own illegitimate child; or

(2) When one spouse seeks to adopt the legitimate child of the other.

Qualification of Adopter; Applicable Law (2001)


A German couple filed a petition for adoption of a minor Filipino child with the
Regional Trial Court of Makati under the provisions of the Child and Youth
Welfare Code which allowed aliens to adopt. Before the petition could be heard, the
Family Code, which repealed the Child and Youth Welfare Code, came into effect.
Consequently, the Solicitor General filed a motion to dismiss the petition, on the
ground that the Family Code prohibits aliens from adopting. If you were the judge,
how will you rule on the motion? (5%)

SUGGESTED ANSWER:
The motion to dismiss the petition for adoption should be denied. The law that
should govern the action is the law in force at the time of filing of the petition. At
that time, it was the Child and Youth Welfare Code that was in effect, not the Family
Code. Petitioners have already acquired a vested right on their qualification to adopt
which cannot be taken away by the Family Code. (Republic v. Miller G.R. No.
125932, April 21, 1999, citing Republic v. Court of Appeals, 205 SCRA 356)

Notes:
In the case of Republic v. Court of Appeals, the Supreme Court ruled that the
law in force at the time of the filing of the petition for adoption shall govern the
qualification of an adoptive parent.

ALTERNATIVE ANSWER:
The motion has to be granted. The new law shall govern their qualification to
adopt and under the new law, the German couple is disqualified from adopting. They
cannot claim that they have already acquired a vested right because adoption is not a
right but a mere privilege. No one acquires a vested right on a privilege.

[Note: If the examinee based his answer on the current law, RA 8552, his answer
should be considered correct. This question is based on the repealed provision of the
Family Code on Adoption.]

Notes:
Now, aliens are qualified to adopt a Filipino child either through the Inter-
Country Adoption Board or by filing a petition with the RTC having jurisdiction over
the child.

Qualifications of Adopter (2000)


Sometime in 1990, Sarah, born a Filipino but by then a naturalized American
citizen, and her American husband Tom, filed a petition in the Regional Trial Court of
Makati, for the adoption of the minor child of her sister, a Filipina. Can the petition be
granted? (5%)
SUGGESTED ANSWER:
It depends. Rules on Adoption effective August 22, 2002 provides the following;
SEC. 4. Who may adopt. –

Section 21. Who May Adopt. – The following may adopt:

(a) Any Filipino citizen at least twenty-five (25) years of age, who is in
possession of full civil capacity and legal rights; has not been convicted of any
crime involving moral turpitude; is of good moral character and can model the
same; is emotionally and psychologically capable of caring for children; at least
sixteen (16) years older than the adoptee; and who is in a position to support and
care for adopted children in keeping with the means of the
family: Provided, That the requirement of sixteen (16)-years difference between
the age of the adopter and the adoptee may be waived when the adopter is the
biological parent of the adoptee, or is the spouse of the adoptee’s parent;

(b) The legal guardian with respect to the ward after the termination of the
guardianship and clearance of financial accountabilities;

(c) The legal guardians with respect to the foster child;

(d) Philippine government officials and employees deployed or stationed


abroad: Provided, That they are able to bring the child with them; and

(e) Foreign nationals who are permanent or habitual residents of the Philippines
for at least five (5) years possessing the same qualifications as above stated for
Filipino nationals prior to filing of the petition: Provided, That they come from a
country with diplomatic relations with the Republic of the Philippines and that
the laws of the adopter’s country will acknowledge the Certificate of Adoption
as valid, acknowledge the child as a legal child of the adopters, and allow entry
of the child into such country as an adoptee: Provided, further, That
requirements of residency may be waived for the following:

(1) A former Filipino citizen, habitually residing in the Philippines, who


seeks to adopt a relative within fourth (4th) civil degree of consanguinity
or affinity; or

(2) One who seeks to adopt the legitimate child of the Filipino spouse; or

(3) One who is married to a Filipino citizen and seeks to adopt jointly with
the spouse a relative within the fourth (4th) degree of consanguinity or
affinity of the Filipino spouse.

Spouses shall jointly adopt, except in the following cases:

(a) If one spouse seeks to adopt the legitimate child of the other; or

(b) If one spouse seeks to adopt own illegitimate


child: Provided, That the other spouse has signified consent thereto;
or
(c) If the spouses are legally separated from each other.

Notes:
Under the Civil Code, an alien who is a former Filipino citizen is qualified to
adopt his relative by consanguinity.

Any alien may adopt a Filipino child provided that he is of good moral
character, that he was not convicted of a crime involving moral turpitude, that he is 16
years older than the adopted, that he is sui juris, that he is capable to support the child,
that his country will allow his adopted to enter therein and that his country and the
Philippines has diplomatic relation.

Under the Inter-Country Adoption Act, aliens may now adopt even though they
are living abroad by filing an application with the Inter-Country Adoption Board.
Consequently, the residency requirement is no longer applicable.

Qualifications of Adopter (2003)


Lina, a former Filipina who became an American citizen shortly after her
marriage to an American husband, would like to adopt in the Philippines, jointly with
her husband, one of her minor brothers. Assuming that all the required consents have
been obtained, could the contemplated joint adoption in the Philippine prosper?
Explain.
SUGGESTED ANSWER:
Yes, Lina and her American husband can jointly adopt a minor brother of Lina
because she and her husband are both qualified to adopt. Lina, as a former Filipino
citizen, can adopt her minor brother under Sec. 7(b)(i) of RA 8552 (Domestic
Adoption Act of 1998), or under Art. 184 (3)(1) of the Family Code. The alien
husband can now adopt under Sec. 7(b) of RA8552. The Supreme Court has held in
several cases that when husband and wife are required to adopt jointly, each one
of them must be qualified to adopt in his or her own right (Republic v. Toledano,
233 SCRA 9 (1994). However, the American husband must comply with the
requirements of the law including the residency requirement of three (3) years.
Otherwise, the adoption will not be allowed.

Notes:
Under the Family Code, an alien may adopt a relative by consanguinity of his
Filipino spouse provided that he is of good moral character, that he was not
convicted of any crime involving moral turpitude, that he is sui juris, that he has the
ability to support the child and that his country allows the child to enter therein.

Successional Rights of Adopted Child (2004)


A Filipino couple, Mr. and Mrs. BM, Jr., decided to adopt YV, an orphan
from St. Claire’s orphanage in New York City. They loved and treated her like a
legitimate child for they have none of their very own. However, BM, Jr., died in an
accident at sea, followed to the grave a year later by his sick father, BM, Sr. Each left
a sizable estate consisting of bank deposits, lands and buildings in Manila. May the
adopted child, YV, inherit from BM, Jr.? May she also inherit from BM, Sr.? Is
there a difference? Why? Explain. (5%)

SUGGESTED ANSWER:
YV can inherit from BM, Jr. The succession to the estate of BM, Jr. is governed
by Philippine law because he was a Filipino when he died (Article 16, Civil Code).
Under Article 1039 of the Civil Code, the capacity of the heir to succeed is governed
by the national law of the decedent and not by the national law of the heir. Hence,
whether or not YV can inherit from BM, Jr. is determined by Philippine law. Under
Philippine law, the adopted inherits from the adopter as a legitimate child of the
adopter.

YV, however, cannot inherit, in his own right, from the father of the adopter,
BM, Sr., because he is not a legal heir of BM, Sr. The legal fiction of adoption exists
only between the adopted and the adopter. (Teotico v. Del Val 13 SCRA 406 [1965]).
Neither may he inherit from BM, Sr. by representing BM, Jr. because in
representation, the representative must be a legal heir not only of the person he is
representing but also of the decedent from whom the represented was supposed to
inherit (Article 973, Civil Code).

THE adopted can now inherit by right of representation from the adopted
grandfather because of the new law which extended the legitimate filiation of adopted
to adopter’s father. RA

Notes:
Under the Civil Code, an adoptee can inherit from the adopter.

In Teotico v. Del Val, it was held that the legal fiction of adoption exists only
between the adopted and the adopter. Thus, the adopted cannot inherit from the father
of the adopter.
FAMILY CODE
Emancipation (1993)
Julio and Lea, both 18 years old, were sweethearts. At a party at the house of a
mutual friend, Lea met Jake, also 18 years old, who showed interest in her. Lea
seemed to entertain Jake because she danced with him many times. In a fit of jealousy,
Julio shot Jake with his father's 38 caliber revolver which, before going to the party he
was able to get from the unlocked drawer inside his father's bedroom. Jake died as a
result of the lone gunshot wound he sustained. His parents sued Julio's parents
for damages arising from quasi-delict. At the time of the incident, Julio was 18
years old living with his parents. Julio's parents moved to dismiss the complaint
against them claiming that since Julio was already of majority age, they were no
longer liable for his acts. 1) Should the motion to dismiss be granted? Why? 2)
What is the liability of Julio's parents to Jake's parents? Explain your answer.

SUGGESTED ANSWER:

1) No, the Motion to Dismiss should not be granted. Article 236 of the Family
Code as amended by Republic Act 6809, provides in the third paragraph that "nothing
in this Code shall be construed to derogate from the duty or responsibility of parents
and guardians for children and wards below twenty-one years of age mentioned in the
second and third paragraphs of Article 2180 of the Civil Code".

2) The liability of Julio's parents to Jake's parents arises from quasi-delict (Arts.
2176 and 2180 Civil Code) and shall cover specifically the following: vicarious
liabilty
a) P50,000.00 for the death of the son;
b) such amount as would correspond to lost earning capacity; and
c) moral damages.

Notes:

Under Article 2180 of the Civil Code, the father is responsible for the damages
caused by the minor children who live in his company but if he died or is incapable,
the mother shall be held responsible therefor. Thus, the parents of Julio shall be civilly
liable for the death of Jake as a result of the criminal act committed by Julio who was
18 years old at the time of the incident.
ARTICLE 2180. The obligation imposed by article 2176 is demandable not
only for one’s own acts or omissions, but also for those of persons for whom one is
responsible.

The father and, in case of his death or incapacity, the mother, are responsible
for the damages caused by the minor children who live in their company.

Guardians are liable for damages caused by the minors or incapacitated persons
who are under their authority and live in their company.

The owners and managers of an establishment or enterprise are likewise


responsible for damages caused by their employees in the service of the branches in
which the latter are employed or on the occasion of their functions.

Employers shall be liable for the damages caused by their employees and
household helpers acting within the scope of their assigned tasks, even though the
former are not engaged in any business or industry.

The State is responsible in like manner when it acts through a special agent; but
not when the damage has been caused by the official to whom the task done properly
pertains, in which case what is provided in article 2176 shall be applicable.

Lastly, teachers or heads of establishments of arts and trades shall be liable for
damages caused by their pupils and students or apprentices, so long as they remain in
their custody.

The responsibility treated of in this article shall cease when the persons herein
mentioned prove that they observed all the diligence of a good father of a family to
prevent damage.

ARTICLE 2176. Whoever by act or omission causes damage to another is


obliged to pay for the damage done if there is fault or negligence. Such fault or
negligence is called a quasi-delict if there is no pre-existing contractual relation
between the parties. (Reengineered)

ARTICLE 2176. Whoever by act or omission causes damage to another, there


being fault or negligence, is obliged to pay for the damage done. Such fault or
negligence, if there is no pre-existing contractual relation between the parties, is called
a quasi-delict and is governed by the provisions of this Chapter. (1902a)
Family Code; Retroactive Application; Vested Rights (2000)
On April 15, 1980, Rene and Angelina were married to each other without a
marriage settlement. In 1985, they acquired a parcel of land in Quezon City. On June
1, 1990, when Angelina was away in Baguio, Rene sold the said lot to Marcelo. Is
the sale void or voidable? (2%)
SUGGESTED ANSWER:
The sale is void. Since the sale was executed in 1990, the Family Code is the law
applicable. Under Article 124 of the FC, the sale of a conjugal property by a spouse
without the consent of the other is void.

Notes:
Under the Family Code, sale of a conjugal property made by one spouse without
the consent of the other shall be void.

Art. 124. The administration and enjoyment of the conjugal partnership shall
belong to both spouses jointly. In case of disagreement, the husband’s decision shall
prevail, subject to recourse to the court by the wife for proper remedy, which must be
availed of within five years from the date of the contract implementing such
decision.

In the event that one spouse is incapacitated or otherwise unable to participate in


the administration of the conjugal properties, the other spouse may assume sole
powers of administration. These powers do not include disposition or
encumbrance without authority of the court or the written consent of the other
spouse. In the absence of such authority or consent, the disposition or
encumbrance shall be void.

However, the transaction shall be construed as a continuing offer on the part of


the consenting spouse and the third person, and may be perfected as a binding contract
upon the acceptance by the other spouse or authorization by the court before the offer
is withdrawn by either or both offerors. (165a)

Art. 124. The administration and enjoyment of the conjugal partnership


shall belong to both spouses jointly. In case of disagreement, the husband’s decision
shall prevail, subject to recourse to the court by the wife for proper remedy, which
must be availed of within five years from the date of the contract implementing such
decision.
In the event that one spouse is incapacitated or otherwise unable to participate in
the administration of the conjugal properties, the other spouse may assume sole
powers of administration. These powers do not include disposition or encumbrance
without authority of the court or the written consent of the other spouse. In the
absence of such authority or consent, the disposition or encumbrance shall be void.
However, the transaction shall be construed as a continuing offer on the part of the
consenting spouse and the third person, and may be perfected as a binding contract
upon the acceptance by the other spouse or authorization by the court before the offer
is withdrawn by either or both offerors. (165a)

ALTERNATIVE ANSWER:
The sale is voidable. The provisions of the Family Code may apply retroactively
but only if such application will not impair vested rights. When Rene and Angelina
got married in 1980, the law that governed their property relations was the New Civil
Code. Under the NCC, as interpreted by the Supreme Court in Heirs of Felipe v.
Aldon, 100 SCRA 628 and reiterated in Heirs of Ayuste v. Malabonga, G.R No,
118784, 2 September 1999, the sale executed by the husband without the consent
of the wife is voidable. The husband has already acquired a vested right on the
voidable nature of dispositions made without the consent of the wife. Hence, Article
124 of the Family Code which makes the sale void does not apply.

Notes:
Under the New Civil Code, sale of conjugal property without the consent of the
other spouse is voidable.

Family Home; Dwelling House (1994)


In 1991, Victor established judicially out of conjugal property, a family home
in Manila worth P200.000.00 and extrajudicially a second family home in Tagaytay
worth P50.000.00. Victor leased the family home in Manila to a foreigner. Victor and
his family transferred to another house of his in Pasig.

Can the two family homes be the subject of execution on a judgment against
Victor's wife for non-payment of the purchase in 1992 of household appliances?

SUGGESTED ANSWER:
The two (2) so-called family homes can be the subject of execution. Neither of
the abodes are considered family homes because for purposes of availing the benefits
under the Family Code, there can only be one (1) family home which is defined as the
"dwelling house" where the husband and the wife and their family actually "reside"
and the land on which it is situated. (Arts. 152 and 161, Family Code)

Notes:
Under the Family Code, for purposes of availing the benefit of a family home,
its beneficiary may establish only one family home. Therefore, since Victor
established more than one family home, neither one of them is considered a family
home.

Art. 161. For purposes of availing of the benefits of a family home as provided
for in this Chapter, a person may constitute, or be the beneficiary of, only one family
home.
Art. 152. The family home is the dwelling house where the family reside, and
the land where the family home is situated; it is established jointly by the husband and
the wife or by an unmarried head of a family (Reengineered)

Art. 152. The family home, constituted jointly by the husband and the wife or by
an unmarried head of a family, is the dwelling house where they and their family
reside, and the land on which it is situated. (223a)

Family; Constitutional Mandates; Divorce (1991)


A. How does the 1987 Constitution strengthen the family as an Institution?
B. Do the Constitutional policy on the family and the provision that
marriage is the foundation of the family and shall be protected by the State bar
Congress from enacting a law allowing divorce in the Philippines?
SUGGESTED ANSWER:
A. Sec, 2, Article II of the Constitution provides that: The State recognizes the
sanctity of family life and shall protect and strengthen the family as a basic
autonomous social institution. It shall equally protect the life of the mother and the life
of the unborn from conception. The natural and primary right and duty of parents in
the rearing of the youth for civic efficiency and the development of moral character
shall receive the support of the Government.

Section I, Article XV, further provides that: The State recognizes the Filipino
family as the foundation of the nation. Accordingly, it shall strengthen its solidarity
and actively promote its total development.
(Note: The Committee recommends that a citation of either one of the provisions
be credited as a complete answer).

SUGGESTED ANSWER:
B, No, the Constitutional policy, as well as the supporting provision, does not
amount to a prohibition to Congress to enact a law on divorce. The Constitution only
meant to help the marriage endure, to "strengthen its solidarity and actively promote
its total development."

ALTERNATIVE ANSWER:
B. Yes. Congress is barred from enacting a law allowing divorce, since Section 2
of Article XV provides: "Sec. 2. Marriage, as an inviolable social institution, is the
foundation of the family and shall be protected by the State." Since marriage is
"Inviolable", it cannot be dissolved by an absolute divorce.

Notes:
Under the Constitution, marriage is an inviolable social institution; it is the
foundation of the family and shall be protected by the State. Thus, Congress cannot
enact law allowing divorce.

Moreover, it provides that the State recognizes the Filipino family as the
foundation of the nation. Accordingly, it shall strengthen its solidarity and actively
promote its total development.

Marriage; Annulment; Effects; Requisites Before Remarriage (1990)


The marriage of H and W was annulled by the competent court. Upon finality
of the judgment of nullity, H began looking for his prospective second mate. He fell in
love with a sexy woman S who wanted to be married as soon as possible, i.e., after a
few months of courtship. As a young lawyer, you were consulted by H.

(a) How soon can H be joined in lawful wedlock to his girlfriend S? Under
existing laws, are there certain requisites that must be complied with before he
can remarry? What advice would you give H?
(b) Suppose that children were born from the union of H and W, what
would be the status of said children? Explain your answer.
c. If the subsequent marriage of H to S was contracted before
compliance with the statutory condition for its validity, what are the rights of the
children of the first marriage (i.e., of H and W) and of the children of the
subsequent marriage (of H and S)? children of second marriage

SUGGESTED ANSWER:
(a) H, or either spouse for that matter, can marry again after complying with the
provisions of Article 52 of the Family Code, namely, there must be a partition and
distribution, of the properties of the spouses, and the delivery of the children's
presumptive legitimes which should be recorded in the appropriate civil registry and
registries of property. H should be so advised.

Notes:
Under the Family Code, after annulment or absolute nullity of marriage, the
husband can marry again provided that the (1) partition, distribution of the
properties were already made, that the (2) presumptive legitimes of the children were
already delivered and that fact were (3) registered in the appropriate civil registry of
properties.

However, with respect to the wife, she has to wait until the expiry of 300 days
from annulment or nullity of marriage before she could remarry. Otherwise, she could
be held liable for contracting a second marriage within the prohibited period.
Art. 52. The judgment of annulment or of absolute nullity of the marriage, the
partition and distribution of the properties of the spouses and the delivery of the
children’s presumptive legitimes shall be recorded in the appropriate civil registry and
registries of property; otherwise, the same shall not affect third persons.

ALTERNATIVE ANSWER:
The following are the requisites prescribed by law and I advise H to comply
with them, namely:

1) If either spouse contracted the marriage in bad faith, his or her share of the
net profits of the community property : or conjugal partnership property shall be
forfeited in favor of the common children or, if there are none, the children of the
guilty spouse by a previous marriage or, in default of children, the innocent spouse;

2) Donations by reason of marriage shall remain valid except that if the donee
contracted the marriage in bad faith, such donations made to said donee are revoked
by operation of law;
3) The spouse who contracted the subsequent marriage in bad faith shall be
disqualified to inherit from the innocent spouse by testate and intestate succession;

4) If both spouses of the subsequent marriage acted in bad faith all donations by
reason of marriage and testamentary dispositions made by one in favor of the other are
revoked by operation of law.

5) The judgment of annulment of the marriage, the partition and distribution of


the properties of the spouses, and the delivery of the children's presumptive legitimes
shall be recorded in the appropriate civil registry and registers of property, (Articles
53. 52, 43. 44. Family Code).

SUGGESTED ANSWER:
(b) The children born from the union of H and W would be legitimate children
if conceived or born before the decree of annulment of the marriage (under Art. 45 of
the Family Code) has become final and executory (Art. 54, Family Code}.

Notes:
(b) The status of the children of H and W would be legitimate children if
conceived or born during the marriage.

Art. 54. The status of the children who were conceived or born before the
judgment of annulment or absolute nullity of the marriage has become final and
executory shall be considered legitimate. Children who were conceived or born of the
subsequent marriage under Article 53 shall likewise be legitimate. (Re-engineered)

Art. 36. A marriage contracted by any party shall be void if at the time of the
celebration one of the parties was psychologically incapacitated to comply with the
essential marital obligations of marriage even if such incapacity becomes manifest
only after its solemnization.(Re-engineered)

Art. 53. Either of the former spouses may marry again after compliance with
the requirements of the immediately preceding Article; otherwise, the subsequent
marriage shall be null and void.

Under Art. 53, the subsequent marriage shall be null and void if the former
spouses do not partition and distribute their property; they do not deliver the
presumptive legitimes of their children.
SUGGESTED ANSWER:

(c) The children of the first marriage shall be considered legitimate children if
conceived or born before the Judgment of annulment of the marriage of H and W
has become final and executory. Children conceived or born of the subsequent
marriage shall likewise be legitimate even if the marriage of H and S be null and void
for failure to comply with the requisites of Article 52 of the Family Code (Article 53,
Family Code). As legitimate children, they have the following rights;

a) To bear the surnames of the father and the mother in conformity with the
provisions of the Civil Code on Surnames;

b) To receive support from their parents, their ascendants, and in proper cases,
their brothers and sisters, in conformity with the provisions of this Code on Support;
and

c) To be entitled to the legitime and other successional rights granted to them by


the Civil Code (Article 174, Family Code).

Marriage; Annulment; Grounds (1991)


One of the grounds for annulment of marriage is that either party, at the time of
their marriage was afflicted with a sexually-transmissible disease, found to be serious
and appears incurable. Two (2) years after their marriage, which took place on 10
October 1988, Bethel discovered that her husband James has a sexually-transmissible
disease which he contracted even prior to their marriage although James did not know
it himself until he was examined two [2) years later when a child was already born to
them. Bethel sues James for annulment of their marriage.

A. James opposes the annulment on the ground that he did not even know
that he had such a disease so that there was no fraud or bad faith on his part.
Decide.

B. Suppose that both parties at the time of their marriage were similarly
afflicted with sexually-transmissible diseases, serious and incurable, and both
knew of their respective infirmities, can Bethel or James sue for annulment of
their marriage?

SUGGESTED ANSWER:
A. The marriage can be annulled, because good faith is not a defense when the
ground is based upon sexually-transmissible disease on the part of either party.

Notes:
The Family Code provides that if either party to the marriage is suffering from a
sexually-transmissible disease, the marriage may be annulled. In this case, the fact that
Bethal and James are afflicted with a sexually-transmissible disease does not remove
the ground for annulment of marriage.

Art. 45. A marriage may be annulled for any of the following causes, existing
at the time of the marriage:

(1) That either party was eighteen years of age; if he or she is below twenty-
one, and the marriage was solemnized without the consent of the parents, guardian
or person having substitute parental authority over the party, the marriage may also
be annulled. 18-21 w/o parental or guardian consent

(2) That either party was of unsound mind unless such party freely cohabited
with the other as husband and wife after coming to reason; unsound mind

(3) That the consent of either party was obtained by fraud unless such party
afterwards freely cohabited with the other as husband and wife with full knowledge of
the facts constituting the fraud;

Fraud/concealment

(4) That the consent of either party was obtained by force, intimidation or
undue influence unless such party thereafter freely cohabited with the other as
husband and wife after the same having disappeared or ceased; vitiation of consent

(5) That either party was physically incapable of consummating the marriage
with the other, and such incapacity continues and appears to be incurable; or
incurable physical incapacity to consummate

(6) That either party was afflicted with a sexually-transmissible disease appears
to be serious and incurable. (85a) serious, incurable sexually-transmitted disease

SUGGESTED ANSWER:
B. Yes, the marriage can still be annulled because the fact that both of them are
afflicted with sexually-transmissible diseases does not efface or nullify the ground.
Alternative Answer:
B. No, the marriage can no longer be annulled, because the fact that both were
afflicted and that both knew of their respective infirmities constitutes a waiver of that
ground.

Marriage; Annulment; Judicial Declaration (1993)


Maria and Luis, both Filipinos, were married by a Catholic priest in Lourdes
Church, Quezon City in 1976, Luis was drunk on the day of his wedding. In fact, he
slumped at the altar soon after the ceremony. After marriage, Luis never had a steady
job because he was drunk most of the time. Finally, he could not get employed at all
because of drunkenness. Hence, it was Maria who had to earn a living to support
herself and her child begotten with Luis. In 1986, Maria filed a petition in the
church matrimonial court in Quezon City to annul her marriage with Luis on the
ground of psychological incapacity to comply with his marital obligation. Her
petition was granted by the church matrimonial court.

1) Can Maria now get married legally to another man under Philippine
laws after her marriage to Luis was annulled by the church matrimonial court?
Explain.

2) What must Maria do to enable her to get married lawfully to another


man under Philippine laws?

SUGGESTED ANSWER:
1) No, Maria cannot validly contract a subsequent marriage without a court
declaration of nullity of the first marriage. The law does not recognize the church
declaration of nullity of a marriage.
2) To enable Maria to get married lawfully to another man, she must obtain a
judicial declaration of nullity of the prior marriage under Article 36 Family Code.

Notes:

Under the Family Code, judicial declaration of nullity of prior marriage is


necessary for a party to contract a second marriage. Therefore, the annulment of their
marriage by the church is not recognized.

Art. 40. What is necessary is final judgement declaring the previous marriage
void when one of the spouses wants to remarry. (Re-engineered)
Art. 40. The absolute nullity of a previous marriage may be invoked for
purposes of remarriage on the basis solely of a final judgment declaring such previous
marriage void.

Marriage; Annulment; Legal Separation; Prescription of Actions (1996)

2) Bert and Baby were married to each other on December 23, 1988. Six
months later, she discovered that he was a drug addict. Efforts to have him
rehabilitated were unsuccessful.

Can Baby ask for annulment of marriage, or legal separation? Explain.

SUGGESTED ANSWER:
No, Baby cannot ask for annulment of her marriage or for legal separation
because both actions had already prescribed.
While concealment of drug addiction existing at the time of marriage
constitutes fraud under Art. 46 of the FC which makes the marriage voidable under
Art. 45 of the FC, the action must, however, be brought within 5 years from the
discovery thereof under Article 47(3), FC, Since the drug addiction of Bert was
discovered by Baby in June 1989, the action had already prescribed in June of 1994.
Although drug addiction is a ground for legal separation under Art. 55(5) and Art. 57
of the FC requires that the action must be brought within 5 years from the occurrence
of the cause. Since Bert had been a drug addict from the time of the celebration of the
marriage, the action for legal separation must have been brought not later than 23
December 1993. Hence, Baby cannot, now, bring the action for legal separation.

Notes:
Under the Family Code, concealment of drug addiction constitutes fraud
which is a ground for annulment of marriage. But the annulment of marriage shall be
filed within 5 years from discovery. Otherwise, the right of action will prescribe.

Art. 46. Any of the following circumstances shall constitute fraud referred to in
Number 3 of the preceding Article:

(1) Non-disclosure of a previous conviction by final judgment of the other party


of a crime involving moral turpitude; previous conviction for moral turpitude

(2) Concealment by the wife of the fact that at the time of the marriage, she was
pregnant by a man other than her husband; pregnant @ marriage by others
(3) Concealment of sexually transmissible disease, regardless of its nature,
existing at the time of the marriage; or sexually transmissible disease regardless of
nature

(4) Concealment of drug addiction, habitual alcoholism or homosexuality or


lesbianism existing at the time of the marriage. Drug addiction, alcoholism,
lesbianism, homosexuality

No other misrepresentation or deceit as to character, health, rank, fortune or


chastity shall constitute such fraud as will give grounds for action for the annulment
of marriage. (86a)

Art. 47. The action for annulment of marriage must be filed by the following
persons and within the periods indicated herein: PRESCRITPIVE PERIOD

(1) For causes mentioned in number 1 of Article 45 by the party whose parent or
guardian did not give his or her consent, within five years after attaining the age of
twenty-one, or by the parent or guardian or person having legal charge of the minor, at
any time before such party has reached the age of twenty-one; by party who lacks
consent after 21; by parent/guardian before MINOR reach 21

(2) For causes mentioned in number 2 of Article 45, by the same spouse, who
had no knowledge of the other’s insanity; or by any relative or guardian or person
having legal charge of the insane, at any time before the death of either party, or by
the insane spouse during a lucid interval or after regaining sanity; by unaffected
spouse/ relative/guardian/person before death of either spouse; by insane spouse
during lucid interval

(3) For causes mentioned in number 3 of Article 45, by the injured party, within
five years after the discovery of the fraud; by injured spouse within 5 yrs of
discovery

(4) For causes mentioned in number 4 of Article 45, by the injured party, within
five years from the time the force, intimidation or undue influence disappeared or
ceased; by injured party within 5yrs from the time force, intimidation, or undue
influence disappeared or ceased

(5) For causes mentioned in number 5 and 6 of Article 45, by the injured party,
within five years after the marriage. (87a)

Marriage; Annulment; Proper Party (1990)


D and G, age 20 and 19, respectively, and both single, eloped and got married to
each other without parental consent in the case of G, a teenaged student of an
exclusive college for girls. Three years later, her parents wanted to seek judicial
annulment on that ground. You were consulted and asked to prepare the proper
complaint. What advice would you give G's parents? Explain your answer.

SUGGESTED ANSWER:
G himself should file the complaint under Article 45 of the Family Code, and no
longer the parents because G is already 22 years of age.
Notes:
Under the Family Code, if a party to the marriage is below 21 years old and the
marriage was solemnized without the consent of his parents, his parents may file a
case to annul the marriage; but if such party is already 21 years of age, he should be
the one to file the case for annulment;

Art. 45. A marriage may be annulled for any of the following causes, existing at
the time of the marriage:

(1) That the party in whose behalf it is sought to have the marriage annulled was
eighteen years of age or over but below twenty-one, and the marriage was solemnized
without the consent of the parents, guardian or person having substitute parental
authority over the party, in that order, unless after attaining the age of twenty-one,
such party freely cohabited with the other and both lived together as husband and
wife;

Art. 46. Any of the following circumstances shall constitute fraud referred to in
Number 3 of the preceding Article:

(1) Non-disclosure of a previous conviction by final judgment of the other party


of a crime involving moral turpitude;

(2) Concealment by the wife of the fact that at the time of the marriage, she was
pregnant by a man other than her husband;

(3) Concealment of sexually transmissible disease, regardless of its nature,


existing at the time of the marriage; or
(4) Concealment of drug addiction, habitual alcoholism or homosexuality or
lesbianism existing at the time of the marriage.

No other misrepresentation or deceit as to character, health, rank, fortune or


chastity shall constitute such fraud as will give grounds for action for the annulment of
marriage. (86a)

Art. 47. The action for annulment of marriage must be filed by the following
persons and within the periods indicated herein:

(1) For causes mentioned in number 1 of Article 45 by the party whose parent
or guardian did not give his or her consent, within five years after attaining the age of
twenty-one, or by the parent or guardian or person having legal charge of the minor,
at any time before such party has reached the age of twenty-one;

(2) For causes mentioned in number 2 of Article 45, by the same spouse, who
had no knowledge of the other’s insanity; or by any relative or guardian or person
having legal charge of the insane, at any time before the death of either party, or by
the insane spouse during a lucid interval or after regaining sanity;

(3) For causes mentioned in number 3 of Article 45, by the injured party,
within five years after the discovery of the fraud;

(4) For causes mentioned in number 4 of Article 45, by the injured party,
within five years from the time the force, intimidation or undue influence disappeared
or ceased;

(5) For causes mentioned in number 5 and 6 of Article 45, by the injured party,
within five years after the marriage. (87a)

Marriage; Annulment; Proper Party (1995)


Yvette was found to be positive for HIV virus, considered sexually
transmissible, serious and incurable. Her boyfriend Joseph was aware of her
condition and yet married her. After two (2) years of cohabiting with Yvette, and in
his belief that she would probably never be able to bear him a healthy child, Joseph
now wants to have his marriage with Yvette annulled. Yvette opposes the suit
contending that Joseph is estopped from seeking annulment of their marriage since he
knew even before their marriage that she was afflicted with HIV virus. Can the
action of Joseph for annulment of his marriage with Yvette prosper? Discuss
fully.
SUGGESTED ANSWER:
No, Joseph knew that Yvette was HIV positive at the time of the marriage. He is,
therefore, not an injured party. The FC gives the right to annul the marriage only to an
injured party. [Art. 47 (5), FC]

Notes:
Under the Family Code, concealment of sexually transmissible disease is a
ground for annulment of marriage. Thus, the action to annul the marriage will not
prosper because there was no concealment.

ALTERNATIVE ANSWER:
The action for annulment can prosper because the prescriptive period of five (5)
years has not yet lapsed. [Art. 45 (6), FC].

Notes:
Under the Family Code, an action for annulment of marriage based on
concealment of sexually transmissible disease shall prescribe in 5 years. Thus, the
action will prosper because only 2 years has elapsed.

Marriage; Divorce Decree; Void Marriages (1992)


In 1989, Maris, a Filipino citizen, married her boss Johnson, an American
citizen, in Tokyo in a wedding ceremony celebrated according to Japanese laws. One
year later, Johnson returned to his native Nevada, and he validly obtained in that state
an absolute divorce from his wife Maris. After Maris received the final judgment of
divorce, she married her childhood sweetheart Pedro, also a Filipino citizen, in a
religious ceremony in Cebu City, celebrated according to the formalities of Philippine
law. Pedro later left for the United States and became naturalized as an American
citizen. Maris followed Pedro to the United States, and after a serious quarrel, Maris
filed a suit and obtained a divorce decree issued by the court in the state of Maryland.
Maris then returned to the Philippines and in a civil ceremony celebrated in Cebu City
according to the formalities of Philippine law, she married her former classmate
Vincent likewise a Filipino citizen.

b) Was the marriage of Maris and Pedro valid when celebrated? Is their
marriage still valid existing now? Reasons.

c) Was the marriage of Maris and Vincent valid when celebrated? Is their
marriage still validly existing now? Reasons.
d) At this point in time, who is the lawful husband of Maris? Reasons.

SUGGESTED ANSWER:

(b) The marriage of Maris and Pedro was valid when celebrated because the
divorce validly obtained by Johnson in Manila capacitated Maris to marry
Pedro. The marriage of Maris and Pedro is still validly existing, because the
marriage has not been validly dissolved by the Maryland divorce [Art. 26, Family
Code).

(c) The marriage of Maris and Vincent is void ab initio because it is a


bigamous marriage contracted by Maris during the subsistence of her marriage
with Pedro (Art 25 and 41, Family Code). The marriage of Maris and Vincent does
not validly exist because Article 26 does not apply. Pedro was not a foreigner at the
time of his marriage with marts and the divorce abroad (in Maryland) was initiated
and obtained not by the alien spouse, but by the Filipino spouse. Hence, the Maryland
divorce did not capacitate Marts to marry Vincent.

(d) At this point in time, Pedro is still the lawful husband of Maris because their
valid marriage has not been dissolved by any valid cause (Art. 26. Family Code)

Notes:
Under Article 26 of the Family Code, all marriages solemnized outside of the
Philippines are valid if valid in that country and they were solemnized according to
the law of such country.

Marriage; Divorce Decrees; Filiation of Children (2005)


In 1985, Sonny and Lulu, both Filipino citizens, were married in the
Philippines. In 1987, they separated, and Sonny went to Canada, where he obtained a
divorce in the same year. He then married another Filipina, Auring, in Canada on
January 1,1988. They had two sons, James and John. In 1990, after failing to hear
from Sonny, Lulu married Tirso, by whom she had a daughter, Verna. In 1991, Sonny
visited the Philippines where he succumbed to heart attack.

a) Discuss the effect of the divorce obtained by Sonny and Lulu in Canada.
(2%)
SUGGESTED ANSWER:

The divorce is not valid. Philippine law does not provide for absolute divorce.
Philippine courts cannot grant it. A marriage between two (2) Filipinos cannot be
dissolved by a divorce obtained abroad. (Garcia v. Redo, G.R. No. 138322, October 2,
2001). Philippine laws apply to Sonny and Lulu. Under Article 15 of the New Civil
Code, laws relating to family rights and duties, status, and capacity of persons are
binding upon citizens of the Philippines wherever they may be. Thus, the marriage of
Sonny and Lulu is still valid and subsisting.

Notes:
In Garcia v. Redo, it was held that divorce obtained by Filipinos abroad is not
recognized in the Philippines.

Under Article 15 of the New Civil Code, laws relating to legal capacity of
persons are binding upon citizens of the Philippines, even though living abroad. Thus,
the marriage of Sonny and Lulu is still valid and subsisting.

b) Explain the status of the marriage between Sonny and Auring. (2%)
SUGGESTED ANSWER:
Since the decree of divorce obtained by Lulu and Sony in Canada is not
recognized here in the Philippines, the marriage between Sonny and Auring is void.
(Art. 35, Family Code) Any marriage subsequently contracted during the lifetime of
the first spouse shall be illegal and void, subject only to the exception in the cases of
absence or where the prior marriage was dissolved or annulled. (Ninal v. Bayadog,
G.R. No. 133778, March 14, 2000) The marriage of Sonny and Auring does not fall
within the exception.

Notes:

Under the Family Code, the Supreme court held that any marriage subsequently
contracted during the existence of the first marriage shall be void. Thus, the Marriage
between Sonny and Auring is void because it is a bigamous marriage since it was
contracted during the existence of marriage between Sonny and Lulu.

Art. 41. A marriage contracted by any person during subsistence of a previous


marriage shall be null and void, unless before the celebration of the subsequent
marriage, the prior spouse had been absent for four consecutive years and the spouse
present has a well-founded belief that the absent spouse was already dead. In case of
disappearance where there is danger of death under the circumstances set forth in the
provisions of Article 391 of the Civil Code, an absence of only two years shall be
sufficient.

For the purpose of contracting the subsequent marriage under the preceding
paragraph the spouse present must institute a summary proceeding as provided in this
Code for the declaration of presumptive death of the absentee, without prejudice to the
effect of reappearance of the absent spouse.

c) Explain the status of the marriage between Lulu and Tirso. (2%)
SUGGESTED ANSWER:
The marriage of Lulu and Tirso is also void. Mere absence of the spouse does
not give rise to a right of the present spouse to remarry. Article 41 of the Family Code
provides for a valid bigamous marriage only where a spouse has been absent for four
consecutive years before the second marriage and the present spouse had a well-
founded belief that the absent spouse is already dead. (Republic v. Nolasco, G.R. No.
94053, March 17, 1993)

Notes:
Under Article 285 of the Civil Code, the present spouse may ask the court to
declare his absent spouse as presumptively dead which may capacitate him to remarry
without prejudice for the reappearance of the absentee. Only after the issuance of
judicial declaration may the present spouse contract marriage.

d) Explain the respective filiation of James, John and Verna. (2%)

SUGGESTED ANSWER:
James, John and Verna are illegitimate children since their parents are not
validly married. Under Article 165 of the Family Code, children conceived and born
outside a valid marriage are illegitimate, unless otherwise provided in this Code.

Notes:
Under Article 165 of the Family Code, children conceived and born outside a
valid marriage are illegitimate.

e) Who are the heirs of Sonny? Explain. (2%)

Suggested answer:
Sonny's heirs include James, John, and Lulu. Article 887 of the Civil Code
provides that the compulsory heirs of the deceased are among others, his widow and
his illegitimate children. The widow referred to in Article 887 is the legal wife of the
deceased. Lulu is still a compulsory heir of Sonny because the divorce obtained by
Sonny in Canada cannot be recognized in the Philippines. The legitime of each
illegitimate child shall consist of one-half of the legitime of a legitimate child. (Art.
176, Family Code)

Marriage; Divorce Decrees; Filipino Spouses becoming Alien (1996)


Flor and Virgillo were married to each other in Roxas City in 198O. In 1984,
Flor was offered a teaching Job in Canada, which she accepted. In 1989, she applied
for and was granted Canadian citizenship. The following year, she sued for divorce
from Virgilio in a Canadian court. After Virgilio was served with summons, the
Canadian court tried the case and decreed the divorce. Shortly thereafter, Flor married
a Canadian. Can Virgilio marry again in the Philippines? Explain.

SUGGESTED ANSWER:
No, Virgilio cannot validly remarry. His case is not covered by Article 26 of the
Family Code, For said Article to be applicable, the spouse who filed for divorce must
be a foreigner at the time of the marriage. Since both of them were Filipinos at the
time of the marriage, the divorce obtained by Flor did not capacitate Virgilio to
remarry. The fact that Flor was already an alien at the time she obtained the divorce
does not give Virgilio the capacity to remarry under Philippine Law.

ALTERNATIVE ANSWERS:
a) Yes, Virgilio can validly remarry. Art. 26 of the FC, merely States the alien
spouse without taking into consideration his or her nationality at the time of the
marriage. While his case is not covered by the letter of Article 26 FC, it is, however,
covered by the spirit of said Article, the injustice to the Filipino spouse sought to be
cured by said Article is present in this case. (Department of Justice Opinion No. 134
Series of 1993).

b) Although the marriage originally involved Filipino citizens, it eventually


became a marriage between an alien and a Filipino after Flor became a Canadian
citizen. Thus, the divorce decree was one obtained by an alien spouse married to a
Filipino. Although nothing is said about whether such divorce did capacitate Flor to
remarry, that fact may as well be assumed since the problem states that she married a
Canadian shortly after obtaining the divorce. Hence, Virgillo can marry again under
Philippine law, pursuant to Art. 26. FC which applies because Flor was already an
alien at the time of the divorce.

Notes:
Under Article 26 of the Family Code, all marriages solemnized outside of the
Philippines are valid if valid there where they are solemnized according to the law in
force in that country.

The rule which allows aliens to divorce and remarry is applicable to an alien
who is a former Filipino.

Art. 26. All marriages solemnized outside the Philippines, in accordance with
the laws in force in the country where they were solemnized, shall also be valid in this
country if valid in that country, except those prohibited under Articles 35 (1), (4), (5)
and (6), 3637 and 38. (17a)

Where a marriage between a Filipino citizen and a foreigner is validly


celebrated and a divorce is thereafter validly obtained abroad by the alien spouse
capacitating him or her to remarry, the Filipino spouse shall have capacity to remarry
under Philippine law. (As amended by Executive Order 227)

Marriage; Divorce Decrees; Filipino Spouses becoming Alien (1999)


Ben and Eva were both Filipino citizens at the time of their marriage in 1967.
When their marriage turned sour, Ben went to a small country in Europe, got himself
naturalized there, and then divorced Eva in accordance with the law of that country,
Later, he returned to the Philippines with his new wife. Eva now wants to know
what action or actions she can file against Ben. She also wants to know if she can
likewise marry again. What advice can you give her? {5%)

SUGGESTED ANSWER:
Considering that Art. 26(2nd par.) contemplates a divorce between a foreigner
and a Filipino, who had such respective nationalities at the time of their marriage, the
divorce in Europe will not capacitate the Filipino wife to remarry. The advice we can
give her is either to file a petition for legal separation, on the ground of sexual
infidelity and of contracting a bigamous marriage abroad, or to file a petition to
dissolve the conjugal partnership or absolute community of property as the case
maybe.
Notes:
Eva cannot file any action against Ben because the divorce obtained by Ben in
Europe is recognized in the Philippines. Eva, however, can marry again because the
divorce obtained by Ben capacitated her to remarry. This is now the rule under Art. 26
of the Family Code.

ALTERNATIVE ANSWER:
Eva may file an action for legal separation on the grounds of sexual infidelity of
her husband and the contracting by her husband of a bigamous marriage abroad.
She may remarry. While a strict interpretation of Article 26 of the Family Code would
capacitate a Filipino spouse to remarry only when the other spouse was a foreigner at
the time of the marriage, the DOJ has issued an opinion (Opinion 134 s. of 1993) that
the same injustice sought to be cured by Article 26 is present in the case of spouses
who were both Filipino at the time of the marriage but one became an alien
subsequently. Said injustice is the anomaly of Eva remaining married to her husband
who is no longer married to her. Hence, said Opinion makes Article 26 applicable to
her case and the divorce obtained abroad by her former Filipino husband would
capacitate her to remarry. To contract a subsequent marriage, all she needs to do is
present to the civil registrar the decree of divorce when she applies for a marriage
license under Article 13 of the Family Code.

Marriage; Donations by Reason of Marriage; Effect of Declaration of Nullity


(1996)
1) On the occasion of Digna's marriage to George, her father gave her a
donation propter nuptias of a car. Subsequently, the marriage was annulled because
of the psychological immaturity of George. May Digna's father revoke the
donation and get back the car? Explain.

SUGGESTED ANSWER:
No, Digna's father may not revoke the donation because Digna was not in bad
faith, applying Art. 86(3) of the Family Code.

Notes:

Under the Family Code, when the marriage is annulled, a donation by reason of
marriage may be revoked by the donor if the donee acted in bad faith.
Art. 86. A donation by reason of marriage may be revoked by the donor in the
following cases:

(1) If the marriage is not celebrated or judicially declared void ab initio except
donations made in the marriage settlements, which shall be governed by Article 81;

(2) When the marriage takes place without the consent of the parents or
guardian, as required by law;

(3) When the marriage is annulled, and the donee acted in bad faith;

(4) Upon legal separation, the donee being the guilty spouse;

5) If it is with a resolutory condition and the condition is complied with;

(6) When the donee has committed an act of ingratitude as specified by the
provisions of the Civil Code on donations in general.

ALTERNATIVE ANSWER:
a) Yes, the donation is revocable. Since the ground for the annulment of the
marriage is the psychological immaturity of George, the judgment was in the nature of
a declaration of nullity under Art. 36 of the FC and, therefore, the donation may be
revoked under Art. 86( 1) of the FC for the reason that the marriage has been
judicially declared void ab initio.
ALTERNATIVE ANSWER:
b) No, the donation cannot be revoked. The law provides that a donation by
reason of marriage may be revoked by the donor if among other cases, the marriage is
judicially declared void ab initio [par. (1) Art. 86. Family Code], or when the marriage
is annulled and the donee acted in bad faith [par. (3), Id.]. Since the problem states
that the marriage was annulled and there is no intimation of bad faith on the part of the
donee Digna, the conclusion is that the donor cannot revoke the donation.

ALTERNATIVE ANSWER:
c) Yes, the donation can be revoked. The ground used in dissolving the marriage
was the psychological immaturity of George, which is not a ground for annulment of
marriage. If this term is equated with psychological incapacity as used in Art. 36 of
the Family Code, then it is a ground for declaration of nullity of the marriage.
Consequently, par. (1) of Art. 86, FC, is the applicable law. Since Art. 86 of the FC
makes no qualification as to who furnished the ground or who was in bad faith in
connection with the nullification of the marriage, the conclusion is that Digna's father
may revoke the donation and get back the car.

Marriage; Grounds; Declaration of Nullity: Annulment: Legal Separation:


Separation of Property (2003)
Which of the following remedies, i.e., (a) declaration of nullity of marriage, (b)
annulment of marriage, (c) legal separation, and/or (d) separation of property, can an
aggrieved spouse avail himself/herself of-
(i) If the wife discovers after the marriage that her husband has aids.
(ii) If the wife goes (to) abroad to work as a nurse and refuses to come home
after the expiration of her three-year contract there.
(iii) If the husband discovers after the marriage that his wife has been a
prostitute before they got married.
(iv) If the husband has a serious affair with his secretary and refuses to stop
notwithstanding advice from relatives and friends.
(v) If the husband beats up his wife every time he comes home drunk. 5%

SUGGESTED ANSWER:
(i) Since AIDS is a serious and incurable sexually-transmissible disease, the wife
may file an action for annulment of the marriage on this ground whether such fact was
concealed or not from the wife, provided that the disease was present at the time of the
marriage. The marriage is voidable even though the husband was not aware that he
had the disease at the time of marriage.
(ii) If the wife refuses to come home for three (3) months from the expiration of
her contract, she is presumed to have abandoned the husband and he may file an
action for judicial separation of property.

If the refusal continues for more than one year from the expiration of her
contract, the husband may file the action for legal separation under Art. 55 (10) of the
Family Code on the ground of abandonment of petitioner by respondent without
justifiable cause for more than one year. The wife is deemed to have abandoned the
husband when she leaves the conjugal dwelling without any intention of returning
(Article 101, FC). The intention not to return cannot be presumed during the 3-year
period of her contract.

(iii) If the husband discovers after the marriage that his wife was a prostitute
before they got married, he has no remedy. No misrepresentation or deceit as to
character, health, rank, fortune or chastity shall constitute fraud as legal ground for an
action for the annulment of marriage (Article 46 FC).
(iv) The wife may file an action for legal separation. The husband’s sexual
infidelity is a ground for legal separation 9Article 55, FC). She may also file an
action for judicial separation of property for failure of her husband to comply with
his martial duty of fidelity (Article 135 (4), 101, FC).

(v) Under Article 55 of the Family Code, an action for legal separation on the
ground of repeated physical violence.

on her person (Article 55 (1), FC). She may also file an action for judicial
separation of property for failure of the husband to comply with his marital duty of
mutual respect (Article 135 (4), Article 101, FC). She may also file an action for
declaration of nullity of the marriage if the husband’s behavior constitute
psychological incapacity existing at the time of the celebration of marriage.

Notes:

(1) Under Article 45 of the Family Code, a marriage may be annulled if one of
the parties is suffering from sexually transmissible disease found to be serious and
incurable. Accordingly, the wife may file an action for annulment of marriage because
her husband is afflicted with AIDS.

(2) Under Article 55 of the Family Code, a petition for legal separation may be
filed on the ground of abandonment of the petitioner.
(3) Under Article 46 of the Family Code, misrepresentation or deceit as to
chastity shall not constitute fraud as a legal ground to annul marriage. Thus, if the
husband discovers after the marriage that his wife was a prostitute before they got
married, he has no remedy.

(4) Under Article 55 of the Family Code, a petition for legal separation may be
filed on the ground of sexual infidelity. Thus, intimate relationship with the Secretary
is a ground for legal separation.

Under Article 55 of the Family Code, an action for judicial separation of


property may be filed for failure to comply with marital duty of fidelity.
(5) Under Article 55 of the Family Code, action for legal separation may be
filed on the ground of repeated physical violence. Therefore, the wife may file an
action for legal separation because her husband always kicks her every time he is
drunk.

Under Article 36 of the Family Code, a marriage contracted by a party who is


psychologically incapacitated to comply with the essential marital obligation of
marriage at the time of its celebration shall be void.

Under Article 135 of the Family Code, an action for judicial separation of
property may be filed for failure to comply with marital duty of mutual respect.
Marriage; Grounds; Nullity; Annulment; Legal Separation (1997)
Under what conditions, respectively, may drug addiction be a ground, if at all,
(a) for a declaration of nullity of marriage, (b) for an annulment of the marriage, and
(c) for legal separation between the spouses?

SUGGESTED ANSWER:
(a) Declaration of nullity of marriage:

1) The drug addiction must amount to psychological incapacity to comply with


the essential obligations of marriage;

2) It must be antecedent (existing at the time of marriage), grave and incurable:

3) The case must be filed before August 1, 1998. Because if they got married
before August 3, 1998, it must be filed before August 1, 1998.

(b) Annulment of the Marriage Contract: 1) The drug addiction must be


concealed; 2) It must exist at the time of marriage; 3) There should be no cohabitation
with full knowledge of the drug addiction; 4) The case is filed within five (5) years
from discovery.

(c) Legal Separation; 1) There should be no condonation or consent to the drug


addiction; 2) The action must be filed within five (5) years from the occurrence of the
cause.

3) Drug addiction arises during the marriage and not at the time of marriage.

Notes:
(a) Drug addiction is a ground for declaration of nullity of marriage if:
1) it amounts to psychological incapacity to comply with the essential
obligations of marriage;
2) It is antecedent (existing at the time of marriage), grave and incurable:

(b) Drug addiction is a ground for annulment marriage if: 1) it was concealed;
2) It exists during the celebration of marriage; 3) There is no cohabitation with full
knowledge of the drug addiction; 4) The case is filed within five (5) years from
discovery.

(c) Drug addiction is a ground for legal separation if: 1) There is no condonation
or consent to the drug addiction; 2) The action is filed within five (5) years from the
occurrence of the cause; and (3) it arises during the marriage and not at the time of the
celebration of marriage.

Marriage; Legal Separation; Declaration of Nullity (2002)

If drug addiction, habitual alcoholism, lesbianism or homosexuality should


occur only during the marriage, would this constitute grounds for a declaration
of nullity or for legal separation, or would they render the marriage voidable?
(1%).

SUGGESTED ANSWER:
In accordance with law, if drug addiction, habitual alcoholism, lesbianism or
homosexuality should occur only during the marriage, they: a) Will not constitute as
ground for declaration of nullity (Art. 36, Family Code); b) Will constitute as grounds
for legal separation (Art. 56, FC) and c) will not constitute as grounds to render the
marriage voidable (Art.45and 46, FC)

Notes:
When drug addiction, habitual alcoholism, lesbianism or homosexuality, among
others, should occur during the marriage, they shall constitute a ground for legal
separation.

If they exist before marriage and concealed by the erring party, they shall
constitute a ground for annulment of marriage.
If they exist before marriage and constitute psychological incapacity to
perform the essential marital of marriage, they shall be a ground for nullity of
marriage.

Marriage; Legal Separation; Grounds; Prescriptive Period (1994)


Rosa and Ariel were married in the Catholic Church of Tarlac, Tarlac on
January 5. 1988. In 1990, Ariel went to Saudi Arabia to work. There, after being
converted into Islam, Ariel married Mystica, Rosa learned of the second marriage of
Ariel on January 1, 1992 when Ariel returned to the Philippines with Mystica. Rosa
filed an action for legal separation on February 5, 1994.

1) Does Rosa have legal grounds to ask for legal separation?


2) Has the action prescribed?

SUGGESTED ANSWER:
1) a) Yes, the abandonment of Rosa by Ariel for more than one (1) year is a
ground for legal separation unless upon returning to the Philippines, Rosa agrees to
cohabit with Ariel which is allowed under the Muslim Code. In this case, there is
condonation.

b) Yes. The contracting of a subsequent bigamous marriage whether in the


Philippines or abroad is a ground for legal separation under Article 55(7) of the
Family Code. Whether the second marriage is valid or not, Ariel having converted
into Islam, is immaterial.

SUGGESTED ANSWER:
2) No. Under Article 57 of the Family Code, the aggrieved spouse must file the
action within five (5) years from the occurrence of the cause. The subsequent
marriage of Ariel could not have occurred earlier than 1990, the time he went to Saudi
Arabia. Hence, Rosa has until 1995 to bring the action under the Family Code.

Notes:
Under Article 55 of the Family Code, a petition for legal separation may be
filed on the ground of sexual infidelity. By contracting a second marriage during the
existence of the prior marriage is sexual infidelity. Thus, the aggrieved spouse may
file a petition for legal separation.
The prescriptive period of an action for legal separation is 5 years from the
existence of the cause of action.

Marriage; Legal Separation; Mutual guilt (2006)


Saul, a married man, had an adulterous relation with Tessie. In one of the
trysts, Saul's wife, Cecile, caught them in flagrante. Armed with a gun, Cecile shot
Saul in a fit of extreme jealousy, nearly killing him. Four (4) years after the incident,
Saul filed an action for legal separation against Cecile on the ground that she
attempted to kill him.

(1) If you were Saul's counsel, how will you argue his case? (2.5%)

SUGGESTED ANSWER:

As the counsel of Saul, I will argue that an attempt by the wife against the life of
the husband is one of the grounds enumerated by the Family Code for legal separation
and there is no need for criminal conviction for the ground to be invoked (Art. 55, par.
9, Family Code).

Notes:
Under the Family Code, a petition for legal separation may be filed based on the
ground of an attempt against the life of the petitioner by his spouse.

(2) If you were the lawyer of Cecile, what will be your defense? (2.5%)

SUGGESTED ANSWER:

As the counsel of Cecile, I will invoke the adultery of Saul. Mutual guilt is a
ground for the dismissal of an action for legal separation (Art. 56, par. 4, Family
Code). The rule is anchored on a well-established principle that one must come to
court with clean hands.

Notes:
Under the Family Code, a petition for legal separation shall be denied if both
parties have given ground for legal separation. This is known as mutual guilt.
(3) If you were the judge, how will you decide the case? (5%)
SUGGESTED ANSWER:
If I were the judge, I will dismiss the action on the ground of mutual guilt of the
parties. The Philippine Constitution protects marriage as an inviolable social
institution (Art. XV, Sec. 2, 1987 Constitution). An action for legal separation
involves public interest and no such decree should be issued if any legal obstacle
thereto appears on record. This is in line with the policy that in case of doubt, the
court shall uphold the validity and sanctity of marriage (Brown v. Yambao, G.R. No.
L-10699, October 18, 1957).

Notes:
In Brown v. Yambao, it was held that in case of doubt, the court shall uphold the
validity of marriage because of constitutional provision which protects marriage as an
inviolable social institution.

Marriage; Non-Bigamous Marriages (2006)


Marvin, a Filipino, and Shelley, an American, both residents of California,
decided to get married in their local parish. Two years after their marriage, Shelley
obtained a divorce in California. While in Boracay, Marvin met Manel, a Filipina,
who was vacationing there. Marvin fell in love with her. After a brief courtship and
complying with all the requirements, they got married in Hongkong to avoid publicity,
it being Marvin's second marriage. Is his marriage to Manel valid? Explain. (5%)

SUGGESTED ANSWER:
Yes. The marriage will not fall under Art. 35(4) of the Family Code on
bigamous marriages, provided that Shelley obtained an absolute divorce, capacitating
her to remarry under her national law. Consequently, the marriage between Marvin
and Manel may be valid as long as it was solemnized and valid in accordance with the
laws of Hongkong [Art. 26, paragraphs 1 and 2, Family Code].

Notes:
A Filipino spouse is capacitated to remarry if his or her alien spouse obtained a
valid divorce in a foreign country.

Art. 26. All marriages solemnized outside the Philippines shall be valid in this
country if the marriages are valid where they were solemnized and in accordance with
the laws in force in that country except those prohibited under Articles 35 (1), (4), (5)
and (6), 3637 and 38.(Reengineered)
Where a marriage between a Filipino citizen and a foreigner is validly
celebrated and a divorce is thereafter validly obtained abroad by the alien spouse
capacitating him or her to remarry, the Filipino spouse shall have capacity to remarry
under Philippine law.

Marriage; Property Relations; Void Marriages (1991)


In June 1985, James married Mary. In September 1988, he also married
Ophelia with whom he begot two (2) children, A and B. In July 1989, Mary died. In
July 1990, he married Shirley and abandoned Ophelia, During their union. James and
Ophelia acquired a residential lot worth P300,000.00.

Ophelia sues James for bigamy and prays that his marriage with Shirley be
declared null and void. James, on the other hand, claims that since his marriage to
Ophelia was contracted during the existence of his marriage with Mary, the
former is not binding upon him, the same being void ab initio he further claims
that his marriage to Shirley is valid and binding as he was already legally
capacitated at the time he married her.
a) Is the contention of James correct?
b) What property Relations governed the union of James and Ophelia?
c) Is the estate of Mary entitled to a share in the residential lot acquired by
James and Ophelia?

SUGGESTED ANSWER:

A. Yes. His marriage to Ophelia is void ab initio because of his subsisting prior
marriage to Mary. His marriage to Shirley, after Mary's death, is valid and binding.

Notes:
For purposes of remarriage, a final judgement declaring a prior marriage as void
is necessary; without such judgement, the absolute nullity of the previous marriage
cannot be invoked.

Art. 40. For purposes of remarriage, the absolute nullity of a previous marriage
may be invoked on the basis of a final judgment only declaring such previous
marriage void. (Reengineered)

ALTERNATIVE ANSWER:
A. No. The contention of James is not correct. Art. 40, Family Code, provides
that the "absolute nullity of a previous marriage may be invoked for purposes of
remarriage on the basis solely of a final judgment declaring such previous marriage
void." It can be said, therefore, that the marriage of James to Shirley is void since his
previous marriage to Ophelia, although itself void, had not yet been judicially declared
void,
ALTERNATIVE ANSWER:
A. No. The contention of James is not correct. He cannot set up as a defense his
own criminal act or wrongdoing-
SUGGESTED ANSWER:
B. The provisions of Art 148 of the Family Code, shall govern: Art. 148. In
cases of cohabitation not falling under the preceding Article, only the properties
acquired by both of the parties through their actual joint contribution of money,
property, or industry shall be owned by them in common in proportion to their
respective contributions. In the absence, of proof to the contrary, their contributions
and corresponding shares are presumed to be equal. The same rule and presumption
shall apply to joint deposits of money and evidences of credit.

Notes:
Under Article 148 of the Family Code, in case of cohabitation with legal
impediment, only properties acquired by them through their actual contribution of
money, property or industry shall be owned by them in common in proportion to
their respective contribution.

Under Article 147 of the Family Code, in case of cohabitation without the
benefit of marriage and without legal impediment, their salaries and wages shall be
owned by them in equal shares following the rules on co-ownership.

Art. 148. In cases of cohabitation not falling under the preceding Article, only
the properties acquired by both of the parties through their actual joint contribution
of money, property, or industry shall be owned by them in common in proportion to
their respective contributions. In the absence of proof to the contrary, their
contributions and corresponding shares are presumed to be equal. The same rule and
presumption shall apply to joint deposits of money and evidences of credit.

If one of the parties is validly married to another, his or her share in the co-
ownership shall accrue to the absolute community or conjugal partnership existing in
such valid marriage. If the party who acted in bad faith is not validly married to
another, his or her shall be forfeited in the manner provided in the last paragraph of
the preceding Article.
The foregoing rules on forfeiture shall likewise apply even if both parties are in bad
faith.

Notes:
In Art. 147, either party to the union is capacitated to marry, while in Art. 148,
either party is not capacitated to marry.

In Art. 147, their wages and salaries shall be owned by them in equal shares and
the property acquired by them through their work and industry shall be governed by
the rules on co-ownership.

On the other hand, under Art. 148, only the properties acquired by both of the
parties through their actual joint contribution of money, property or industry shall be
owned by them in common in proportion to their respective contribution.

SUGGESTED ANSWER:
C. It should be distinguished when the property was acquired.
1. If it was acquired before Mary's death, the estate of Mary is entitled to 1/2
of the share of James.
2. If it was acquired after Mary's death, there will be no share at all for the
estate of Mary.

Notes:
Yes, the estate of Mary is entitled to a share in the residential lot acquired by
James and Ophelia because the contribution of James to buy such lot is conjugal
money of James and Mary. When Mary died, one-half of the contribution shall go to
the estate of Mary.

Marriage; Psychological Incapacity (1996)


On April 15, 1983, Jose, an engineer, and Marina, a nurse, were married to
each other in a civil ceremony in Boac. Marinduque. Six months after their marriage,
Jose was employed in an oil refinery in Saudi Arabia for a period of three years. When
he returned to the Philippines, Marina was no longer living in their house, but in
Zamboanga City, working in a hospital. He asked her to come home, but she refused
to do so, unless he agreed not to work overseas anymore because she cannot stand
living alone. He could not agree as in fact, he had signed another three year contract.
When he returned in 1989, he could not locate Marina anymore. In 1992, Jose filed an
action served by publication in a newspaper of general circulation. Marina did not file
any answer, a possible collusion between the parties was ruled out by the Public
Prosecutor. Trial was conducted and Marina neither appeared nor presented
evidence in her favor. If you were the judge, will you grant the annulment.
Explain.

SUGGESTED ANSWER:
As judge, I will not grant the annulment. The facts do not show any taint of
personality disorder on the part of the wife Marina so as to lend substance to her
husband's averment of psychological incapacity within the meaning of Art 36 of the
Family Code. In Santos vs. CA (240 SCRA 20), this particular ground for nullity of
marriage was held to be limited only to the most serious cases of personality
disorders (clearly demonstrative of utter sensitivity or inability to give meaning and
significance to the marriage. Marina's refusal to come home to her husband unless he
agreed not to work overseas, far from being indicative of an insensitivity to the
meaning of marriage, or of a personality disorder, actually shows a sensitive
awareness on her part of the marital duty to live together as husband and wife.
Mere refusal to rejoin her husband when he did not accept the condition imposed by
her does not furnish any basis for concluding that she was suffering from
psychological incapacity to discharge the essential marital obligations.

Mere intention to live apart does not fall under Art. 36, FC. Furthermore, there is
no proof that the alleged psychological incapacity existed at the time of the marriage.
Notes:
In Santos vs. CA, the Supreme Court held that psychological incapacity may be
a ground for nullity of marriage if it is a serious case of personality disorder such that
a spouse is unable to give the meaning and significance of marriage.
( When a question calls for psychological incapacity, the answer must be in the
negative).

Marriage; Psychological Incapacity (2006)


Gemma filed a petition for the declaration of nullity of her marriage with Arnell
on the ground of psychological incapacity. She alleged that after 2 months of their
marriage, Arnell showed signs of disinterest in her, neglected her and went abroad. He
returned to the Philippines after 3 years but did not even get in touch with her. Worse,
they met several times in social functions but he snubbed her. When she got sick, he
did not visit her even if he knew of her confinement in the hospital. Meanwhile,
Arnell met an accident which disabled him from reporting for work and earning a
living to support himself. Will Gemma's suit prosper? Explain. (5%)
SUGGESTED ANSWER:
No, Gemma's suit will not prosper. Even if taken as true, the grounds, singly or
collectively, do not constitute "psychological incapacity." In Santos v. CA, G.R. No.
112019, January 4, 1995, the Supreme Court clearly explained that "psychological
incapacity must be characterized by (a) gravity, (b) juridical antecedence, and (c)
incurability"(Ferraris v. Ferraris, G.R. No. 162368, July 17, 2006; Choa v. Choa,
G.R. No. 143376, November 26, 2002). The illness must be shown as downright
incapacity or inability to perform one's marital obligations, not a mere refusal,
neglect, difficulty or much less, ill will. Moreover, as ruled in Republic v. Molina,
GR No. 108763, February 13, 1997, it is essential that the husband is capable of
meeting his marital responsibilities due to psychological and not physical
illness(Antonio v. Reyes, G.R. No. 155800, March 10, 2006; Republic
v. Quintero-Hamano, G.R. No. 149498, May 20, 2004). Furthermore, the condition
complained of did not exist at the time of the celebration of marriage.
Notes:
In Republic v. Molina, it was held that it is essential that the husband is
incapable of meeting his marital responsibilities due to psychological and not physical
illness, and it is not merely a refusal or neglect to do the same.

In Santos v. CA, the Supreme Court clearly explained that "psychological


incapacity as a ground for nullity of marriage must be (a) serious, (b) juridically
antecedent (existing before marriage), and (c) incurable.

(When a question is asked in the bar exams regarding psychological incapacity,


the safest answer is not to grant the petition or to uphold the validity of marriage).

Art. 36. A marriage contracted by any party shall be void if either party was
psychologically incapacitated to comply with the essential marital obligations of
marriage at the time of the celebration even if such incapacity becomes manifest only
after its solemnization.(Reengineered)

Marriage; Psychological Incapacity (2006)


Article 36 of the Family Code provides that a marriage contracted by any party
who, at the time of the celebration, was psychologically incapacitated to comply with
the essential marital obligations of marriage, shall be void. Choose the spouse listed
below who is psychologically incapacitated. Explain. (2.5%) a) Nagger b) Gay or
Lesbian c) Congenital sexual pervert d) Gambler e) Alcoholic
SUGGESTED ANSWER:
The best answers are B and C. To be sure, the existence and concealment of
these conditions at the inception of marriage renders the marriage contract voidable
(Art. 46, Family Code). They may serve as indicia of psychological incapacity,
depending on the degree and severity of the disorder (Santos v. CA, G.R. No.
112019, Jan. 4, 1995).

Hence, if the condition of homosexuality, lesbianism or sexual perversion,


existing at the inception of the marriage, is of such a degree as to prevent any form of
sexual intimacy, any of them may qualify as a ground for psychological incapacity.
The law provides that the husband and wife are obliged to live together, observe
mutual love, respect and fidelity (Art. 68, Family Code). The mandate is actually the
spontaneous, mutual affection between the spouses. In the natural order, it is sexual
intimacy which brings the spouses wholeness and oneness (Chi Ming Tsoi v. CA,
G.R. No. 119190, January 16,1997).

Notes:
In the case of Chi Ming Tsoi v. CA, the Supreme Court held that sexual
intimacy brings the spouses to oneness and wholeness. Therefore, if the condition of
homosexuality, lesbianism or sexual perversion, existing at the time of marriage,
prevents sexual intimacy, anyone of them may be a ground for nullity of marriage.

ALTERNATIVE ANSWER:
None of them are necessarily psychologically incapacitated. Being a nagger, etc.
are at best only physical manifestations indicative of psychological incapacity. More
than just showing the manifestations of incapacity, the petitioner must show that the
respondent is incapacitated to comply with the essential marital obligations of
marriage and that it is also essential that he must be shown to be incapable of doing so
due to some psychological, not physical illness (Republic v. Quintero-Hamano,
G.R. No. 149498, May 20, 2004).

ALTERNATIVE ANSWER:
A congenital sexual pervert may be psychologically incapacitated if his
perversion incapacitates him from discharging his marital obligations. For instance, if
his perversion is of such a nature as to preclude any normal sexual activity with his
spouse.

Marriage; Requisites (1995)


Isidro and Irma, Filipinos, both 18 years of age, were passengers of Flight No.
317 of Oriental Airlines. The plane they boarded was of Philippine registry. While
en route from Manila to Greece some passengers hijacked the plane, held the chief
pilot hostage at the cockpit and ordered him to fly instead to Libya. During the
hijacking Isidro suffered a heart attack and was on the verge of death. Since Irma was
already eight months pregnant by Isidro, she pleaded to the hijackers to allow the
assistant pilot to solemnize her marriage with Isidro. Soon after the marriage, Isidro
expired. As the plane landed in Libya Irma gave birth. However, the baby died a few
minutes after complete delivery. Back in the Philippines Irma immediately filed a
claim for inheritance. The parents of Isidro opposed her claim contending that the
marriage between her and Isidro was void ab initio on the following grounds: (a)
they had not given their consent to the marriage of their son; (b) there was no
marriage license; (c) the solemnizing officer had no authority to perform the
marriage; and, (d) the solemnizing officer did not file an affidavit of marriage
with the proper civil registrar.

1. Resolve each of the contentions ([a] to [d]) raised by the parents of Isidro.
Discuss fully.

SUGGESTED ANSWER:
1. (a) The fact that the parents of Isidro and of Irma did not give their consent to
the marriage did not make the marriage void ab initio. The marriage is merely
voidable under Art 45 of the FC.
(b) Absence of marriage license did not make the marriage void ab initio. Since
the marriage was solemnized in articulo mortis, it was exempt from the license
requirement under Art. 31 of the FC.
(c) On the assumption that the assistant pilot was acting for and in behalf of the
airplane chief who was under disability, and by reason of the extraordinary and
exceptional circumstances of the case [ie. hostage situation), the marriage was
solemnized by an authorized officer under Art. 7 (3) and Art. 31. of the FC.
(d) Failure of the solemnizing officer to file the affidavit of marriage did not
affect the validity of the marriage. It is merely an irregularity which may subject the
solemnizing officer to sanctions.

Notes:
1. (a) Under Article 45 of the Family Code, a marriage between 18 years old or
over but below 21 without the consent of the parents is voidable, hence, can be
annulled.
(b) Under Article 31 of the Family Code, a marriage in articulo mortis is exempt
from license.

(c) Under Article 31 of the Family Code, a marriage in articulo mortis between
passengers or crew members may solemnized by an airplane pilot.

(d) Failure of the solemnizing officer to file the affidavit of marriage did not
affect the validity of the marriage. It is merely an irregularity which may subject the
solemnizing officer to sanctions.

ALTERNATIVE ANSWER:
Considering that the solemnizing officer has no authority to perform the
marriage because under Art. 7 the law authorizes only the airplane chief, the marriage
is void, hence, a, c, and d are immaterial.

Notes:
Under Article 31 of the Family Code, marriage in articulo mortis between
passengers or crew members may solemnized by an airplane pilot.

Marriage; Requisites (1999)


What is the status of the following marriages and why?
(a) A marriage between two 19-year olds without parental consent, (2%)
voidable
(b) A marriage between two 21-year olds without parental advice. (2%)
irregularity affecting only formal requisite, valid
(c) A marriage between two Filipino first cousins in Spain where such marriage
is valid. (2%) void, against public policy
(d) A marriage between two Filipinos in Hongkong before a notary public. (2%)
valid, if valid in HK
(e) A marriage solemnized by a town mayor three towns away from his
jurisdiction, (2%) valid

SUGGESTED ANSWER:
(a) The marriage is voidable. The consent of the parties to the marriage was
defective. Being below 21 years old, the consent of the parties is not full without the
consent of their parents. The consent of the parents of the parties to the marriage is
indispensable for its validity.

Notes:
Under the Family Code, consent of the parties to the marriage who are below 21
but above 18 is not full so it needs the consent of the parents; otherwise, it is voidable.

SUGGESTED ANSWER:
(b) Between 21 years old, the marriage is valid despite the absence of parental
advice, because such absence is merely an irregularity affecting a formal requisite
i.e., the marriage license and does not affect the validity of the marriage itself. This is
without prejudice to the civil, criminal, or administrative liability of the party
responsible therefor.

Notes:
Marriage between 21 years old without parental advice is valid because its
absence is merely an irregularity in the formal requisite of marriage. But one who is
liable for the absence of parental advice shall be held civilly, criminally or
administratively liable.

SUGGESTED ANSWER:
(c) By reason of public policy, the marriage between Filipino first cousins is
void [Art. 38, par. (1), Family Code], and the fact that it is considered a valid marriage
in a foreign country in this case, Spain— does not validate it, being an exception to
the general rule in Art. 96 of said Code which accords validity to all marriage
solemnized outside the Philippine x x x and valid there as such.

Notes:
Under Article 38 of the Family Code, marriage between first cousin is void by
reason of public policy.
This marriage is still void even though valid where celebrated.

Under Article 26 of the Family Code, marriages outside of the Philippines are
valid if valid where celebrated; however, marriages between first cousin is void
although valid where celebrated.
ALTERNATIVE ANSWER
The marriage is void. Under Article 96 of the Family Code, a marriage valid
where celebrated is valid in the Philippines except those marriages enumerated in said
Article which marriages will remain void even though valid where solemnized. The
marriage between first cousins is one of those marriages enumerated therein, hence, it
is void even though valid in Spain where it was celebrated.
By reason of Art. 15 in relation to Article 38 of the Civil Code, which applies to
Filipinos wherever they are, the marriage is void.
SUGGESTED ANSWER:
(d) It depends. If the marriage before the notary public is valid under Hongkong Law,
the marriage is valid in the Philippines. Otherwise, the marriage that is invalid in
Hongkong will be invalid in the Philippines. If it is not prohibited in the Philippines
and valid abroad, it is valid

ALTERNATIVE ANSWER:
If the two Filipinos believed in good faith that the Notary Public is authorized to
solemnize marriage, then the marriage is valid.

SUGGESTED ANSWER:
(e) Under the Local Government Code, a town mayor may validly solemnize a
marriage but said law is silent as to the territorial limits for the exercise by a town
mayor of such authority. However, by analogy, with the authority of members of the
Judiciary to solemnize a marriage, it would seem that the mayor did not have the
requisite authority to solemnize a marriage outside of his territorial jurisdiction.
Hence, the marriage is void, unless it was contracted with either or both parties
believing in good faith that the mayor had the legal authority to solemnize this
particular marriage (Art 35, par 2 Family Code).

ALTERNATIVE ANSWER:
The marriage is valid. Under the Local Government Code, the authority of a
mayor to solemnize marriages is not restricted within his municipality implying that
he has the authority even outside the territory thereof. Hence, the marriage he
solemnized outside his municipality is valid. And even assuming that his authority is
restricted within his municipality, such marriage will nevertheless, be valid because
solemnizing the marriage outside said municipality is a mere irregularity applying by
analogy the case of Navarro v Domagtoy, 259 Scra 129. In this case, the Supreme
Court held that the celebration by a judge of a marriage outside the jurisdiction of his
court is a mere irregularity that did not affect the validity of the marriage
notwithstanding Article 7 of the Family Code which provides that an incumbent
member of the judiciary is authorized to solemnize marriages only within the court’s
jurisdiction.

Notes:
Applying by analogy, In Navarro v Domagtoy, it was held that the marriage is
valid although the judge solemnized the marriage outside of the jurisdiction of the
court. There is only irregularity in the formal requisite of marriage; it does not affect
its validity. But the solemnizing officer will be administratively liable. Moreover, the
law granting authority to the Mayor in solemnizing marriages did not restrict said
Mayor’s authority within its territory.

(Discussed by Dean Albano).

Marriage; Requisites; Marriage License (1996)


On Valentine's Day 1996, Ellas and Fely, both single and 25 years of age, went
to the city hall where they sought out a fixer to help them obtain a quickie marriage.
For a fee, the fixer produced an ante-dated marriage license for them, Issued by the
Civil Registrar of a small remote municipality. He then brought them to a licensed
minister in a restaurant behind the city hall, and the latter solemnized their marriage
right there and then.

1) Is their marriage valid, void or voidable? Explain. Valid, as to the


license it is only irregular

SUGGESTED ANSWER:
The marriage is valid. The irregularity in the issuance of a valid license does
not adversely affect the validity of the marriage. The marriage license is valid because
it was in fact issued by a Civil Registrar (Arts. 3 and 4. FC).

Notes:
Under the Family Code, irregularity in the issuance of a marriage license does
not adversely affect the marriage; therefore, the marriage is valid.

ATIVE ANSWER:
It depends. If both or one of the parties was a member of the religious sect of the
solemnizing officer, the marriage is valid. If none of the parties is a member of the
sect and both of them were aware of the fact, the marriage is void. They cannot claim
good faith in believing that the solemnizing officer was authorized because the scope
of the authority of the solemnizing officer is a matter of law. If, however, one of the
parties believed in good faith that the other was a member of the sect, then the
marriage is valid under Article 35 (2), FC. In that case, the party in good faith is acting
under a mistake of fact, not a mistake of law,
2) Would your answer be the same if it should turn out that the marriage
license was spurious? Explain. No, marriage is void because license is void, hence
absence of formal re quisites
SUGGESTED ANSWER:
No, the answer would not be the same. The marriage would be void because of
the absence of a formal requisite. In such a case, there was actually no valid marriage
license.

Notes:
Under the law, if the marriage license is spurious, the marriage is void because
of absence of the formal requisite of marriage.

Marriage; Requisites; Marriage License (2002)


On May 1, 1978 Facundo married Petra, by whom he had a son Sotero. Petra
died on July 1, 1996, while Facundo died on January 1, 2002. Before his demise,
Facundo had married, on July 1, 2002, Quercia. Having lived together as husband
and wife since July 1, 1990, Facundo and Quercia did not secure a marriage license
but executed the requisite affidavit for the purpose. To ensure that his inheritance
rights are not adversely affected by his father second marriage, Sotero now brings a
suit to seek a declaration of the nullity of the marriage of Facundo and Quercia,
grounded on the absence of a valid marriage license. Quercia contends that there was
no need for a marriage license in view for her having lived continuously with Facundo
for five years before their marriage and that has Sotero has no legal personality to seek
a declaration of nullity of the marriage since Facundo is now deceased.

A. Is the marriage of Facundo and Quercia valid, despite the absence of a


marriage license? Explain. (2%) void, not exempted from marriage license since
during cohabitation, Facundo is not capacitated to marry Quercia because his
marriage with Petra was still subsisting at the time of cohabitation.

SUGGESTED ANSWER:
A. The marriage with Quercia is void. The exemption from the requirement of a
marriage license under Art, 34, Family Code, requires that the man and woman must
have lived together as husband and wife for at least five years and without any legal
impediment to marry each other during those five years. The cohabitation of Facundo
and Quercia for six years from 1990 to July 1, 1996 when Petra died was one with a
legal impediment hence, not in compliance with the requirement of law. On other
hand, the cohabitation thereafter until the marriage on July 1, 2000, although free from
legal impediment, did not meet the 5-year cohabitation requirement. (if second
marriage was celebrated during NCC)

Notes:
Under the Family Code, marriage between a man and a woman who live
together as husband and wife for 5 years without legal impediment is exempt from
marriage license.

ALTERNATIVE ANSWER:
The marriage of Facundo and Quercia is VALID. The second marriage was
solemnized on July 1, 2000, when the Family code was already affective. The family
Code took effect on August 3, 1988. Under the Family Code, no marriage license
is required if the parties have been cohabiting for the period of five years and
there is no legal impediment. There must no legal impediment ONLY AT THE
TIME OF THE SOLEMNIZATION OF THE MARRIAGE, and not the whole five
years period. This is clearly the intent of the code framers (see Minutes of the 150th
joint Civil Code of the Family Law Committees held on August 9, 1986). Also, in
Manzano V. Sanchez, AM NO. MT –00-129, March 8, 2001, the Supreme Court said
that, as one of the requisites for the exception to apply, there must be no legal
impediment at the time of the marriage. The Supreme Court did not say that the legal
impediment must exist all throughout the five-year period. This is different from the
case of Nināl V. Bayadog, (328 SCRA 122 [2000]). In the said case, the situation
occurred during the Relations of the new Civil Code where Article 76 thereof clearly
provides that during the five-year cohabitation, the parties must be unmarried. This is
not so anymore in the Family Code. The Change in the Family Code is significant. If
the second marriage occurred before the effectivity of the Family Code, the answer
would that be that the marriage is void.

Notes:

If marriage was celebrated before family code, there must be no legal impediment
within the entire 5-year period of cohabitation for a marriage to be exempt from
marriage license

If marriage was celebrated during family code, the law requires the parties to have no
legal impediment only at the time of solemnization of marriage.

B. Does Sotero have the personality to seek the declaration of nullity of the
marriage, especially now that Facundo is already deceased? Explain. (3%)

SUGGESTED ANSWER:
B. A void marriage may be questioned by any interested party in any proceeding
where the resolution of the issue is material. Being a compulsory heir, Soterro has the
personality to question the validity of the marriage of Facundo and Quercia.
Otherwise, his participation in the estate on Facundo would be affected. (Ninãl V.
Bayadog, 328 SCRA 122 [2000] ).

Notes:

GR: Void marriage may inly be assailed by the husband or wife


EXP: By any real party in interest, only in the ff cases:
1. Nullity of marriage cases commenced before the effectivity of AM no.
02-11-10 (March 15, 2003)
2. Marriages celebrated during the effectivity of civil code
3. A party to a previous marriage may seek the nullity of the subsequent
marriage on the ground that the subsequent marriage is bigamous

Marriage; Requisites; Solemnizing Officers (1994)

1} The complete publication of the Family Code was made on August 4,


1987. On September 4, 1987, Junior Cruz and Gemma Reyes were married
before a municipal mayor. Was the marriage valid? Valid, LGC, Arts. 444 and
445

2) Suppose the couple got married on September 1, 1994 at the Manila


Hotel before the Philippine Consul General to Hongkong, who was on vacation in
Manila. The couple executed an affidavit consenting to the celebration of the
marriage at the Manila Hotel. Is the marriage valid? Void, not authorized if
celebrated within Philippijnes

SUGGESTED ANSWER:
1) Yes, the marriage is valid. The Family Code took effect on August 3, 1988.
At the time of the marriage on September 4, 1987, municipal mayors were empowered
to solemnize marriage under the Civil Code of 1950.

2) The marriage is not valid. Consuls and vice-consuls are empowered to


solemnize marriages between Philippine citizens abroad in the consular office of the
foreign country to which they were assigned and have no power to solemnize
marriage on Philippine soil.

b) A Philippine consul is authorized by law to solemnize marriages abroad


between Filipino citizens. He has no authority to solemnize a marriage in the
Philippines. Consequently, the marriage in question is void, unless either or both of
the contracting parties believed in good faith that the consul general had authority to
solemnize their marriage in which case the marriage is valid.

Notes:
1) From the time of the effectivity of FC to the time of effectivity of Local
government code, mayors are prohibited. Also incumbent judiciary member; priest,
rabbi, imam, or minister of any church/ religious sectors; consul general, consul, vice-
consul, mayors

2) Under the Family Code, consuls and vice-consuls are empowered to


solemnize marriages between Philippine citizens abroad in the consular office of
the foreign country to which they were assigned and have no power to solemnize
marriage on Philippine soil.

b) A Philippine consul is authorized by law to solemnize marriages abroad


between Filipino citizens. He has no authority to solemnize a marriage in the
Philippines. Consequently, the marriage in question is void, unless either or both of
the contracting parties believed in good faith that the consul general had authority to
solemnize their marriage in which case the marriage is valid.

Marriage; Requisites; Void Marriage (1993)


A and B, both 18 years old, were sweethearts studying in Manila. On August 3,
1988, while in first year college, they eloped. They stayed in the house of a mutual
friend in town X, where they were able to obtain a marriage license. On August 30,
1988, their marriage was solemnized by the town mayor of X in his office.
Thereafter, they returned to Manila and continued to live separately in their respective
boarding houses, concealing from their parents, who were living in the province what
they had done. In 1992, after graduation from college, A and B decided to break their
relation and parted ways. Both went home to their respective towns to live and work.

1) Was the marriage of A and B solemnized on August 30, 1988 by the town
mayor of X in his office a valid marriage? Explain your answer.

2) Can either or both of them contract marriage with another person


without committing bigamy? Explain your answer. No, they must first file a
proper proceeding to terminate their marriage as their marriage is considered valid

SUGGESTED ANSWER:
1) The marriage of A and B is void because the solemnizing officer had no legal
authority to solemnize the marriage. But if either or both parties believed in good faith
that the solemnizing officer had the legal authority to do so, the marriage is voidable
because the marriage between the parties, both below 21 years of age, was solemnized
without the consent of the parents. (Art. 35, par. (2) and Art. 45 par. (1), Family Code)
2) Either or both of the parties cannot contract marriage in the Philippines with
another person without committing bigamy, unless there is compliance with the
requirements of Article 52 Family Code, namely: there must be a judgment of
annulment or absolute nullity of the marriage, partition and distribution of the
properties of the spouses and the delivery of their children's presumptive legitimes,
which shall be recorded in the appropriate Civil Registry and Registry of Property,
otherwise the same shall not affect third persons and the subsequent marriage shall be
null and void. (Arts. 52 and 53. Family Code)

Notes:
1) Under the Family Code, a town mayor has no authority to solemnize
marriage. Until the Local Government unit took effect, in which case, the mayor is
now authorize.

2) Under the Family Code, when the marriage is void or voidable, a spouse who
wants to contract a second marriage must file case for the judicial declaration of such
marriage as void or voidable; otherwise, he is liable for bigamy. In addition, there
must be a partition of the properties of spouses and delivery of the presumptive
legitimes; such facts shall be registered in the Civil registry and Registry of
Property.

Marriage; Void Marriages (2004)


A. BONI and ANNE met while working overseas. They became sweethearts
and got engaged to be married on New Year’s Eve aboard a cruise ship in the
Caribbean. They took the proper license to marry in New York City, where there is a
Filipino consulate. But as planned the wedding ceremony was officiated by the
captain of the Norwegian-registered vessel in a private suite among selected friends.
Back in Manila, Anne discovered that Boni had been married in Bacolod City 5
years earlier but divorced in Oslo only last year. His first wife was also a Filipina but
now based in Sweden. Boni himself is a resident of Norway where he and Anne plan
to live permanently.

Anne retains your services to advise her on whether her marriage to Boni is
valid under Philippine law? Void, if boni is a filipino because there is no divorce
here, hence his first marriage was never terminated and his subsequent marriage with
Anne is deemed void for being bigamous.
License i

Is there anything else she should do under the circumstances? (5%)

SUGGESTED ANSWER:
If Boni is still a Filipino citizen, his legal capacity is governed by Philippine
Law (Art. 15 Civil Code). Under Philippine Law, his marriage to Anne is void
because of a prior existing marriage which was not dissolved by the divorce decreed
in Oslo. Divorce obtained abroad by a Filipino is not recognized.

If Boni was no longer a Filipino citizen, the divorce is valid. Hence, his marriage
to Anne is valid if celebrated in accordance with the law of the place where it was
celebrated. Since the marriage was celebrated aboard a vessel of Norwegian registry,
Norwegian law applies. If the Ship Captain has authority to solemnize the marriage
aboard his ship, the marriage is valid and shall be recognized in the Philippines. As to
the second question, if Boni is still a Filipino, Anne can file an action for
declaration of nullity of her marriage to him.

Notes:
If not prohibited by Philippine law and valid of in accordance with the law
of the place where it was celebrated, valid

Marriage; Void Marriages (2006)


Gigi and Ric, Catholics, got married when they were 18 years old. Their
marriage was solemnized on August 2, 1989 by Ric's uncle, a Baptist Minister, in
Calamba, Laguna. He overlooked the fact that his license to solemnize marriage
expired the month before and that the parties do not belong to his congregation. After
5 years of married life and blessed with 2 children, the spouses developed
irreconcilable differences, so they parted ways. While separated, Ric fell in love with
Juliet, a 16 year-old sophomore in a local college and a Seventh-Day Adventist. They
decided to get married with the consent of Juliet's parents. She presented to him a
birth certificate showing she is 18 years old. Ric never doubted her age much less the
authenticity of her birth certificate. They got married in a Catholic church in Manila.
A year after, Juliet gave birth to twins, Aissa and Aretha.

(1) What is the status of the marriage between Gigi and Ric — valid,
voidable or void? Explain. (2.5%) valid, if parties believed that such solemnizing
officer is indeed vested with authority to solemnize

SUGGESTED ANSWER:
Even if the Minister's license expired, the marriage is valid if either or both Gigi
and Ric believed in good faith that he had the legal authority to solemnize marriage.
While the authority of the solemnizing officer is a formal requisite of marriage, and at
least one of the parties must belong to the solemnizing officer's church, the law
provides that the good faith of the parties cures the defect in the lack of authority
of the solemnizing officer (Art. 35 par. 2, Family Code; Sempio-Diy, p. 34; Rabuya,
The Law on Persons and Family Relations, p. 208). The absence of parental consent
despite their having married at the age of 18 is deemed cured by their continued
cohabitation beyond the age of 21. At this point, their marriage is valid (See Art. 45,
Family Code).

Notes:
the law provides that the good faith of the parties cures the defect in the lack of
authority of the solemnizing officer

Under the Family Code, irregularity in the formal requisites of marriage will not
render the marriage void or voidable. The expiration of the authority of the Baptist
Minister to solemnize is merely irregularity in the formal requisite. Thus, the marriage
between Gigi and Ric is valid.

(2) What is the status of the marriage between Ric and Juliet — valid,
voidable or void? (2.5%) void, bigamous
SUGGESTED ANSWER:
The marriage between Juliet and Ric is void. First of all, the marriage is a
bigamous marriage not falling under Article 41 [Art. 35(4) Family Code], A
subsisting marriage constitutes a legal impediment to remarriage. Secondly, Juliet is
below eighteen years of age. The marriage is void even if consented to by her parents
[Art. 35(1), Family Code]. The fact that Ric was not aware of her real age is
immaterial.

Notes:
Under the Family Code, a subsequent marriage contracted during the existence
of the previous marriage is void.

(3) Suppose Ric himself procured the falsified birth certificate to persuade
Juliet to marry him despite her minority and assured her that everything is in
order. He did not divulge to her his prior marriage with Gigi. What action, if
any, can Juliet take against him? Explain. (2.5%)

SUGGESTED ANSWER:
Juliet can file an action for the declaration of nullity of the marriage on the
ground that he willfully caused loss or injury to her in a manner that is contrary to
morals, good customs and public policy [Art. 21, New Civil Code]. She may also
bring criminal actions for seduction, falsification, illegal marriage and bigamy against
Ric.

Notes:
Under Article 4 of the Family Code, absence of the essential requisite shall
render the marriage void ab initio. The marriage between Ric and Julieta is void
because Julieta is only 16 years old.

(4) If you were the counsel for Gigi, what action/s will you take to enforce
and protect her interests? Explain. (2.5%)

SUGGESTED ANSWER:
I would file an action to declare the marriage between Juliet and Ric null and
void ab initio and for Ric's share in the co-ownership of that marriage to be forfeited
in favor and considered part of the absolute community in the marriage between Gigi
and Ric [Arts. 148 Family Code]. I would also file an action for damages against Ric
on the grounds that his acts constitute an abuse of right and they are contrary to law
and morals, causing damages to Gigi (See Arts 19, 20, 21, New Civil Code).

Notes:
Under Article 21 of the Civil Code, any person who willfully causes injury to
another in a manner that is contrary to morals, good customs or public policy shall
compensate the latter.
Marriage; Void Marriages; Psychological Incapacity (2002)
A. Give a brief definition or explanation of the term “psychological
incapacity” as a ground for the declaration of nullity of a marriage. (2%) not a
medical condition but a legal condition (Tan-Andal). Has character of incurabilty,
juridical antecedent, and gravity to the extent the spouse affected is incapacitated to
perform essential marital obligations.

B. If existing at the inception of marriage, would the state of being of


unsound mind or the concealment of drug addiction, habitual alcoholism,
homosexuality or lesbianism be considered indicia of psychological incapacity?
Explain. (2%) depending on the severity, concealment is ground for annulment
SUGGESTED ANSWER:
A.” PSYCHOLOGICAL INCAPACITY” is a mental disorder of the most
serious type showing the incapability of one or both spouses to comply the essential
marital obligations of love, respect, cohabitation, mutual help and support, trust and
commitment. It must be characterized by Juridical antecedence, gravity and
incurability and its root causes must be clinically identified or examined. (Santos v.
CA, 240 SCRA 20 [1995]).

B. In the case of Santos v. Court of Appeals, 240 SCRA 20 (1995), the


Supreme Court held that being of unsound mind, drug addiction, habitual alcoholism,
lesbianism or homosexuality may be indicia of psychological incapacity, depending
on the degree of severity of the disorder. However, the concealment of drug
addiction, habitual alcoholism, lesbianism or homosexuality is a ground of annulment
of marriage.

Notes:
In Santos v. CA, the Supreme Court held that psychological incapacity is a
mental disorder of the most serious type; it shows incapability of one or both spouses
to comply with the essential marital obligations of love, respect, cohabitation, mutual
help and support, trust and commitment. It must be characterized by Juridical
antecedence, gravity and incurability; its root causes must be clinically identified or
examined.
B. In one case, the Supreme Court held that being of unsound mind, drug
addiction, habitual alcoholism, lesbianism or homosexuality may be indicia of
psychological incapacity, depending on the degree of severity of the disorder.
However, the concealment of drug addiction, habitual alcoholism, lesbianism or
homosexuality is a ground of annulment of marriage. In addition, if drug addiction,
habitual alcoholism prevents sexual intimacy, a petition for nullity of marriage may be
file because it is the sexual intimacy that makes oneness and wholeness of the
spouses.

PARENTAL AND SPECIAL PARENTAL AUTHORITY

Parental Authority; Child under 7 years of age (2006)


Under Article 213 of the Family Code, no child under 7 years of age shall be
separated from the mother unless the court finds compelling reasons to order
otherwise.
(1) Explain the rationale of this provision. (2.5%)

SUGGESTED ANSWER:
The rationale of the 2nd paragraph of Article 213 of the Family Code is to avoid
the tragedy of a mother who sees her baby torn away from her. It is said that the
maternal affection and care during the early years of the child are generally needed by
the child more than paternal care (Hontiveros v. IAC, G.R. No. 64982, October 23,
1984; Tolentino, Commentaries and Jurisprudence on the Civil Code, Volume
One, pp. 718-719). The general rule is that a child below 7 years old shall not be
separated from his mother due to his basic need for her loving care (Espiritu v. C.A.,
G.R. No. 115640, March 15,1995).

Notes:
In Hontiveros v. IAC, the Supreme Court held that the reason why a child below
7 years old shall not be separated from his mother is that maternal care during the
early years of the child is generally needed by the child more than paternal care.

(2) Give at least 3 examples of "compelling reasons" which justify the


taking away from the mother's custody of her child under 7 years of age. (2.5%)

SUGGESTED ANSWER:
a. The mother is insane (Sempio-Diy, Handbook on the Family Code of the
Philippines, pp. 296-297);
The mother is sick with a disease that is communicable and might endanger the
health and life of the child;
The mother has been maltreating the child;
The mother is engaged in prostitution;
The mother is engaged in adulterous relationship;
The mother is a drug addict;
The mother is a habitual drunk or an alcoholic;

Notes:
The following are the examples of compelling reasons which justify the taking
away from the mother's custody of her child under 7 years of age:
a. The mother is insane;
b. The mother is sick with a disease that is communicable and might endanger
the health and life of the child;
c. The mother is maltreating the child;
d. The mother is engaged in prostitution;
e. The mother is engaged in adulterous relationship;
f. The mother is a drug addict;
g. The mother is an alcoholic.

Parental Authority; Special Parental Authority; Liability of Teachers (2003)


If during class hours, while the teacher was chatting with other teachers in the
school corridor, a 7 year old male pupil stabs the eye of another boy with a ball pen
during a fight, causing permanent blindness to the victim, who could be liable for
damages for the boy’s injury: the teacher, the school authorities, or the guilty
boy’s parents? Explain.

SUGGESTED ANSWER:
The school, its administrators, and teachers have special parental authority and
responsibility over the minor child while under their supervision, instruction or
custody (Article 218, FC). They are principally and solidarily liable for the damages
caused by the acts or omissions of the unemancipated minor unless they exercised the
proper diligence required under the circumstances (Article 219, FC). In the problem,
the TEACHER and the SCHOOL AUTHORITIES are liable for the blindness of the
victim, because the student who cause it was under their special parental authority and
they were negligent. They were negligent because they were chatting in the corridor
during the class period when the stabbing incident occurred. The incident could have
been prevented had the teacher been inside the classroom at that time. The guilty
boy’s PARENTS are subsidiarily liable under Article 219 of the Family Code.

Notes:
Under Article 129 of the Family Code, the school, its administrator and teachers
have the authority and responsibility over a minor while under their custody and they
shall be held principally and solidarily liable for damages caused by the acts or
omission of such minor.

Art. 218. The school, its administrators and teachers, or the individual, entity or
institution engaged in child are shall have special parental authority and responsibility
over the minor child while under their supervision, instruction or custody.

Authority and responsibility shall apply to all authorized activities whether


inside or outside the premises of the school, entity or institution. (349a)

Art. 129. Those given the authority and responsibility under the preceding
Article shall be principally and solidarily liable for damages caused by the acts or
omissions of the unemancipated minor. The parents, judicial guardians or the persons
exercising substitute parental authority over said minor shall be subsidiarily liable.

The respective liabilities of those referred to in the preceding paragraph shall not
apply if it is proved that they exercised the proper diligence required under the
particular circumstances.

All other cases not covered by this and the preceding articles shall be governed
by the provisions of the Civil Code on quasi-delicts.

Parental Authority; Substitute vs. Special (2004)


Distinguish briefly but clearly between: Substitute parental authority and
special parental authority.

SUGGESTED ANSWER:
In substitute parental authority, the parents lose their parental authority in favor
of the substitute who acquires it to the exclusion of the parents.

In special parental authority, the parents or anyone exercising parental authority


does not lose parental authority. Those who are charged with special parental
authority exercise such authority only during the time that the child is in their custody
or supervision.
Substitute parental authority displaces parental authority while special parental
authority concurs with parental authority.
Notes:

Under the substitute parental authority, the parents of the minor lose parental
authority in favour of the substitute, while in special parental authority, the
authority of the parental authority of the parents concurs with the one who has special
parental authority.

In special parental authority, the teachers, among others, have the authority over
a minor while the latter is under the custody of the former, while in substitute parental
authority, the substitute has the authority over the minor in place of the parent’s
authority.

Paternity & Filiation (1999)


(a) Two (2) months after the death of her husband who was shot by unknown
criminal elements on his way home from office, Rose married her childhood
boyfriend, and seven (7) months after said marriage, she delivered a baby. In the
absence of any evidence from Rose as to who is her child's father, what status does the
law give to said child? Explain. (2%)

SUGGESTED ANSWER:
(a) The child is legitimate of the second marriage under Article 168(2) of the
Family Code which provides that a "child born after one hundred eighty days
following the celebration of the subsequent marriage is considered to have been
conceived during such marriage, even though it be born within three hundred days
after the termination of the former marriage."

Notes:

Under the Family Code, when the marriage is terminated and the mother
contracted a subsequent marriage within 300 days, the following rules shall govern:

1) A child is considered to have been conceived during the former marriage if he


was born before 180 days has elapsed after the solemnization of the subsequent
marriage.
2) A child is considered to have been conceived during the subsequent marriage
if he was born after 180 days from the celebration of the subsequent marriage.

Art. 168. If the marriage is terminated and the mother contracted another
marriage within three hundred days after such termination of the former marriage,
these rules shall govern in the absence of proof to the contrary:

(1) If after the solemnization of the subsequent marriage, a child is born within
180 days from solemnization, that child is considered as conceived during the
former marriage, provided it is born within three hundred days after the termination
of the former marriage;

(2) A child who was born after one hundred eighty days after the celebration of
the subsequent marriage is considered to have been conceived during the subsequent
marriage, even though it was born within the three hundred days after the termination
of the former marriage. (Reengineered)

Art. 168. If the marriage is terminated and the mother contracted another
marriage within three hundred days after such termination of the former marriage,
these rules shall govern in the absence of proof to the contrary:

(1) A child born before one hundred eighty days after the solemnization of the
subsequent marriage is considered to have been conceived during the former marriage,
provided it be born within three hundred days after the termination of the former
marriage;

(2) A child born after one hundred eighty days following the celebration of the
subsequent marriage is considered to have been conceived during such marriage, even
though it be born within the three hundred days after the termination of the former
marriage.

Paternity & Filiation; Proofs (1999)


(b) Nestor is the illegitimate son of Dr. Perez. When Dr. Perez died, Nestor
intervened in the settlement of his father's estate, claiming that he is the illegitimate
son of said deceased, but the legitimate family of Dr. Perez is denying Nestor's claim.
What evidence or evidences should Nestor present so that he may receive his
rightful share in his father's estate? (3%) birth certificate
SUGGESTED ANSWER:
(b) To be able to inherit, the illegitimate filiation of Nestor must have been
admitted by his father in any of the following:
(1) the record of birth appearing in the civil register,
(2) a final judgment,
(3) a public document signed by the father, or
(4) a private handwritten document signed by the father (Article 17S in
relation to Article 172 of the Family Code).

Art. 172. The filiation of legitimate children is established by any of the


following:

(1) The record of birth appearing in the civil register or a final judgment; or

(2) An admission of legitimate filiation in a public document or a private


handwritten instrument and signed by the parent concerned.

In the absence of the foregoing evidence, the legitimate filiation shall be proved
by:

(1) That the child is in open and continuous possession of the status of a
legitimate child; or

(2) Any other means allowed by the Rules of Court and special laws.

Notes:
The evidences to prove filiation of legitimate and illegitimate children are the
same.
Para 2 is secondary evidence.

Paternity & Filiation; Artificial Insemination; Formalities (2006)


Ed and Beth have been married for 20 years without children. Desirous to have
a baby, they consulted Dr. Jun Canlas, a prominent medical specialist on human
fertility. He advised Beth to undergo artificial insemination. It was found that Ed’s
sperm count was inadequate to induce pregnancy. Hence, the couple looked for a
willing donor. Andy the brother of Ed, readily consented to donate his sperm. After a
series of test, Andy's sperm was medically introduced into Beth's ovary. She became
pregnant and 9 months later, gave birth to a baby boy, named Alvin.

(1) Who is the Father of Alvin? Explain. (2.5%)


SUGGESTED ANSWER:
Andy is the biological father of Alvin being the source of the sperm. Andy is the
legal father of Alvin because there was neither consent nor ratification to the
artificial insemination. Under the law, children conceived by artificial insemination
are legitimate children of the spouses, provided, that both of them authorized or
ratified the insemination in a written instrument executed and signed by both of them
before the birth of the child (Art. 164, Family Code).

Notes:
Art. 164. Children conceived or born during the marriage of the parents are
legitimate.

Children who was conceived by artificial insemination of the wife with the
sperm of the husband or that of a donor or both are likewise legitimate children of
the husband and his wife, provided, that the husband and the wife authorized or
ratified such insemination in a written instrument executed and signed by them
before the birth of the child. The instrument shall be recorded in the civil registry
together with the birth certificate of the child. (Reengineered)

(2) What are the requirements, if any, in order for Ed to establish his
paternity over Alvin. (2.5%)

SUGGESTED ANSWER:
The following are the requirements for Ed to establish his paternity over Alvin:
The artificial insemination has been authorized or ratified by the spouses in a written
instrument executed and signed by them before the birth of the child; and The written
instrument is recorded in the civil registry together with the birth certificate of the
child (Art. 164, 2nd paragraph, Family Code).

Paternity & Filiation; Common-Law Union (2004)


A. RN and DM, without any impediment to marry each other, had been living
together without benefit of church blessings. Their common-law union resulted in the
birth of ZMN. Two years later, they got married in a civil ceremony. Could ZMN be
legitimated? Reason. (5%)

SUGGESTED ANSWER:
ZMN was legitimated by the subsequent marriage of RN and DM because at
the time he was conceived, RN and DM could have validly married each other. Under
the Family Code children conceived and born outside of wedlock of parents who, at
the time of the former's conception, were not disqualified by any impediment to marry
each other are legitimated by the subsequent marriage of the parents.
Notes:

Art. 177. Children conceived and born outside of wedlock of parents may be
legitimated if both of them were not disqualified by any impediment to marry each
other at the time of the conception of the children.

Art. 178. Legitimation shall take place by a subsequent valid marriage between
parents. The annulment of a voidable marriage shall not affect the legitimation.

Paternity & Filiation; Proofs; Limitations; Adopted Child (1995)


Abraham died intestate on 7 January 1994 survived by his son Braulio.
Abraham's older son Carlos died on 14 February 1990. Danilo who claims to be an
adulterous child of Carlos intervenes in the proceedings for the settlement of the
estate of Abraham in representation of Carlos. Danilo was legally adopted on 17
March 1970 by Carlos with the consent of the " latter's wife.

1. Under the Family Code, how may an illegitimate filiation be proved?


Explain. same with legitimate filiation; birth record, judgment, public/private
document of admission of paternity
2. As lawyer for Danilo, do you have to prove Danilo's illegitimate filiation?
Explain.
3. Can Danilo inherit from Abraham in representation of his father Carlos?
Explain.
SUGGESTED ANSWER:
1. Under Art. 172 in relation to Art. 173 and Art. 175 of the FC, the filiation of
illegitimate children may be established in the same way and by the same evidence as
legitimate children. Art. 172 provides that the filiation of legitimate children is
established by any of the following: (1) the record of birth appearing in the civil
register or a final Judgment; or (2) an admission of legitimate filiation in a public
document or a private handwritten instrument and signed by the parent concerned. In
the absence of the foregoing evidence, the legitimate filiation shall be proved by: (1)
the open and continuous possession of the status of a legitimate child; or (2) any
other means allowed by the Rules of Court and special laws.

Notes:
1. The filiation of illegitimate children may be established in the same way the
legitimate filiation is established. Filiation of legitimate children is established by any
of the following:
(1) the record of birth appearing in the civil register; or
(2) a final Judgment; or
(3) an admission of legitimate filiation in a public document; or
(4) a private handwritten instrument and signed by the parent.

In the absence of the foregoing evidence, the legitimate filiation shall be proved
by:
(1) the open and continuous possession of the status of a legitimate child; or
(2) any other means allowed by the Rules of Court and special laws.

SUGGESTED ANSWER:
2. No. Since Danilo has already been adopted by Carlos, he ceased to be an
illegitimate child. An adopted child acquires all the rights of a legitimate child under
Art, 189 of the FC.

SUGGESTED ANSWER:
3. No, he cannot. Danilo cannot represent Carlos as the latter's adopted child in
the inheritance of Abraham because adoption did not make Danilo a legitimate
grandchild of Abraham. Adoption is personal between Carlos and Danilo. He cannot
also represent Carlos as the latter's illegitimate child because in such case he is barred
by Art. 992 of the NCC from inheriting from his illegitimate grandfather Abraham.

Under the new law on adoption, an adopted is given an extended legitimate


filiation with the father of the adopter. Hence, adopted may now inherit by right of
representation to the adoptive grandfather. However this is only applicable to
legitimate children.

Notes:
The legal fiction is between the adopted and the adopting parent only.
Under Art. 992 of the New Civil Code, an illegitimate child cannot inherit by
intestacy from the legitimate children and relatives of his father or mother.
ARTICLE 992. An illegitimate child has no right to inherit ab intestato from the
legitimate children and relatives of his father or mother; nor shall such children or
relatives inherit in the same manner from the illegitimate child.

ALTERNATIVE ANSWER:
An adopted child's successional rights do not include the right to represent his
deceased adopter in the inheritance of the latter's legitimate parent, in view of Art. 973
which provides that in order that representation may take place, the representative
must himself be capable of succeeding the decedent. Adoption by itself did not render
Danilo an heir of the adopter's legitimate parent. Neither does his being a grandchild
of Abraham render him an heir of the latter because as an illegitimate child of Carlos,
who was a legitimate child of Abraham, Danilo is incapable of succeeding Abraham
under Art. 992 of the Code.

Paternity & Filiation; Recognition of illegitimate Child (2005)


Steve was married to Linda, with whom he had a daughter, Tintin. Steve
fathered a son with Dina, his secretary of 20 years, whom Dina named Joey, born on
September 20, 1981. Joey's birth certificate did not indicate the father's name. Steve
died on August 13, 1993, while Linda died on December 3, 1993, leaving their
legitimate daughter, Tintin, as sole heir. On May 16, 1994, Dina filed a case on behalf
of Joey, praying that the latter be declared an acknowledged illegitimate son of Steve
and that Joey be given his share in Steve's estate, which is now being solely held by
Tintin. Tintin put up the defense that an action for recognition shall only be filed
during the lifetime of the presumed parents and that the exceptions under Article
285 of the Civil Code do not apply to him since the said article has been repealed by
the Family Code. In any case, according to Tintin, Joey's birth certificate does not
show that Steve is his father.

a) Does Joey have a cause of action against Tintin for recognition and
partition? Explain. (2%)

SUGGESTED ANSWER:
No, Joey does not have a cause of action against Tintin for recognition and
partition. Under Article 175 of the Family Code, as a general rule, an action for
compulsory recognition of an illegitimate child can be brought at any time during
the lifetime of the child. However, if the action is based on "open and continuous
possession of the status of an illegitimate child, the same can be filed during the
lifetime of the putative father."

In the present case, the action for compulsory recognition was filed by Joey's
mother, Dina, on May 16,1994, after the death of Steve, the putative father. The action
will prosper if Joey can present his birth certificate that bears the signature of his
putative father. However, the facts clearly state that the birth certificate of Joey did not
indicate the father's name. A birth certificate not signed by the alleged father cannot
be taken as a record of birth to prove recognition of the child, nor can said birth
certificate be taken as a recognition in a public instrument. (Reyes v. Court of
Appeals, G.R. No. 39537, March 19, 1985) Consequently, the action filed by Joey's
mother has already prescribed.

Notes:
Under the Family Code, an action for recognition of an illegitimate child shall be
brought during his or her lifetime if the evidence to be presented is birth certificate,
among others. If the evidence is based on the possession of a status of an illegitimate
child, it shall be brought during the lifetime of the putative father.

Basis Action to impugn Action to claim


legitimacy legitimacy
Art. 166, FC Art. 173, FC
REMEDY Action to impugn Action to claim legitimacy
legitimacy of illegitimacy or compulsory
recognition
REAL PARTY IN GR: Husband GR: Child
INTEREST XPns: Heirs, in cases XPns: Heirs of the child in
where: cases where:
1. Husband died before 1. Child died in the
the expiration of the state of insanity
period of bringing an 2. Child died during
action; minority
2. Husband died after Note: Must be filed within
filing a complaint, 5 years
without having
desisted
3. Child was born after
death of the husband
PRESCRIPTION 1 year—husband resides in GR: during the lifetime of
the same municipality the child (proof: birth
where birth took place record, private hand
2 years- husband does not written or public
reside in the same instrument of admission of
municipality or city paternity by father, judicial
3 years—husband is living decree)
abroad
EXPn: During the lifetime
of putative father (in cases
where the action for
recognition of illegitimate
child by open and
continuous possession of
status

b) Are the defenses set up by Tintin tenable? Explain. (2%)

SUGGESTED ANSWER:
Yes, the defenses of Tintin are tenable. In Tayag v. Court of Appeals (G.R.
No. 95229, June 9,1992), a complaint to compel recognition of an illegitimate child
was brought before effectivity of the Family Code by the mother of a minor child
based on "open and continuous possession of the status of an illegitimate child." The
Supreme Court held that the right of action of the minor child has been vested by the
filing of the complaint in court under the regime of the Civil Code and prior to the
effectivity of the Family Code. The ruling in Tayag v. Court of Appeals finds no
application in the instant case. Although the child was born before the effectivity of
the Family Code, the complaint was filed after its effectivity. Hence, Article 175 of
the Family Code should apply and not Article 285 of the Civil Code.

c) Supposing that Joey died during the pendency of the action, should the
action be dismissed? Explain. (2%)

SUGGESTED ANSWER:
If Joey died during the pendency of the action, the action should still be
dismissed because the right of Joey or his heirs to file the action has already
prescribed. (Art. 175, Family Code)

Notes:
Under the Article 175 of the Family Code, an action to establish filiation shall be
brought during the lifetime of the child if the evidence showing filiation is a birth
certificate signed by the putative father, a judgement or handwritten instrument signed
by the father; but when the evidence presented is the possession of status of an
illegitimate child, and not a birth certificate, among others, it shall be brought during
the lifetime of the putative father. Therefore, the action of Joey has already
prescribed.

Paternity & Filiation; Rights of Legitimate Children (1990)


B and G (college students, both single and not disqualified to marry each other)
had a romantic affair, G was seven months in the family way as of the graduation of
B. Right after graduation B went home to Cebu City. Unknown to G, B had a
commitment to C (his childhood sweetheart) to marry her after getting his college
degree. Two weeks after B marriage in Cebu City, G gave birth to a son E in Metro
Manila. After ten years of married life in Cebu, B became a widower by the sudden
death of C in a plane crash. Out of the union of B and C, two children, X and Y were
born. Unknown to C while on weekend trips to Manila during the last 5 years of their
marriage, B invariably visited G and lived at her residence and as a result of which,
they renewed their relationship. A baby girl F was born to B and G two years before
the death of C. Bringing his family later to Manila, B finally married G. Recently, G
died. What are the rights of B's four children: X and Y of his first marriage; and
E and F, his children with G? Explain your answer.

SUGGESTED ANSWER:
Under the facts stated, X and Y are legitimate children of B and C. E is the
legitimate children of B and G. E is the legitimated child of B&G. F is the illegitimate
child of B and G. As legitimate children of B and C, X and Y have the following
rights:

1) To bear the surnames of the father and the mother, in conformity with the
provisions of the Civil Code on Surnames;
2) To receive support from their parents, their ascendants, and in proper cases,
their brothers and sisters, in conformity with the provisions of the Family Code on
Support; and
3) To be entitled to the legitime and other successional rights granted to them by
the Civil Code. (Article 174, Family Code).

E is the legitimated child of B and G. Under Art. 177 of the Family Code, only
children conceived and born outside of wedlock of parents who, at the time of the
conception of the former, were not disqualified by any impediment to marry each
other may be legitimated. E will have the same rights as X and Y.
F is the illegitimate child of B and G. F has the right to use the surname of G,
her mother, and is entitled to support as well as the legitime consisting of 1/2 of that
of each of X, Y and E. (Article 176, Family Code)

Presumptive Legitime (1999)

What do you understand by "presumptive legitime", in what case or cases must


the parent deliver such legitime to the children, and what are the legal effects in each
case if the parent fails to do so? (5%)

SUGGESTED ANSWER:
PRESUMPTIVE LEGITIME is not defined in the law. Its definition must have
been taken from Act 2710, the Old Divorce Law, which required the delivery to the
legitimate children of "the equivalent of what would have been due to them as their
legal portion if said spouse had died intestate immediately after the dissolution of the
community of property." As used in the Family Code, presumptive legitime is
understood as the equivalent of the legitimate children's legitimes assuming that the
spouses had died immediately after the dissolution of the community of property.

Presumptive legitime is required to be delivered to the common children of the


spouses when the marriage is annulled or declared void ab initio and possibly, when
the conjugal partnership or absolute community is dissolved as in the case of legal
separation. Failure of the parents to deliver the presumptive legitime will make their
subsequent marriage null and void under Article 53 of the Family Code.

Notes:
Under the Old Divorce Law, presumptive legitime is the delivery of the legal
portion of the estate of a deceased who died intestate to the common children.

Presumptive legitime is required to be delivered to the common children when


the marriage is annulled or declared void ab initio and when the conjugal partnership
or community property is dissolved in case of legal separation. For failure to deliver
such legitime, the subsequent marriage of either parent is null and void.

Property Relations; Absolute Community (1994)


Paulita left the conjugal home because of the excessive drinking of her husband,
Alberto. Paulita, out of her own endeavor, was able to buy a parcel of land which she
was able to register under her name with the addendum "widow." She also acquired
stocks in a listed corporation registered in her name. Paulita sold the parcel of land to
Rafael, who first examined the original of the transfer certificate of title.

1) Has Alberto the right to share in the shares of stock acquired by Paulita?
Yes, no decree yet of legal separation
2) Can Alberto recover the land from Rafael? yes

SUGGESTED ANSWER:
1. a) Yes. The Family Code provides that all property acquired during the
marriage, whether the acquisition appears to have been made, contracted or
registered in the name of one or both spouses, is presumed to be absolute
community property unless the contrary is proved.

b) Yes. The shares are presumed to be absolute community property having been
acquired during the marriage despite the fact that those shares were registered only in
her name. Alberto's right to claim his share will only arise, however, at dissolution.

c) The presumption is still that the shares of stock are owned in common.
Hence, they will form part of the absolute community or the conjugal partnership
depending on what the property Relations is.
d) Since Paulita acquired the shares of stock by onerous title during the
marriage, these are part of the conjugal or absolute community property, as the case
maybe (depending on whether the marriage was celebrated prior to or after the
effectivity of the Family Code). Her physical separation from her husband did not
dissolve the community of property. Hence, the husband has a right to share in the
shares of stock.

Notes:

Art. 116. All property acquired during the marriage is presumed to be conjugal
even though the acquisition is registered in the name of one or both spouses, unless
the contrary is proved. (Reengineered)

Art. 117. The following are conjugal partnership properties:

(1) Those that are acquired by onerous title during the marriage at the expense
of the common fund, despite that the acquisition is for the partnership, or for only one
of the spouses;
(2) Those obtained from the labor, industry, work or profession of either or
both of the spouses;

(3) The fruits, natural, industrial, or civil, due or received during the
marriage from the common property, as well as the net fruits from the exclusive
property of each spouse;

(4) The share of either spouse in the hidden treasure which the law awards to
the finder or owner of the property where the treasure is found;

(5) Those acquired through occupation such as fishing or hunting;

(6) Livestock existing upon the dissolution of the partnership in excess of the
number of each kind brought to the marriage by either spouse; and

(7) Those which are acquired by chance, such as winnings from gambling or
betting. However, losses therefrom shall be borne exclusively by the loser-spouse.

SUGGESTED ANSWER:
2) a) Under a community of property, whether absolute or relative, the
disposition of property belonging to such community is void if done by just one
spouse without the consent of the other or authority of the proper court. However,
the land was registered in the name of Paulita as "widow". Hence, the buyer has the
right to rely upon what appears in the record of the Register of Deeds and should,
consequently, be protected. Alberto cannot recover the land from Rafael but would
have the right of recourse against his wife

b) The parcel of land is absolute community property having been acquired


during the marriage and through Paulita's industry despite the registration being only
in the name of Paulita. The land being community property, its sale to Rafael without
the consent of Alberto is void. However, since the land is registered in the name of
Paulita as widow, there is nothing in the title which would raise a suspicion for Rafael
to make inquiry. He, therefore, is an innocent purchaser for value from whom the
land may no longer be recovered.

c) No. Rafael is an innocent purchaser in good faith who, upon relying on the
correctness of the certificate of title, acquires rights which are to be protected by the
courts.

Under the established principles of land registration law, the presumption is that
the transferee of registered land is not aware of any defect in the title of the property
he purchased. (See Tojonera v. Court of Appeals, 103 SCRA 467). Moreover, the
person dealing with registered land may safely rely on the correctness of its certificate
of title and the law will in no way oblige him to go behind the certificate to determine
the condition of the property. [Director of Lands v. Abache, et al. 73 Phil. 606). No
strong considerations of public policy have been presented which would lead the
Court to reverse the established and sound doctrine that the buyer in good faith of a
registered parcel of land does not have to look beyond the Torrens Title and search for
any hidden defect or inchoate right which may later invalidate or diminish his right to
what he purchased. (Lopez v. Court of Appeals, 189 SCRA 271)

d) The parcel of land is absolute community property having been acquired


during the marriage and through Paulita's industry despite registration only in the
name of Paulita. The land being community property, its sale to Rafael without the
consent of Alberto is void.

Notes:
In Lopez v. Court of Appeals, it was held that the buyer in good faith of a
registered parcel of land does not have to look beyond the Torrens Title and search for
any hidden defect.

In Director of Lands v. Abache, it was held that a person dealing with registered
land may safely rely on the correctness of its certificate of title and the law will in no
way oblige him to go behind the certificate to determine the condition of the property.

Property Relations; Ante Nuptial Agreement (1995)


Suppose Tirso and Tessie were married on 2 August 1988 without executing
any ante nuptial agreement. One year after their marriage, Tirso while supervising
the clearing of Tessie's inherited land upon the latter's request, accidentally found the
treasure not in the new river bed but on the property of Tessie. To whom shall the
treasure belong? Explain.

SUGGESTED ANSWER:
Since Tirso and Tessie were married before the effectivity of the Family Code,
their property relation is governed by conjugal partnership of gains. Under Art. 54 of
the Civil Code, the share of the hidden treasure which the law awards to the finder or
the proprietor belongs to the conjugal partnership of gains. The one-half share
pertaining to Tessie as owner of the land, and the one-half share pertaining to Tirso as
finder of the treasure, belong to the conjugal partnership of gains.

Notes:

Under the Civil Code, whoever owns the land, building, or other property on
which it is found is the owner of the hidden treasure.

ARTICLE 438. Hidden treasure belongs to the owner of the land, building, or
other property on which it is found.

Nevertheless, when the discovery is made on the property of another, or of the


State or any of its subdivisions, and by chance, one-half thereof shall be allowed to
the finder. If the finder is a trespasser, he shall not be entitled to any share of the
treasure.

If the things found be of interest to science or the arts, the State may acquire
them at their just price, which shall be divided in conformity with the rule stated.
(Notes: Judge Bernardo said when the question asks who owns hidden treasure,
the answer must be the owner of the land. That is enough. But if you want to impress
the examiner, give the exception).

Property Relations; Conjugal Partnership of Gains (1998)


In 1970, Bob and Issa got married without executing a marriage settlement. In
1975, Bob inherited from his father a residential lot upon which, in 1981, he
constructed a two-room bungalow with savings from his own earnings. At that time,
the lot was worth P800.000.00 while the house, when finished cost P600,000.00. In
1989 Bob died, survived only by his wife, Issa and his mother, Sofia. Assuming that
the relative values of both assets remained at the same proportion:

1. State whether Sofia can rightfully claim that the house and lot are not
conjugal but exclusive property of her deceased son. [3%]
2. Will your answer be the same if Bob died before August 3, 1988? [2%]

SUGGESTED ANSWER:
1. Since Bob and Sofia got married In 1970, then the law that governs is the
New Civil Code (Persons), in which case, the property relations that should be applied
as regards the property of the spouses is the system of relative community or conjugal
partnership of gains (Article 119, Civil Code). By conjugal partnership of gains, the
husband and the wife place in a common fund the fruits of their separate property and
the income from their work or Industry (Article 142, Civil Code). In this instance, the
lot inherited by Bob in 1975 is his own separate property, he having acquired the
same by lucrative title (par. 2, Art. 148, Civil Code).

However, the house constructed from his own savings in 1981 during the
subsistence of his marriage with Issa is conjugal property and not exclusive property
in accordance with the principle of "reverse accession" provided for in Art. 158, Civil
Code.

Notes:
Under Article 92 of the Family Code, property acquired by gratuitous title by
either spouse shall be his or her exclusive property. Therefore, the property inherited
by Bob from his father is his exclusive property.
The Family Code took effect on August 31, 1988.

ANOTHER ANSWER:
1. Sofia, being her deceased son's legal heir concurring with his surviving
spouse (Arts. 985, 986 and 997, Civil Code), may rightfully claim that the house and
lot are not conjugal but belong to the hereditary estate of Bob. The value of the land
being more than the cost of the improvement (Art. 120, Family Code).

Notes:
Under Article 120 of the Family Code, in case of improvement of the exclusive
property of either spouse using the fund from conjugal property, the exclusive
property becomes conjugal if the improvement is more than the value of the exclusive
property “reverse accession”. But the owner-spouse shall be reimbursed. If the
improvement is less than the exclusive property, the entire property shall belong to the
owner-spouse but he has to reimburse the conjugal property.

Conjugal Property:
1. Acquired by onerous title during the marriage with conjugal funds
2. Livestock in excess of what was brought to the marriage
3. Acquired by chance such as winning from gambling or betting
4. Obtained from labor, industry, work or profession of either or both spouses
5. Fruits of conjugal property due or received during the marriage and net fruits of
separate propert
6. Share of either spouse in hidden treasure
7. Those acquired through occupation such as hunting or fishing
SUGGESTED ANSWER:
2. Yes, the answer would still be the same. Since Bob and Issa contracted their
marriage way back in 1970, then the property relations that will govern is still the
relative community or conjugal partnership of gains (Article 119, Civil Code). It will
not matter if Bob died before or after August 3. 1988 (sic) (effectivity date of the
Family Code], what matters is the date when the marriage was contracted. As Bob
and Issa contracted their marriage way back in 1970. the property relation that governs
them is still the conjugal partnership of gains. (Art. 158, Civil Code)

ANOTHER ANSWER:
2. If Bob died before August 3, 1988 (sic) which is the date the Family Code
took effect, the answer will not be the same. Art. 158. Civil Code, would then apply.
The land would then be deemed conjugal, along with the house, since conjugal funds
were used in constructing it. The husband's estate would be entitled to a
reimbursement of the value of the land from conjugal partnership funds.

Property Relations; Marriage Settlement; Conjugal Partnership of Gains (2005)


Gabby and Mila got married at Lourdes Church in Quezon City on July 10,
1990. Prior thereto, they executed a marriage settlement whereby they agreed on the
regime of conjugal partnership of gains. The marriage settlement was registered in
the Register of Deeds of Manila, where Mila is a resident. In 1992, they jointly
acquired a residential house and lot, as well as a condominium unit in Makati. In
1995, they decided to change their property relations to the regime of complete
separation of property. Mila consented, as she was then engaged in a lucrative
business. The spouses then signed a private document dissolving their conjugal
partnership and agreeing on a complete separation of property. Thereafter, Gabby
acquired a mansion in Baguio City, and a5-hectare agricultural land in Oriental
Mindoro, which he registered exclusively in his name. In the year 2000, Mila's
business venture failed, and her creditors sued her for P10,000,000.00. After obtaining
a favorable judgment, the creditors sought to execute on the spouses' house and lot
and condominium unit, as well as Gabby's mansion and agricultural land.
a) Discuss the status of the first and the amended marriage settlements.
(2%)

SUGGESTED ANSWER:
The marriage settlement between Gabby and Mila adopting the regime of
conjugal partnership of gains still subsists. It is not dissolved by the mere agreement
of the spouses during the marriage. It is clear from Article 134 of the Family Code
that in the absence of an express declaration in the marriage settlement, the separation
of property between the spouses during the marriage shall not take place except by
judicial order.

Notes:
Under Article 134 of the Family Code, judicial order is necessary to change a
regime of property that will govern the regime of property of the spouses.

Art. 134. In the absence of an express declaration in the marriage settlements,


the separation of property between spouses during the marriage shall not take place
except by judicial order. Such judicial separation of property may either be voluntary
or for sufficient cause.

b) Discuss the effects of the said settlements on the properties acquired by the
spouses. (2%)

SUGGESTED ANSWER:
The regime of conjugal partnership of gains governs the properties acquired by
the spouses. All the properties acquired by the spouses after the marriage belong to the
conjugal partnership. Under Article 116 of the Family Code, even if Gabby registered
the mansion and 5-hectare agricultural land exclusively in his name, still they are
presumed to be conjugal properties, unless the contrary is proved.

c) What properties may be held answerable for Mila's obligations? Explain.


(2%)

ALTERNATIVE ANSWER:
Since all the properties are conjugal, they can be held answerable for Mila's
obligation if the obligation redounded to the benefit of the family. (Art. 121 [3],
Family Code) However, the burden of proof lies with the creditor claiming against the
properties. (Ayala Investment v. Court of Appeals, G.R. No. 118305, February
12,1998, reiterated in Homeowners Savings & Loan Bank v. Dailo, G.R. No.
153802, March 11, 2005)

ALTERNATIVE ANSWER:
Except for the residential house which is the family home, all other properties of
Gabby and Mila may be held answerable for Mila's obligation. Since the said
properties are conjugal in nature, they can be held liable for debts and obligations
contracted during the marriage to the extent that the family was benefited or where the
debts were contracted by both spouses, or by one of them, with the consent of the
other.

A family home is a dwelling place of a person and his family. It confers upon a
family the right to enjoy such property, which must remain with the person
constituting it as a family home and his heirs. It cannot be seized by creditors except
in special cases. (Taneo, Jr. v. Court of Appeals, G.R. No. 108532, March 9, 1999)

Notes:
In Taneo, Jr. v. Court of Appeals, it was held that the conjugal partnership of
gains shall be liable for the debt of either spouse if the debt was incurred with the
consent of the other spouse or without the consent, but the debt redounded to the
benefits of the family. The burden of proving that the debt redounded to the
benefit of the family lies on the creditor

Property Relations; Marriage Settlements (1991)


Bar Candidates Patricio Mahigugmaon and Rowena Amor decided to marry
each other before the last day of the 1991 Bar Examinations. They agreed to execute a
Marriage Settlement. Rowena herself prepared the document in her own handwriting.
They agreed on the following: (1) a conjugal partnership of gains; (2) each donates to
the other fifty percent (50%) of his/her present property, (3) Rowena shall administer
the conjugal partnership property; and (4) neither may bring an action for the
annulment or declaration of nullity of their marriage. Both signed the agreement in the
presence of two (2) witnesses. They did not, however, acknowledge it before a
notary public.

A. As to form, is the Marriage Settlement valid? May it be registered in the


registry of property? If not, what steps must be taken to make it registerable?

B. Are the stipulations valid?

C. If the Marriage Settlement is valid as to form and the above stipulations


are likewise valid, does it now follow that said Marriage Settlement is valid and
enforceable? Yes, but at the moment of marriage celebration

SUGGESTED ANSWER:
A. Yes, it is valid as to form because it is in writing. No, it cannot be registered
in the registry of property because it is not a public document. To make it
registerable, it must be reformed and has to be notarized.
SUGGESTED ANSWER:
B. Stipulations (1) and (3) are valid because they are not contrary to law.
Stipulation (4) is void because it is contrary to law. Stipulation (2) is valid up to 1/5
of their respective present properties but void as to the excess (Art 84, Family
Code).

Notes:

(1) An agreement that the regime of the spouses is a conjugal partnership of


gains is valid because it is not contrary to law;

(2) That each donates to the other fifty percent (50%) of his/her present property
is valid only up to 1/5 of their respective property but void as to the rest because it is
contrary to law;

(3) That Rowena shall administer the conjugal partnership property is not valid
because both spouses shall administer the property; and

(4) That neither may bring an action for the annulment or declaration of nullity
of their marriage is not valid because it is contrary to law.

.Art. 84. The future spouses cannot donate to each other in their marriage
settlements more than one-fifth of their present property If the future spouses agree
upon a regime other than the absolute community of property. Any excess shall be
considered void.

Art. 84. If the future spouses agree upon a regime other than the absolute
community of property, they cannot donate to each other in their marriage
settlements more than one-fifth of their present property. Any excess shall be
considered void.

Donations of future property shall be governed by the provisions on


testamentary succession and the formalities of wills.
Marriage settlement is valid if it is in writing. It must be in public document for
it to be registered in the civil registry.

SUGGESTED ANSWER:
C. No. on September 15, 1991, the marriage settlement is not yet valid and
enforceable until the celebration of the marriage, to take place before the last day of
the 1991 bar Examinations.
Notes:
The marriage settlement is valid and enforceable when the marriage has been
celebrated.

Property Relations; Marriage Settlements (1995)


On 10 September 1988 Kevin, a 26-year old businessman, married Karla, a
winsome lass of 18. Without the knowledge of their parents or legal guardians,
Kevin and Karla entered into an ante-nuptial contract the day before their marriage
stipulating that conjugal partnership of gains shall govern their marriage. At the
time of their marriage Kevin's estate was worth 50 Million while Karla's was valued at
2 Million. A month after their marriage Kevin died in a freak helicopter accident. He
left no will, no debts, no obligations. Surviving Kevin, aside from Karla, are his only
relatives: his brother Luis and first cousin Lilia.
1) What property Relations governed the marriage of Kevin and Karla?
Explain.
2) Determine the value of the estate of Kevin,
3) Who are Kevin's heirs?
4) How much is each of Kevin's heirs entitled to inherit?

SUGGESTED ANSWER:
1. Since the marriage settlement was entered into without the consent and
without the participation of the parents (they did not sign the document), the marriage
settlement is invalid applying Art. 78, F.C. which provides that a minor who
according to law may contract marriage may also enter into marriage settlements but
they shall be valid only if the person who may give consent to the marriage are made
parties to the agreement. (Karla was still a minor at the time the marriage settlement
was executed in September 1988 because the law, R.A. 6809, reducing the age of
majority to 18 years took effect on 18 December 1989). The marriage settlement being
void, the property Relations governing the marriage is, therefore, absolute
community of property, under Art. 75 of the FC.

2. All the properties which Kevin and Karla owned at the time of marriage
became community property which shall be divided equally between them at
dissolution. Since Kevin owned 50 Million and Karla. 2 Million, at the time of the
marriage, 52 Million constituted their community property. Upon the death of Kevin,
the community was dissolved and half of the 52 Million or 26 Million is his share in
the community. This 26 Million therefore is his estate.
3. Karla and Luis are the Intestate heirs of Kevin.

4. They are entitled to share the estate equally under Article 1001 of the NCC.
Therefore. Karla gets 13 Million and Luis gets 13 Million.

Notes:
Under Art. 75 of the FC, when the regime of property agreed upon is void, the
system of absolute community of property shall govern. Since Karla was a minor and
her parents did not participate in the marriage settlement, the marriage settlement is
void. Therefore, the regime of absolute community of property shall govern the
property regime of Kevin and Karla. Was there marriage in the first place?????
Under Article 78 of the Family Code, the father or mother, among others, shall
give his consent to the marriage and be made a party to the marriage settlement if one
or both parties to the marriage are minor.

Art. 78. A minor who according to law may contract marriage may also execute
his or her marriage settlements, but they shall be valid only if the father, mother,
surviving parent or guardian, or persons

If the marriage is celebrated without the consent of the parents, among other, the
marriage is voidable.

If the settlement was made without the participation of the parents, the regime is
void and the

having legal charge of them, consent to the marriage and they are made parties
to the agreement.(Reengineered)

ARTICLE 1001. When brothers and sisters or their children survive with the
widow or widower, the latter shall be entitled to one-half of the inheritance and the
brothers and sisters or their children to the other half. (Reengineered)

ARTICLE 1001. Should brothers and sisters or their children survive with the
widow or widower, the latter shall be entitled to one-half of the inheritance and the
brothers and sisters or their children to the other half

Art. 78. A minor who according to law may contract marriage may also execute
his or her marriage settlements, but they shall be valid only if the persons designated
in Article 14 to give consent to the marriage are made parties to the agreement, subject
to the provisions of Title IX of this Code. (120a)
Property Relations; Obligations; Benefit of the Family (2000)
As finance officer of K and Co., Victorino arranged a loan of P5 Million from
PNB for the corporation. However, he was required by the bank to sign a
Continuing Surety Agreement to secure the repayment of the loan. The corporation
failed to pay the loan, and the bank obtained a judgment against it and Victorino,
jointly and severally. To enforce the judgment, the sheriff levied on a farm owned
by the conjugal partnership of Victorino and his wife Elsa. Is the levy proper or
not? (3%)

SUGGESTED ANSWER:
The levy is not proper there being no showing that the surety agreement
executed by the husband redounded to the benefit of the family. An obligation
contracted by the husband alone is chargeable against the conjugal partnership only
when it was contracted for the benefit of the family. When the obligation was
contracted on behalf of the family business the law presumes that such obligation
will redound to the benefit of the family. However, when the obligation was to
guarantee the debt of a third party, as in the problem, the obligation is presumed for
the benefit of the third party, not the family. Hence, for the obligation under the
surety agreement to be chargeable against the partnership, it must be proven that the
family was benefited and that the benefit was a direct result of such agreement,
(Ayala Investment v. Ching, 286 SCRA 272)

Notes:
Under the Family Code, a loan contracted by the husband alone is chargeable
against the conjugal property if the loan redounded to the benefit of the family. In this
case, the husband executed a surety loan which did not redound to the benefit of the
family. Thus, the levy on the conjugal property is not proper.

In Ayala Investment v. Ching, it was held that for the obligation under the surety
agreement to be chargeable against the conjugal partnership, it must be proven that the
family was benefited.

Property Relations; Unions without Marriage (1992)


In 1989, Rico, then a widower forty (40) years of age, cohabited with Cora, a
widow thirty (30) years of age. While living together, they acquired from their
combined earnings a parcel of riceland.

After Rico and Cora separated, Rico lived together with Mabel, a maiden
sixteen (16) years of age. While living together, Rico was a salaried employee and
Mabel kept house for Rico and did full-time household chores for him. During their
cohabitation, a parcel of coconut land was acquired by Rico from his savings.

After living together for one (1) year, Rico and Mabel separated. Rico then met
and married Letty, a single woman twenty-six (26) years of age. During the marriage
of Rico and Letty, Letty bought a mango orchard out of her own personal earnings.

a) Who would own the riceland, and what property Relations governs the
ownership? Explain. Both of them, Art. 147, properties acquired by joint
efforts
b) Who would own the coconut land, and what property Relations governs
the ownership? Explain. Rico, Art. 148, no presumption of joint efforts
c) Who would own the mango orchard, and what property Relations
governs the ownership? Explain. community property, ACP

SUGGESTED ANSWER:
(a) Rico and Cora are the co-owners of the riceland. The Relations is that of co-
ownership (Art. 147, Family Code, first paragraph).
(Optional Addendum: However, after Rico's marriage to Letty, the half interest
of Rico in the riceland will then become absolute community property of Rico and
Letty.)

(b) Rico is the exclusive owner of the coconut land. The Relations is a
sole/single proprietorship (Art. 148. Family Code, first paragraph is applicable, and
not Art. 147 Family Code).
(Optional Addendum: However, after Rico's marriage to Letty, the coconut land
of Rico will then become absolute community property of Rico and Letty.)

(c) Rico and Letty are the co-owners. The Relations is the Absolute Community
of Property (Arts, 75, 90 and 9l, Family Code).

Notes:
Under Article 147 of the Family Code, when a man and a woman who have the
capacity to marry each other live together as husband and wife without the benefit of
marriage, their wages and salaries shall be owned by them in equal share and the
property they acquired shall be governed by the rules on co-ownership.
Under Article 148 of the Family Code, when a man and a woman who have
legal impediment to marry each other live together as husband and wife, only
property which they acquired through their effort or industry shall be owned by them
in common.

Property Relations; Unions without Marriage (1997)


Luis and Rizza, both 26 years of age and single, live exclusively with each other
as husband and wife without the benefit of marriage, Luis is gainfully employed,
Rizza is not employed, stays at home, and takes charge of the household chores. After
living together for a little over twenty years, Luis was able to save from his salary
earnings during that period the amount of P200,000.00 presently deposited in a bank.
A house and lot worth P500,000.00 was recently purchased for the same amount by
the couple. Of the P500.000.00 used by the common-law spouses to purchase the
property, P200.000.00 had come from the sale of palay harvested from the hacienda
owned by Luis and P300,000.00 from the rentals of a building belonging to Rizza. In
fine, the sum of P500.000.00 had been part of the fruits received during the period of
cohabitation from their separate property, a car worth P100.000.00. being used by the
common-law spouses, was donated Just months ago to Rizza by her parents. Luis and
Rizza now decide to terminate their cohabitation, and they ask you to give them your
legal advice on the following:
(a) How, under the law should the bank deposit of P200,000.00 the house
and lot valued at P500.000.00 and the car worth P100.000.00 be allocated to
them? Co-ownership governs, except the car

(b) What would your answer be (to the above question) had Luis and Rizza
been living together all the time, ie., since twenty years ago, under a valid
marriage? same

SUGGESTED ANSWER:
a) Art. 147 of the Family Code provides in part that when a man and a woman
who are capacitated to marry each other, live exclusively with each other as husband
and wife without the benefit of marriage or under a void marriage, their wages and
salaries shall be owned by them in equal shares and the property acquired by both of
them through their work or industry shall be governed by the rules of coownership. In
the absence of proof to the contrary, properties acquired while they lived together
shall be presumed to have been obtained by their Joint efforts, worker Industry, and
shall be owned by them in equal shares. A party who did not participate in the
acquisition by the other party of any property shall be deemed to have contributed
jointly in the acquisition thereof if the former's efforts consisted in the care and
maintenance of the family and of the household. Thus:
1) the wages and salaries of Luis in the amount of P200,000.00 shall be
divided equally between Luis and Rizza.

2) the house and lot valued at P500.000.00 having been acquired by both of
them through work or industry shall be divided between them in proportion to their
respective contribution, in consonance with the rules on co-ownership. Hence, Luis
gets 2\5 while Rizza gets 3\5 of P500.000.00.
3) the car worth P100,000.00 shall be exclusively owned by Rizza, the same
having been donated to her by her parents.

Notes:
Under Article 147 of the Family Code, when a man and woman live together as
husband and wife without the benefit of marriage and there is no legal impediment
thereto, their wages and salaries shall be owned by them in equal shares; the property
acquired by them through their effort and industry shall be governed by the rules on
co-ownership.

Under Article 148 of the Family Code, when a man and a woman live together
as husband and wife without the benefit of marriage and there is legal impediment
thereto, only the properties acquired by them through their actual joint contribution of
money, industry property shall be owned by them in common in proportion to their
respective contributions.

SUGGESTED ANSWER:

(b) The property relations between Luis and Rizza, their marriage having been
celebrated 20 years ago (under the Civil Code) shall be governed by the conjugal
partnership of gains, under which the husband and wife place in a common fund the
proceeds, products, fruits and income from their separate properties and those
acquired by either or both spouses through their efforts or by chance, and upon
dissolution of the marriage or of the partnership, the net gains or benefits obtained by
either or both spouse shall be divided equally between them (Art. 142. Civil Code).
Thus:
1) The salary of Luis deposited in the bank in the amount of P200.000.00 and
the house and lot valued at P500,000.00 shall be divided equally between Luis and
Rizza.
2) However, the car worth P100.000,00 donated to Rizza by her parents shall be
considered to her own paraphernal property, having been acquired by lucrative title
(par. 2, Art. 148, Civil Code).

Property Relations; Unions without Marriage (2000)


For five years since 1989, Tony, a bank Vice-president, and Susan, an
entertainer, lived together as husband and wife without the benefit of marriage
although they were capacitated to many each other. Since Tony's salary was more than
enough for their needs, Susan stopped working and merely "kept house". During that
period, Tony was able to buy a lot and house in a plush subdivision. However, after
five years, Tony and Susan decided to separate.

a) Who will be entitled to the house and lot? (3%) Art. 147, they jointly own
it

SUGGESTED ANSWER:
Tony and Susan are entitled to the house and lot as co-owners in equal shares.
Under Article 147 of the Family Code, when a man and a woman who are capacitated
to marry each other lived exclusively with each other as husband and wife, the
property acquired during their cohabitation are presumed to have been obtained by
their joint efforts, work or industry and shall be owned by them in equal shares. This
is true even though the efforts of one of them consisted merely in his or her care and
maintenance of the family and of the household.

b) Would it make any difference if Tony could not marry Susan because he
was previously married to Alice from whom he is legally separated? (2%)

SUGGESTED ANSWER:
Yes, it would make a difference. Under Article 148 of the Family Code, when
the parties to the cohabitation could not marry each other because of an impediment,
only those properties acquired by both of them through their actual joint contribution
of money, property, or Industry shall be owned by them in common in proportion to
their respective contributions. The efforts of one of the parties in maintaining the
family and household are not considered adequate contribution in the acquisition of
the properties.

Since Susan did not contribute to the acquisition of the house and lot, she has no
share therein. If Tony cohabited with Susan after his legal separation from Alice, the
house and lot is his exclusive property. If he cohabited with Susan before his legal
separation from Alice, the house and lot belongs to his community or partnership
with Alice.

Notes:
Under Article 148 of the Family Code, when a man and a woman live together
as husband and wife without the benefit of marriage and there is legal impediment
thereto, the property acquired by them are owned by them in common if both
contributed to the purchase of the property; if only one of the parties bought the
property, it is his. The other one has no share from that property.

SUCCESSION

Amount of Successional Rights (2004)


Mr. XT and Mrs. YT have been married for 20 years. Suppose the wife, YT,
died childless, survived only by her husband, XT. What would be the share of XT
from her estate as inheritance? Why? Explain. (5%)

SUGGESTED ANSWER:
Under the Civil Code, the widow or widower is a legal and compulsory heir of
the deceased spouse. If the widow is the only surviving heir, there being no legitimate
ascendants, descendants, brothers, and sisters, nephews and nieces, she gets the entire
estate.
Notes:
Under the Civil Code, if the widow or widower is the only surviving heir and
there is no legitimate ascendants, descendants, brothers and sisters, nephews and
nieces, she or he gets the entire estate.

Barrier between illegitimate & legitimate relatives (1993)


A is the acknowledged natural child of B who died when A was already 22
years old. When B's full blood brother, C, died he (C) was survived by his widow and
four children of his other brother D. Claiming that he is entitled to inherit from his
father's brother C, A brought suit to obtain his share in the estate of C. Will his
action prosper?

SUGGESTED ANSWER:
No, the action of A will not prosper. On the premise that B, C and D are
legitimate brothers, as an illegitimate child of B, A cannot inherit in intestacy from C
who is a legitimate brother of B. Only the wife of C in her own right and the
legitimate relatives of C (i.e. the children of D as C's legitimate nephews inheriting as
collateral relatives) can inherit in intestacy. (Arts. 992, 1001, 1OO5 and 975, Civil
Code)

Notes:
Under Article 992 of the Civil Code, an illegitimate child has no right to inherit
in intestacy from the legitimate relatives of his father. In this case, A is an illegitimate
child of B. Therefore, A cannot inherit from C who is the legitimate brother of A. The
barrier between illegitimate and legitimate relatives shall apply.

Aquino case: illegitimate child can now inherit by right of representation to their
grandparents

ARTICLE 992. An illegitimate child has no right to inherit in intestacy from the
legitimate children and relatives of his father or mother; in the same manner, such
children or relatives have no right to inherit in testacy from such an illegitimate child.

ARTICLE 992. An illegitimate child has no right to inherit ab intestato from


the legitimate children and relatives of his father or mother; nor shall such children or
relatives inherit in the same manner from the illegitimate child.
ALTERNATIVE ANSWER:
The action of A will not prosper. Being an illegitimate, he is barred by Article
992 of the Civil Code from inheriting ab intestato from the legitimate relatives of his
father.

Barrier between illegitimate & legitimate relatives (1996)


Cristina the illegitimate daughter of Jose and Maria, died intestate, without
any descendant or ascendant. Her valuable estate is being claimed by Ana, the
legitimate daughter of Jose, and Eduardo, the legitimate son of Maria. Is either,
both, or neither of them entitled to inherit? Explain.

SUGGESTED ANSWER:
Neither Ana nor Eduardo is entitled to inherit of ab intestato from Cristina. Both
are legitimate relatives of Cristina's illegitimate parents and therefore they fall under
the prohibition prescribed by Art. 992, NCC (Manuel v. Ferrer, 242 SCRA 477; Diaz
v. Court of Appeals, 182 SCRA 427).

Notes:
Under Article 992 of the Civil Code, legitimate children cannot inherit from
illegitimate children. In this case, Cristina is an illegitimate child of Jose and Maria.
Therefore, the legitimate daughter of Jose and the legitimate son of Maria cannot
inherit from Cristina.

Collation (1993)
Joaquin Reyes bought from Julio Cruz a residential lot of 300 square meters in
Quezon City for which Joaquin paid Julio the amount of P300,000.00, When the deed
was about to be prepared, Joaquin told Julio that it be drawn in the name of Joaquina
Roxas, his acknowledged natural child. Thus, the deed was so prepared and
executed by Julio. Joaquina then built a house on the lot where she, her husband and
children resided. Upon Joaquin's death, his legitimate children sought to recover
possession and ownership of the lot, claiming that Joaquina Roxas was but a trustee
of their father. Will the action against Joaquina Roxas prosper?

SUGGESTED ANSWER:
Yes, because there is a presumed donation in favor of Joaquina under Art. 1448
of the Civil Code (De los Santos v. Reyes, 27 January 1992, 206 SCRA 437).
However, the donation should be collated to the hereditary estate and the legitime of
the other heirs should be preserved.

Notes:
Under Article 1448 of the Civil Code, when a person paid a property but the title
is conveyed to his child, there is a presumed donation. In this case, Joaquin paid the
property but the title was drawn in the name of Joaquina who is an illegitimate
daughter of Joaquin. Donation is presumed. The property must be collated upon the
death of Joaquin. Therefore, the action against Joaquina will prosper.

ARTICLE 1061. To determine the legitime of each heir for purposes of


partition, every compulsory heir must bring into the mass of the estate any property or
right which he may have received from the decedent during the lifetime of the latter
by way of donation. (Reengineered)
ARTICLE 1448. There is an implied trust when property is sold, and the title
is granted to one party but the price is paid by another. The former is the trustee, while
the latter is the beneficiary.

However, if the person to whom the title is conveyed is a child of the one who
paid the price of the sale, no trust is implied by law; it is presumed that there is a gift
in favor of that child. (Reengineered)

SUGGESTED ANSWER:
Yes, because there is a presumed donation in favor of Joaquina under Art. 1448
of the Civil Code (De los Santos v. Reyes, 27 January 1992, 206 SCRA 437).
However, the donation should be collated to the hereditary estate and the legitime of
the other heirs should be preserved.

ARTICLE 1061. Every compulsory heir, who succeeds with other compulsory
heirs, must bring into the mass of the estate any property or right which he may have
received from the decedent, during the lifetime of the latter, by way of donation, or
any other gratuitous title, in order that it may be computed in the determination of the
legitime of each heir, and in the account of the partition.

ARTICLE 1448. There is an implied trust when property is sold, and the legal
estate is granted to one party but the price is paid by another for the purpose of having
the beneficial interest of the property. The former is the trustee, while the latter is the
beneficiary. However, if the person to whom the title is conveyed is a child, legitimate
or illegitimate, of the one paying the price of the sale, no trust is implied by law, it
being disputably presumed that there is a gift in favor of the child.
ALTERNATIVE ANSWER:
Yes, the action against Joaquina Roxas will prosper, but only to the extent of the
aliquot hereditary rights of the legitimate children as heirs. Joaquina will be entitled to
retain her own share as an illegitimate child, (Arts. 1440 and 1453. Civil Code; Art.
176, F. C.)

Disinheritance vs. Preterition (1993)


Maria, to spite her husband Jorge, whom she suspected was having an affair
with another woman, executed a will, unknown to him, bequeathing all the
properties she inherited from her parents, to her sister Miguela. Upon her death, the
will was presented for probate. Jorge opposed probate of the will on the ground that
the will was executed by his wife without his knowledge, much less consent, and that
it deprived him of his legitime. After all, he had given her no cause for disinheritance,
added Jorge in his opposition.
How will you rule on Jorge's opposition to the probate of Maria's will, if
you were the Judge?

SUGGESTED ANSWER:
As Judge, I shall rule as follows: Jorge's opposition should be sustained in part
and denied in part. Jorge's omission as spouse of Maria is not preterition of a
compulsory heir in the direct line. Hence, Art. 854 of the Civil Code does not apply,
and the institution of Miguela (since she is not a compulsory heir but only voluntary
heir) as heir is valid, but only to the extent of the free portion of one-half. Jorge is still
entitled to one-half of the estate as his legitime. (Art. 1001, Civil Code)

Notes:
Under Article 1001 of the Civil Code, if a surviving spouse survived with
brothers and sisters of the deceased or the children of the brothers and sisters, the
surviving spouse shall get one-half of the inheritance and the other half shall go to the
brothers and sisters or their children. Therefore, Jorge is entitled to one-half of the
estate of Maria.

ALTERNATIVE ANSWERS:
a) As Judge, I shall rule as follows: Jorge's opposition should be sustained in
part and denied in part. This is a case of ineffective disinheritance under Art, 918 of
the Civil Code, because the omission of the compulsory heir Jorge by Maria was
intentional. Consequently, the institution of Miguela as heir is void only insofar as the
legitime of Jorge is prejudiced. Accordingly, Jorge is entitled to his legitime of one-
half of the estate, and Miguela gets the other half.

b) As Judge, I shall rule as follows: Jorge's opposition should be sustained. This


is a case of preterition under Article 854 Civil Code, the result of the omission of
Jorge as compulsory heir having the same right equivalent to a legitimate child "in the
direct line" is that total intestacy will arise, and Jorge will inherit the entire estate.

c) As Judge, I shall rule as follows: the opposition should be denied since it is


predicated upon causes not recognized by law as grounds for disallowance of a wll, to
wit: that the will was made without his knowledge; that the will was made without his
consent; and
3. that it has the effect of depriving him of his legitime, which is a ground that
goes into the intrinsic validity of the will and need not be resolved during the probate
proceedings. However, the opposition may be entertained for, the purpose of securing
to the husband his right to the legitime on the theory that the will constitutes an
ineffective disinheritance under Art. 918 of the Civil Code.

d) As Judge, I shall rule as follows: Jorge is entitled to receive his legitime from
the estate of his wife. He was not disinherited in the will even assuming that he gave
ground for disinheritance, hence, he is still entitled to his legitime. Jorge, however,
cannot receive anything from the free portion. He cannot claim preterition as he is not
a compulsory heir in the direct line. There being no preterition, the institution of the
sister was valid and the only right of Jorge is to claim his legitime.

Notes:
Under Article 918 of the Civil Code, disinheritance which is not one of the
causes set forth in this Code shall annul the institution of heirs to the extent that it may
prejudice the person disinherited; but the devises and legacies and other testamentary
dispositions shall be valid if they do not impair the legitime.

Under the Civil Code, suspicion of having an affair with other woman is not a
ground for disinheriting a spouse. Consequently, there is ineffective disinheritance in
this case. Therefore, the opposition of Jorge must be sustained.

ARTICLE 918. If the cause is not specified, or the purpose is not proved, or the
reason is one which is not mentioned in the Civil Code, disinheritance shall annul the
institution of heirs to the extent that it may prejudice the heir who is disinherited; but
the devises and legacies and other testamentary dispositions shall be valid to an extent
which will not impair the legitime.(Reengineered)

ARTICLE 918. Disinheritance without a specification of the cause, or for a


cause the truth of which, if contradicted, is not proved, or which is not one of those set
forth in this Code, shall annul the institution of heirs insofar as it may prejudice the
person disinherited; but the devises and legacies and other testamentary dispositions
shall be valid to such extent as will not impair the legitime.

ARTICLE 921. The following shall be sufficient causes for disinheriting a


spouse:
(1) When the spouse has been convicted of an attempt against the life of the
testator, his or her descendants, or ascendants;

(2) When the spouse has accused the testator of a crime for which the law
prescribes imprisonment of six years or more, and the accusation has been found to be
false;

(3) When the spouse by fraud, violence, intimidation, or undue influence cause
the testator to make a will or to change one already made;

(4) When the spouse has given cause for legal separation;

(5) When the spouse has given grounds for the loss of parental authority;

(6) Unjustifiable refusal to support the children or the other spouse.


These grounds must be specified in a valid will

Disinheritance; Ineffective (1999)


Mr. Palma, widower, has three daughters D, D-l and D-2. He executes a Will
disinheriting D because she married a man he did not like , and instituting daughters
D-1 and D-2 as his heirs to his entire estate of P 1,000,000.00, Upon Mr, Palma's
death, how should his estate be divided? Explain. (5%)

SUGGESTED ANSWER:
This is a case of ineffective disinheritance because marrying a man that the
father did not approve of is not a ground for disinheriting D. Therefore, the institution
of D-l and D-2 shall be annulled insofar as it prejudices the legitime of D, and the
institution of D-l and D-2 shall only apply on the free portion in the amount of
P500,000.00. Therefore, D, D-l and D-2 will get their legitimes of P500.000.00
divided into three equal parts and D-l and D-2 will get a reduced testamentary
disposition of P250,000.00 each. Hence, the shares will be:
D P166,666.66
D-l P166,666.66 + P250.000.00
D-2 P166,666.66 + P250,000.00

Notes:
The value of the estate of Palma is P1,000,000.00. The P500,000.00 is the free
portion. The legitime of D-1, D-2 and D-3 is, likewise, P500,000.00.
Under Article 918 of the Civil Code, disinheritance which is based on a cause
that is not set forth in the Civil Code shall annul the institution of heirs to the extent
that the person disinherited is prejudiced. Since marrying a man who the father does
not like is not a ground for disinheritance, the institution of the other children of the
testator is void. Therefore, their legitime, which is half of the estate of their father will
be divided equally among the three. But the free portion shall go only to D1 and D2
because D is only entitled to his legitime.

ARTICLE 919. The following shall be sufficient causes for the disinheritance of
children and descendants, legitimate as well as illegitimate:

(1) When a child or descendant has been found guilty of an attempt against the
life of the testator, his or her spouse, descendants, or ascendants;

(2) When a child or descendant has accused the testator of a crime for which the
law prescribes imprisonment for six years or more, if the accusation has been found
groundless;

(3) When a child or descendant has been convicted of adultery or concubinage


with the spouse of the testator;

(4) When a child or descendant by fraud, violence, intimidation, or undue


influence causes the testator to make a will or to change one already made;

(5) A refusal without justifiable cause to support the parent or ascendant who
disinherits such child or descendant;

(6) Maltreatment of the testator by word or deed, by the child or descendant;

(7) When a child or descendant leads a dishonorable or disgraceful life;

(8) Conviction of a crime which carries with it the penalty of civil interdiction.

Disinheritance; Ineffective; Preterition (2000)


In his last will and testament, Lamberto 1) disinherits his daughter Wilma
because "she is disrespectful towards me and raises her voice talking to me ", 2) omits
entirely his spouse Elvira, 3) leaves a legacy of P100,000.00 to his mistress Rosa and
P50,000.00 to his driver Ernie and 4) institutes his son Baldo as his sole heir. How
will you distribute his estate of P1,000,000.00? (5%)

SUGGESTED ANSWER:
The disinheritance of Wilma was ineffective because the ground relied upon by
the testator does not constitute maltreatment under Article 919(6) of the New Civil
Code. Hence, the testamentary provisions in the will shall be annulled but only to the
extent that her legitime was impaired.

The total omission of Elvira does not constitute preterition because she is not a
compulsory heir in the direct line. Only compulsory heirs in the direct line may be the
subject of preterition. Not having been preterited, she will be entitled only to her
legitime.

The legacy in favor of Rosa is void under Article 1028 for being in consideration
of her adulterous relation with the testator. She is, therefore, disqualified to receive the
legacy of 100,000 pesos. The legacy of 50,000 pesos in favor of Ernie is not
inofficious not having exceeded the free portion. Hence, he shall be entitled to receive
it.

The institution of Baldo, which applies only to the free portion, shall be
respected. In sum, the estate of Lamberto will be distributed as follows:
Baldo-----------------450,000 Wilma---------------250,000 Elvira-----------------250,000
Ernie-----------------50,000
1,000,000

Notes:
Disinheritance is ineffective if it is based upon the ground which is not provided
in the Civil Code; it annuls the institution of heirs when the right of the disinherited is
prejudiced. But the legacies and devisees shall be respected.

The word omission means preterition; spouse cannot be the subject of preterition
because he or she is not a compulsory heir in the direct line.
Under 739 of the Civil Code, donation between persons who are guilty of
adultery or concubinage at the time of the donation shall be void. This prohibition
applies to donation mortis causa.

ARTICLE 1028. The prohibitions mentioned in article 739, concerning


donations inter vivos shall apply to testamentary provisions.

ARTICLE 739. The following donations shall be void:


(1) Those made between persons who were guilty of adultery or concubinage
at the time of the donation;

(2) Those made between persons found guilty of the same criminal offense, in
consideration thereof;

(3) Those made to a public officer or his wife, descendants and ascendants, by
reason of his office.

In the case referred to in No. 1, the action for declaration of nullity may be
brought by the spouse of the donor or donee; and the guilt of the donor and donee
may be proved by preponderance of evidence in the same action.

Notes:
Under Article 739 of the Civil Code, the spouse of the donor or donee may file
an action to nullify the donation.
Only a compulsory heir in the direct line may be omitted from the institution of
heir. If a spouse is omitted, he or she is not preterited because he or she is not a
compulsory heir in the direct line. The only way to deprive a spouse of his share in the
estate of another is to disinherit him or her.

ALTERNATIVE ANSWER:
The disinheritance of Wilma was effective because disrespect of, and raising of
voice to her father constitute maltreatment under Article 919(6) of the New Civil
Code. She is, therefore, not entitled to inherit anything. Her inheritance will go to the
other legal heirs. The total omission of Elvira is not preterition because she is not a
compulsory heir in the direct line. She will receive only her legitime. The legacy in
favor of Rosa is void under Article1028 for being in consideration of her adulterous
relation with the testator. She is, therefore, disqualified to receive the legacy. Ernie
will receive the legacy in his favor because it is not inofficious. The institution of
Baldo, which applies only to the free portion, will be respected. In sum, the estate of
Lamberto shall be distributed as follows:
Heir Legitime Legacy Institution TOTAL Baldo 500,000 200.000 700,000 Elvira
250,000 250,000 Ernie 50,000 50,000 TOTAL 750,000 50,000 200,000 1,000,000

Notes:
Under Article 919 of the Family Code, maltreatment such as raising of voice
upon the testator is, among others, a ground for disinheritance.
ANOTHER ALTERNATIVE ANSWER:
Same answer as the first Alternative Answer except as to distribution. Justice
Jurado solved this problem differently. In his opinion, the legitime of the heir who
was disinherited is distributed among the other compulsory heirs in proportion to their
respective legitimes, while his share in the intestate portion. If any, is distributed
among the other legal heirs by accretion under Article 1018 of the NCC in proportion
to their respective intestate shares. In sum the distribution shall be as follows:
Heir Legitime Distribution
of Legacy Institution TOTAL
Wilma’s
Legitime
Baldo 250,0000 125,000 200,000 575,000
Wilm (250.000)
a
Elvira 250,000 125.000 375.000
Ernie 50,000 50.000
TOT 50,00
AL 500,000 250,000 0 200,000 1,000,000

Heirs; Intestate Heirs; Reserva Troncal (1995)


Isidro and Irma, Filipinos, both 18 years of age, were passengers of Flight No.
317 of Oriental Airlines. The plane they boarded was of Philippine registry. While en
route from Manila to Greece some passengers hijacked the plane, held the chief pilot
hostage at the cockpit and ordered him to fly instead to Libya. During the hijacking
Isidro suffered a heart attack and was on the verge of death. Since Irma was already
eight months pregnant by Isidro, she pleaded to the hijackers to allow the assistant
pilot to solemnize her marriage with Isidro. Soon after the marriage, Isidro expired. As
the plane landed in Libya Irma gave birth.

However, the baby died a few minutes after complete delivery. Back in the
Philippines Irma Immediately filed a claim for inheritance . The parents of Isidro
opposed her claim contending that the marriage between her and Isidro was void ab
initio on the following grounds: (a) they had not given their consent to the marriage
of their son; (b) there was no marriage license; (c) the solemnizing officer had no
authority to perform the marriage; and, (d) the solemnizing officer did not file an
affidavit of marriage with the proper civil registrar.

2. Does Irma have any successional rights at all? Discuss fully.


SUGGESTED ANSWER:
2. Irma succeeded to the estate of Isidro as his surviving spouse to the estate of
her legitimate child. When Isidro died, he was succeeded by his surviving wife Irma,
and his legitimate unborn child. They divided the estate equally between them, the
child excluding the parents of Isidro. An unborn child is considered born for all
purposes favorable to it provided it is born later. The child was considered born
because, having an intra-uterine life of more than seven months, it lived for a few
minutes after its complete delivery. It was legitimate because it was born within the
valid marriage of the parents. Succession is favorable to it. When the child died, Irma
inherited the share of the child. However, the share of the child in the hands of Irma is
subject to reserva troncal for the benefit of the relatives of the child within the third
degree of consanguinity and who belong to the line of Isidro.
Notes:
Under the Civil Code, the ascendant is obliged to reserve the property, which he
has acquired by gratuitous title from another ascendant, or a brother or sister by
operation of law, for the benefit of relatives within the third degree of the person from
whom the property came from.

ARTICLE 891. The ascendant who inherits from his descendant any property
which the latter may have acquired by gratuitous title from another ascendant, or a
brother or sister, is obliged to reserve such property as he may have acquired by
operation of law for the benefit of relatives who are within the third degree and who
belong to the line from which said property came.

ALTERNATIVE ANSWER:
If the marriage is void. Irma has no successional rights with respect to Isidro but
she would have successional rights with respect to the child.

Heirs; Intestate Heirs; Shares (2003)


Luis was survived by two legitimate children, two illegitimate children, his
parents, and two brothers. He left an estate of P1 million. Luis died intestate. Who
are his intestate heirs, and how much is the share of each in his estate?

SUGGESTED ANSWER:
The intestate heirs are the two (2) legitimate children and the two (2) illegitimate
children. In intestacy the estate of the decedent is divided among the legitimate and
illegitimate children such that the share of each illegitimate child is one -half the
share of each legitimate child.

Their share are : For each legitimate child – P333,333.33 For each illegitimate
child – P166,666.66

(Article 983, New Civil Code; Article 176, Family Code)

Notes:
Under Article 985 of the Civil Code, the legitime of each illegitimate child shall
be equal to one-half of the share of each legitimate child.
ARTICLE 983. If illegitimate children survive with legitimate children, the
shares of the former shall be in the proportions prescribed by article 895.

Intestate Succession (1992)


F had three (3) legitimate children: A, B, and C. B has one (1) legitimate child
X. C has two (2) legitimate children: Y and Z. F and A rode together in a car and
perished together at the same time in a vehicular accident, F and A died, each of them
leaving substantial estates in intestacy.

a) Who are the intestate heirs of F? What are their respective fractional
shares?

b) Who are the intestate heirs of A? What are their respective fractional
shares?

c) If B and C both predeceased F, who are F’s intestate heirs? What are
their respective fractional shares? Do they inherit in their own right or by
representation?

d) If B and C both repudiated their shares in the estate of F who are F's
intestate heirs? What are their respective fractional shares? Do they inherit in
their own right or by representation? Explain your answer,

SUGGESTED ANSWER:
(a) B = ½

(b) B = 1/2 Z = 1/4 by representation of C C= 1/2 Article 982 of the Civil


Code provides that grandchildren inherit by right of representation.
(c) X = 1/2 by representation of B C=l/2 Y = 1/4 by representation of C

(d) X - 1/3 in his own right Y- 1/3 in his own right 2 - 1/3 in his own right
Article 977 of the Civil Code provides that heirs who repudiate their share cannot be
represented.

Notes:

ARTICLE 977. Heirs who repudiate their share may not be represented.

ARTICLE 982. The grandchildren and other descendants shall inherit by right of
representation, and if any one of them should have died, leaving several heirs, the
portion pertaining to him shall be divided among the latter in equal portions. Per
stirpes

Intestate Succession (1997)


"T" died intestate on 1 September 1997.He was survived by M (his mother), W
(his widow), A and B (his legitimate children), C (his grandson, being the
legitimate son of B), D (his other grandson, being the son of E who was a legitimate
son of, and who predeceased, "T"), and F (his grandson, being the son of G, a
legitimate son who repudiated the inheritance from "T"). His distributable net
estate is P120.000.00. How should this amount be shared in intestacy among the
surviving heirs?

SUGGESTED ANSWER:
The legal heirs are A, B, D, and W. C is excluded by B who is still alive. D
inherits in representation of E who predeceased. F is excluded because of the
repudiation of G, the predecessor. M is excluded by the legitimate children of T. The
answer may be premised on two theories: the Theory of Exclusion and the Theory of
Concurrence.

Under the Theory of Exclusion the legitimes of the heirs are accorded them and
the free portion will be given exclusively to the legitimate descendants. Hence under
the Exclusion Theory: A will get P20.000.00. and P 13.333.33 (1/3 of the free portion)
B will get P 20,000.00. and P13. 333.33 (1/3 of the free portion) D will get
P20.000.00. and P13. 333.33 (1/3 of the free portion) W, the widow is limited to the
legitime of P20.000.00 Under the Theory of Concurrence. In addition to their
legitimes, the heirs of A, B, D and W will be given equal shares in the free portions:
A: P20.000.00 plus P10.000.00 (1 /4 of the free portion) B: P20,000.00 plus
P10.000.00 (l/4 of the free portlon) C: P20,000.00 plus P10.000.00 (1/4 of the free
portion) W: P20,000.00 plus P10,000.00 (l/4 of the free portion) Alternative
Answer:
Shares in Intestacy T - decedent Estate: P120.000.00 Survived by: M -
Mother.....................None W - Widow.............................P 30,000.00 A -
Son.................................P 30,000.00 B - Son.........................P30.000.00 C - Grandson
(son of B).............None D - Grandson (son of E who predeceased T)................P
30,000.00 F - Grandson (son of G who repudiated the Inheritance
from"T").......................None

Explanation:
a) The mother (M) cannot inherit from T because under Art. 985 the ascendants
shall inherit in default of legitimate children and descendants of the deceased.
b) The widow's share is P30.000.00 because under Art, 996 it states that if the
widow or widower and legitimate children or descendants are left, the surviving
spouse has in the succession the same share as that of
each of the children, c) C has no share because his father is still alive hence succession
by representation shall not apply (Art. 975).
d) D inherits P30.000 which is the share of his father E who predeceased T by
virtue of Art. 981 on the right of representation.
e) F has no share because his father G repudiated the inheritance. Under Article
977 heirs who repudiate their share may not be represented.

Notes:
Under Article 975 of the Civil Code, when nephews and nieces of the deceased
survive, they shall inherit alone; but when they survive with their uncles and aunts,
they shall inherit by right of representation, representing their parent who may be a
brother or sister of the deceased.

ARTICLE 975. When children of one or more brothers or sisters of the deceased
survive, they shall inherit from the latter by representation, if they survive with their
uncles or aunts. But if they alone survive, they shall inherit in equal portions. Per
stirpes, per capita

ARTICLE 977. Heirs who repudiate their share may not be represented.
ARTICLE 985. In default of legitimate children and descendants of the
deceased, his parents and ascendants shall inherit from him, to the exclusion of
collateral relatives.

ARTICLE 996. If a widow or widower and legitimate children or descendants


are left, the surviving spouse has in the succession the same share as that of each of
the children. Theory of inclusion

Intestate Succession (1998)


Enrique died, leaving a net hereditary estate of P1.2 million. He is survived by
his widow, three legitimate children, two legitimate grandchildren sired by a
legitimate child who predeceased him, and two recognized illegitimate children.
Distribute the estate in intestacy. [5%]

SUGGESTED ANSWER:
Under the theory of Concurrence, the shares are as follows: A (legitimate
child) = P200,000 B (legitimate child) = P200,000 C (legitimate child) = P200,000 D
(legitimate child) = O (predeceased] E (legitimate child of D) = P100,000 - by right
of representation F (legitimate child of D) = P100,000 - by right of representation
G (illegitimate child) = P100,000 - 1/2 share of the legitimate child H (illegitimate
child) = P100,000 - 1/2 share of the legitimate child W (Widow) = P200.000 - same
share as legitimate child

Notes:
When a child predeceased a parent, the children of that child can succeed by
right of representation to the estate of the parent who died intestate.

ANOTHER ANSWER:
Under the theory of Exclusion the free portion (P300,000) is distributed only
among the legitimate children and is given to them in addition to their legitime. All
other Intestate heirs are entitled only to their respective legitimes. The distribution is
as follows:

Legitime Free Portion Total


A [legitimate child) P150.000 + P 75,000 - P225.000 B {legitimate child)
P150.000 + P150.000 - P225.000 C (legitimate child) P150.000 + P 75.000 -
P225.000 D (legitimate child) 0 0 0 E (legitimate child of D) P 75,000 + P35.500 -
P112,500 F (legitimate child of D) P 75.000 + P 37.500 - P112,500 G (illegitimate
child) P 75.000 0 -P 75,500 H (illegitimate child) P 75.000 0 - P 75,500 W
(Widow) P150,000 0 -P150.000

Intestate Succession (1998)


Tessie died survived by her husband Mario, and two nieces, Michelle and
Jorelle, who are the legitimate children of an elder sister who had predeceased her.
The only property she left behind was a house and lot worth two million pesos, which
Tessie and her husband had acquired with the use of Mario's savings from his income
as a doctor. How much of the property or its value, if any, may Michelle and
Jorelle claim as their hereditary shares? [5%]

SUGGESTED ANSWER:
Article 1001 of the Civil Code provides, "Should brothers and sisters or their
children survive with the widow or widower, the latter shall be entitled to one-half of
the inheritance and the brothers and sisters or their children to the other half." Tessie's
gross estate consists of a house and lot acquired during her marriage, making it part of
the community property. Thus, one-half of the said property would have to be set
aside as Mario's conjugal share from the community property. The other half,
amounting to one million pesos, is her conjugal share (net estate), and should be
distributed to her intestate heirs. Applying the above provision of law, Michelle and
Jorelle, Tessie's nieces, are entitled to one-half of her conjugal share worth one million
pesos, or 500,000 pesos, while the other one-half amounting to P500,000 will go to
Mario, Tessie's surviving spouse. Michelle and Jorelle are then entitled to P250,000
pesos each as their hereditary share.

Notes:

ARTICLE 1001. Should brothers and sisters or their children survive with the
widow or widower, the latter shall be entitled to one-half of the inheritance and the
brothers and sisters or their children to the other half.

Intestate Succession (1999)


Mr. and Mrs. Cruz, who are childless, met with a serious motor vehicle
accident with Mr. Cruz at the wheel and Mrs. Cruz seated beside him, resulting in the
instant death of Mr. Cruz. Mrs. Cruz was still alive when help came but she also died
on the way to the hospital. The couple acquired properties worth One Million
(P1,000,000.00) Pesos during their marriage, which are being claimed by the parents
of both spouses in equal shares. Is the claim of both sets of parents valid and why?
(3%)

SUGGESTED ANSWER:
(a) No, the claim of both parents is not valid. When Mr. Cruz died, he was
succeeded by his wife and his parents as his intestate heirs who will share his estate
equally. His estate was 0.5 Million pesos which is his half share in the absolute
community amounting to 1 Million Pesos. His wife, will, therefore, inherit O.25
Million Pesos and his parents will inherit 0.25 Million Pesos.

When Mrs. Cruz died, she was succeeded by her parents as her intestate heirs.
They will inherit all of her estate consisting of her 0.5 Million half share in the
absolute community and her 0.25 Million inheritance from her husband, or a total of
0.750 Million Pesos.

In sum, the parents of Mr. Cruz will inherit 250,000 Pesos while the parents of
Mrs. Cruz will inherit 750,000 Pesos.

Intestate Succession (2000)


Eugenio died without issue, leaving several parcels of land in Bataan. He was
survived by Antonio, his legitimate brother; Martina, the only daughter of his
predeceased sister Mercedes; and five legitimate children of Joaquin, another
predeceased brother. Shortly after Eugenio's death, Antonio also died, leaving three
legitimate children. Subsequently, Martina, the children of Joaquin and the children of
Antonio executed an extrajudicial settlement of the estate of Eugenio, dividing it
among themselves. The succeeding year, a petition to annul the extrajudicial
settlement was filed by Antero, an illegitimate son of Antonio, who claims he is
entitled to share in the estate of Eugenio. The defendants filed a motion to
dismiss on the ground that Antero is barred by Article 992 of the Civil Code
from inheriting from the legitimate brother of his father. How will you resolve
the motion? (5%)

SUGGESTED ANSWER:
The motion to dismiss should not be granted. Article 992 does not apply. Antero
is not claiming any inheritance from Eugenio. He is claiming his share in the
inheritance of his father consisting of his father's share in the inheritance of Eugenio
(Dela Merced v. Dela Merced, Gr No. 126707, 25 February 1999).

Notes:
In the case of Dela Merced v. Dela Merced, the Supreme Court held that Article
992 of the Civil Code does not apply to a situation which an illegitimate child is
claiming the share of his father to the estate of the brother of his father. The barrier
between illegitimate and legitimate relatives is not applicable in this case.

Under Article 992 of the Civil Code, an illegitimate child has no right to inherit
intestate from the legitimate children and relatives of his father or mother; in the same
way, the legitimate children and relatives of his father or mother cannot inherit from
the illegitimate child.

ARTICLE 992. An illegitimate child has no right to inherit ab intestato from the
legitimate children and relatives of his father or mother; nor shall such children or
relatives inherit in the same manner from the illegitimate child.

ALTERNATIVE ANSWER:
It depends. If Antero was not acknowledged by Antonio, the motion to dismiss
should be granted because Antero is not a legal heir of Antonio. If Antero was
acknowledged, the motion should be denied because Article 992 is not applicable.
This is because Antero is claiming his inheritance from his illegitimate father, not
from Eugenio.

Intestate Succession; Reserva Troncal (1999)


Mr. Luna died, leaving an estate of Ten Million (P1 0,000,000.00) Pesos. His
widow gave birth to a child four months after Mr. Luna's death, but the child died five
hours after birth. Two days after the child's death, the widow of Mr. Luna also died
because she had suffered from difficult childbirth. The estate of Mr. Luna is now
being claimed by his parents, and the parents of his widow? (5%)

SUGGESTED ANSWER:
Half of the estate of Mr. Luna will go to the parents of Mrs. Luna as their
inheritance from Mrs. Luna, while the other half will be inherited by the parents of
Mr. Luna as the reservatarios of the reserved property inherited by Mrs. Luna from
her child.

When Mr. Luna died, his heirs were his wife and the unborn child. The unborn
child inherited because the inheritance was favorable to it and it was born alive later
though it lived only for five hours. Mrs. Luna inherited half of the 10 Million estate
while the unborn child inherited the other half. When the child died, it was survived
by its mother, Mrs. Luna. As the only heir, Mrs. Luna inherited, by operation of law,
the estate of the child consisting of its 5 Million inheritance from Mr. Luna. In the
hands of Mrs. Luna, what she inherited from her child was subject to reserva troncal
for the benefit of the relatives of the child within the third degree of consanguinity and
who belong to the family of Mr. Luna, the line where the property came from.

When Mrs. Luna died, she was survived by her parents as her only heirs. Her
parents will inherit her estate consisting of the 5 Million she inherited from Mr. Luna.
The other 5 Million she inherited from her child will be delivered to the parents of Mr.
Luna as beneficiaries of the reserved property.
In sum, 5 Million Pesos of Mr. Luna's estate will go to the parents of Mrs. Luna,
while the other 5 Million Pesos will go to the parents of Mr. Luna as reservatarios.
Notes:
When Mr. Luna died, he is survived by his widow and his son; the 5 million will
go to his son and the other 5 million will go to his widow; when the child died, the 5
million was inherited by the widow of Mr. Luna subject to the rule on reserva troncal.
When the widow of Mr. Luna died, the 5 million shall go to the parents of Mr. Luna
who are within the third degree relative of propositus.

ARTICLE 891. The ascendant who inherits from his descendant any property
which the latter may have acquired by gratuitous title from another ascendant, or a
brother or sister, is obliged to reserve such property as he may have acquired by
operation of law for the benefit of relatives who are within the third degree and who
belong to the line from which said property came.

ALTERNATIVE ANSWER:
If the child had an intra-uterine life of not less than 7 months, it inherited from
the father. In which case, the estate of 10M will be divided equally between the child
and the widow as legal heirs. Upon the death of the child, its share of 5M shall go by
operation of law to the mother, which shall be subject to reserva troncal. Under Art.
891, the reserva is in favor of relatives belonging to the paternal line and who are
within 3 degrees from the child. The parents of Mr, Luna are entitled to the reserved
portion which is 5M as they are 2 degrees related from child. The 5M inherited by
Mrs. Luna from Mr. Luna will be inherited from her by her parents.

However, if the child had intra-uterine life of less than 7 months, half of the
estate of Mr. Luna, or 5M, will be inherited by the widow (Mrs. Luna), while the other
half, or 5M, will be inherited by the parents of Mr. Luna. Upon the death of Mrs.
Luna, her estate of 5M will be inherited by her own parents.
Legitime (1997)
"X", the decedent, was survived by W (his widow), A (his son), B (a
granddaughter, being the daughter of A) and C and D (the two acknowledged
illegitimate children of the decedent). "X" died this year (1997) leaving a net estate of
P180,000.00. All were willing to succeed, except A who repudiated the inheritance
from his father, and they seek your legal advice on how much each can expect to
receive as their respective shares in the distribution of the estate. Give your
answer.

SUGGESTED ANSWER:
The heirs are B, W, C and D. A inherits nothing because of his renunciation. B
inherits a legitime of P90.000.00 as the nearest and only legitimate descendant,
inheriting in his own right not by representation because of A's renunciation. W gets a
legitime equivalent to one-half (1 / 2) that of B amounting to P45.000. C and D each
gets a legitime equivalent to one-half (1/2) that of B amounting to P45.000.00 each.
But since the total exceeds the entire estate, their legitimes would have to be reduced
corresponding to P22.500.00 each (Art. 895. CC). The total of all of these amounts to
P180.000.00.

Notes:
Under Article 895 of the Civil Code, an illegitimate child will get equivalent to
one-half of the share of each legitimate child.

ARTICLE 998. If a widow or widower survives with illegitimate children, such


widow or widower shall be entitled to one-half of the inheritance, and the
illegitimate children or their descendants, whether legitimate or illegitimate, to the
other half.

ALTERNATIVE ANSWER:
INTESTATE SUCCESSION ESTATE: P180,000.00 W- (widow gets 1/2
share) P90.000.00 (Art. 998) A- (son who repudiated his inheritance) None Art. 977)
B - (Granddaughter) None C - (Acknowledged illegitimate child) P45.000.00
(Art.998) D - (Acknowledged illegitimate child) P45,000.00 (Art. 998) The
acknowledged illegitimate child gets 1/2 of the share of each legitimate child.

Notes:
ARTICLE 998. If a widow or widower survives with illegitimate children, such
widow or widower shall be entitled to one-half of the inheritance, and the
illegitimate children or their descendants, whether legitimate or illegitimate, to the
other half.

Legitime; Compulsory Heirs (2003)


Luis was survived by two legitimate children, two illegitimate children, his
parents, and two brothers. He left an estate of P1 million. Who are the compulsory
heirs of Luis, how much is the legitime of each, and how much is the free portion
of his estate, if any?

SUGGESTED ANSWER:
The compulsory heirs are the two legitimate children and the two illegitimate
children. The parents are excluded by the legitimate children, while the brothers are
not compulsory heirs at all. Their respective legitimate are: a) The legitime of the two
(2) legitimate children is one half (1/2) of the estate (P500,000.00) to be divided
between them equally, or P250,000.00 each. b) The legitimate of each illegitimate
child is one-half (1/2) the legitime of each legitimate child or
P125,000.00.

c) Since the total legitime of the compulsory heirs is P750,000.00, the


balance of P250,000.00 is the free portion.

Legitime; Compulsory Heirs vs. Secondary Compulsory Heirs (2005)


Emil, the testator, has three legitimate children, Tom, Henry and Warlito; a
wife named Adette; parents named Pepe and Pilar; an illegitimate child, Ramon;
brother, Mark; and a sister, Nanette. Since his wife Adette is well-off, he wants to
leave to his illegitimate child as much of his estate as he can legally do. His estate has
an aggregate net amount of Pl,200,000.00, and all the above-named relatives are still
living. Emil now comes to you for advice in making a will. How will you distribute
his estate according to his wishes without violating the law on testamentary
succession? (5%)

SUGGESTED ANSWER:
P600,000.00 — legitime to be divided equally between Tom, Henry and Warlito
as the legitimate children. Each will be entitled to P200,000.00. (Art. 888, Civil
Code) P100,000.00 -- share of Ramon the illegitimate child. Equivalent to 1/2 of the
share of each legitimate child. (Art. 176, Family Code) P200,000.00 — Adette the
wife. Her share is equivalent to the share of one legitimate child. (Art. 892, par. 2,
Civil Code) Pepe and Pilar, the parents are only secondary compulsory heirs and
they cannot inherit if the primary compulsory heirs (legitimate children) are alive.
(Art. 887, par. 2, Civil Code)
Brother Mark and sister Nanette are not compulsory heirs since they are not included
in the enumeration under Article 887 of the Civil Code.

The remaining balance of P300,000.00 is the free portion which can be given to
the illegitimate child Ramon as an instituted heir. (Art. 914, Civil Code) If so given by
the decedent, Ramon would receive a total of P400,000.00.

Notes:

Under 887 of the Civil Code, legitimate children and descendants are
compulsory heirs of their legitimate parents and ascendants.

ARTICLE 888. The legitime of legitimate children and descendants consists of


one-half of the hereditary estate of the father and of the mother.

The latter may freely dispose of the remaining half, subject to the rights of
illegitimate children and of the surviving spouse as hereinafter provided.

ARTICLE 892. If only one legitimate child or descendant of the deceased


survives, the widow or widower shall be entitled to one-fourth of the hereditary estate.
In case of a legal separation, the surviving spouse may inherit if it was the deceased
who had given cause for the same.

If there are two or more legitimate children or descendants, the surviving spouse
shall be entitled to a portion equal to the legitime of each of the legitimate children or
descendants.

In both cases, the legitime of the surviving spouse shall be taken from the
portion that can be freely disposed of by the testator.

Preterition (2001)
Because her eldest son Juan had been pestering her for capital to start a
business, Josefa gave him P100,000. Five years later, Josefa died, leaving a last will
and testament in which she instituted only her four younger children as her sole
heirs. At the time of her death, her only property left was P900,000.00 in a bank. Juan
opposed the will on the ground of preterition. How should Josefa's estate be
divided among her heirs? State briefly the reason(s) for your answer. (5%)
SUGGESTED ANSWER:
There was no preterition of the oldest son because the testatrix donated 100,000
pesos to him. This donation is considered an advance on the son's inheritance. There
being no preterition, the institutions in the will shall be respected but the legitime of
the oldest son has to be completed if he received less. After collating the donation of
P100.000 to the remaining property of P900,000, the estate of the testatrix is
P1,000,000. Of this amount, one-half or P500,000, is the legitime of the legitimate
children and it follows that the legitime of one legitimate child is P100,000. The
legitime, therefore, of the oldest son is P100,000. However, since the donation given
him was P100,000, he has already received in full his legitime and he will not receive
anything anymore from the decedent. The remaining P900,000, therefore, shall go to
the four younger children by institution in the will, to be divided equally among them.
Each will receive P225,000.

Notes:
The will cannot be opposed when there is no preterition; it must be respected.

There is no preterition to speak of when an heir has received a donation from the
testator. The donation is, under the law, considered as an advance of his inheritance.

ARTICLE 1061. Every compulsory heir must bring into the mass of the estate
any property or right he received from the decedent during the lifetime of the latter by
way of donation to determine the legitime of each heir. (Reengineered)

Meaning, the donation must be collated.

ALTERNATIVE ANSWER:
Assuming that the donation is valid as to form and substance, Juan cannot
invoke preterition because he actually had received a donation inter vivos from the
testatrix (III Tolentino 188,1992 ed.). He would only have a right to a completion of
his legitime under Art. 906 of the Civil Code. The estate should be divided equally
among the five children who will each receive P225,000.00 because the total
hereditary estate, after collating the donation to Juan (Art. 1061, CC), would be P1
million. In the actual distribution of the net estate, Juan gets nothing while his siblings
will get P225,000.00 each.

Notes:
ARTICLE 906. Any compulsory heir may demand that his legitime be fully
satisfied if the testator has left by any title less than the legitime he is entitled.
ARTICLE 906. Any compulsory heir to whom the testator has left by any title
less than the legitime belonging to him may demand that the same be fully satisfied.

Preterition; Compulsory Heir (1999)


(a) Mr, Cruz, widower, has three legitimate children, A, B and C. He
executed a Will instituting as his heirs to his estate of One Million (P1,000,000.00)
Pesos his two children A and B, and his friend F. Upon his death, how should Mr.
Cruz's estate be divided? Explain. (3%)

(b) In the preceding question, suppose Mr. Cruz instituted his two children
A and B as his heirs in his Will, but gave a legacy of P 100,000.00 to his friend F.
How should the estate of Mr, Cruz be divided upon his death? Explain, (2%)

SUGGESTED ANSWER:
(a) Assuming that the institution of A, B and F were to the entire estate, there
was preterition of C since C is a compulsory heir in the direct line. The preterition
will result in the total annulment of the institution of heirs. Therefore, the institution of
A, B and F will be set aside and Mr. Cuz's estate will be divided, as in intestacy,
equally among A, B and C as follows: A - P333,333.33; B - P333.333.33; and C -
P333,333.33.

(b) On the same assumption as letter (a), there was preterition of C. Therefore,
the institution of A and B is annulled but the legacy of P100.000.00 to F shall be
respected for not being inofficious. Therefore, the remainder of P900.000.00 will be
divided equally among A, B and C.

Notes:
Only the institution of the heir is annulled; the devise and legacy shall be
respected. Under the Civil Code, when a compulsory heir is preterited, the institution
of heir is annulled but the legacy and devise shall be respected. They may be
inofficious but it may be reduced to the extent that the legitime of the compulsory heir
is affected.

(b) On the same assumption as letter (a), there was preterition of C. Therefore,
the institution of A and B is annulled but the legacy of P100.000.00 to F shall be
respected for not being inofficious. Therefore, the remainder of P900.000.00 will be
divided equally among A, B and C.
Proceedings; Intestate Proceedings; Jurisdiction (2004)
In his lifetime, a Pakistani citizen, ADIL, married three times under Pakistani
law. When he died an old widower, he left behind six children, two sisters, three
homes, and an estate worth at least 30 million pesos in the Philippines. He was born
in Lahore but last resided in Cebu City, where he had a mansion and where two of
his youngest children now live and work. Two of his oldest children are farmers in
Sulu, while the two middle-aged children are employees in Zamboanga City. Finding
that the deceased left no will, the youngest son wanted to file intestate proceedings
before the Regional Trial Court of Cebu City. Two other siblings objected, arguing
that it should be in Jolo before a Shari’a court since his lands are in Sulu. But Adil’s
sisters in Pakistan want the proceedings held in Lahore before a Pakistani court.

Which court has jurisdiction and is the proper venue for the intestate
proceedings? The law of which country shall govern succession to his estate?
(5%)

SUGGESTED ANSWER:
In so far as the properties of the decedent located in the Philippines are
concerned, they are governed by Philippine law (Article 16, Civil Code). Under
Philippine law, the proper venue for the settlement of the estate is the domicile of the
decedent at the time of his death. Since the decedent last resided in Cebu City, that is
the proper venue for the intestate settlement of his estate.

However, the successional rights to the estate of ADIL are governed by


Pakistani law, his national law, under Article 16 of the Civil Code.

Notes:
Under Article 16 of the Civil Code, real property and personal property are
subject to the law of the country where they are situated. Therefore, the proper venue
for estate proceeding is in the place where the properties are situated.

Succession; Death; Presumptive Legitime (1991)


a) For purposes of succession, when is death deemed to occur or take place?
b) May succession be conferred by contracts or acts inter vivos? Illustrate.
Yes, provided such acts inter vivos conferring succession are irrevocable,
otherwise it is considered as acts mortis causa
c) Is there any law which allows the delivery to compulsory heirs of their
presumptive legitimes during the lifetime of their parents? If so, in what
instances? Yes, after a judicial declaration of nullity of marriage if in cases of
annulment
SUGGESTED ANSWER:
A. Death as a fact is deemed to occur when it actually takes place. Death is
presumed to take place in the circumstances under Arts. 390-391 of the Civil Code.
The time of death is presumed to be at the expiration of the 10 year period as
prescribed by Article 390 and at the moment of disappearance under Article 391.

B. Under Art. 84 of the Family Code amending Art 130 of the Civil Code,
contractual succession is no longer possible since the law now requires that donations
of future property be governed by the provisions on the testamentary succession and
formalities of wills.

Notes:
1) Under the Civil Code, for purposes of succession, death is deemed to occur
after an absence for 10 years; if he disappeared at the age of seventy-five, he is
deemed dead after an absence 5 years.

In case of a person who is a member of the armed forces and who participated in
a war, he is presumed dead after an absence of 4 years. Further, a person who lost
during a voyage whether by sea or by air is presumed dead after an absence for 4
years. In addition, a person who was in a danger of death is presumed dead after an
absence for 4 years.

2) Under Article 1080 of the Civil Code, if a person makes a partition of his
estate by an act inter vivos, or by will, such partition shall be respected, insofar as it
does not prejudice the legitime of the compulsory heirs. Therefore, succession may be
conferred by act inter vivos.

3) Under Article 52 of the Civil Code, the judgment of annulment or of


absolute nullity of the marriage, the partition and distribution of the properties of the
spouses and the delivery of the children’s presumptive legitimes shall be recorded in
the appropriate civil registry and registries of property; otherwise, the same shall not
affect third persons. Therefore, therefore, the legitimes of the compulsory heirs must
be delivered to them during the lifetime of their parents.
ARTICLE 390. After an absence of seven years, it being unknown whether or
not the absentee still lives, he shall be presumed dead for all purposes, except for
those of succession.

The absentee shall not be presumed dead for the purpose of opening his
succession till after an absence of ten years. If he disappeared after the age of
seventy-five years, an absence of five years shall be sufficient in order that his
succession may be opened. (n)

ARTICLE 391. The following shall be presumed dead for all purposes,
including the division of the estate among the heirs:

(1) A person on board a vessel lost during a sea voyage, or an aeroplane which is
missing, who has not been heard of for four years since the loss of the vessel or
aeroplane;

(2) A person in the armed forces who has taken part in war, and has been
missing for four years;

(3) A person who has been in danger of death under other circumstances and his
existence has not been known for four years.

ARTICLE 1080. Should a person make a partition of his estate by an act inter
vivos, or by will, such partition shall be respected, insofar as it does not prejudice the
legitime of the compulsory heirs.

A parent who, in the interest of his or her family, desires to keep any agricultural,
industrial, or manufacturing enterprise intact, may avail himself of the right granted
him in this article, by ordering that the legitime of the other children to whom the
property is not assigned, be paid in cash.

Art. 51. In said partition, the value of the presumptive legitimes of all common
children, computed as of the date of the final judgment of the trial court, shall be
delivered in cash, property or sound securities, unless the parties, by mutual
agreement judicially approved, had already provided for such matters.

The children or their guardian or the trustee of their property may ask for the
enforcement of the judgment.

The delivery of the presumptive legitimes herein prescribed shall in no way


prejudice the ultimate successional rights of the children accruing upon the death of
either of both of the parents; but the value of the properties already received under the
decree of annulment or absolute nullity shall be considered as advances on their
legitime. (n)

Art. 52. The judgment of annulment or of absolute nullity of the marriage, the
partition and distribution of the properties of the spouses and the delivery of the
children’s presumptive legitimes shall be recorded in the appropriate civil registry and
registries of property; otherwise, the same shall not affect third persons.
ALTERNATIVE ANSWER:
B. In the case of Coronado vs.CA(l91 SCRA81), it was ruled that no property
passes under a will without its being probated, but may under Article 1058 of the Civil
Code of 1898, be sustained as a partition by an act inter vivos [Many-Oy vs. CA
144SCRA33).

And in the case of Chavez vs, IAC 1191 SCRA211), it was ruled that while the
law prohibits contracts upon future inheritance, the partition by the parent, as provided
in Art. 1080 is a case expressly authorized by law. A person has two options in
making a partition of his estate: either by an act inter vivos or by will. If the partition
is by will, it is imperative that such partition must be executed in accordance with the
provisions of the law on wills; if by an act inter vivos, such partition may even be oral
or written, and need not be in the form of a will, provided the legitime is not
prejudiced.

"Where several sisters execute deeds of sale over their 1 /6 undivided share of
the paraphernal property of their mother, in favor of another sister, with their mother
not only giving her authority thereto but even signing said deeds, there is a valid
partition inter vivos between the mother and her children which cannot be revoked by
the mother. Said deeds of sale are not contracts entered into with respect to future
inheritance. "It would be unjust for the mother to revoke the sales to a son and to
execute a simulated sale in favor of a daughter who already benefited by the
partition."
SUGGESTED ANSWER:
C. Yes, under Arts. 51 and 52 of the New Family Code. In case of legal
separation, annulment of marriage, declaration of nullity of marriage and the
automatic termination of a subsequent marriage by the reappearance of the absent
spouse, the common or community property of the spouses shall be dissolved and
liquidated. Art. 51. In said partition, the value of the presumptive legitimes of all
common children, computed as of the date of the final judgment of the trial court,
shall be delivered in cash, property or sound securities, unless the parties, by mutual
agreement, judicially approved, had already provided for such matters.

The children of their guardian, or the trustee of their property, may ask for the
enforcement of the judgment. The delivery of the presumptive legitimes herein
prescribed shall in no way prejudice the ultimate successional rights of the children
accruing upon the death of either or both of the parents; but the value of the properties
already received under the decree of annulment or absolute nullity shall be considered
as advances on their legitime.

Art. 52. The judgment of annulment or of absolute nullity of the marriage, the
partition and distribution of the properties of the spouses, and the delivery of the
children's presumptive legitimes shall be recorded in the appropriate civil registry and
registries of property; otherwise, the same shall not affect third persons.

Wills; Codicil; Institution of Heirs; Substitution of Heirs (2002)


By virtue of a Codicil appended to his will, Theodore devised to Divino a tract
of sugar land, with the obligation on the part of Divino or his heirs to deliver to
Betina a specified volume of sugar per harvest during Betina’s lifetime. It is also
stated in the Codicil that in the event the obligation is not fulfilled, Betina should
immediately seize the property from Divino or latter’s heirs and turn it over to
Theodore’s compulsory heirs. Divino failed to fulfill the obligation under the Codicil.
Betina brings suit against Divino for the reversion of the tract of land.

a) Distinguish between modal institution and substation of heirs. (3%)


b) Distinguish between simple and fideicommissary substitution of heirs
(2%)
c) Does Betina have a cause of action against Divino? Explain (5%)

SUGGESTED ANSWER:
A. A MODAL INSTITUTION is the institution of an heir made for a certain
purpose or cause (Arts. 871 and 882, NCC). SUBSTITUTION is the appointment of
another heir so that he may enter into the inheritance in default of the heir originality
instituted. (Art. 857, NCC).

B. In a SIMPLE SUBSTITUTION of heirs, the testator designates one or more


persons to substitute the heirs instituted in case such heir or heirs should die before
him, or should not wish or should be incapacitated to accept the inheritance. In a
FIDEICOMMISSARY SUBSTITUTION, the testator institutes a first heir and
charges him to preserve and transmit the whole or part of the inheritance to a second
heir. In a simple substitution, only one heir inherits. In a fideicommissary substitution,
both the first and second heirs inherit. (Art. 859 and 869, NCC)

C. Betina has a cause of action against Divino. This is a case of a testamentary


disposition subject to a mode and the will itself provides for the consequence if the
mode is not complied with. To enforce the mode, the will itself gives Betina the right
to compel the return of the property to the heirs of Theodore. (Rabadilla v.
Conscoluella, 334 SCRA 522 [2000] GR 113725, 29 June 2000).

Notes:
A) Under Civil Code, a modal institution of an heir is the institution of an heir
made for a certain purpose or cause, while substitution is the appointment of
another heir so that he may enter into the inheritance in default of the heir originally
instituted heir.

B) In a simple substitution of heirs, the testator designates one or more persons


to substitute the heirs originally instituted if the heir or heirs should die before him,
or should not wish or should be incapacitated to accept the inheritance.

On the other hand, in a fideicommissary substitution, the testator institutes a


first heir who is obliged to preserve the inheritance and transmit it to a second heir.

In a simple substitution, only one heir inherits. In a fideicommissary


substitution, both the first and second heirs inherit.

C) Betina has a cause of action against Divino. This is a case of a testamentary


disposition subject to a mode and the will itself provides for the consequence if the
condition is not complied with. To enforce the mode, the will itself gives Betina the
right to compel the return of the property to the heirs of Theodore.

ARTICLE 857. Substitution is the appointment of another heir so that he may


enter into the inheritance in default of the heir originally instituted.

ARTICLE 859. The testator may designate one or more persons to substitute
the instituted heirs when such heirs die before him, or do not wish, or are
incapacitated to accept the inheritance.
ARTICLE 863. A fideicommissary substitution is one where the fiduciary or
first heir is instituted; he is entrusted with the obligation to preserve and to transmit
to a second heir the whole or part of the inheritance.

It shall be valid and shall take effect, provided such substitution does not go
beyond one degree from the heir originally instituted and that the fiduciary or first heir
and the second heir are living at the time of the death of the testator.

ARTICLE 871. The institution of an heir may be made conditionally, or for a


certain purpose or cause. (Modal institution)

(Notes: example of the phrase that the substitution does not go beyond one
degree from the heir originally instituted is that the first heir is the mother and the
second heir is the daughter).

Wills; Formalities (1990)


(1) If a will is executed by a testator who is a Filipino citizen, what law will
govern if the will is executed in the Philippines? What law will govern if the will
is executed in another country? Explain your answers.

(2) If a will is executed by a foreigner, for instance, a Japanese, residing in


the Philippines, what law will govern if the will is executed in the Philippines?
And what law will govern if the will is executed in Japan, or some other country,
for instance, the U.S.A.? Explain your answers.

SUGGESTED ANSWER:
(1) a. If the testator who is a Filipino citizen executes his will in the Philippines,
Philippine law will govern the formalities.
b. If said Filipino testator executes his will in another country, the law of the
country where he maybe or Philippine law will govern the formalities. (Article 815,
Civil Code}

Notes:
ARTICLE 815. When a Filipino is in a foreign country, he is authorized to
make a will in any of the forms established by the law of the country in which he may
be. Such will may be probated in the Philippines.
SUGGESTED ANSWER:

(2) a. If the testator is a foreigner residing in the Philippines and he executes his
will in the Philippines, the law of the country of which he is a citizen or Philippine
law will govern the formalities.

b. If the testator is a foreigner and executes his will in a foreign country, the law
of his place of residence or the law of the country of which he is a citizen or the law of
the place of execution, or Philippine law will govern the formalities (Articles 17. 816.
817. Civil Code).

Notes:

ARTICLE 17. The forms and solemnities of contracts, wills, and other public
instruments shall be governed by the laws of the country where they are executed.

When the acts referred to are executed before the diplomatic or consular
officials of the Republic of the Philippines in a foreign country, the solemnities
established by Philippine laws shall be observed in their execution.

Prohibitive laws concerning persons, their acts or property, and those which
have for their object public order, public policy and good customs shall not be
rendered ineffective by laws or judgments promulgated, or by determinations or
conventions agreed upon in a foreign country.

ARTICLE 816. The will of an alien who is abroad produces effect in the
Philippines if made with the formalities prescribed by the law of the place in which he
resides, or according to the formalities observed in his country, or in conformity with
the Civil Code. (Re-engineered)

ARTICLE 817. A will made in the Philippines by a citizen of another country,


which is executed in accordance with the law of his country, and which might be
proved and allowed by the law of his own country, shall have the same effect as if
executed according to the laws of the Philippines. (Re-engineered)

POSSIBLE ADDITIONAL ANSWERS:

a. In the case of a Filipino citizen, Philippine law shall govern substantive


validity whether he executes his will in the Philippines or in a foreign country.
b. In the case of a foreigner, his national law shall govern substantive validity
whether he executes his will in the Philippines or in a foreign country.
Notes:
The two answers have no basis.

Wills; Holographic Wills; Insertions & Cancellations (1996)

Vanessa died on April 14, 1980, leaving behind a holographic will which is
entirely written, dated and signed in her own handwriting. However, it contains
insertions and cancellations which are not authenticated by her signature. For this
reason, the probate of Vanessa's will was opposed by her relatives who stood to inherit
by her intestacy. May Vanessa's holographic will be probated? Explain.

SUGGESTED ANSWER:
Yes, the will as originally written may be probated. The insertions and
alterations were void since they were not authenticated by the full signature of
Vanessa, under Art. 814, NCC. The original will, however, remains valid because a
holographic will is not invalidated by the unauthenticated insertions or alterations
(Ajero v. CA, 236 SCRA 468].

Notes:

ARTICLE 814. In case of any insertion, cancellation, erasure or alteration in


a holographic will, the testator must authenticate the same by his full signature.

ALTERNATIVE ANSWER:
It depends. As a rule, a holographic will is not adversely affected by Insertions
or cancellations which were not authenticated by the full signature of the testator
(Ajero v. CA, 236 SCRA 468). However, when the insertion or cancellation amounts
to revocation of the will, Art.814 of the NCC does not apply but Art. 830. NCC. Art.
830 of the NCC does not require the testator to authenticate his cancellation for the
effectivity of a revocation effected through such cancellation (Kalaw v. Relova, 132
SCRA 237). In the Kalaw case, the original holographic will designated only one heir
as the only substantial provision which was altered by substituting the original heir
with another heir. Hence, if the unauthenticated cancellation amounted to a revocation
of the will, the will may not be probated because it had already been revoked.

Wills; Holographic Wills; Witnesses (1994)


On his deathbed, Vicente was executing a will. In the room were Carissa,
Carmela, Comelio and Atty. Cimpo, a notary public. Suddenly, there was a street
brawl which caught Comelio's attention, prompting him to look out the window.
Cornelio did not see Vicente sign a will. Is the will valid?

SUGGESTED ANSWERS:
a) Yes, the will is valid. The law does not require a witness to actually see the
testator sign the will. It is sufficient if the witness could have seen the act of signing
had he chosen to do so by casting his eyes to the proper direction.

b) Yes, the will is valid. Applying the "test of position", although Comelio did
not actually see Vicente sign the will, Cornelio was in the proper position to see
Vicente sign if Cornelio so wished.

Notes:
The phrase that the testator must sign the will in the presence of the three
witnesses is satisfied even though one of them is not casting his eyes when the testator
signs the will for as long as he can see him if he wants to.

ARTICLE 805. Every will, other than a holographic will, must be subscribed at
the end thereof by the testator himself or by the testator’s name written by some other
person in his presence, and by his express direction, and attested and subscribed by
three or more credible witnesses in the presence of the testator and of one another.

Wills; Joint Wills (2000)


Manuel, a Filipino, and his American wife Eleanor, executed a Joint Will in
Boston, Massachusetts when they were residing in said city. The law of Massachusetts
allows the execution of joint wills. Shortly thereafter, Eleanor died. Can the said Will
be probated in the Philippines for the settlement of her estate? (3%)

SUGGESTED ANSWER:
Yes, the will may be probated in the Philippines insofar as the estate of Eleanor
is concerned. While the Civil Code prohibits the execution of Joint wills here and
abroad, such prohibition applies only to Filipinos. Hence, the joint will which is valid
where executed is valid in the Philippines but only with respect to Eleanor. Under
Article 819, it is void with respect to Manuel whose joint will remains void in the
Philippines despite being valid where executed.
ALTERNATIVE ANSWER:
The will cannot be probated in the Philippines, even though valid where
executed, because it is prohibited under Article 818 of the Civil Code and declared
void under Article 819, The prohibition should apply even to the American wife
because the Joint will is offensive to public policy. Moreover, it is a single juridical
act which cannot be valid as to one testator and void as to the other.

Notes:
Under the Civil Code, two or more persons cannot make a will jointly in the
same instrument. Therefore, the will executed by the Filipino husband and an
American wife cannot be probated in the Philippines.

(The alternative answer is more accurate; the suggested answer shows that
answering the bar exam in any way is acceptable provided there is a valid and logical
arguments).

ARTICLE 818. Two or more persons cannot make a will jointly, or in the same
instrument, either for their reciprocal benefit or for the benefit of a third person.

ARTICLE 819. Wills, prohibited by the preceding article, executed by Filipinos


in a foreign country shall not be valid in the Philippines, even though authorized by
the laws of the country where they may have been executed.

Wills; Probate; Intrinsic Validity (1990)


H died leaving a last will and testament wherein it is stated that he was legally
married to W by whom he had two legitimate children A and B. H devised to his
said forced heirs the entire estate except the free portion which he gave to X who was
living with him at the time of his death.
In said will he explained that he had been estranged from his wife W for more
than 20 years and he has been living with X as man and wife since his separation from
his legitimate family.
In the probate proceedings, X asked for the issuance of letters testamentary in
accordance with the will wherein she is named sole executor. This was opposed by W
and her children.

(a) Should the will be admitted in said probate proceedings?


(b) Is the said devise to X valid? no
(c) Was it proper for the trial court to consider the intrinsic validity of the
provisions of said will? Explain your answers, yes, jurisprudence permits

SUGGESTED ANSWER:
(a) Yes, the will may be probated if executed according to the formalities
prescribed by law.
(b) The institution giving X the free portion is not valid, because the prohibitions
under Art. 739 of the Civil Code on donations also apply to testamentary dispositions
(Article 1028, Civil Code), Among donations which are considered void are those
made between persons who were guilty of adultery or concubinage at the time of the
donation.

(c) As a general rule, the will should be admitted in probate proceedings if all
the necessary requirements for its extrinsic validity have been met and the court
should not consider the intrinsic validity of the provisions of said will . However, the
exception arises when the will in effect contains only one testamentary disposition. In
effect, the only testamentary disposition under the will is the giving of the free portion
to X, since legitimes are provided by law. Hence, the trial court may consider the
intrinsic validity of the provisions of said will. (Nuguid v. Nuguid, et al.. No. L23445,
June 23, 1966, 17 SCRA; Nepomuceno v. CA, L-62952, Nepomuceno v. CA, L-
62952, 9 October 1985. 139 SCRA 206).

Notes:

(a) Yes, the will may be probated if executed according to the formalities
prescribed by law.

(b) Under the Civil Code, a man and a woman who are guilty of adultery or
concubinage cannot make donation inter vivos and mortis causa between
themselves.

ARTICLE 739. The following donations shall be void:

(1) Those made between persons who were guilty of adultery or concubinage
at the time of the donation;

(2) Those made between persons found guilty of the same criminal offense, in
consideration thereof;
(3) Those made to a public officer or his wife, descendants and ascendants, by
reason of his office.

In the case referred to in No. 1, the action for declaration of nullity may be
brought by the spouse of the donor or donee; and the guilt of the donor and donee may
be proved by preponderance of evidence in the same action.

ARTICLE 1028. The prohibitions mentioned in article 739, concerning


donations inter vivos shall apply to testamentary provisions.

(c) Under the Civil Code, if a will was executed according to law, it can be
probated and the probate court cannot pass upon the intrinsic validity of the will.

Lost or Destroyed Wills; Probate; Notarial and Holographic Wills (1997)


Johnny, with no known living relatives, executed a notarial will giving all his
estate to his sweetheart. One day, he had a serious altercation with his sweetheart. A
few days later, he was introduced to a charming lady who later became a dear friend.
Soon after, he executed a holographic will expressly revoking the notarial will and so
designating his new friend as sole heir. One day when he was clearing up his desk,
Johnny mistakenly burned, along with other papers, the only copy of his holographic
will. His business associate, Eduardo knew well the contents of the will which was
shown to him by Johnny the day it was executed. A few days after the burning
incident, Johnny died. Both wills were sought to be probated in two separate
petitions. Will either or both petitions prosper?

ADDITIONAL ANSWERS:
In the case of Gan vs. Yap (104 Phil 509), the execution and the contents of a
lost or destroyed holographic will may not be proved by the bare testimony of
witnesses who have seen or read such will. The will itself must be presented otherwise
it shall produce no effect. The law regards the document itself as material proof of
authenticity. Moreover, in order that a will may be revoked by a subsequent will, it is
necessary that the latter will be valid and executed with the formalities required for the
making of a will. The latter should possess all the requisites of a valid will whether it
be ordinary or a holographic will, and should be probated in order that the revocatory
clause thereof may produce effect. In the case at bar, since the holographic will itself
cannot be presented, it cannot therefore be probated. Since it cannot be
probated, it cannot revoke the notarial will previously written by the decedent.
(2) On the basis of the Rules of Court, Rule 76, Sec. 6, provides that no will
shall be proved as a lost or destroyed will unless its provisions are clearly and
distinctly proved by at least two (2) credible witnesses. Hence, if we abide strictly by
the two-witness rule to prove a lost or destroyed will, the holographic will which
Johnny allegedly mistakenly burned, cannot be probated, since there is only one
witness, Eduardo, who can be called to testify as to the existence of the will. If the
holographic will, which purportedly, revoked the earlier notarial will cannot be proved
because of the absence of the required witness, then the petition for the probate of the
notarial will should prosper.

Notes:
In Gan vs. Yap, the Supreme Court held that the revocatory clause revoking the
previous will is effective only if the subsequent will can be probated. Therefore, the
notarial will of Johnny can be probated since his holographic will which contained the
revocatory clause cannot be probated because it was unintentionally burned by
Johnny.

Under the Rules of Court, two witnesses are necessary to prove a lost or
destroyed will. Therefore, the holographic will of Johnny cannot be probated since
only one witness, Eduardo who read the contents thereof before it was mistakenly
burned by the testator.

Section 6. Proof of lost or destroyed will. Certificate thereupon. — A lost or


destroyed will may be proved if its execution and validity are established, and it is
proved that it is in existence at the time of the death of the testator, or it is shown that
it is fraudulently or accidentally destroyed in the lifetime of the testator without his
knowledge, or its provisions are clearly and distinctly proved by at least two (2)
credible witnesses.

Wills; Revocation of Wills; Dependent Relative Revocation (2003)


Mr. Reyes executed a will completely valid as to form. A week later, however,
he executed another will which expressly revoked his first will, which he tore his
first will to pieces. Upon the death of Mr. Reyes, his second will was presented for
probate by his heirs, but it was denied probate due to formal defects. Assuming that
a copy of the first will is available, may it now be admitted to probate and given
effect? Why?

SUGGESTED ANSWER:
Yes, the first will may be admitted to probate and given effect. When the testator
tore first will, he was under the mistaken belief that the second will was perfectly
valid and he would not have destroyed the first will had he known that the second
will is not valid. The revocation by destruction therefore is dependent on the validity
of the second will. Since it turned out that the second will was invalid, the tearing of
the first will did not produce the effect of revocation. This is known as the doctrine of
dependent relative revocation (Molo v. Molo, 90 Phil 37.)

Notes:
In Molo v. Molo, the Supreme Court held that the first will is not revoked by the
second will if it turned out that the subsequent will cannot revoke the previous will
because it is invalid due to the noncompliance of the requisites of a valid will. This
is known as the doctrine of dependent relative revocation. Accordingly, the first will
can be probated.

(This concept was discussed by Judge Bernardo)

ALTERNATIVE ANSWERS:
No, the first will cannot be admitted to probate. While it is true that the first will
was successfully revoked by the second will because the second will was later denied
probate, the first will was, nevertheless, revoked when the testator destroyed it after
executing the second invalid will.

Wills; Testamentary Disposition (2006)


Don died after executing a Last Will and Testament leaving his estate valued at
P12 Million to his common-law wife Roshelle. He is survived by his brother Ronie
and his half-sister Michelle.

(1) Was Don's testamentary disposition of his estate in accordance with the
law on succession? Whether you agree or not, explain your answer. Explain.

SUGGESTED ANSWER:
Yes, Don's testamentary disposition of his estate is in accordance with the law
on succession. Don has no compulsory heirs not having ascendants, descendants nor a
spouse [Art. 887, New Civil Code]. Brothers and sisters are not compulsory heirs.
Thus, he can bequeath his entire estate to anyone who is not otherwise incapacitated
to inherit from him. A common-law wife is not incapacitated under the law, as Don is
not married to anyone.
Notes:
Under Art. 887, New Civil Code, a testator may bequeath his entire estate to
anyone he wishes provided the latter is not disqualified to inherit from the former
and the testator has no compulsory heir.

(2) If Don failed to execute a will during his lifetime, as his lawyer, how will
you distribute his estate? Explain. (2.5%)

SUGGESTED ANSWER:
After paying the legal obligations of the estate, I will give Ronie, as full-blood
brother of Don, 2/3 of the net estate, twice the share of Michelle, the half-sister who
shall receive 1/3. Roshelle will not receive anything as she is not a legal heir [Art.
1006 New Civil Code].

Notes:
Under the Civil Code, brothers of the full blood will get twice the share of the
brothers of the half blood if they survive together.

ARTICLE 1006. Should brothers and sisters of the full blood survive together
with brothers and sisters of the half blood, the former shall be entitled to a share
double that of the latter.

(3) Assuming he died intestate survived by his brother Ronie, his half-sister
Michelle, and his legitimate son Jayson, how will you distribute his estate?
Explain. (2.5%)
SUGGESTED ANSWER:
SUGGESTED ANSWER:
Jayson will be entitled to the entire P12 Million as the brother and sister will be
excluded by a legitimate son of the decedent. This follows the principle of proximity,
where "the nearer excludes the farther."

(4) Assuming further he died intestate, survived by his father Juan, his
brother Ronie, his half-sister Michelle, and his legitimate son Jayson, how will
you distribute his estate? Explain. (2.5%)

SUGGESTED ANSWER:
Jayson will still be entitled to the entire P12 Million as the father, brother and
sister will be excluded by a legitimate son of the decedent [Art. 887, New Civil Code].
This follows the principle that the descendants exclude the ascendants from
inheritance.

Notes:

ARTICLE 887. The following are compulsory heirs:

(1) Legitimate children and descendants, with respect to their legitimate parents
and ascendants;

(2) In default of the foregoing, legitimate parents and ascendants, with respect to
their legitimate children and descendants;

(3) The widow or widower;

(4) Acknowledged natural children, and natural children by legal fiction;

(5) Other illegitimate children referred to in article 287.

Compulsory heirs mentioned in Nos. 3, 4 and 5 are not excluded by those in


Nos. 1 and 2; neither do they exclude one another.

In all cases of illegitimate children, their filiation must be duly proved.

The father or mother of illegitimate children of the three classes mentioned, shall
inherit from them in the manner and to the extent established by this Code. (807a)

Wills; Testamentary Intent (1996)


Alfonso, a bachelor without any descendant or ascendant, wrote a last will and
testament in which he devised." all the properties of which I may be possessed at the
time of my death" to his favorite brother Manuel. At the time he wrote the will, he
owned only one parcel of land. But by the time he died, he owned twenty parcels of
land. His other brothers and sisters insist that his will should pass only the parcel of
land he owned at the time it was written, and did not cover his properties acquired,
which should be by intestate succession. Manuel claims otherwise. Who is correct?
Explain.

SUGGESTED ANSWER:
Manuel is correct because under Art. 793, NCC, property acquired after the
making of a will shall only pass thereby, as if the testator had possessed it at the time
of making the will, should it expressly appear by the will that such was his intention.
Since Alfonso's intention to devise all properties he owned at the time of his death
expressly appears on the will, then all the 20 parcels of land are included in the devise.

Notes:
Under the Civil Code, property acquired after the making of a will shall be
included therein if it is expressly appear in the will that it is the intention of the
testator. Therefore, the property acquired by the testator after the making of the will
shall be given to his brother, Manuel because that was his intention since he used the
word all. Moreover, the 20 parcels of land shall be given to Manuel.

ARTICLE 793. Property acquired after the making of a will shall only pass
thereby, as if the testator had possessed it at the time of making the will, should it
expressly appear by the will that such was his intention.

DONATION

ARTICLE 725. Donation is an act of liberality whereby a person disposes


gratuitously of a thing or right in favor of another, who accepts it.

Donation vs. Sale (2003)


a) May a person sell something that does not belong to him? Explain.
b) May a person donate something that does not belong to him? Explain.
5%

SUGGESTED ANSWER:
(a) Yes, a person may sell something which does not belong to him. For the sale
to be valid, the law does not require the seller to be the owner of the property at the
time of the sale. (Article 1434, NCC). If the seller cannot transfer ownership over the
thing sold at the time of delivery because he was not the owner thereof, he shall be
liable for breach of contact.

Notes:
Under the Civil Code, when a person who is not the owner of a thing sells or
alienates and delivers it, such title passes by operation of law to the buyer or grantee.
Therefore, a person may sell something which does not belong to him.
Under the Civil Code, donation does not comprehend future property. Therefore,
a person cannot donate something which does not belong to him.

(b) As a general rule, a person cannot donate something which he cannot dispose
of at the time of the donation (Article 751, New Civil Code).

ARTICLE 751. Donations cannot comprehend future property.

By future property is understood anything which the donor cannot dispose of at


the time of the donation.

ARTICLE 1434. When a person who is not the owner of a thing sells or
alienates and delivers it, and later the seller or grantor acquires title thereto, such title
passes by operation of law to the buyer or grantee.

Donations; Condition; Capacity to Sue (1996)


Sometime in 1955, Tomas donated a parcel of land to his stepdaughter Irene,
subject to the condition that she may not sell, transfer or cede the same for twenty
years. Shortly thereafter, he died. In 1965, because she needed money for medical
expenses, Irene sold the land to Conrado. The following year, Irene died, leaving as
her sole heir a son by the name of Armando. When Armando learned that the land
which he expected to inherit had been sold by Irene to Conrado, he filed an action
against the latter for annulment of the sale, on the ground that it violated the restriction
imposed by Tomas. Conrado filed a motion to dismiss, on the ground that
Armando did not have the legal capacity to sue. If you were the Judge, how will
you rule on this motion to dismiss? Explain.

SUGGESTED ANSWER:
As judge, I will grant the motion to dismiss. Armando has no personality to
bring the action for annulment of the sale to Conrado. Only an aggrieved party to the
contract may bring the action for annulment thereof (Art. 1397. NCC). While
Armando is heir and successor-in-interest of his mother (Art. 1311, NCC), he
[standing in place of his mother) has no personality to annul the contract. Both are not
aggrieved parties on account of their own violation of the condition of, or restriction
on, their ownership imposed by the donation. Only the donor or his heirs would have
the personality to bring an action to revoke a donation for violation of a condition
thereof or a restriction thereon. (Garrido u. CA, 236 SCRA 450). Consequently, while
the donor or his heirs were not parties to the sale, they have the right to annul the
contract of sale because their rights are prejudiced by one of the contracting parties
thereof [DBP v. CA, 96 SCRA 342; Teves vs. PHHC. 23 SCRA 114]. Since
Armando is neither the donor nor heir of the donor, he has no personality to bring the
action for annulment.

Notes:
Under the Civil Code, only an aggrieved party to the contract may bring the
action for annulment thereof.
Moreover, only heirs of the donor, Tomas, can annul the donation. Therefore,
Conrado cannot annul the donation for noncompliance of the condition because he is
not an heir of Tomas.

ARTICLE 1397. The action for the annulment of contracts may be instituted
by all who are thereby obliged principally or subsidiarily. However, persons who
are capable cannot allege the incapacity of those with whom they contracted; nor can
those who exerted intimidation, violence, or undue influence, or employed fraud, or
caused mistake base their action upon these flaws of the contract.

ARTICLE 764. The donation shall be revoked at the instance of the donor, when
the donee fails to comply with any of the conditions which the former imposed upon
the latter.

In this case, the property donated shall be returned to the donor, the alienations
made by the donee and the mortgages imposed thereon by him being void, with the
limitations established, with regard to third persons, by the Mortgage Law and the
Land Registration laws.

This action shall prescribe after four years from the noncompliance with the
condition, may be transmitted to the heirs of the donor, and may be exercised against
the donee’s heirs.

In DBP v. CA, the Supreme Court held that only the donor or heirs of the donor
has the capacity to annul the donation for failure to comply with the condition. Since
Armando is neither the donor nor an heir of the donor, he has no personality to bring
the action for annulment of the sale.

Moreover, the contract of sale is already perfected between Irene and Conrado.
Thus, Armando is bound by the contract of sale because he is an heir of the seller,
Irene.
ARTICLE 1311. Contracts take effect only between the parties, their assigns and
heirs, except in case where the rights and obligations arising from the contract are not
transmissible by their nature, or by stipulation or by provision of law. The heir is not
liable beyond the value of the property he received from the decedent.

If a contract should contain some stipulation in favor of a third person, he may


demand its fulfillment provided he communicated his acceptance to the obligor before
its revocation. A mere incidental benefit or interest of a person is not sufficient. The
contracting parties must have clearly and deliberately conferred a favor upon a third
person.

ALTERNATIVE ANSWER:
As judge, I will grant the motion to dismiss. Non-compliance with a condition
imposed by a donor gives rise to an action to revoke the donation under Art. 764,
NCC. However, the right of action belongs to the donor and is transmissible to his
heirs, and may be exercised against the donee's heirs. Since Armando is an heir of the
donee, not of the donor, he has no legal capacity to sue for revocation of the donation.
Although he is not seeking such revocation but an annulment of the sale which his
mother, the donee, had executed in violation of the condition imposed by the donor,
an action for annulment of a contract may be brought only by those who are
principally or subsidiarily obliged thereby (Art. 1397, NCC). As an exception to the
rule, it has been held that a person not so obliged may nevertheless ask for annulment
if he is prejudiced in his rights regarding one of the contracting parties (DBP us. CA.
96 SCRA 342 and other cases) and can show the detriment which would result to
him from the contract in which he had no intervention, (Teves vs. PHHC, 23 SCRA
1141).

Such detriment or prejudice cannot be shown by Armando. As a forced heir,


Armando's interest in the property was, at best, a mere expectancy. The sale of the
land by his mother did not impair any vested right. The fact remains that the
premature sale made by his mother (premature because only half of the period of the
ban had elapsed) was not voidable at all, none of the vices of consent under Art. 139
of the NCC being present. Hence, the motion to dismiss should be granted.

Donations; Conditions; Revocation (1991)


Spouses Michael and Linda donated a 3-hectare residential land to the City of
Baguio on the condition that the city government would build thereon a public park
with a boxing arena, the construction of which shall commence within six (6) months
from the date the parties ratify the donation. The donee accepted the donation and the
title to the property was transferred in its name. Five years elapsed but the public park
with the boxing arena was never started. Considering the failure of the donee to
comply with the condition of the donation, the donor-spouses sold the property to
Ferdinand who then sued to recover the land from the city government. Will the
suit prosper?

SUGGESTED ANSWER:
Ferdinand has no right to recover the land. It is true that the donation was
revocable because of breach of the conditions. But until and unless the donation
was revoked, it remained valid. Hence, Spouses Michael and Linda had no right to
sell the land to Ferdinand. One cannot give what he does not have. What the donors
should have done first was to have the donation annulled or revoked. And after that
was done, they could validly have disposed of the land in favor of Ferdinand.
Notes:
Under the Civil Code, an annullable contract is valid until annulled. Therefore,
the spouses-donor cannot sell the property donated without first annulling the
contract.

ALTERNATIVE ANSWER:
A. Until the contract of donation has been resolved or rescinded under Article
1191 of the Civil Code or revoked under Art. 764 of the Civil Code, the donation
stands effective and valid. Accordingly, the sale made by the donor to Ferdinand
cannot be said to have conveyed title to Ferdinand, who, thereby, has no cause of
action for recovery of the land acting for and in his behalf.

B. The donation is onerous, and being onerous, what applies is the law on
contracts, and not the law on donation (De Luna us. Abrigo, 81 SCRA 150).
Accordingly, the prescriptive period for the filing of such an action would be the
ordinary prescriptive period for contacts which may either be six or ten depending
upon whether it is verbal or written. The filing of the case five years later is within the
prescriptive period and, therefore, the action can prosper.

Alternative Answer:

The law on donation lays down a special prescriptive period in the case of
breach of condition, which is four years from non-compliance thereof (Article 764
Civil Code). Since the action has prescribed, the suit will not prosper.

ARTICLE 764. The donation shall be revoked at the instance of the donor,
when the donee fails to comply with any of the conditions which the former imposed
upon the latter.
The property donated shall be returned to the donor; any alienations made by
the donee and the mortgages imposed thereon by him are void.

This action shall prescribe after four years from the noncompliance with the
condition; it may be transmitted to the heirs of the donor, and may be exercised
against the donee’s heirs. (Reengineered)
(The case of De Luna v. Abrigo is not in accordance with this provision).

Donations; Effect; illegal & immoral conditions (1997)


Are the effects of illegal and immoral conditions on simple donations the
same as those effects that would follow when such conditions are imposed on
donations con causa onerosa?

SUGGESTED ANSWER:
No, they don't have the same effect. Illegal or impossible conditions in simple
and remuneratory donations shall be considered as not imposed. Hence the donation is
valid. The donation will be considered as simple or pure. The condition or mode is
merely an accessory disposition, and its nullity does not affect the donation, unless it
clearly appears that the donor would not have made the donation without the mode or
condition.

Donations con causa onerosa is governed by law on obligations and contracts,


under which an impossible or Illicit condition annuls the obligation dependent upon
the condition where the condition is positive and suspensive. If the impossible or
illicit condition is negative, it is simply considered as not written, and the obligation is
converted into a pure and simple one. However, in order that an illegal condition may
annul a contract, the impossibility must exist at the time of the creation of the
obligation; a supervening impossibility does not affect the existence of the obligation.
Notes:
Simple donation is a donation which does not impose a condition. On the other
hand, onerous donation is one which imposes a condition like building a boxing
complex on the donated land.

Imposition of illegal or impossible condition in a simple donation make such


condition as if not written. But imposition of illicit or impossible condition on the
onerous or con causa onerosa donation make the donation ineffective. It is governed
by the Law on Obligations and Contracts.
The illegal and impossible condition imposed on onerous donation shall annul
the obligation. Thus, the donation is not effective.

ADDITIONAL ANSWER:
No. In simple or pure donation, only the illegal or impossible condition is
considered not written but the donation remains valid and becomes free from
conditions. The condition or mode being a mere accessory disposition. Its nullity does
not affect the donation unless it clearly appears that the donor would not have made
the donation without the mode or condition. On the other hand, onerous donation is
governed by the rules on contracts. Under Article 1183, Impossible or illegal
conditions shall annul the obligation which depends upon them. In these cases, both
the obligation and the condition are void.

Notes:
The illegal or impossible condition is considered not written in simple and pure
donation. Thus, the donation remains effective.
However, the impossible and illegal condition shall annul the obligation which
depends upon them in case of onerous donation.

Under the Civil Code, an impossible or illegal condition shall annul an


obligation if such obligation depends upon that condition.

ARTICLE 1183. Impossible conditions, those contrary to good customs or


public policy and those prohibited by law shall annul the obligation which depends
upon them. If the obligation is divisible, that part thereof which is not affected by the
impossible or unlawful condition shall be valid.

The condition not to do an impossible thing shall be considered as not having


been agreed upon.

Donations; Formalities; Mortis Causa (1990)


B donated to M a parcel of land in 1980. B made the deed of donation, entitled
“Donation Inter Vivos,” in a public instrument and M accepted the donation in the
same document. It was provided in the deed that the land donated shall be
immediately delivered to M and that M shall have the right to enjoy the fruits fully.
The deed also provided that B was reserving the right to dispose of said land during
his (B’s) lifetime, and that M shall not register the deed of donation until after B’s
death. Upon B’s death, W, B’s widow and sole heir, filed an action for the
recovery of the donated land, contending that the donation made by B is a
donation mortis causa and not a donation inter vivos. Will said action prosper?
Explain your answer.

SUGGESTED ANSWER:
Yes, the action will prosper. The donation is a donation mortis causa because the
reservation is to dispose of all the property donated and, therefore, the donation is
revocable at will. Accordingly, the donation requires the execution of a valid will,
either notarial or holographic (Arts 755, 728 NCC).

Notes:
Under the Civil Code, when a donation is to take effect upon the death of the
donor, it is a donation mortis causa; it shall comply with the formalities of a will. In
this case, the donation is a donation mortis causa because the donor reserved the right
to dispose of the property while he is still alive. Therefore, the action filed by the
widow for the annulment of the donation will prosper because it does not comply with
the formalities of a will.
ARTICLE 728. Donations which are to take effect upon the death of the donor
partake of the nature of testamentary provisions, and shall be governed by the rules
established in the Title on Succession.

ARTICLE 755. The right to dispose of some of the things donated, or of some
amount which shall be a charge thereon, may be reserved by the donor; but if he
should die without having made use of this right, the property or amount reserved
shall belong to the donee.

Donations; Formalities; Mortis Causa (1998)


Ernesto donated in a public instrument a parcel of land to Demetrio, who
accepted it in the same document. It is there declared that the donation shall take
effect immediately, with the donee having the right to take possession of the land and
receive its fruits but not to dispose of the land while Ernesto is alive as well as for ten
years following his death. Moreover, Ernesto also reserved in the same deed his
right to sell the property should he decide to dispose of it at any time - a right which
he did not exercise at all. After his death, Ernesto's heirs seasonably brought an
action to recover the property, alleging that the donation was void as it did not
comply with the formalities of a will. Will the suit prosper? [5%]

SUGGESTED ANSWER:
Yes, the suit will prosper as the donation did not comply with the
formalities of a will. In this instance, the fact that the donor did not intend to transfer
ownership or possession of the donated property to the donee until the donor's death,
would result in a donation mortis causa and in this kind of disposition, the formalities
of a will should be complied with, otherwise, the donation is void. In this Instance,
donation mortis causa embodied only in a public instrument without the formalities of
a will could not have transferred ownership of disputed property to another.

Notes:
If the donation partakes of a donation mortis causa, it must comply with the
formalities of a will; otherwise, the donation is void.
Yes, the suit will prosper because the donation is void since it did not comply
with the formalities of a will. The condition that the donee cannot dispose the property
while the donor is alive means that the donation is a donation mortis causa. Thus, it
must comply with the formalities of a will to become effective.

ALTERNATIVE ANSWER:
One of the essential distinctions between a donation inter vivos and a donation
mortis causa is that while the former is irrevocable, the latter is revocable. In the
problem given, all the clauses or conditions mentioned in the deed of donation, except
one, are consistent with the rule of irrevocability and would have sustained the view
that the donation is inter vivos and therefore valid. The lone exception is the clause
which reserves the donor's right to sell the property at any time before his death. Such
a reservation has been held to render the donation revocable and, therefore, becomes a
donation mortis causa (Puig vs. Penqflorida, 15 SCRA 276, at p. 286). That the right
was not exercised is immaterial; its reservation was an implied recognition of the
donor's power to nullify the donation anytime he wished to do so. Consequently, it
should have been embodied in a last will and testament. The suit for nullity will thus
prosper.

Notes:
In the case of Puig vs. Penqflorida, the Supreme Court held that donation mortis
causa is revocable while donation inter vivos is irrevocable.
Donations; Inter Vivos; Acceptance (1993)
On January 21, 1986, A executed a deed of donation inter vivos of a parcel of
land to Dr. B who had earlier constructed thereon a building in which researches on
the dreaded disease AIDS were being conducted. The deed, acknowledged before a
notary public, was handed over by A to Dr. B who received it. A few days after, A
flew to Davao City. Unfortunately, the airplane he was riding crashed on landing
killing him. Two days after the unfortunate accident, Dr. B, upon advice of a lawyer,
executed a deed acknowledged before a notary public accepting the donation. Is the
donation effective? Explain your answer.
SUGGESTED ANSWER:
No, the donation is not effective. The law requires that the separate acceptance
of the donee of an immovable must be done in a public document during the lifetime
of the donor (Art. 746 & 749, Civil Code) In this case, B executed the deed of
acceptance before a notary public after the donor had already died.

Notes:
Under the Civil Code, acceptance of a donation must be during the lifetime of
the donor and the donee; otherwise, the donation is void.
ARTICLE 746. Acceptance must be made during the lifetime of the donor and
of the donee.

ARTICLE 749. When you donate an immovable, it must be in public


instrument; otherwise, the donation is not valid. (Reengineered)

Donations; Perfection (1998)


On July 27, 1997, Pedro mailed in Manila a letter to his brother, Jose, a
resident of Ilollo City, offering to donate a vintage sports car which the latter had
long been wanting to buy from the former. On August 5, 1997, Jose called Pedro by
cellular phone to thank him for his generosity and to inform him that he was sending
by mail his letter of acceptance. Pedro never received that letter because it was never
mailed. On August 14, 1997, Pedro received a telegram from Iloilo informing him that
Jose had been killed in a road accident the day before (August 13, 1997)

1. Is there a perfected donation? [2%]

2. Will your answer be the same if Jose did mail his acceptance letter but it
was received by Pedro in Manila days after Jose's death? [3%]

SUGGESTED ANSWER:
1. None. There is no perfected donation. Under Article 748 of the Civil Code,
the donation of a movable may be made orally or in writing. If the value of the
personal property donated exceeds five thousand pesos, the donation and the
acceptance shall be made in writing. Assuming that the value of the thing donated, a
vintage sports car, exceeds P5,000.00 then the donation and the acceptance must be in
writing. In this instance, the acceptance of Jose was not in writing, therefore, the
donation is void. Upon the other hand, assuming that the sports car costs less than
P5,000.00 then the donation maybe oral, but still, the simultaneous delivery of the car
is needed and there being none, the donation was never perfected.

Notes:

1) Under the Civil Code, donation and acceptance must be made during the
lifetime of the donor and the donee. Therefore, the donation is not perfected because
the acceptance was not communicated to the donor until he died.

2) Under the Civil Code, donation and acceptance must be made during the
lifetime of the donor and the donee. Therefore, the answer is the same because the
acceptance was made after the death of the donor.

ARTICLE 748. The donation of a movable may be made orally or in writing.

An oral donation requires the simultaneous delivery of the thing or of the


document representing the right donated.

SUGGESTED ANSWER:

2. Yes, the answer is the same. If Jose's mail containing his acceptance of the
donation was received by Pedro after the former's death, then the donation is still void
because under Article 734 of the Civil Code, the donation is perfected the moment
the donor knows of the acceptance by the donee. The death of Jose before Pedro
could receive the acceptance indicates that the donation was never perfected. Under
Article 746 acceptance must be made during the lifetime of both the donor and the
donee.

Notes:
In this case, the donation is not perfected because the law requires that the
donation and the acceptance must be made during the lifetime of the donor and the
donee. Since the donee died before the donor received the acceptance, the donation is
not perfected.

Donations; Requisites; Immovable Property


Anastacia purchased a house and lot on installments at a housing project in
Quezon City. Subsequently, she was employed in California and a year later, she
executed a deed of donation, duly authenticated by the Philippine Consulate in Los
Angeles, California, donating the house and lot to her friend Amanda. The latter
brought the deed of donation to the owner of the project and discovered that Anastacia
left unpaid installments and real estate taxes. Amanda paid these so that the donation
in her favor can be registered in the project owner's office. Two months later,
Anastacia died, leaving her mother Rosa as her sole heir. Rosa filed an action to
annul the donation on the ground that Amanda did not give her consent in the
deed of donation or in a separate public instrument. Amanda replied that the
donation was an onerous one because she had to pay unpaid installments and
taxes; hence her acceptance may be implied. Who is correct? (2%)

SUGGESTED ANSWER:
Rosa is correct because the donation is void. The property donated was an
immovable. For such donation to be valid, Article 749 of the New Civil Code
requires both the donation and the acceptance (of real property) to be in a public
instrument. There being no showing that Amanda's acceptance was made in a public
instrument, the donation is void. The contention that the donation is onerous and,
therefore, need not comply with Article 749 for validity is without merit. The donation
is not onerous because it did not impose on Amanda the obligation to pay the balance
on the purchase price or the arrears in real estate taxes. Amanda took it upon herself to
pay those amounts voluntarily. For a donation to be onerous, the burden must be
imposed by the donor on the donee. In the problem, there is no such burden imposed
by the donor on the donee. The donation not being onerous, it must comply with the
formalities of Article 749.

Notes:
Under the Civil Code, donation and acceptance shall be in a public instrument
if the object of donation is immovable; otherwise, it is not valid. In this case, the
donation is not valid because it was not accepted by Amanda. Therefore, Rosa is
correct; the donation can be annulled.

ARTICLE 749. In order that the donation of an immovable may be valid, it


must be made in a public document, specifying therein the property donated and the
value of the charges which the donee must satisfy.

The acceptance may be made in the same deed of donation or in a separate


public document, but it shall not take effect unless it is done during the lifetime of
the donor.
If the acceptance is made in a separate instrument, the donor shall be notified
thereof in an authentic form, and this step shall be noted in both instruments.

ALTERNATIVE ANSWER:
Neither Rosa nor Amanda is correct. The donation is onerous only as to the
portion of the property corresponding to the value of the installments and taxes paid
by Amanda.

The portion in excess thereof is not onerous. The onerous portion is governed by
the rules on contracts which do not require the acceptance by the donee to be in any
form. The onerous part, therefore, is valid. The portion which is not onerous must
comply with Article 749 of the New Civil Code which requires the donation and the
acceptance thereof to be in a public instrument in order to be valid. The acceptance
not being in a public instrument, the part which is not onerous is void and Rosa may
recover it from Amanda.

Notes:
An onerous donation is one which imposes a burden on the donee like the
payment of taxes, among others.

An onerous donation is governed by the law on contracts; it does not need the
requirement of acceptance. If the donation is not onerous, acceptance of the donee
during the lifetime of the donor is necessary to make the contract effective.

Notes:
When what is donated is movable, the donation and acceptance shall be in
writing if the value of the movable exceeds P 5,000.00. If it does not exceed P
5,000.00, it does not need to be in writing but the oral donation and the delivery shall
be simultaneous; otherwise, it is not effective.

Donations; Unregistered; Effects; Non-Compliance; Resolutory Condition (2006)


Spouses Alfredo and Racquel were active members of a religious
congregation. They donated a parcel of land in favor of that congregation in a duly
notarized Deed of Donation, subject to the condition that the Minister shall construct
thereon a place of worship within 1 year from the acceptance of the donation. In an
affidavit he executed on behalf of the congregation, the Minister accepted the
donation. The Deed of Donation was not registered with the Registry of Deeds.

However, instead of constructing a place of worship, the Minister constructed a


bungalow on the property he used as his residence. Disappointed with the Minister,
the spouses revoked the donation and demanded that he vacate the premises
immediately. But the Minister refused to leave, claiming that aside from using the
bungalow as his residence, he is also using it as a place for worship on special
occasions.

Under the circumstances, can Alfredo and Racquel evict the Minister and
recover possession of the property? If you were the couple's counsel, what action
you take to protect the interest of your clients? (5%)

ALTERNATIVE ANSWER:
Yes, Alfredo and Racquel can bring an action for ejectment against the Minister
for recovery of possession of the property. An action for annulment of the donation,
reconveyance and damages should be filed to protect the interests of my client. The
donation is an onerous donation and therefore shall be governed by the rules on
contracts. Because there was no fulfillment or compliance with the condition which is
resolutory in character, the donation may now be revoked and all rights which the
donee may have acquired under it shall be deemed lost and extinguished (Central
Philippine University, G.R. No. 112127, July 17,1995).

Notes:
Under the Civil Code, donation and acceptance shall be in a public instrument if
the object thereof is immovable; otherwise, it is void. Therefore, Alfredo and Racquel
can evict the Minister and recover possession of the property because the done, the
Minister, did not accept the donation in a public instrument which made the donation
void.

ARTICLE 764. The donation shall be revoked at the instance of the donor,
when the donee fails to comply with any of the conditions which the former imposed
upon the latter.

The property donated shall be returned to the donor; any alienations made by
the donee and the mortgages imposed thereon by him are void.
This action shall prescribe after four years from the noncompliance with the
condition; it may be transmitted to the heirs of the donor, and may be exercised
against the donee’s heirs. (Reengineered)

ALTERNATIVE ANSWER:
No, an action for ejectment will not prosper. I would advice Alfredo and
Racquel that the Minister, by constructing a structure which also serves as a place of
worship, has pursued the objective of the donation. His taking up residence in the
bungalow may be regarded as a casual breach and will not warrant revocation of the
donation. Similarily, therefore, an action for revocation of the donation will be denied
(C. J. Yulo & Sons, Inc. v. Roman Catholic Bishop, G.R. No. 133705, March 31,
2005; Heirs ofRozendo Sevilla v. De Leon, G.R. No. 149570, March 12, 2004).

Donations; Validity; Effectivity; for Unborn Child (1999)


Elated that her sister who had been married for five years was pregnant for the
first time, Alma donated P100,000.00 to the unborn child. Unfortunately, the baby
died one hour after delivery. May Alma recover the P100.000.00 that she had
donated to said baby before it was born considering that the baby died? Stated
otherwise, is the donation valid and binding? Explain. (5%)

SUGGESTED ANSWER:
The donation is valid and binding, being an act favorable to the unborn child, but
only if the baby had an intra-uterine life of not less than seven months and provided
there was due acceptance of the donation by the proper person representing said
child. If the child had less than seven months of intra-uterine life, it is not deemed
born since it died less than 24 hours following its delivery, in which ease the donation
never became effective since the donee never became a person, birth being
determinative of personality.

Notes:
The donation is valid and binding because it is an act favorable to the unborn
child provided the baby had an intra-uterine life of at least seven months and there
was due acceptance of the donation by the proper person representing said child.
(Reengineered)

If the child had less than seven months of intra-uterine life, it is not deemed
born since it died less than 24 hours following its delivery, in which ease the donation
never became effective since the donee never became a person, birth being
determinative of personality.
ALTERNATIVE ANSWER:
Even if the baby had an intra-uterine life of more than seven months and the
donation was properly accepted, it would be void for not having conformed with the
proper form. In order to be valid, the donation and acceptance of personal property
exceeding five thousand pesos should be in writing. (Article 748, par. 3)
Notes:
Under the Civil Code, a donation of movable which exceeds P 5,000.00. shall be
in writing; also, the acceptance shall be in writing.

ARTICLE 748. The donation of a movable may be made orally or in writing.

An oral donation requires the simultaneous delivery of the thing or of the


document representing the right donated.

If the value of the personal property donated exceeds five thousand pesos, the
donation and the acceptance shall be made in writing. Otherwise, the donation shall be
void.

Donations; with Resolutory Condition (2003)


In 1950, Dr. Alba donated a parcel of land to Central University on condition
that the latter must establish a medical college on the land to be named after him. In
the year 2000, the heirs of Dr. Alba filed an action to annul the donation and for the
reconveyance of the property donated to them for the failure, after 50 years, of the
University to establish on the property a medical school named after their father. The
University opposed the action on the ground of prescription and also because it
had not used the property for some purpose other than that stated in the
donation. Should the opposition of the University to the action of Dr. Alba’s heirs
be sustained? Explain.

SUGGESTED ANSWER:
The donation may be revoked. The non-establishment of the medical college on
the donated property was a resolutory condition imposed on the donation by the
donor. Although the Deed of Donation did not fix the time for the establishment of the
medical college, the failure of the donee to establish the medical college after fifty
(50) years from the making of the donation should be considered as occurrence of the
resolutory condition, and the donation may now be revoked. While the general rule is
that in case the period is not fixed in the agreement of the parties, the period must be
fixed first by the court before the obligation may be demanded, the period of fifty (50)
years was more than enough time for the donee to comply with the condition.
Hence, in this case, there is no more need for the court to fix the period because such
procedure with the condition. (Central Philippine University v. CA. 246 SCRA 511).

Notes:
Under the Civil Code, if a period to comply with the obligation is not fixed and
it can be inferred that parties intended a period, a court may fix it. Therefore, the heirs
of Dr. Alba cannot revoke as yet the donation without allowing the court to fix it first.

(The SC ruled in a mysterious way in this case).


ANOTHER SUGGESTED ANSWER:
The donation may not as yet revoked. The establishment of a medical college is
not a resolutory or suspensive condition but a “charge”, obligation”, or a “mode”. The
non- compliance with the charge or mode will give the donor the right to revoke the
donation within four (4) years from the time the charge was supposed to have
been complied with, or to enforce the charge by specific performance within ten(10)
years from the time the cause of action accrued. Inasmuch as the time to establish the
medical college has not been fixed in the Deed of Donation, the donee is not yet
default in his obligation until the period is fixed by order of the court under Article
1197 of the New Civil Code. Since the period has not been fixed as yet, the donee is
not yet default, and therefore the donor has no cause of action to revoke the donation.
(Dissenting opinion of Davide, CJ, Central Philippine University v. Court of
Appeals, 246 SCRA 511 [1995])

Notes:
Under the Civil Code, courts may fix the period in an obligation if it is not fixed
and when it can be inferred that a period is intended by the parties. Therefore, the
heirs of Dr. Alba cannot revoke the donation without allowing the court to first fix the
period to comply with the condition to establish a medical college.

ARTICLE 1197. If the obligation does not fix a period, but from its nature and
the circumstances it can be inferred that a period was intended, the courts may fix
the duration thereof.

The courts shall also fix the duration of the period when it depends upon the will
of the debtor.
In every case, the courts shall determine such period as may under the
circumstances have been probably contemplated by the parties. Once fixed by the
courts, the period cannot be changed by them.

PROPERTY

Accretion; Alluvion (2001)


For many years, the Rio Grande river deposited soil along its bank, beside the
titled land of Jose. In time, such deposit reached an area of one thousand square
meters. With the permission of Jose, Vicente cultivated the said area. Ten years later,
a big flood occurred in the river and transferred the 1000 square meters to the opposite
bank, beside the land of Agustin. The land transferred is now contested by Jose
and Agustin as riparian owners and by Vicente who claims ownership by
prescription. Who should prevail? Why? (5%)

SUGGESTED ANSWER:
Jose should prevail. The disputed area, which is an alluvion, belongs by right of
accretion to Jose, the riparian owner (Art. 457 CC). When, as given in the problem,
the very same area" was "transferred" by flood waters to the opposite bank, it became
an avulsion and ownership thereof is retained by Jose who has two years to remove it
(Art. 459, CC). Vicente's claim based on prescription is baseless since his possession
was by mere tolerance of Jose and, therefore, did not adversely affect Jose's
possession and ownership (Art. 537, CC). Inasmuch as his possession is merely that of
a holder, he cannot acquire the disputed area by prescription.

Notes:
Jose should prevail because under the Civil Code, avulsion should belong to the
riparian owner where the land gradually transferred as a result of the current of the
rivers for so many years. When such land deposit suddenly transferred to the land of
Agustin, it becomes avulsion. Jose retained the ownership of the land under the law.
Vicente cannot claim ownership over the land because possession by mere tolerance
does not prescribe since he did not possess the land in the concept of an owner.

ARTICLE 457. Accretion which gradually receive from the effects of the
current of the waters belong to the owners of lands where the accretion attached. (This
is called alluvium). (Reengineered)
ARTICLE 459. Whenever the current of a river, creek or torrent segregates a
portion of land from one estate and transfers it to another estate, the owner of the land
where the segregated portion came from retains the ownership of it, provided that he
removes the same within two years. (This is known as avulsion. The transfer of the
land is sudden). (Reengineered)

ARTICLE 537. Acts merely tolerated, and those executed clandestinely and
without the knowledge of the possessor of a thing, or by violence, do not affect
possession.

Accretion; Alluvion (2003)


Andres is a riparian owner of a parcel of registered land. His land, however, has
gradually diminished in area due to the current of the river, while the registered land
of Mario on the opposite bank has gradually increased in area by 200square meters.

(a) Who has the better right over the 200-square meter area that has been
added to Mario’s registered land, Mario or may a third person acquire said 200-
square meter land by prescription?

SUGGESTED ANSWER:
a. Mario has a better right over the 200 square meters increase in area by reason
of accretion, applying Article 457 of the New Civil Code, which provides that “to the
owners of lands adjoining the banks of rivers belong the accretion which they
gradually received from the effects of the current of the waters”.

Andres cannot claim that the increase in Mario’s land is his own, because such is
an accretion and not result of the sudden detachment of a known portion of his land
and its attachment to Mario’s land, a process called “avulsion”. He can no longer
claim ownership of the portion of his registered land which was gradually and
naturally eroded due to the current of the river, because he had lost it by operation of
law. That portion of the land has become part of the public domain.

Notes:
Under the Civil Code, owners of the lands adjoining the bank of rivers are the
owners of the accretion which the lands gradually receive from the effect of the
current of waters.
ARTICLE 457. To the owners of lands adjoining the banks of rivers belong the
accretion which they gradually receive from the effects of the current of the waters.

SUGGESTED ANSWER:
b. Yes, a third party may acquire by prescription the 200 square meters, increase
in area, because it is not included in the Torrens Title of the riparian owner. Hence,
this does not involve the imprescriptibility conferred by Section 47, P.D. No. 1529.
The fact that the riparian land is registered does not automatically make the accretion
thereto a registered land. (Grande v. CA, 115 521 (1962); Jagualing v. CA, 194
SCRA 607 (1991).

Builder; Good Faith (1992)


A owns a parcel of residential land worth P500,000.00. Unknown to A, a
residential house costing P 100,000.00 is built on the entire parcel by B who claims
ownership of the land. Answer all the following questions based on the premise that B
is a builder in good faith and A is a landowner in good faith.

a) May A acquire the house built by B? If so, how?

b) If the land increased in value to P500,000.00 by reason of the building of


the house thereon, what amount should be paid by A in order to acquire the
house from B?

c) Assuming that the cost of the house was P90,000.00 and not P100,000.00,
may A require B to buy the land?

d) If B voluntarily buys the land as desired by A, under what circumstances


may A nevertheless be entitled to have the house removed?

e) In what situation may a "forced lease" arise between A and B. and what
terms and conditions would govern the lease?
Give reasons for your answers.

SUGGESTED ANSWER:
(a) Yes, A may acquire the house build by B by paying indemnity to B. Article
448 of the Civil Code provides that the owner of the land on which anything has been
built, sown or planted in good faith, shall have the right to appropriate as his own
the works, sowing or planting, after payment of the indemnity provided for in
Articles 546 and 546 of the Civil Code.
(b) A should pay B the sum of P50,000. Article 548 of the Civil Code provides
that useful expenses shall be refunded to the possessor in good faith with the right of
retention, the person who has defeated him in the possession having the option of
refunding the amount of the expenses or of paying the increase in value which the
thing may have acquired by reason thereof. The increase in value amounts to
P50,000.00.

(c) Yes, A may require B to buy the land. Article 448 of the Civil Code provides
that the owner of the land on which anything has been built in good faith shall have
the right to oblige the one who built to pay the price of the land if its value is not
considerably more than that of the building,

(d) If B agrees to buy land but fails to pay, A can have the house removed
( Depra vs. Dumlao, 136 SCRA 475).

(e) Article 448 of the Civil Code provides that the builder cannot be obliged to
buy the land if its value is considerably more than that of the building. In such case, he
shall pay reasonable rent, if the owner of the land does not choose to appropriate the
building after proper indemnity. The parties shall agree upon the terms of the lease
and in case of disagreement, the court fix the terms thereof.

Notes:

a) Under the Civil Code, a landowner may acquire the improvement upon
payment of indemnity if the builder or planter was in good faith.

b) Under the Civil Code, the landowner shall reimburse the planter or builder
in good faith. Therefore, the amount to be paid must be the building cost and not the
fair market value of the building.

c) Under the Civil Code, if the value of the land is not considerably higher than
the value of the building, the landowner may require the builder to pay the price of the
land.

d) Under the Civil Code, if the builder has chosen to buy the land buy he failed
to pay the price, the landowner may oblige the builder to remove the house.
e) Under the Civil Code, if the landowner does not want to buy the improvement
and the builder cannot be compelled to buy the land because the amount of the same is
considerably more than the building, they shall enter into a forced lease. If they
cannot agree as to the terms of the lease, the court shall fix the term of the lease.

ARTICLE 448. The owner of the land on which anything has been built, sown
or planted in good faith, shall have the right to appropriate as his own the works,
sowing or planting, after payment of the indemnity provided for in articles 546 and
548, or to oblige the one who built or planted to pay the price of the land, and the one
who sowed, the proper rent. However, the builder or planter cannot be obliged to buy
the land if its value is considerably more than that of the building or trees. In such
case, he shall pay reasonable rent, if the owner of the land does not choose to
appropriate the building or trees after proper indemnity. The parties shall agree upon
the terms of the lease and in case of disagreement, the court shall fix the terms thereof.

ARTICLE 546. Necessary expenses shall be refunded to every possessor; but


only the possessor in good faith may retain the thing until he has been reimbursed
therefor.

Useful expenses shall be refunded only to the possessor in good faith with the
same right of retention, the person who has defeated him in the possession having the
option of refunding the amount of the expenses or of paying the increase in value
which the thing may have acquired by reason thereof. (453a)

ARTICLE 548. Expenses for pure luxury or mere pleasure shall not be refunded
to the possessor in good faith; but he may remove the ornaments with which he has
embellished the principal thing if it suffers no injury thereby, and if his successor in
the possession does not prefer to refund the amount expended.

Builder; Good Faith vs. Bad Faith (1999)


(a) Because of confusion as to the boundaries of the adjoining lots that they
bought from the same subdivision company, X constructed a house on the adjoining
lot of Y in the honest belief that it is the land that he bought from the subdivision
company. What are the respective rights of X and Y with respect to X's house? (3%)

(b) Suppose X was in good faith but Y knew that X was constructing on his
(Y's) land but simply kept quiet about it, thinking perhaps that he could get X's house
later. What are the respective rights of the parties over X's house in this case? (2%)
SUGGESTED ANSWER:
(a) The rights of Y, as owner of the lot, and of X, as builder of a house thereon,
are governed by Art. 448 of the Civil Code which grants to Y the right to choose
between two remedies: (a) appropriate the house by indemnifying X for its value
plus whatever necessary expenses the latter may have incurred for the preservation
of the land, or (b) compel X to buy the land if the price of the land is not
considerably more than the value of the house. If it is, then X cannot be obliged to buy
the land but he shall pay reasonable rent, and in case of disagreement, the court shall
fix the terms of the lease.

SUGGESTED ANSWER:
(b) Since the lot owner Y is deemed to be in bad faith (Art 453), X as the builder
in good faith may (a) remove the house and demand indemnification for damages
suffered by him, or (b) demand payment of the value of the house plus reparation for
damages (Art 447, in relation to Art 454). Y continues as owner of the lot and
becomes, under the second option, owner of the house as well, after he pays the sums
demanded.

Builder; Good Faith vs. Bad Faith (2000)


In good faith, Pedro constructed a five-door commercial building on the land of
Pablo who was also in good faith. When Pablo discovered the construction, he opted
to appropriate the building by paying Pedro the cost thereof. However, Pedro insists
that he should be paid the current market value of the building, which was much
higher because of inflation.

1) Who is correct Pedro or Pablo?(1%)

2) In the meantime that Pedro is not yet paid, who is entitled to the rentals
of the building, Pedro or Pablo? (1%)

SUGGESTED ANSWER:
Pablo is correct. Under Article 448 of the New Civil Code in relation to Article
546, the builder in good faith is entitled to a refund of the necessary and useful
expenses incurred by him, or the increase in value which the land may have acquired
by reason of the improvement, at the option of the landowner. The builder is entitled
to a refund of the expenses he incurred, and not to the market value of the
improvement The case of Pecson v. CA, 244 SCRA 407, is not applicable to the
problem. In the Pecson case, the builder was the owner of the land who later lost the
property at a public sale due to non-payment of taxes. The Court ruled that Article 448
does not apply to the case where the owner of the land is the builder but who later lost
the land; not being applicable, the indemnity that should be paid to the buyer must be
the fair market value of the building and not just the cost of construction thereof.
The Court opined in that case that to do otherwise would unjustly enrich the new
owner of the land.

ALTERNATIVE ANSWER:
Pedro is correct. In Pecson vs. CA, it was held that Article 546 of the New Civil
Code does not specifically state how the value of useful improvements should be
determined in fixing the amount of indemnity that the owner of the land should pay to
the builder in good faith. Since the objective of the law is to adjust the rights of the
parties in such manner as "to administer complete justice to both of them in such a
way as neither one nor the other may enrich himself of that which does not belong to
him", the Court ruled that the basis of reimbursement should be the fair market
value of the building.

Notes:
Under the Civil Code, the landowner shall pay the building cost, and not the
current market value of the building since the word used by the law is reimburse.

SUGGESTED ANSWER:

2) Pablo is entitled to the rentals of the building. As the owner of the land, Pablo
is also the owner of the building being an accession thereto. However, Pedro who is
entitled to retain the building is also entitled to retain the rentals. He, however, shall
apply the rentals to the indemnity payable to him after deducting reasonable cost of
repair and maintenance.

ALTERNATIVE ANSWER:
Pablo is entitled to the rentals. Pedro became a possessor in bad faith from the
time he learned that the land belongs to Pablo. As such, he loses his right to the
building, including the fruits thereof, except the right of retention.

Builder; Good Faith vs. Bad Faith; Accession (2000)


a) Demetrio knew that a piece of land bordering the beach belonged to Ernesto.
However, since the latter was studying in Europe and no one was taking care of the
land, Demetrio occupied the same and constructed thereon nipa sheds with tables and
benches which he rented out to people who want to have a picnic by the beach. When
Ernesto returned, he demanded the return of the land. Demetrio agreed to do so after
he has removed the nipa sheds. Ernesto refused to let Demetrio remove the nipa
sheds on the ground that these already belonged to him by right of accession.
Who is correct? (3%)

SUGGESTED ANSWER:
Ernesto is correct, Demetrio is a builder in bad faith because he knew
beforehand that the land belonged to Ernesto, under Article 449 of the New Civil
Code, one who builds on the land of another loses what is built without right to
indemnity. Ernesto becomes the owner of the nipa sheds by right of accession. Hence,
Ernesto is well within his right in refusing to allow the removal of the nipa sheds.

Notes:
Under Article 449 of the New Civil Code, one who builds in bad faith on the
land of another loses what is built without right to indemnity.

ARTICLE 449. He who builds, plants or sows in bad faith on the land of
another, loses what is built, planted or sown without right to indemnity.

Builder; Good Faith vs. Bad Faith; Presumption (2001)


Mike built a house on his lot in Pasay City. Two years later, a survey disclosed
that a portion of the building actually stood on the neighboring land of Jose, to the
extent of 40 square meters. Jose claims that Mike is a builder in bad faith because he
should know the boundaries of his lot, and demands that the portion of the house
which encroached on his land should be destroyed or removed. Mike replies that
he is a builder in good faith and offers to buy the land occupied by the building
instead.

1) Is Mike a builder in good faith or bad faith? Why? (3%)


2) Whose preference should be followed? Why? (2%)

SUGGESTED ANSWER:
1) Yes, Mike is a builder in good faith. There is no showing that when he built
his house, he knew that a portion thereof encroached on Jose's lot. Unless one is
versed in the science of surveying, he cannot determine the precise boundaries or
location of his property by merely examining his title. In the absence of contrary
proof, the law presumes that the encroachment was done in good faith [Technogas
Phils, v. CA, 268 SCRA 5, 15 (1997)].
2} None of the preferences shall be followed. The preference of Mike cannot
prevail because under Article 448 of the Civil Code, it is the owner of the land who
has the option or choice, not the builder. On the other hand, the option belongs to
Jose, he cannot demand that the portion of the house encroaching on his land be
destroyed or removed because this is not one of the options given by law to the owner
of the land. The owner may choose between the appropriation of what was built
after payment of indemnity, or to compel the builder to pay for the land if the
value of the land is not considerably more than that of the building. Otherwise,
the builder shall pay rent for the portion of the land encroached.

ALTERNATIVE ANSWER:
1) Mike cannot be considered a builder in good faith because he built his house
without first determining the corners and boundaries of his lot to make sure that his
construction was within the perimeter of his property. He could have done this with
the help of a geodetic engineer as an ordinary prudent and reasonable man would do
under the circumstances.
2) Jose's preference should be followed. He may have the building removed at
the expense of Mike, appropriate the building as his own, oblige Mike to buy the land
and ask for damages in addition to any of the three options. (Articles 449, 450, 451,
CC)

CONTRACT OF SURETY

Chattel Mortgage vs. Pledge (1999)


Distinguish a contract of chattel mortgage from a contract of pledge. (2%)

SUGGESTED ANSWER:
In a contract of CHATTEL MORTGAGE possession belongs to the creditor,
while in a contract of PLEDGE possession belongs to the debtor.
A chattel mortgage is a formal contract while a pledge is a real contract.
A contract of chattel mortgage must be recorded in a public instrument to bind
third persons while a contract of pledge must be in a public instrument containing
description of the thing pledged and the date thereof to bind third persons.

Notes:
In a contract of chattel mortgage, the personal property is not delivered to the
creditor, while in pledge, the personal property is delivered to the creditor.
A contract of pledge is a real contract, while a chattel mortgage is a formal
contract.
To bind third persons, the contract of pledge must be in a public instrument,
while the contract of chattel mortgage should be recorded in the appropriate
government agency.

Chattel Mortgage; Immovables (1994)


Vini constructed a building on a parcel of land he leased from Andrea. He
chattel mortgaged the land to Felicia. When he could not pay Felicia. Felicia initiated
foreclosure proceedings. Vini claimed that the building he had constructed on the
leased land cannot be validly foreclosed because the building was, by law, an
immovable. Is Vini correct?
No. Vini is not correct.
Supreme court has ruled in one case that a building, which by law is an
immovable property, may become subject of chattel mortgage if the parties had
stipulated so. Under the New Civil Code, a contract has the obligatory force of a law
and the parties must abide by it.
Here, Vini chattel mortgaged the building to Felicia to secure her lease on the
latter’s land. The building therefore is treated as a movable property under the
chattel mortgage and the parties are bound by it. They can no longer disavow their
contract on account of estoppel by deed. However, third parties are not bound by
this stipulation.
Hence, Vini’s contention is not correct

SUGGESTED ANSWERS:
a) The Chattel Mortgage is void and cannot be foreclosed because the building is
an immovable and cannot be an object of a chattel mortgage.
b) It depends. If the building was intended and is built of light materials, the
chattel mortgage may be considered as valid as between the parties and it may be
considered in respect to them as movable property, since it can be removed from one
place to another. But if the building is of strong material and is not capable of being
removed or transferred without being destroyed, the chattel mortgage is void and
cannot be foreclosed.
c) If it was the land which Vini chattel mortgaged, such mortgage would be
void, or at least unenforceable, since he was not the owner of the land. If what was
mortgaged as a chattel is the building, the chattel mortgage is valid as between the
parties only, on grounds of estoppel which would preclude the mortgagor from
assailing the contract on the ground that its subject-matter is an immovable. Therefore
Vini's defense is untenable, and Felicia can foreclose the mortgage over the building,
observing, however, the procedure prescribed for the execution of sale of a judgment
debtor's immovable under Rule 39, Rules of Court, specifically, that the notice of
auction sale should be published in a newspaper of general circulation.

d) The problem that Vini mortgaged the land by way of a chattel mortgage is
untenable. Land can only be the subject matter of a real estate mortgage and only an
absolute owner of real property may mortgage a parcel of land. (Article 2085 (2) Civil
Code). Hence, there can be no foreclosure.

But on the assumption that what was mortgaged by way of chattel mortgage was
the building on leased land, then the parties are treating the building as chattel. A
building that is not merely superimposed on the ground is an immovable property and
a chattel mortgage on said building is legally void but the parties cannot be allowed to
disavow their contract on account of estoppel by deed. However, if third parties are
involved such chattel mortgage is void and has no effect.

Notes:

Under the Civil Code, the mortgagor must be the absolute owner of the thing
mortgaged. Therefore, the land subject of the chattel mortgage cannot be foreclosed
not due to the claim of Vini; but it cannot be foreclose because Vini is not the owner
thereof.

ARTICLE 2085. The following requisites are essential to the contracts of


pledge and mortgage:

(1) That they be constituted to secure the fulfillment of a principal obligation;

(2) That the pledgor or mortgagor be the absolute owner of the thing pledged
or mortgaged;

(3) That the persons constituting the pledge or mortgage have the free disposal
of their property, and in the absence thereof, that they be legally authorized for the
purpose.

Third persons who are not parties to the principal obligation may secure the
latter by pledging or mortgaging their own property. guaranty
Chattel Mortgage; Immovables (2003)
X constructed a house on a lot which he was leasing from Y. Later, X executed a
chattel mortgage over said house in favor of Z as security for a loan obtained from the
latter. Still later, X acquired ownership of the land where his house was constructed,
after which he mortgaged both house and land in favor of a bank, which mortgage was
annotated on the Torrens Certificate of Title. When X failed to pay his loan to the
bank, the latter, being the highest bidder at the foreclosure sale, foreclosed the
mortgage and acquired X’s house and lot. Learning of the proceedings conducted
by the bank, Z is now demanding that the bank reconvey to him X’s house or pay
X’s loan to him plus interests. Is Z’s demand against the bank valid and
sustainable? Why? 5%

SUGGESTED ANSWER:
No, Z’s demand is not valid. A building is immovable or real property whether it
is erected by the owner of the land, by a usufructuary, or by a lessee. It may be treated
as a movable by the parties to chattel mortgage but such is binding only between them
and not on third parties (Evangelista v. Alto Surety Col. inc. 103 Phil. 401 [1958]).

In this case, since the bank is not a party to the chattel mortgage, it is not bound
by it, as far as the Bank is concerned, the chattel mortgage, does not exist. Moreover,
the chattel mortgage does not exist. Moreover, the chattel mortgage is void because it
was not registered. Assuming that it is valid, it does not bind the Bank because it was
not annotated on the title of the land mortgaged to the bank. Z cannot demand that the
Bank pay him the loan Z extended to X, because the Bank was not privy to such loan
transaction.

Notes:
In the case of Evangelista v. Alto Surety Col. inc., the Supreme Court held that
when the subject of the chattel mortgage is a real property, it does not bind a third
party. Therefore, the bank as the third party may foreclose the property mortgaged to
it although it was previously the subject of the chattel mortgage.

ANOTHER SUGGESTED ANSWER:


No, Z’s demand against the bank is not valid. His demand that the bank
reconvey to him X’s house presupposes that he has a real right over the house. All that
Z has is a personal right against X for damages for breach of the contract of loan.
The treatment of a house, even if built on rented land, as movable property is
void insofar as third persons, such as the bank, are concerned. On the other hand, the
Bank already had a real right over the house and lot when the mortgage was annotated
at the back of the Torrens title. The bank later became the owner in the foreclosure
sale. Z cannot ask the bank to pay for X’s loan plus interest. There is no privity of
contract between Z and the bank.
ALTERNATIVE ANSWER:
The answer hinges on whether or not the bank is an innocent mortgagee in good
faith or a mortgagee in bad faith. In the former case, Z’s demand is not valid. In the
latter case, Z’s demand against the bank is valid and sustainable.

Under the Torrens system of land registration, every person dealing with
registered land may rely on the correctness of the certificate of title and the law will
not in any way oblige to him to look behind or beyond the certificate in order to
determine the condition of the title. He is not bound by anything not annotated or
reflected in the certificate. If he proceeds to buy the land or accept it as a collateral
relying on the certificate, he is considered a buyer or a mortgagee in good faith. On
this ground, the Bank acquires a clean title to the land and the house.

However, a bank is not an ordinary mortgagee. Unlike private individuals, a


bank is expected to exercise greater care and prudence in its dealings. The
ascertainment of the condition of a property offered as collateral for a loan must be a
standard and indispensable part of its operation. The bank should have conducted
further inquiry regarding the house standing on the land considering that it was
already standing there before X acquired the title to the land. The bank cannot be
considered as a mortgagee in good faith. On this ground, Z’s demand against the Bank
is valid and sustainable.

Chattel Mortgage; Possession (1993)


A, about to leave the country on a foreign assignment, entrusted to B his brand
new car and its certificate of registration. Falsifying A's signature. B sold A's car to C
for P200,000.00. C then registered the car in his name. To complete the needed
amount, C borrowed P100.000.00 from the savings and loan association in his
office, constituting a chattel mortgage on the car. For failure of C to pay the amount
owed, the savings and loan association filed in the RTC a complaint for collection
with application for issuance of a writ of replevin to obtain possession of the vehicle
so that the chattel mortgage could be foreclosed. The RTC issued the writ of replevin.
The car was then seized from C and sold by the sheriff at public auction at which the
savings and loan association was the lone bidder. Accordingly, the car was sold to it.
A few days later, A arrived from his foreign assignment. Learning of what happened
to his car, A sought to recover possession and ownership of it from the savings and
loan association. Can A recover his car from the savings and loan association?
Explain your answer.

SUGGESTED ANSWER:
Under the prevailing rulings of the Supreme Court, A can recover the car from
the Savings and Loan Association provided he pays the price at which the
Association bought the car at a public auction. Under that doctrine, there has been an
unlawful deprivation by B of A of his car and, therefore, A can recover it from any
person in possession thereof. But since it was bought at a public auction in good
faith by the Savings and Loan Association, he must reimburse the Association at
the price for which the car was bought.

ALTERNATIVE ANSWER:
Yes, A can recover his car from the Savings and Loan Association. In a Chattel
Mortgage, the mortgagor must be the absolute owner of the thing mortgaged.
Furthermore, the person constituting the mortgage must have the free disposal of
the property, and in the absence thereof, must be legally authorized for the
purpose. In the case at bar, these essential requisites did not apply to the mortgagor B,
hence the Chattel Mortgage was not valid.

Notes:
Under the Civil Code, the owner of the movable who was unlawfully deprived
of its possession may recover such movable from the possessor thereof. In this case, A
was unlawfully deprived of possession of his car by the loan association. Therefore, A
can recover the car from the bank.

Chattel Mortgage; Preference of Creditors (1995)


Lawrence, a retired air force captain, decided to go into the air transport
business. He purchased an aircraft in cash except for an outstanding balance of
P500,000.00. He incurred an indebtedness of P300,000.00 for repairs with an
aircraft repair company. He also borrowed P1 Million from a bank for additional
capital and constituted a chattel mortgage on the aircraft to secure the loan.
While on a test flight the aircraft crashed causing physical injuries to a third
party who was awarded damages of P200,000.00. Lawrence's insurance claim for
damage to the aircraft was denied thus leaving him nothing else but the aircraft which
was then valued only at P1 Million. Lawrence was declared insolvent.
Assuming that the aircraft was sold for Pl Million, give the order of
preference of the creditors of Lawrence and distribute the amount of P1 Million.

SUGGESTED ANSWER:
Assuming that the aircraft was sold for P1 Million, there is no order of
preference. The P1 Million will all go to the bank as a chattel mortgagee because a
chattel mortgage under Art. 2241 (4) NCC defeats Art. 2244 (12) and (14}. Art. 2241
(3) and (5) are not applicable because the aircraft is no longer in the possession of the
creditor.

Notes:
Under Article 2241 of the New Civil Code, the unpaid price of the movable is
preferred, then the cost of chattel mortgage and then the expenses for the repair of the
aircraft.

ARTICLE 2241. With reference to specific movable property of the debtor, the
following claims or liens shall be preferred:

(1) Duties, taxes and fees due thereon to the State or any subdivision thereof;

(2) Claims arising from misappropriation, breach of trust, or malfeasance by


public officials committed in the performance of their duties, on the movables, money
or securities obtained by them;

(3) Claims for the unpaid price of movables sold, on said movables, so long as
they are in the possession of the debtor, up to the value of the same; and if the
movable has been resold by the debtor and the price is still unpaid, the lien may be
enforced on the price; this right is not lost by the immobilization of the thing by
destination, provided it has not lost its form, substance and identity; neither is the right
lost by the sale of the thing together with other property for a lump sum, when the
price thereof can be determined proportionally;

(4) Credits guaranteed with a pledge so long as the things pledged are in the
hands of the creditor, or those guaranteed by a chattel mortgage, upon the things
pledged or mortgaged, up to the value thereof;

(5) Credits for the making, repair, safekeeping or preservation of personal


property, on the movable thus made, repaired, kept or possessed;

(6) Claims for laborers’ wages, on the goods manufactured or the work done;
(7) For expenses of salvage, upon the goods salvaged;

(8) Credits between the landlord and the tenant, arising from the contract of
tenancy on shares, on the share of each in the fruits or harvest;

(9) Credits for transportation, upon the goods carried, for the price of the
contract and incidental expenses, until their delivery and for thirty days thereafter;

(10) Credits for lodging and supplies usually furnished to travellers by hotel
keepers, on the movables belonging to the guest as long as such movables are in the
hotel, but not for money loaned to the guests;

(11) Credits for seeds and expenses for cultivation and harvest advanced to the
debtor, upon the fruits harvested;

(12) Credits for rent for one year, upon the personal property of the lessee
existing on the immovable leased and on the fruits of the same, but not on money or
instruments of credit;

(13) Claims in favor of the depositor if the depositary has wrongfully sold the
thing deposited, upon the price of the sale.

In the foregoing cases, if the movables to which the lien or preference attaches
have been wrongfully taken, the creditor may demand them from any possessor,
within thirty days from the unlawful seizure. (1922a)

ARTICLE 2242. With reference to specific immovable property and real


rights of the debtor, the following claims, mortgages and liens shall be preferred,
and shall constitute an encumbrance on the immovable or real right:

(1) Taxes due upon the land or building;

(2) For the unpaid price of real property sold, upon the immovable sold;

(3) Claims of laborers, masons, mechanics and other workmen, as well as of


architects, engineers and contractors, engaged in the construction, reconstruction or
repair of buildings, canals or other works, upon said buildings, canals or other works;

(4) Claims of furnishers of materials used in the construction, reconstruction, or


repair of buildings, canals or other works, upon said buildings, canals or other works;

(5) Mortgage credits recorded in the Registry of Property, upon the real estate
mortgaged;
(6) Expenses for the preservation or improvement of real property when the law
authorizes reimbursement, upon the immovable preserved or improved;

(7) Credits annotated in the Registry of Property, in virtue of a judicial order, by


attachments or executions, upon the property affected, and only as to later credits;

(8) Claims of co-heirs for warranty in the partition of an immovable among


them, upon the real property thus divided;

(9) Claims of donors or real property for pecuniary charges or other conditions
imposed upon the donee, upon the immovable donated;

(10) Credits of insurers, upon the property insured, for the insurance premium
for two years.

EASEMENT

Easement vs. Usufruct (1995)


1. What is easement? Distinguish easement from usufruct.
2. Can there be (a) an easement over a usufruct? (b) a usufruct over an
easement? (c) an easement over another easement? Explain.

SUGGESTED ANSWER:
1. An EASEMENT or servitude is an encumbrance imposed upon an
immovable for the benefit of another immovable belonging to a different owner. (Art.
613, NCC) USUFRUCT gives a right to enjoy the property of another with the
obligation of preserving its form and substance, unless the title constituting it or the
law otherwise provides. (Art. 562, NCC).

Notes:
1) Under the Civil Code, an easement is an encumbrance imposed upon an
immovable for the benefit of another immovable belonging to a different owner, while
a usufruct is a right to enjoy the property of another with the obligation of
preserving its form and substance.
2) a.There is no easement over a usufruct because the former can be established
on immovable only, while the latter can be established on either movable or
immovable.
b. There is no usufruct over an easement because the former has no limit as to
its uses, while the latter is used for specific purpose only.

ARTICLE 613. An easement or servitude is an encumbrance imposed upon an


immovable for the benefit of another immovable belonging to a different owner.

The immovable in favor of which the easement is established is called the


dominant estate; that which is subject thereto, the servient estate.

ALTERNATIVE ANSWER:
Easement is an encumbrance imposed upon an immovable for the benefit of
another immovable belonging to a different owner in which case it is called real or
predial easement, or for the benefit of a community or group of persons in which case
it is known as a personal easement.
The distinctions between usufruct and easement are:
a) Usufruct includes all uses of the property and for all purposes, including jus
fruendi. Easement is limited to a specific use.
b) usufruct may be constituted on immovable or movable property. Easement
may be constituted only on an immovable property.
c) Easement is not extinguished by the death of the owner of the dominant
estate while usufruct is extinguished by the death of the usufructuary unless a
contrary intention appears.
d) An easement contemplates two (2) estates belonging to two (2) different
owners; a usufruct contemplates only one property (real or personal) whereby the
usufructuary uses and enjoys the property as well as its fruits, while another owns the
naked title during the period of the usufruct.
e) A usufruct may be alienated separately from the property to which it
attaches, while an easement cannot be alienated separately from the property to
which it attaches.

NOTE: It is recommended by the Committee that any two (2) distinctions should
be given full credit.
Notes:

Under Article 613 of the New Civil Code, easement or servitude is an


encumbrance imposed upon an immovable for the benefit of another immovable
belonging to a different owner.
The immovable for which the easement is established is called the dominant
estate; the immovable on which the easement is established is the servient estate.
The distinctions between an easement and a usufruct are: Easement involves 2
real properties, while usufruct may involve one real or one personal property;
easement is for a specific use of a real property, while usufruct allows all uses of real
or personal property. Easement is constituted on real property only, while usufruct
may be constituted on real or personal property.

SUGGESTED ANSWER:
2. (a) There can be no easement over a usufruct. Since an easement may be
constituted only on a corporeal immovable property, no easement may be
constituted on a usufruct which is not a corporeal right. Corporeal immovable
property
(b) There can be no usufruct over an easement. While a usufruct maybe created
over a right, such right must have an existence of its own independent of the
property. A servitude cannot be the object of a usufruct because it has no
existence independent of the property to which It attaches. Independent existence

ALTERNATIVE ANSWERS:
There cannot be a usufruct over an easement since an easement presupposes two
(2) tenements belonging to different persons and the right attaches to the tenement and
not to the owner. While a usufruct gives the usufructuary a right to use, right to enjoy,
right to the fruits, and right to possess, an easement gives only a limited use of the
servient estate.
However, a usufruct can be constituted over a property that has in its favor an
easement or one burdened with servitude. The usufructuary will exercise the easement
during the period of usufruct.
(c) There can be no easement over another easement for the same reason as in
(a). An easement, although it is a real right over an immovable, is not a corporeal
right. There is a Roman maxim which says that: There can be no servitude over
another servitude.

Easement; Effects; Discontinuous Easements; Permissive Use (2005)


Don was the owner of an agricultural land with no access to a public road. He
had been passing through the land of Ernie with the latter's acquiescence for over 20
years. Subsequently, Don subdivided his property into 20 residential lots and sold
them to different persons. Ernie blocked the pathway and refused to let the buyers
pass through his land.
a) Did Don acquire an easement of right of way? Explain. (2%)

ALTERNATIVE ANSWER:
No, Don did not acquire an easement of right of way. An easement of right of
way is discontinuous in nature — it is exercised only if a man passes over
somebody's land. Under Article 622 of the Civil Code, discontinuous easements,
whether apparent or not,. The Supreme Court, in Abellana, Sr. v. Court of Appeals
(G.R. No. 97039, April 24, 1992), ruled that an easement of right of way being
discontinuous may only be acquired by virtue of a title in nature is not acquirable by
prescription. Further, possession of the easement by Don is only permissive, tolerated
or with the acquiescence of Ernie. It is settled in the case of Cuaycong v. Benedicto
(G.R. No. 9989, March 13, 1918) that a permissive use of a road over the land of
another, no matter how long continued, will not create an easement of way by
prescription.

Notes:
ARTICLE 622. Continuous nonapparent easements, and discontinuous ones,
whether apparent or not, may be acquired only by virtue of a title.
In Abellana, Sr. v. Court of Appeals, it was held that an easement of right of way
is discontinuous and may be acquired by virtue of a title only; it is not acquirable by
prescription.

In Cuaycong v. Benedicto, it was held that a permissive use of a road over the
land of another will not create an easement of way by prescription no matter how long
it was used.

ALTERNATIVE ANSWER:
Yes, Don acquired an easement of right of way. An easement that is continuous
and apparent can be acquired by prescription and title. According to Professor
Tolentino, an easement of right of way may have a continuous nature if there is a
degree of regularity to indicate continuity of possession and that if coupled with an
apparent sign, such easement of way may be acquired by prescription.

ALTERNATIVE ANSWER:
Yes, Ernie could close the pathway on his land. Don has not acquired an
easement of right of way either by agreement or by judicial grant. Neither did the
buyers. Thus, establishment of a road or unlawful use of the land of Ernie would
constitute an invasion of possessory rights of the owner, which under Article 429 of
the Civil Code may be repelled or prevented. Ernie has the right to exclude any person
from the enjoyment and disposal of the land. This is an attribute of ownership that
Ernie enjoys.

Notes:
Under the Civil Code, the lawful owner of a thing has the right to exclude any
person from the enjoyment thereof. Therefore, Ernie could close the pathway because
it is his.

ARTICLE 429. The owner or lawful possessor of a thing has the right to exclude
any person from the enjoyment and disposal thereof. For this purpose, he may use
such force as may be reasonably necessary to repel or prevent an actual or
threatened unlawful physical invasion or usurpation of his property.

ALTERNATIVE ANSWER:
Yes, Ernie may close the pathway, subject however, to the rights of the lot
buyers. Since there is no access to the public road, this results in the creation of a legal
easement. The lot buyers have the right to demand that Ernie grant them a right of
way. In turn, they have the obligation to pay the value of the portion used as a right
of way, plus damages.

c) What are the rights of the lot buyers, if any? Explain. (2%)

SUGGESTED ANSWER:
Prior to the grant of an easement, the buyers of the dominant estate have no
other right than to compel grant of easement of right of way. Since the properties of
the buyers are surrounded by other immovables and has no adequate outlet to a
public highway and the isolation is not due to their acts, buyers may demand an
easement of a right of way provided proper indemnity is paid and the right of way
demanded is the shortest and least prejudicial to Ernie. (Villanueva v. Velasco, G.R.
No. 130845, November 27, 2000).

Notes:
In Villanueva v. Velasco, the Supreme Court held that the owner of immovable
which has no adequate outlet to a public highway may demand a right of way
provided he pays proper indemnity.

Easement; Nuisance; Abatement (2002)


Lauro owns an agricultural land planted mostly with fruit trees. Hernando
owns an adjacent land devoted to his piggery business, which is two (2) meters higher
in elevation. Although Hernando has constructed a waste disposal lagoon for his
piggery, it is inadequate to contain the waste water containing pig manure, and it often
overflows and inundates Lauro’s plantation. This has increased the acidity of the soil
in the plantation, causing the trees to wither and die. Lauro sues for damages caused
to his plantation. Hernando invokes his right to the benefit of a natural easement
in favor of his higher estate, which imposes upon the lower estate of Lauro the
obligation to receive the waters descending from the higher estate. Is Hernando
correct? (5%)

SUGGESTED ANSWER:
Hernando is wrong. It is true that Lauro’s land is burdened with the natural
easement to accept or receive the water which naturally and without interruption of
man descends from a higher estate to a lower estate. However, Hernando has
constructed a waste disposal lagoon for his piggery and it is this waste water that
flows downward to Lauro’s land. Hernando has, thus, interrupted the flow of water
and has created and is maintaining a nuisance. Under Act. 697 NCC, abatement of a
nuisance does not preclude recovery of damages by Lauro even for the past existence
of a nuisance. The claim for damages may also be premised in Art. 2191(4) NCC.

Notes:

Under the Civil Code, proprietors shall be responsible for damages caused by
emanations from tubes or canals. Therefore, Hernando shall be held liable for
damages because he increased the burden of the estate of Lauro because of the
overflowing of pig manure from the lagoons which he built.

ARTICLE 2191. Proprietors shall also be responsible for damages caused:

(1) By the explosion of machinery which has not been taken care of with due
diligence, and the inflammation of explosive substances which have not been kept in a
safe and adequate place;

(2) By excessive smoke, which may be harmful to persons or property;

(3) By the falling of trees situated at or near highways or lanes, if not caused by
force majeure;

(4) By emanations from tubes, canals, sewers or deposits of infectious matter,


constructed without precautions suitable to the place.
ARTICLE 637. Lower estates are obliged to receive the waters which
naturally and without the intervention of man descend from the higher estates; they
are also obliged to receive stones or earth which the waters carry with them.

ARTICLE 697. The abatement of a nuisance does not preclude the right of any
person injured to recover damages for its past existence.

ANOTHER ANSWER:
Hernando is not correct. Article 637 of the New Civil Code provides that the
owner of the higher estate cannot make works which will increase the burden on the
servient estate. (Remman Enterprises, Inc. v. CA, 330 SCRA 145 [2000]). The
owner of the higher estate may be compelled to pay damages to the owner of the
lower estate.

Notes:
Under the Civil Code, the owner of the higher estate cannot construct works
which will increase the burden on the servient estate. Therefore, Hernando shall be
held liable for damages because he increased the burden of the estate of Lauro because
of the overflowing of pig manure from the lagoons which he built.

Article 637 of the New Civil Code provides that the owner of the higher estate
cannot make works which will increase the burden on the servient estate.

In Remman Enterprises, Inc. v. CA, it was held that the owner the higher estate
may be compelled to pay damages to the owner of the lower estate.

ARTICLE 637. Lower estates are obliged to receive the waters which naturally
and without the intervention of man descend from the higher estates; they are also
obliged to receive stones or earth which the waters carry with them.

The owner of the lower estate cannot construct works which will impede this
easement; neither can the owner of the higher estate make works which will increase
the burden. (Reengineered)

Easements; Classification (1998)


Distinguish between:
1. Continuous and discontinuous easements; |2%]
2. Apparent and non-apparent easements; and [2%]
3. Positive and negative easements. [1%]
SUGGESTED ANSWER:
1. CONTINUOUS EASEMENTS are those the use of which is or may be
incessant, without the intervention of any act of man, while DISCONTINUOUS
EASEMENTS are those which are used at intervals and depend upon the acts of man.
(Art. 615, Civil Code)

Notes:
Continuous easements are easements the use of which is incessant, while
discontinuous easements are easements the use of which is at intervals.

SUGGESTED ANSWER:
2. APPARENT EASEMENTS are those which are made known and are
continually kept in view by external signs that reveal the use and enjoyment of the
same, while NONAPPARENT EASEMENTS are those which show no external
indication of their existence. (Art. 615, Civil Code)

Notes:
Apparent easements are easements that have external signs, while non-apparent
easements are easements that have no external signs.

SUGGESTED ANSWER:
3. POSITIVE EASEMENTS are those which impose upon the owner of the
servient estate the obligation of allowing something to be done or of doing it himself,
while NEGATIVE EASEMENTS are those which prohibit the owner of the servient
estate from doing something which he could lawfully do if the easement did not exist.
(Art. 615. Civil Code)

Notes:
Positive easements are easements that impose an obligation to do something
upon the owner of the servient estate, while negative easements are easements which
impose an obligation not to do something upon the owner of the servient estate.

Easements; Right of Way (1993)


Tomas Encarnacion's 3,000 square meter parcel of land, where he has a plant
nursery, is located just behind Aniceta Magsino's two hectare parcel land. To enable
Tomas to have access to the highway, Aniceta agreed to grant him a road right of
way a meter wide through which he could pass. Through the years Tomas' business
flourished which enabled him to buy another portion which enlarged the area of his
plant nursery. But he was still landlocked. He could not bring in and out of his plant
nursery a jeep or delivery panel much less a truck that he needed to transport his
seedlings. He now asked Aniceta to grant him a wider portion of her property, the
price of which he was willing to pay, to enable him to construct a road to have access
to his plant nursery. Aniceta refused claiming that she had already allowed him a
previous road right of way. Is Tomas entitled to the easement he now demands
from Aniceta?

SUGGESTED ANSWER:
Art. 651 of the Civil Code provides that the width of the easement must be
sufficient to meet the needs of the dominant estate, and may accordingly change
from time to time. It is the need of the dominant estate which determines the width of
the passage. These needs may vary from time to time. As Tomas' business grows, the
need for use of modern conveyances requires widening of the easement.

Notes:
Under the Civil Code, the width of the easement must be sufficient to meet the
needs of the dominant estate. Therefore, Anecita must give Tomas a wider portion of
her property.
ARTICLE 651. The width of the easement of right of way shall be that which is
sufficient for the needs of the dominant estate, and may accordingly be changed from
time to time.

ALTERNATIVE ANSWER:
The facts show that the need for a wider right of way arose from the increased
production owing to the acquisition by Tomas of an additional area. Under Art. 626 of
the Civil Code, the easement can be used only for the immovable originally
contemplated. Hence, the increase in width is justified and should have been granted.

ARTICLE 626. The owner of the dominant estate cannot use the easement
except for the benefit of the immovable originally contemplated. Neither can he
exercise the easement in any other manner than that previously established.

Easements; Right of Way (2000)


The coconut farm of Federico is surrounded by the lands of Romulo. Federico
seeks a right of way through a portion of the land of Romulo to bring his coconut
products to the market. He has chosen a point where he will pass through a housing
project of Romulo. The latter wants him to pass another way which is one
kilometer longer. Who should prevail? (5%)
SUGGESTED ANSWER:
Romulo will prevail. Under Article 650 of the New Civil Code, the easement of
right of way shall be established at the point least prejudicial to the servient estate and
where the distance from the dominant estate to a public highway is the shortest. In
case of conflict, the criterion of least prejudice prevails over the criterion of shortest
distance. Since the route chosen by Federico will prejudice the housing project of
Romulo, Romulo has the right to demand that Federico pass another way even though
it will be longer.

Notes:
Under the Civil Code, the right of way shall be established at the point least
prejudicial to the servient estate. Therefore, Federico cannot pass through the housing
project of Romulo.

ARTICLE 650. The easement of right of way shall be established at the point
least prejudicial to the servient estate, and, insofar as consistent with this rule, where
the distance from the dominant estate to a public highway may be the shortest.

Easements; Right of Way; Inseparability (2001)

Emma bought a parcel of land from Equitable-PCI Bank, which acquired the
same from Felisa, the original owner. Thereafter, Emma discovered that Felisa had
granted a right of way over the land in favor of the land of Georgina, which had no
outlet to a public highway, but the easement was not annotated when the servient
estate was registered under the Torrens system. Emma then filed a complaint for
cancellation of the right of way, on the ground that it had been extinguished by
such failure to annotate. How would you decide the controversy? (5%)

SUGGESTED ANSWER:
The complaint for cancellation of easement of right of way must fail. The failure
to annotate the easement upon the title of the servient estate is not among the grounds
for extinguishing an easement under Art. 631 of the Civil Code. Under Article 617,
easements are inseparable from the estate to which they actively or passively
belong. Once it attaches, it can only be extinguished under Art. 631, and they exist
even if they are not stated or annotated as an encumbrance on the Torrens title of
the servient estate. (II Tolentino 326, 1987 ed.)

Notes:
The complaint for cancellation of easement of right of way must fail because the
failure to annotate the easement upon the title of the servient estate is not among the
grounds for extinguishing an easement under Article 631 of the Civil Code.

ARTICLE 631. Easements are extinguished:

(1) By merger in the same person of the ownership of the dominant and servient
estates;

(2) By nonuser for ten years; with respect to discontinuous easements, this
period shall be computed from the day on which they ceased to be used; and, with
respect to continuous easements, from the day on which an act contrary to the same
took place;

(3) When either or both of the estates fall into such condition that the easement
cannot be used; but it shall revive if the subsequent condition of the estates or
either of them should again permit its use, unless when the use becomes possible,
sufficient time for prescription has elapsed, in accordance with the provisions of the
preceding number;

(4) By the expiration of the term or the fulfillment of the condition, if the
easement is temporary or conditional;

(5) By the renunciation of the owner of the dominant estate;

(6) By the redemption agreed upon between the owners of the dominant and
servient estates.

ALTERNATIVE ANSWER:
Under Section 44, PD No. 1529, every registered owner receiving a certificate of
title pursuant to a decree of registration, and every subsequent innocent purchaser for
value, shall hold the same free from all encumbrances except those noted on said
certificate. This rule, however, admits of exceptions.

Under Act 496, as amended by Act No. 2011, and Section 4, Act 3621, an
easement if not registered shall remain and shall be held to pass with the land until
cutoff or extinguished by the registration of the servient estate.

However, this provision has been suppressed in Section 44, PD No. 1529. In
other words, the registration of the servient estate did not operate to cut-off or
extinguish the right of way. Therefore, the complaint for the cancellation of the right
of way should be dismissed.

Easements; Right of Way; Requisites (1996)


David is the owner of the subdivision in Sta. Rosa, Laguna, without an access
to the highway. When he applied for a license to establish the subdivision, David
represented that he will purchase a rice field located between his land and the
highway, and develop it into an access road. But when the license was already
granted, he did not bother to buy the rice field, which remains unutilized until the
present. Instead, he chose to connect his subdivision with the neighboring subdivision
of Nestor, which has an access to the highway. Nestor allowed him to do this,
pending negotiations on the compensation to be paid. When they failed to arrive at an
agreement, Nestor built a wall across the road connecting with David's subdivision.
David filed a complaint in court, for the establishment of an easement of right of way
through the subdivision of Nestor which he claims to be the most adequate and
practical outlet to the highway.
1) What are the requisites for the establishment of a compulsory easement
of a right of way?

SUGGESTED ANSWER:
Art. 649, NCC. The owner, or any person who by virtue of a real right may
cultivate or use any immovable which is surrounded by other immovables
pertaining to other persons and without adequate outlet to a public highway, is
entitled to demand a right of way through the neighboring estates, after payment of the
property indemnity.

Should this easement be established in such a manner that its use may be
continuous for all the needs of the dominant estate, establishing a permanent passage,
the indemnity shall consist of the value of the land occupied and the amount of the
damage caused to the servient estate.

In case the right of way is limited to the necessary passage for the cultivation of
the estate surrounded by others and for the gathering of its crops through the servient
estate without a permanent way, the indemnity shall consist in the payment of the
damage cause by such encumbrance.

This easement is not compulsory if the isolation of the immovable is due to the
proprietor's own acts. (564a). The easement of right of way shall be established at the
point least prejudicial to the servient estate, and insofar as consistent with this rule,
where the distance from the dominant estate to a public highway may be the shortest
(Art. 650, NCC: Vda. de Baltazar v. CA. 245 SCRA 333}

Notes:
The requisites for a compulsory easement of right of way are:
(a) the dominant estate is surrounded by other immovables and is without an
adequate outlet to a public street or highway;
(b) proper indemnity must be paid;
(c) the isolation must not be due to the acts of the owner of the dominant estate;
and
(d) the right of way claimed is at a point least prejudicial to the servient estate.

ARTICLE 649. The owner, or any person who by virtue of a real right may
cultivate or use any immovable, which is surrounded by other immovables pertaining
to other persons and without adequate outlet to a public highway, is entitled to
demand a right of way through the neighboring estates, after payment of the
proper indemnity.

Should this easement be established in such a manner that its use may be
continuous for all the needs of the dominant estate, establishing a permanent
passage, the indemnity shall consist of the value of the land occupied and the
amount of the damage caused to the servient estate.

In case the right of way is limited to the necessary passage for the cultivation of
the estate surrounded by others and for the gathering of its crops through the servient
estate without a permanent way, the indemnity shall consist in the payment of the
damage caused by such encumbrance.

This easement is not compulsory if the isolation of the immovable is due to the
proprietor’s own acts.

ALTERNATIVE ANSWER:
The requisites for a compulsory easement of right of way are: (a) the dominant
estate is surrounded by other immovables and is without an adequate outlet to a public
street or highway; (b) proper indemnity must be paid; (c) the isolation must not be
due to the acts of the owner of the dominant estate; and (d) the right of way claimed is
at a point least prejudicial to the servient estate and, insofar as is consistent with
this rule, where the distance to the street or highway is shortest.
2) Is David entitled to a right of way in this case? Why or why not?

SUGGESTED ANSWER:
No, David is not entitled to the right of way being claimed. The isolation of his
subdivision was due to his own act or omission because he did not develop into an
access road the rice field which he was supposed to purchase according to his own
representation when he applied for a license to establish the subdivision (Floro us.
Llenado, 244 SCRA713).

Notes:
In Floro v. Llenado, the Supreme Court held that when the isolation of his estate
was due to his own act, the proprietor shall not be entitled to the right of way.
Therefore, David is not entitled to the right of way because the isolation of his estate
was due to his failure to buy the rice field as he promised when he applied for a
license for subdivision.

EJECTMENT

Ejectment Suit v. Cancellation of Title (2005)


In an ejectment case filed by Don against Cesar, can the latter ask for the
cancellation of Don's title considering that he (Cesar) is the rightful owner of the
lot? Explain. (2%)

SUGGESTED ANSWER:
Cesar cannot ask for the cancellation of Don's title even if he is the rightful
owner of the lot. In an action for ejectment, the only issue involved is one of
possession de facto, the purpose of which is merely to protect the owner from any
physical encroachment from without. The title of the land or its ownership is not
involved, for if a person is in actual possession thereof, he is entitled to be maintained
and respected in it even against the owner himself. (Garcia v. Anas, G.R. No. L-
20617, May 31, 1965)

Since the case filed by Don against Cesar is an ejectment case, the latter cannot
ask for the cancellation of Don's title. He has to file the proper action where the issue
of ownership over the property can be raised.

Notes:
In Garcia v. Anas, it was held that an action for ejectment can be maintained
even against the owner of the property because the issue involved is one of
possession and issue of ownership is not involved.

Ejectment Suit; Commodatum (2006)


Alberto and Janine migrated to the United States of America, leaving behind
their 4 children, one of whom is Manny. They own a duplex apartment and allowed
Manny to live in one of the units. While in the United States, Alberto died. His
widow and all his children executed an Extrajudicial Settlement of Alberto's estate
wherein the 2door apartment was assigned by all the children to their mother, Janine.
Subsequently, she sold the property to George. The latter required Manny to sign a
prepared Lease Contract so that he and his family could continue occupying the unit.
Manny refused to sign the contract alleging that his parents allowed him and his
family to continue occupying the premises.

If you were George's counsel, what legal steps will you take? Explain. (5%)

SUGGESTED ANSWER:
If I were George's counsel, I would first demand that Manny vacate the
apartment. If Manny refuses, I will file an ejectment suit. When Manny was allowed
by his parents to occupy the premises, without compensation, the contract of
commodatum was created. Upon the death of the father, the contract was extinguished
as it is a purely personal contract. As the new owner of the apartment George is
entitled to exercise his right of possession over the same.

Notes:
Under the Civil Code, commodatum is purely personal in character;
consequently, the death of the bailor or the bailee extinguishes the contract. Therefore,
Many shall vacate the duplex department because his contract with his parents which
allowed him to stay in that apartment was already extinguished when his father died.

ARTICLE 1939. Commodatum is purely personal in character. Consequently:

(1) The death of either the bailor or the bailee extinguishes the contract;

(2) The bailee can neither lend nor lease the object of the contract to a third
person. However, the members of the bailee’s household may make use of the thing
loaned, unless there is a stipulation to the contrary, or unless the nature of the thing
forbids such use.
Extra-Judicial Partition; Fraud (1990)
X was the owner of a 10,000 square meter property. X married Y and out of
their union, A, B and C were born. After the death of Y, X married Z and they begot
as children, D, E and F. After the death of X, the children of the first and second
marriages executed an extrajudicial partition of the aforestated property on May 1,
1970. D, E and F were given a one thousand square meter portion of the property.
They were minors at the time of the execution of the document. D was 17 years old, E
was 14 and F was 12; and they were made to believe by A, B and C that unless
they sign the document, they will not get any share. Z was not present then. In
January 1974, D, E and F filed an action in court to nullify the suit alleging they
discovered the fraud only in 1973.

(a) Can the minority of D, E and F be a basis to nullify the partition?


Explain your answer.

(b) How about fraud? Explain your

SUGGESTED ANSWER:
(a) Yes, minority can be a basis to nullify the partition because D, E and F were
not properly represented by their parents or guardians at the time they contracted
the extrajudicial partition. (Articles 1327. 1391, Civil Code).

(b) In the case of fraud, when through insidious words or machinations of one
party the other is induced to enter into the contract without which he would not have
agreed to, the action still prosper because under Art, 1391 of the Civil Code, in case of
fraud, the action for annulment may be brought within four years from the discovery
of the fraud.

Notes:
Under the Civil Code, minors cannot give consent to a contract. Therefore, the
extrajudicial partition can be annulled because D, E and F were minors when they
contracted the extrajudicial partition.

Under the Civil Code, in case of fraud, a contract may be nullified. Therefore,
the contract of extrajudicial partition can be nullified because of fraud perpetrated by
A, B and C who made D, E and F believed the they will not get any share unless they
will sign the contract of extrajudicial partition.

ARTICLE 1327. The following cannot give consent to a contract:


(1) Unemancipated minors;

(2) Insane or demented persons, and deaf-mutes who do not know how to write.

ARTICLE 1391. The action for annulment shall be brought within four years.

This period shall begin:

In cases of intimidation, violence or undue influence, from the time the defect
of the consent ceases.

In case of mistake or fraud, from the time of the discovery of the same.

And when the action refers to contracts entered into by minors or other
incapacitated persons, from the time the guardianship ceases

Hidden Treasure (1995)


Tim came into possession of an old map showing where a purported cache of
gold bullion was hidden. Without any authority from the government Tim
conducted a relentless search and finally found the treasure buried in a new river bed
formerly part of a parcel of land owned by spouses Tirso and Tessie. The old river
which used to cut through the land of spouses Ursula and Urbito changed its course
through natural causes. To whom shall the treasure belong? Explain.

SUGGESTED ANSWER:
The treasure was found in a property of public dominion, the new river bed.
Since Tim did not have authority from the government and, therefore, was a
trespasser, he is not entitled to the one-half share allotted to a finder of hidden
treasure. All of it will go to the State. In addition, under Art. 438 of the NCC in order
that the finder be entitled to the 1/2 share, the treasure must be found by chance, that
is by sheer luck. In this case, since Tim found the treasure not by chance but because
he relentlessly searched for it, he is not entitled to any share in the hidden treasure.
Notes:
Under the Civil Code, one-half of the hidden treasure belongs to the finder
provided he is not a trespasser. Therefore, Tim who is the finder of the hidden treasure
is not entitled to ½ because he was a trespasser since he did not ask authority from the
government which owns the river bed where the treasure was found.
For a finder to be entitled to ½ of treasure found on a land of public dominion,
ff req must be complied:
1. Authority from the government, hence he should not be a trespasser
2. Found by chance and not by relentless search

ALTERNATIVE ANSWER:
The law grants a one-half share to a finder of hidden treasure provided he is not
a trespasser and the finding is by chance. It is submitted that Tim is not a trespasser
despite his not getting authority from the government, because the new river bed
where he found the treasure is property for public use (Art. 420 NCC), to which the
public has legitimate access. The question, therefore, boils down to whether or not the
finding was by chance in view of the fact that Tim "conducted a relentless search"
before finding the treasure. The strict or literal view holds that deliberate or intentional
search precludes entitlement to the one-half share allotted by law to the finder since
the phrase "by chance" means "by accident", meaning an unexpected discovery. The
liberal view, however, would sustain Tim's right to the allocated share interpreting the
phrase in question as meaning "by a stroke of good fortune", which does not rule out
deliberate or intentional search. It is submitted that the liberal view should prevail
since in practical reality, hidden treasure is hardly ever found without conscious effort
to find it, and the strict view would tend to render the codal provision in question
illusory.

Notes:
Under the Civil Code, hidden treasure belongs to the owner of the land where it
is found. Therefore, the hidden treasure is owned by the government since it was
found in the river bed owned by the government.

ARTICLE 438. Hidden treasure belongs to the owner of the land, building, or
other property where it is found.

Nevertheless, when the hidden treasure is found on the property of another or of


the State or any of its subdivisions, one-half thereof shall be allowed to the finder if
the same is found by chance. If the finder is a trespasser, he shall not be entitled to
any share of the treasure.

If the things found are of interest to science or the arts, the State may acquire
them at their just price, which shall be divided in conformity with the rule stated.

Hidden Treasures (1997)


Marcelino, a treasure hunter as just a hobby, has found a map which appears to
indicate the location of hidden treasure. He has an idea of the land where the treasure
might possibly be found. Upon inquiry, Marcelino learns that the owner of the land,
Leopoldo, is a permanent resident of Canada, Nobody, however, could give him
Leopoldo's exact address. Ultimately, anyway, he enters the land and conducts a
search. He succeeds.

Leopoldo learning of Marcelino's "find", seeks to recover the treasure from


Marcelino but the latter is not willing to part with it. Failing to reach an agreement,
Leopoldo sues Marcelino for the recovery of the property. Marcelino contests the
action. How would you decide the case?

SUGGESTED ANSWER:
I would decide in favor of Marcelino since he is considered a finder by chance
of the hidden treasure, hence, he is entitled to one-half (1/2) of the hidden
treasure. While Marcelino may have had the intention to look for the hidden treasure,
still he is a finder by chance since it is enough that he tried to look for it. By chance in
the law does not mean sheer luck such that the finder should have no intention at all to
look for the treasure. By chance means good luck, implying that one who intentionally
looks for the treasure is embraced in the provision. The reason is that it is extremely
difficult to find hidden treasure without looking for it deliberately. Marcelino is not a
trespasser since there is no prohibition for him to enter the premises, hence, he is
entitled to half of the treasure.

ALTERNATIVE ANSWERS:
1. Marcelino did not find the treasure by chance because he had a map, he knew
the location of the hidden treasure and he intentionally looked for the treasure, hence,
he is not entitled to any part of the treasure.
2. Marcelino appears to be a trespasser and although there may be a question of
whether he found it by chance or not, as he has found the hidden treasure by means of
a treasure map, he will not be entitled to a finder's share. The hidden treasure shall
belong to the owner.
3. The main rule is that hidden treasure belongs to the owner of the land,
building or other property on which it is found. If it is found by chance by a third
person and he is not a trespasser, he is entitled to one-half (1/2). If he is a trespasser,
he loses everything.

Notes:
Under the Civil Code, hidden treasure belongs to the owner of the land, among
others. However, the finder is entitled to one-half if he is not a trespasser. In this case,
the finder is a trespasser. Thus, he is not entitled to one-half of the treasure.
Mortgage; Pactum Commissorium (1999)

(a) X borrowed money from Y and gave a piece of land as security by way
of mortgage. It was expressly agreed between the parties in the mortgage
contract that upon nonpayment of the debt on time by X, the mortgaged land
would already belong to Y. If X defaulted in paying, would Y now become the
owner of the mortgaged land? Why? (3%)

(b) Suppose in the preceding question, the agreement between X and Y was
that if X failed to pay the mortgage debt on time, the debt shall be paid with the
land mortgaged by X to Y. Would your answer be the same as in the preceding
question? Explain. (3%)

SUGGESTED ANSWER:

(a) No, Y would not become the owner of the land. The stipulation is in the
nature of pactum commissorium which is prohibited by law. The property should be
sold at public auction and the proceeds thereof applied to the indebtedness. Any
excess shall be given to the mortgagor.

SUGGESTED ANSWER:

(b) No, the answer would not be the same. This is a valid stipulation and does
not constitute pactum commissorium. In pactum commissorium, the acquisition is
automatic without need of any further action. In the instant problem another act is
required to be performed, namely, the conveyance of the property as payment
(dacion en pago).

Notes:
Under the Civil Code, there is dacion en pago when the obligation is paid by the
delivery of a thing which was not agreed upon.

Mortgage; No Pactum Commissorium


To secure a loan obtained from a rural bank, Purita assigned her leasehold
rights over a stall in the public market in favor of the bank. The deed of assignment
provides that in case of default in the payment of the loan, the bank shall have
the right to sell Purita's rights over the market stall as her attorney-in-fact, and
to apply the proceeds to the payment of the loan.
1) Was the assignment of leasehold rights a mortgage or a cession? Why?
(3%)
2) Assuming the assignment to be a mortgage, does the provision giving the
bank the power to sell Purita's rights constitute pactum commissorium or not?
Why? (2%)

SUGGESTED ANSWER:
1) The assignment was a mortgage, not a cession, of the leasehold rights. A
cession would have transferred ownership to the bank. However, the grant of authority
to the bank to sell the leasehold rights in case of default is proof that no such
ownership was transferred and that a mere encumbrance was constituted. There would
have been no need for such authority had there been a cession.

Notes:
Under the Civil Code, there is pactum commissorium when the property will
automatically be owned by the creditor upon default in payment of the obligation by
the debtor.

SUGGESTED ANSWER:
2) No, the clause in question is not a pactum commissorium. It is pactum
commissorium when default in the payment of the loan automatically vests ownership
of the encumbered property in the bank. In the problem given, the bank does not
automatically become owner of the property upon default of the mortgagor. The bank
has to sell the property and apply the proceeds to the indebtedness.
Notes:
When an agreement states that the ownership of a property is automatically
vested in the creditor when the debtor failed to pay his obligation, it is pactum
commissorium and is prohibited by law.

Mortgage; Right of Redemption vs. Equity of Redemption (1999)


Are the right of redemption and the equity of redemption given by law to a
mortgagor the same? Explain. (2%)

SUGGESTED ANSWER:
The equity of redemption is different from the right of redemption. EQUITY
OF REDEMPTION is the right of the mortgagor after judgment in a judicial
foreclosure to redeem the property by paying to the court the amount of the
judgment debt before the sale or confirmation of the sale.
On the other hand, RIGHT OF REDEMPTION is the right of the mortgagor
to redeem the property sold at an extra-judicial foreclosure by paying to the buyer
in the foreclosure sale the amount paid by the buyer within one year from such sale.

Notes:
Equity of redemption is a right to redeem the mortgaged property in a judicial
foreclosure; the mortgagor pays to the court the amount of the judgement debt before
the sale of the mortgaged property.

On the other hand, right of redemption is the right to redeem the mortgaged
property in extrajudicial foreclosure. The mortgagor pays to the buyer of the
property in public auction the amount he paid for the property. This right must be
exercised within one year from the sale.

Nuisance; Family House; Not Nuisance per se (2006)


A drug lord and his family reside in a small bungalow where they sell shabu and
other prohibited drugs. When the police found the illegal trade, they immediately
demolished the house because according to them, it was a nuisance per se that should
be abated. Can this demolition be sustained? Explain. (5%)

SUGGESTED ANSWER:
No, the demolition cannot be sustained. The house is not a nuisance per se or at
law as it is not an act, occupation, or structure which is a nuisance at all times and
under any circumstances, regardless of location or surroundings. A nuisance per
se is a nuisance in and of itself, without regard to circumstances [Tolentino, p. 695,
citing Wheeler v. River Falls Power Co., 215 Ala. 655, 111 So. 907].

Notes:
In Wheeler v. River Falls Power Co., it was held that a nuisance per se is a
nuisance in itself at all times without regard to circumstances. It can summarily be
abated. Thus, a house used for selling drugs is not a nuisance per se because it is not a
structure which is a nuisance itself without regard to circumstances.
Under the Civil Code, anything which endangers the safety of others, offends
senses, disregard decency or morality, obstructs free passage of any street or
public highways or impairs the use of property is nuisance.
ARTICLE 694. A nuisance is any act, omission, establishment, business,
condition of property, or anything else which:

(1) Injures or endangers the health or safety of others; or

(2) Annoys or offends the senses; or

(3) Shocks, defies or disregards decency or morality; or

(4) Obstructs or interferes with the free passage of any public highway or
street, or any body of water; or

(5) Hinders or impairs the use of property.

Nuisance; Public Nuisance vs. Private Nuisance (2005)


State with reason whether each of the following is a nuisance, and if so, give its
classification, whether public or private: Article 694 of the Civil Code defines
nuisance as any act, omission, establishment, business, condition or property, or
anything else which injures or endangers the health or safety of others, or annoys or
offends the senses, or shocks, defies or disregards decency or morality or obstructs or
interferes with the free passage of any public highway or street or any body of water
or hinders or impairs the use of property.

It is a public nuisance if it affects a community or neighborhood or any


considerable number of persons. It is a direct encroachment upon public rights or
property which results injuriously to the public. It is a private nuisance, if it affects
only a person or small number of persons. It violates only private rights.

a) A squatter's hut (1%)

If constructed on public streets or riverbeds, it is a public nuisance because


it obstructs the free use by the public of said places. (City of Manila v. Garcia,
G.R. No. L-26053, February 21,1967) If constructed on private land, it is a private
nuisance because it hinders or impairs the use of the property by the owner.

b) A swimming pool (1%)

Suggested Answer:
This is not a nuisance in the absence of any unusual condition or artificial
feature other than the mere water. In Hidalgo Enterprises v. Balandan (G.R. No. L-
3422, June 13, 1952), the Supreme Court ruled that a swimming pool is but a
duplication of nature — thus, could not be considered as a nuisance.

c) A house of prostitution (1%)

Irrespective of its location and how its business is conducted, it is a nuisance


since it defies, shocks and disregards decency and morality. It is a public nuisance
because of its injury to the public.

d) A noisy or dangerous factory in a private land (1%)

If the noise injuriously affects the health and comfort of ordinary people in the
vicinity to an unreasonable extent, it is a nuisance. It is a public nuisance because
there is a tendency to annoy the public. (Velasco v. Manila Electric Co., G.R. No.
L-18390, August 6, 1971)

e) Uncollected garbage (1%)

Suggested Answer:
It will become a nuisance if it substantially impairs the comfort and enjoyment
of the adjacent occupants. The annoyance and the smell must be substantial as to
interfere sensibly with the use and enjoyment by persons of ordinary sensibilities. It is
a public nuisance because of its injury to the public.

Notes:
Public nuisance is one which affects the community or a considerable number
of persons.
On the other hand, private nuisance is one which affects a small number of
persons.

ARTICLE 694. A nuisance is any act, omission, establishment, business,


condition of property, or anything else which:

(1) Injures or endangers the health or safety of others; or

(2) Annoys or offends the senses; or

(3) Shocks, defies or disregards decency or morality; or


(4) Obstructs or interferes with the free passage of any public highway or
street, or any body of water; or

(5) Hinders or impairs the use of property.

ARTICLE 695. Nuisance is either public or private. A public nuisance affects a


community or neighborhood or any considerable number of persons, although the
extent of the annoyance, danger or damage upon individuals may be unequal. A
private nuisance is one which does not affect the community, etc.

Notes:
A public nuisance affects a community or a considerable number of persons,
while a private nuisance affects a small number of persons and does not affect a
community.

CO-OWNERSHIP

Ownership; Co-Ownership (1992)


A, B and C are the co-owners in equal shares of a residential house and lot.
During their co-ownership, the following acts were respectively done by the co-
owners:
1) A undertook the repair of the foundation of the house, then tilting to one
side, to prevent the house from collapsing.
2) B and C mortgaged the house and lot to secure a loan.
3) B engaged a contractor to build a concrete fence all around the lot.
4) C built a beautiful grotto in the garden.
5) A and C sold the land to X for a very good price.

(a) Is A's sole decision to repair the foundation of the house binding on B and C?
May A require B and C to contribute their 2/3 share of the expense? Reasons. yes

(b) What is the legal effect of the mortgage contract executed by B and C?
Reasons. Affects only their portion

(c) Is B's sole decision to build the fence binding upon A and C? May B require
A and C to contribute their 2/ 3 share of the expense? Reasons. Embellishment or
improvement
(d) Is C's sole decision to build the grotto binding upon A and B? May C require
A and B to contribute their 2/ 3 share of the expense? Reasons. No.

(e) What are the legal effects of the contract of sale executed by A, C and X?
Reasons. Valid and enforceable as their share

SUGGESTED ANSWER:
(a) Yes. A's sole decision to repair the foundation is binding upon B and C. B
and C must contribute 2/3 of the expense. Each co-owner has the right to compel the
other co-owners to contribute to the expense of preservation of the thing (the house)
owned in common in proportion to their respective interests (Arts. 485 and 488, Civil
Code).
Notes:

ARTICLE 485. The share of the co-owners, in the benefits as well as in the
charges, shall be proportional to their respective interests. Any stipulation in a
contract to the contrary shall be void.

ARTICLE 488. Each co-owner shall have a right to compel the other co-owners
to contribute to the expenses of preservation of the thing or right owned in common
and to the taxes. Any one of the latter may exempt himself from this obligation by
renouncing so much of his undivided interest as may be equivalent to his share of the
expenses and taxes. No such waiver shall be made if it is prejudicial to the co-
ownership.

ARTICLE 489. Repairs for preservation may be made at the will of one of the
co-owners, but he must, if practicable, first notify his co-owners of the necessity for
such repairs. Expenses to improve or embellish the thing shall be decided upon by
a majority as determined in article 492.
Expenses for preservation of the thing owned in common shall be paid by all co-
owner in proportion to their interest in the thing.

SUGGESTED ANSWER:
(b) The mortgage shall not bind the 1/3 right and interest of A and shall be
deemed to cover only the rights and interests of B and C in the house and lot. The
mortgage shall be limited to the portion (2/3) which may be allotted to B and C in
the partition (Art. 493, Civil Code).
Notes:
Whoever mortgages the thing owned in common without the consent of the
other co-owners shall be solely liable therefor.

ARTICLE 493. Each co-owner shall have the full ownership of his part and of
the fruits and benefits pertaining thereto; he may therefore alienate, assign or
mortgage it; he may substitute another person in its enjoyment, except when
personal rights are involved.

But the effect of the alienation or the mortgage, with respect to the co-owners,
shall be limited to the portion which may be allotted to him in the division upon the
termination of the co-ownership. (Reengineered)

SUGGESTED ANSWER:
(c) B's sole decision to build the concrete fence is not binding upon A and C.
Expenses to improve the thing owned in common must be decided upon by a
majority of the co-owners who represent the controlling interest (Arts. 489 and
492. Civil Code).

Notes:
Under the Civil Code, expenses to improve the thing in common shall be
decided by a majority of the co-owners who represent the controlling interest.
Therefore, the decision to build a fence is not binding upon A and C who were not
consulted before the fence was constructed.
ARTICLE 489. Repairs for preservation may be made at the will of one of the
co-owners, but if practicable, he must first notify his co-owners of the necessity for
such repairs.
Expenses to improve or embellish the thing shall be decided upon by a
majority as determined in article 492.

ARTICLE 492. For the administration and better enjoyment of the thing owned
in common, the resolutions of the majority of the co-owners shall be binding.

There shall be no majority unless the resolution is approved by the co-owners


who represent the controlling interest in the object of the co-ownership.

Should there be no majority, or should the resolution of the majority be seriously


prejudicial to those interested in the property owned in common, the court, at the
instance of an interested party, shall order such measures as it may deem proper,
including the appointment of an administrator.
Whenever a part of the thing belongs exclusively to one of the co-owners, and
the remainder is owned in common, the preceding provisions shall apply only to the
part owned in common.

ARTICLE 493. Each co-owner shall have the full ownership of his part and of
the fruits and benefits pertaining thereto, and he may therefore alienate, assign or
mortgage it, and even substitute another person in its enjoyment, except when
personal rights are involved. But the effect of the alienation or the mortgage, with
respect to the co-owners, shall be limited to the portion which may be allotted to him
in the division upon the termination of the co-ownership.

Notes:
Under Article 493 of the Civil Code, each co-owner shall have the full
ownership of his part; he may assign, alienate or mortgage his part in the co-owned
property. Therefore, the contract to mortgage the co-owned property shall be decided
by all co-owners.

SUGGESTED ANSWER:
(d) C's sole decision to build the grotto is not binding upon A and B who cannot
be required to contribute to the expenses for the embellishment of the thing owned in
common if not decided upon by the majority of the co-owners who represent the
controlling interest (Arts. 489 and 492, Civil Code).

Notes:
Under the Civil Code, expenses to embellish the thing in common shall be
decided by a majority of the co-owners who represent the controlling interest.
Therefore, the decision to put up a grotto in the co-ownership shall be decided by a
majority of the co-owners who represent the controlling interest.

SUGGESTED ANSWER:
(e) The sale to X shall not bind the 1/3 share of B and shall be deemed to cover
only the 2/3 share of A and C in the land (Art. 493, Civil Code). B shall have the
right to redeem the 2/3 share sold to X by A and C since X is a third person (Art.
1620, Civil Code). Legal redemption

Notes:
Under the Civil Code, a co-owner has the right to redeem the co-ownership in
proportion to his share. Therefore, B shall have the right to redeem the 2/3 of the co-
ownership which is proportionate to his share therein.

ARTICLE 493. Each co-owner shall have the full ownership of his part and of
the fruits and benefits pertaining thereto, and he may therefore alienate, assign or
mortgage it, and even substitute another person in its enjoyment, except when
personal rights are involved. But the effect of the alienation or the mortgage, with
respect to the co-owners, shall be limited to the portion which may be allotted to him
in the division upon the termination of the co-ownership.

ARTICLE 1620. A co-owner of a thing may exercise the right of redemption in


case the shares of all the other co-owners or of any of them, are sold to a third
person. If the price of the alienation is grossly excessive, the redemptioner shall pay
only a reasonable one.

Should two or more co-owners desire to exercise the right of redemption, they
may only do so in proportion to the share they may respectively have in the thing
owned in common.

Ownership; Co-Ownership; Prescription (2000)


In 1955, Ramon and his sister Rosario inherited a parcel of land in Albay from
their parents. Since Rosario was gainfully employed in Manila, she left Ramon alone
to possess and cultivate the land. However, Ramon never shared the harvest with
Rosario and was even able to sell one-half of the land in 1985 by claiming to be the
sole heir of his parents. Having reached retirement age in 1990 Rosario returned to
the province and upon learning what had transpired, demanded that the remaining half
of the land be given to her as her share. Ramon opposed, asserting that he has
already acquired ownership of the land by prescription, and that Rosario is
barred by laches from demanding partition and reconveyance. Decide the
conflicting claims. (5%)

SUGGESTED ANSWER:
Ramon is wrong on both counts: prescription and laches. His possession as co-
owner did not give rise to acquisitive prescription. Possession by a co-owner is
deemed not adverse to the other co-owners but is, on the contrary, deemed
beneficial to them (Pongon v. CA, 166 SCRA 375). Ramon's possession will
become adverse only when he has repudiated the co-ownership and such
repudiation was made known to Rosario. Assuming that the sale in 1985 where
Ramon claimed he was the sole heir of his parents amounted to a repudiation of
the co-ownership, the prescriptive period began to run only from that time. Not more
than 30 years having lapsed since then, the claim of Rosario has not as yet prescribed.
The claim of laches is not also meritorious. Until the repudiation of the co-ownership
was made known to the other co-owners, no right has been violated for the said co-
owners to vindicate. Mere delay in vindicating the right, standing alone, does not
constitute laches.

Notes:
In Pongon v. CA, the Supreme Court held that possession of co-owners shall not
amount to prescription unless an act of repudiation was made known to other co-
owners in which case, the claim of ownership of the others will prescribe in 30 years.
Therefore, the claim of Rosario has not yet elapsed since the act of repudiation was
made known to her when Ramon sold the co-ownership in 1985.

ARTICLE 494. No co-owner shall be obliged to remain in the co-ownership.


Each co-owner may demand at any time the partition of the thing owned in
common, insofar as his share is concerned.

Nevertheless, an agreement to keep the thing undivided for a certain period of


time, not exceeding ten years, shall be valid. This term may be extended by a new
agreement.

A donor or testator may prohibit partition for a period which shall not exceed
twenty years.

Neither shall there be any partition when it is prohibited by law.

No prescription shall run in favor of a co-owner or co-heir against his co-


owners or co-heirs so long as he expressly or impliedly recognizes the co-
ownership.

ALTERNATIVE ANSWER:
Ramon has acquired the land by acquisitive prescription, and because of laches
on the part of Rosario. Ramon's possession of the land was adverse because he
asserted sole ownership thereof and never shared the harvest therefrom. His adverse
possession having been continuous and uninterrupted for more than 30 years, Ramon
has acquired the land by prescription. Rosario is also guilty of laches not having
asserted her right to the harvest for more than 40 years.
Notes:
When a co-owner asserted sole ownership of the thing in common and he never
shared the harvest therefrom to other co-owners, this act may be repudiation. That co-
owner may acquire the property by acquisitive prescription if his possession is
continuous, uninterrupted for more than 30 years from repudiation.

Ownership; Co-Ownership; Prescription (2002)


Senen and Peter are brothers. Senen migrated to Canada early while still a
teenager. Peter stayed in Bulacan to take care of their widowed mother and continued
to work on the Family farm even after her death. Returning to the country some thirty
years after he had left, Senen seeks a partition of the farm to get his share as the only
co-heir of Peter. Peter interposes his opposition, contending that acquisitive
prescription has already set in and that estoppel lies to bar the action for
partition, citing his continuous possession of the property for at least 10 years,
for almost 30 years in fact. It is undisputed that Peter has never openly claimed
sole ownership of the property. If he ever had the intention to do so, Senen was
completely ignorant of it. Will Senen’s action prosper? Explain. (5%).

SUGGESTED ANSWER:
Senen’s action will prosper. Article 494 of the New Civil Code provides that “no
prescription shall run in favor of a co-owner or co-heir against his co-owners or co-
heirs so long as he expressly or impliedly recognizes the coownership and does not
notified Senen of his having repudiated the same.

Notes:
Under Article 494 of the Civil Code, no prescription shall run in favor of a co-
owner against his co-owners for as long as he expressly or impliedly recognizes the
co-ownership. In this case, there was no act of Peter which would repudiate the co-
ownership. Therefore, the act of Sinen will prosper.

ARTICLE 494. No co-owner shall be obliged to remain in the co-ownership.


Each co-owner may demand at any time the partition of the thing owned in
common, insofar as his share is concerned.
Nevertheless, an agreement to keep the thing undivided for a certain period of
time, not exceeding ten years, shall be valid. This term may be extended by a new
agreement.

A donor or testator may prohibit partition for a period which shall not exceed
twenty years.

Neither shall there be any partition when it is prohibited by law.

No prescription shall run in favor of a co-owner or co-heir against his co-


owners or co-heirs so long as he expressly or impliedly recognizes the co-
ownership.

ALTERNATIVE ANSWER:
Senen’s action will prosper. This is a case of implied trust. (Art 1441, NCC) For
purposes of prescription under the concept of an owner (Art. 540, NCC). There is no
such concept here. Peter was a co-owner, he never claimed sole ownership of the
property. He is therefore estopped under Art. 1431, NCC.

Notes:

Under the Civil Code, only possession in the concept of owner can ripen into
ownership. In this case, Peter still recognizes co-ownership. His possession of the
land was not in the concept of owner for the entire land. Therefore, the action for
partition filed by Senen will propser.

ARTICLE 540. Only the possession acquired and enjoyed in the concept of
owner can serve as title for acquiring dominion.

ARTICLE 1441. Trusts are either express or implied. Express trusts are created
by the intention of the trustor or of the parties. Implied trusts come into being by
operation of law.

Ownership; Co-Ownership; Redemption (1993)


In 1937, A obtained a loan of P20,000.00 from the National City Bank of New
York, an American-owned bank doing business in the Philippines. To guarantee
payment of his obligation, A constituted a real estate mortgage on his 30 hectare
parcel of agricultural land. In 1939, before he could pay his obligation, A died
intestate leaving three children B, a son by a first marriage, and C and D, daughters by
a second marriage. In 1940, the bank foreclosed the mortgage for non-payment of the
principal obligation. As the only bidder at the extrajudicial foreclosure sale, the bank
bought the property and was later issued a certificate of sale. The war supervened in
1941 without the bank having been able to obtain actual possession of the property
which remained with A's three children who appropriated for themselves the income
from it. In 1948, B bought the property from the bank using the money he
received as back pay from the U. S. Government, and utilized the same in
agribusiness. In 1960, as B's business flourished, C and D sued B for partition and
accounting of the income of the property, claiming that as heirs of their father
they were co-owners thereof and offering to reimburse B for whatever he had
paid in purchasing the property from the bank. In brief, how will you answer the
complaint of C and D, if you were engaged by B as his counsel?

SUGGESTED ANSWER:
As counsel of B, I shall answer the complaint as follows: When B bought the
property, it was not by a right of redemption since the period therefore had
already expired. Hence, B bought the property in an independent unconditional
sale. C and D are not co-owners with B of the property. Therefore, the suit of C and D
cannot prosper.
Notes:
B did not redeem the land from the bank but bought it since the right of
redemption has already expired. The bank foreclosed and bought, as the highest
bidder, the foreclosed land in 1941. B bought the land from the bank in 1948. Under
the law, the mortgagor has the right to redeem the foreclosed land within one year
from the sale in extrajudicial foreclosure by paying the buyer of the amount he paid
for the property. It is clear that the one-year period of redemption has already expired
when B bought the land. Therefore, the claim of C and D is bereft of merit. C and D
and A are no longer co-owners since the property were already foreclosed and their
of redemption had already expired.

ALTERNATIVE ANSWER:
As counsel of B, I shall answer the complaint as follows: From the facts
described, it would appear that the Certificate of sale has not been registered. The
one-year period of redemption begins to run from registration. In this case, it has
not yet even commenced. Under the Rules of Court, the property may be released by
the Judgment debtor or his successor in interest. (Sec. 29, Rule 27). It has been held
that this includes a joint owner. (Ref. Magno vs.Ciola, 61 Phil. 80).
Notes:
Under the Civil Code, in extrajudicial foreclosure, right of redemption shall be
exercised within the period of one year from sale or registration of the certificate of
sale. In this case, the right of redemption has not yet commence to run since the
certificate of sale is not yet registered. Therefore, C and D can file an action for
partition.

Ownership; Co-Ownership; Redemption (2000)


Ambrosio died, leaving his three daughters, Belen, Rosario and Sylvia a
hacienda which was mortgaged to the Philippine National Bank. Due to the failure of
the daughters to pay the bank, the latter foreclosed the mortgage and the hacienda was
sold to it as the highest bidder. Six months later, Sylvia won the grand prize at the
lotto and used part of it to redeem the hacienda from the bank. Thereafter, she took
possession of the hacienda and refused to share its fruits with her sisters, contending
that it was owned exclusively by her, having bought it from the bank with her own
money. Is she correct or not? (3%) /no.

SUGGESTED ANSWER:
Sylvia is not correct. The 3 daughters are the co-owners of the hacienda being
the only heirs of Ambrosio. When the property was foreclosed, the right of
redemption belongs also to the 3 daughters. When Sylvia redeemed the entire
property before the lapse of the redemption period, she also exercised the right of
redemption of her co-owners on their behalf. As such she is holding the shares of
her two sisters in the property, and all the fruits corresponding thereto, in trust for
them. Redemption by one co-owner inures to the benefit of all (Adille v. CA.157
SCRA 455). Sylvia, however, is entitled to be reimbursed the shares of her two sisters
in the redemption price.

Notes:
In Adille v. CA, the Supreme Court ruled that right of redemption belongs to the
co-owner. In this case, Sylvia redeemed the mortgaged property six month from the
sale in extrajudicial foreclosure. Therefore, she is not correct in claiming that she
owns the land because the other co-owners have the right to redeemed too. However,
Sylvia has to be reimbursed in the amount proportionate to the shares of the other co-
owners.

Ownership; Co-Ownership; Redemption (2002)


Antonio, Bart, and Carlos are brothers. They purchased from their parents
specific portions of a parcel of land as evidenced by three separates deeds of sale,
each deed referring to a particular lot in metes and bounds. When the deeds were
presented for registration, the Register of Deeds could not issue separate certificates of
Title that it had to be issued, therefore, in the names of three brothers as co-owners of
the entire property. The situation has not changed up to now, but each of the brothers
has been receiving rentals exclusively from the lot actually purchased by him.
Antonio sells his lot to a third person, with notice to his brothers. To enable the buyer
to secure a new title in his name, the deed of sale was made to refer to undivided
interest in the property of the seller (Antonio), with the metes and bounds of the lot
sold being stated. Bart and Carlos reacted by signifying their exercise of their
right of redemption as co owners. Antonio in his behalf and in behalf of his
buyer, contends that they are no longer co-owners, although the title covering the
property has remained in their names as such. May Bart and Carlos still redeem
the lot sold by Antonio? Explain. (5%)
SUGGESTED ANSWER:
No, they may not redeem because there was no Coownership among Antonio,
Bart, and Carlos to start with. Their parents already partitioned the land in selling
separate portions to them. The situation is the same as in the case Si v. Court of
Appeals, (342 SCRA 653 [2000]). There is already a partition of the land, hence
co-ownership is extinguished and there is no redemption to speak of

Notes:
In Si v. Court of Appeals, it was held that there is no co-ownership to speak of
when the land is bought by brothers from their parents especially so when each of the
brothers has been receiving rentals exclusively from the lot actually purchased by
him. Thus, right of redemption does not exist among the brothers.

Possession (1998)
Using a falsified manager's check, Justine, as the buyer, was able to take
delivery of a second hand car which she had just bought from United Car Sales Inc.
The sale was registered with the Land Transportation Office. A week later, the seller
learned that the check had been dishonored, but by that time, Justine was nowhere to
be seen. It turned out that Justine had sold the car to Jerico, the present possessor who
knew nothing about the falsified check. In a suit by United Car Sales, Inc. against
Jerico for recovery of the car, plaintiff alleges it had been unlawfully deprived of its
property through fraud and should, consequently, be allowed to recover it without
having to reimburse the defendant for the price the latter had paid. Should the suit
prosper? [5%] innocent purchaser for value
SUGGESTED ANSWER:
The suit should prosper as to the recovery of the car. However, since Jerico was
not guilty of any fraud and appears to be an innocent purchaser for value, he should be
reimbursed for the price he paid. This is without prejudice to United Car Sales, Inc.
right of action against Justine. As between two innocent parties, the party causing
the injury should suffer the loss. Therefore, United Car Sales, Inc. should suffer
the loss.

Notes:

Under the Civil Code, one who has been unlawfully deprived of movable may
recover it from the person in possession of the same. Therefore, the United Car Sales,
Inc. may recover the car from Justine because it was unlawfully deprived of its car
through fraud. But Justine must be reimbursed because he appears to be an innocent
purchaser for value pursuant to the principle of unjust enrichment.

ARTICLE 559. The possession of movable property acquired in good faith is


equivalent to a title. Nevertheless, one who has lost any movable or has been
unlawfully deprived thereof, may recover it from the person in possession of the
same.

(Note: Go beyond a perfect score).

SUGGESTED ANSWER:
Yes, the suit will prosper because the criminal act of estafa should be deemed to
come within the meaning of unlawful deprivation under Art. 559, Civil Code, as
without it plaintiff would not have parted with the possession of its car.

SUGGESTED ANSWER:
No, the suit will not prosper. The sale is valid and Jerico is a buyer in good faith.

SUGGESTED ANSWER:
Under the law on Sales, when the thing sold is delivered by the seller to the
buyer without reservation of ownership, the ownership is transferred to the buyer.
Therefore in the suit of United Car Sales, Inc. against Jerico for the recovery of the
car, the plaintiff should not be allowed to recover the car without reimbursing the
defendant for the price that the latter paid. (EDCA Publishing and Distributing
Corp. vs. Santos, 184 SCRA 614, April 26, 1990)
IMMOVABLE PROPERTY

Property; Real vs. Personal Property (1995)


Salvador, a timber concessionaire, built on his lot a warehouse where he
processes and stores his timber for shipment. Adjoining the warehouse is a furniture
factory owned by NARRAMIX of which Salvador is a majority stockholder.
NARRAMIX leased space in the warehouse where it placed its furniture-making
machinery.
1. How would you classify the furniture-making machinery as property
under the Civil Code? Explain. movable, being a placed by a person not the
owner
2. Suppose the lease contract between Salvador and NARRAMIX stipulates
that at the end of the lease the machinery shall become the property of the lessor,
will your answer be the same? Explain. real, because such stipu transfers
ownership of machne to the owner of land. Hence, it is as if it ws the owner of the
land who placed the same

SUGGESTED ANSWER:
1. The furniture-making machinery is movable property because it was not
installed by the owner of the tenement. To become immovable under Art. 415 (5) of
the NCC, the machinery must be installed by the owner of the tenement.

Notes:
Under Article 415 of the Civil Code, if machineries are installed by the owner
of the tenement, they are immovable. In this case, the warehouse is owned by
Salvador; NARRAMIX place a machinery in that warehouse. Therefore, the
machinery is not immovable since it was not place by the owner of the warehouse.

ALTERNATIVE ANSWER:
It depends on the circumstances of the case. If the machinery was attached in a
fixed manner, in such a way that it cannot be separated from the tenement without
breaking the material or causing deterioration thereof, it is immovable property [Art.
415 (3), NCC]. However, if the machinery can be transported from place to place
without impairment of the tenement to which they were fixed, then it is movable
property. [Art. 416 (4), NCC]

SUGGESTED ANSWER:
2. It is immovable property. When there is a provision in the lease contract
making the lessor, at the end of the lease, owner of the machinery installed by the
lessee, the said machinery is considered to have been installed by the lessor through
the lessee who acted merely as his agent. Having been installed by the owner of the
tenement, the machinery became immovable under Art. 415 of the NCC. (Davao
Sawmill v. Castillo 61 Phil. 709)

Notes:
In Davao Sawmill v. Castillo, the Supreme Court held that machineries installed
by the owner of the tenement are immovable. Likewise, when it is installed by the
agent of the owner, it is also immovable. In this case, the machinery was installed by
the lessee but in the contract of lease, the lessor will become the owner of the
machinery at the end of the lease; thus, the lessee is considered as the agent of the
owner of the tenement. Therefore, the machinery is immovable.

Property; Real vs. Personal Property (1997)


Pedro is the registered owner of a parcel of land situated in Malolos, Bulacan. In
1973, he mortgaged the land to the Philippine National Bank (PNB) to secure a loan
of P100.000.00. For Pedro's failure to pay the loan, the PNB foreclosed on the
mortgage in 1980, and the land was sold at public auction to PNB for being the
highest bidder. PNB secured title thereto in 1987.

In the meanwhile, Pedro, who was still in possession of the land, constructed a
warehouse on the property. In 1988, the PNB sold the land to Pablo, the Deed of Sale
was amended in 1989 to include the warehouse. Pedro, claiming ownership of the
warehouse, files a complaint to annul the amended Deed of Sale before the Regional
Trial Court of Quezon City, where he resides, against both the PNB and Pablo. The
PNB filed a motion to dismiss the complaint for improper venue contending that
the warehouse is real property under Article 415(1) of the Civil Code and
therefore the action should have instead been filed in Malolos, Bulacan. Pedro
claims otherwise. The question arose as to whether the warehouse should be
considered as real or as personal property. If consulted, what would your legal
advice be?

SUGGESTED ANSWER:
The warehouse which is a construction adhered to the soil is an immovable by
nature under Art. 415 (1) and the proper venue of any case to recover ownership of the
same, which is what the purpose of the complaint to annul the amended Deed of Sale
amounts to, should be the place where the property is located, or the RTC of
Bulacan.

Notes:
Under Article 415 of the Civil Code, constructions adhered to the soil are
immovable. Therefore, the warehouse is immovable since it is adhered to the soil.

ADDITIONAL ANSWERS:
1. Buildings are always immovable property, and even in the instances where
the parties to a contract seem to have dealt with it separate and apart from the land on
which it stood in no wise does it change its character as immovable property. A
building is an immovable even if not erected by the owner of the land. The only
criterion is union or incorporation with the soil. (Ladera vs. Hodges (CA) 48 O.G.
4374) (Reyes and Puno, Outline of Philippine Civil Law, Vol. 2. p.7) immovable
by incorporation or by nature

Notes:
Under Article 415 of the Civil Code, buildings are always immovable.
Lands, buildings, road, and constructions adhered to the soil are real properties by
nature

Sower; Good Faith/ Bad Faith (2000)


Felix cultivated a parcel of land and planted it to sugar cane, believing it to be
his own. When the crop was eight months old, and harvestable after two more months,
a resurvey of the land showed that it really belonged to Fred. What are the options
available to Fred? (2%)

SUGGESTED ANSWER:
As to the pending crops planted by Felix in good faith, Fred has the option of
allowing Felix to continue the cultivation and to harvest the crops, or to continue the
cultivation and harvest the crops himself. In the latter option, however, Felix shall
have the right to a part of the expenses of cultivation and to a part of the net
harvest, both in proportion to the time of possession. (Art. 545 NCC),

Notes:
Under the Civil Code, if good faith ceases and there are natural or industrial
fruits, the planter in good faith shall have the right to be paid of a part of expenses of
cultivation; he has also a right to the net harvest.
Moreover, the owner of the thing may allow the possessor in good faith to
continue the cultivation and harvest the fruit to indemnify him of his expenses for
cultivation; if he refuses the concession, he shall lose the right to be indemnified for
expenses of cultivation.

ARTICLE 545. If at the time the good faith ceases, there should be any natural
or industrial fruits, the possessor shall have a right to a part of the expenses of
cultivation, and to a part of the net harvest, both in proportion to the time of the
possession.

The charges shall be divided on the same basis by the two possessors.

The owner of the thing may, should he so desire, give the possessor in good faith
the right to finish the cultivation and gathering of the growing fruits, as an indemnity
for his part of the expenses of cultivation and the net proceeds; the possessor in good
faith who for any reason whatever should refuse to accept this concession, shall lose
the right to be indemnified in any other manner.

ARTICLE 546. Necessary expenses shall be refunded to every possessor; but


only the possessor in good faith may retain the thing until he has been reimbursed
therefor.

Useful expenses shall be refunded only to the possessor in good faith with the
same right of retention, the person who has defeated him in the possession having the
option of refunding the amount of the expenses or of paying the increase in value
which the thing may have acquired by reason thereof.

ALTERNATIVE ANSWER:
Since sugarcane is not a perennial crop. Felix is considered a sower in good
faith. Being so, Art. 448 applies. The options available to Fred are: (a) to appropriate
the crop after paying Felix the indemnity under Art. 546, or (b) to require Felix to pay
rent.

Usufruct (1997)
On 1 January 1980, Minerva, the owner of a building, granted Petronila a
usufruct over the property until 01 June 1998 when Manuel, a son of Petronila, would
have reached his 30th birthday. Manuel, however, died on 1 June 1990 when he was
only 26 years old. Minerva notified Petronila that the usufruct had been extinguished
by the death of Manuel and demanded that the latter vacate the premises and deliver
the same to the former. Petronila refused to vacate the place on the ground that
the usufruct in her favor would expire only on 1 June 1998 when Manuel would
have reached his 30th birthday and that the death of Manuel before his 30th
birthday did not extinguish the usufruct. Whose contention should be accepted?

SUGGESTED ANSWER:
Petronila's contention is correct. Under Article 606 of the Civil Code, a usufruct
granted for the time that may elapse before a third person reaches a certain age shall
subsist for the number of years specified even if the third person should die unless
there is an express stipulation in the contract that states otherwise. In the case at bar,
there is no express stipulation that the consideration for the usufruct is the existence of
Petronila's son. Thus, the general rule and not the exception should apply in this case.

Notes:
Under the Civil Code, a usufruct granted for a certain period until a third person
may reach a certain age shall continue to subsist for the number of years specified
even if the third person dies; but if the usufruct is expressly granted for the
consideration of the existence of such third person, it may cease when the third parson
dies.

ARTICLE 606. A usufruct granted for the time that may elapse before a third
person attains a certain age, shall subsist for the number of years specified, even if the
third person should die before the period expires, unless such usufruct has been
expressly granted only in consideration of the existence of such person.

ALTERNATIVE ANSWER:
This is a usufruct which is clearly intended for the benefit of Manuel until he
reaches 30 yrs. of age with Petronila serving only as a conduit, holding the property in
trust for his benefit. The death of Manuel at the age of 26 therefore, terminated the
usufruct.

LAND TRANSFER & DEEDS

ACQUISITION OF LANDS

Acquisition of Lands; Citizenship Requirement (2003)


In 1970, the spouses Juan and Juana de la Cruz, then Filipinos, bought the
parcel of unregistered land in the Philippines on which they built a house which
became their residence. In 1986, they migrated to Canada and became Canadian
citizens. Thereafter, in 1990, they applied, opposed by the Republic, for the
registration of the aforesaid land in their names. Should the application of the
spouses de la Cruz be granted over the Republic’s opposition? Why? 5%

SUGGESTED ANSWER:
Yes, the application should be granted. As a rule, the Constitution prohibits
aliens from owning private lands in the Philippines. This rule, however, does not
apply to the spouses Juan and Juana de la Cruz because at the time they acquired
ownership over the land, albeit imperfect, they were still Filipino citizens. The
application for registration is a mere confirmation of the imperfect title which the
spouses have already acquired before they became Canadian citizens. (Republic v.
CA, 235 SCRA 567 [1994]).

Notes:
In Republic v. CA, the Supreme Court held that former Filipinos who acquired
lands while still Filipinos shall be allowed to register their lands. Therefore, the
application of spouses de la Cruz shall be granted over the opposition of the Republic
because they purchased such land when they were still Filipinos.

NOTICE OF ADVERSE CLAIM

Adverse Claims; Notice of Levy (1998)


Section 70 of Presidential Decree No. 1529, concerning adverse claims on
registered land, provides a 30-day period of effectivity of an adverse claim, counted
from the date of its registration. Suppose a notice of adverse claim based upon a
contract to sell was registered on March 1, 1997 at the instance of the BUYER, but
on June 1, 1997, or after the lapse of the 30-day period, a notice of levy on execution
in favor of a JUDGMENT CREDITOR was also registered to enforce a final
judgment for money against the registered owner. Then, on June 15, 1997 there
having been no formal cancellation of his notice of adverse claim, the BUYER pays to
the seller-owner the agreed purchase price in full and registers the corresponding deed
of sale. Because the annotation of the notice of levy is carried over to the new title in
his name, the BUYER brings an action against the JUDGMENT CREDITOR to
cancel such annotation, but the latter claims that his lien is superior because it
was annotated after the adverse claim of the BUYER had ipso facto ceased to be
effective. Will the suit prosper? [5%]
SUGGESTED ANSWER:
The suit will prosper. While an adverse claim duly annotated at the back of a
title under Section 7O of P.D. 1529 is good only for 30 days, cancellation thereof is
still necessary to render it ineffective, otherwise, the inscription thereof will
remain annotated as a lien on the property. While the life of adverse claim is 30
days under P.D. 1529, it continues to be effective until it is cancelled by formal
petition filed with the Register of Deeds. The cancellation of the notice of levy is
justified under Section 108 of P.D. 1529 considering that the levy on execution cannot
be enforced against the buyer whose adverse claim against the registered owner was
recorded ahead of the notice of levy on execution.

Notes:
Under the law, notice of adverse claim is effective until cancelled; although the
law states that a notice of adverse claim is effective for 30 days, an action for
cancellation for the notice of adverse claim is necessary. Therefore, the action to
cancel the notice of adverse claim will prosper because it is superior to notice of levy
on execution.

Under Section 70 of P.D. 1529, While an adverse claim duly annotated at the
back of a title is good only for 30 days, cancellation thereof is still necessary to
render it ineffective, otherwise, the inscription thereof will remain annotated as a lien
on the property. It continues to be effective until it is cancelled by formal petition filed
with the Register of Deeds.
In this case, the levy on execution cannot be enforced against the buyer whose
adverse claim against the registered owner was recorded ahead of the notice of levy on
execution.
Under P.D. 1529, An adverse claim serves as a warning to third parties dealing
with said property that someone is claiming an interest on the same or a better right
than the registered owner thereof.

NOTICE OF LIS PENDENS

Annotation of Lis Pendens; When Proper (2001)


Mario sold his house and lot to Carmen for P1 million payable in five (5) equal
annual installments. The sale was registered and title was issued in Carmen's name.
Carmen failed to pay the last three installments and Mario filed an action for
collection, damages and attorneys fees against her. Upon filing of the complaint,
he caused a notice of lis pendens to be annotated on Carmen's title. Is the notice
of lis pendens proper or not? Why? (5%)

SUGGESTED ANSWER:
The notice of lis pendens is not proper for the reason that the case filed by Mario
against Carmen is only for collection, damages, and attorney's fees.

Annotation of a lis pendens can only be done in cases involving recovery of


possession of real property, or to quiet title or to remove cloud thereon, or for
partition or any other proceeding affecting title to the land or the use or occupation
thereof. The action filed by Mario does not fall on anyone of these.

Notes:
Under the law, notice of lis pendens is proper when the action affects title to real
property. Therefore, the notice of lis pendens is not proper because the action filed by
Mario is not for the recovery of possession of real property but for collection of sum
of money, among others.

Foreshore Lands (2000)


Regina has been leasing foreshore land from the Bureau of Fisheries and
Aquatic Resources for the past 15 years. Recently, she learned that Jorge was able to
obtain a free patent from the Bureau of Agriculture, covering the same land, on the
basis of a certification by the District Forester that the same is already "alienable
and disposable". Moreover, Jorge had already registered the patent with the Register
of Deeds of the province, and he was issued an Original Certificate of Title for the
same. Regina filed an action for annulment of Jorge's title on the ground that it
was obtained fraudulently. Will the action prosper? (2%)

SUGGESTED ANSWER:
An action for the annulment of Jorge's Original Certificate of Title will prosper
on the following grounds:

(1) Under Chapter IX of C .A, No. 141, otherwise known as the Public Land
Act, foreshore lands are disposable for residential, commercial, industrial, or similar
productive purposes, and only by lease when not needed by the government for public
service.
(2) If the land is suited or actually used for fishpond or aquaculture purposes, it
comes under the Jurisdiction of the Bureau of Fisheries and Aquatic Resources
(BFAR) and can only be acquired by lease. (P.D. 705)

(3) Free Patent is a mode of concession under Section 41, Chapter VII of the
Public Land Act, which is applicable only for agricultural lands.

(4) The certificate of the district forester that the land is already "alienable and
disposable" simply means that the land is no longer needed for forest purposes, but the
Bureau of Lands could no longer dispose of it by free patent because it is already
covered by a lease contract between BFAR and Regina. That contract must be
respected.

(5) The free patent of Jorge is highly irregular and void ab initio, not only
because the Bureau has no statutory authority to issue a free patent over a
foreshore area, but also because of the false statements made in his sworn application
that he has occupied and cultivated the land since July 4, 1945, as required by the free
patent law. Under Section 91 of the Public Land Act, any patent concession or title
obtained thru false representation is void ab initio. In cases of this nature, it is the
government that shall institute annulment proceedings considering that the suit carries
with it a prayer for the reversion of the land to the state. However, Regina is a party in
interest and the case will prosper because she has a lease contract for the same land
with the government.

Notes:
Under the law, foreshore land is disposable for lease only. Moreover, a free
patent is applicable in agricultural land only. Therefore, the free patent of Jorge on the
foreshore land is void.

In this case, Jorge was granted free patent of foreshore land which is not allowed
because only agricultural land is subject to a free patent. Thus, the free patent of Jorge
is highly irregular, so it is void ab initio.

FRAUD AND MIRROR DOCTRINE IN ACQUIRING LANDS

Forgery; Innocent Purchaser; Holder in Bad Faith (2005)


Rod, the owner of an FX taxi, found in his vehicle an envelope containing TCT
No. 65432 over a lot registered in Cesar's name. Posing as Cesar, Rod forged
Cesar's signature on a Deed of Sale in Rod's favor. Rod registered the said
document with the Register of Deeds, and obtained a new title in his name. After
a year, he sold the lot to Don, a buyer in good faith and for value, who also registered
the lot in his name.
a) Did Rod acquire title to the land? Explain. (2%)

SUGGESTED ANSWER:
No, Rod did not acquire title to the land. The inscription in the registry, to be
effective, must be made in good faith. The defense of indefeasibility of a Torrens Title
does not extend to a transferee who takes the certificate of title with notice of a flaw.
A holder in bad faith of a certificate of title is not entitled to the protection of the law,
for the law cannot be used as a shield for frauds. (Samonte v. Court of Appeals,
G.R. No. 104223, July 12, 2001)

In the case at bar, Rod only forged Cesar's signature on the Deed of Sale. It is
very apparent that there was bad faith on the part of Rod from the very beginning. As
such, he is not entitled to the protection of the Land Registration Act.

Notes:
In Samonte v. Court of Appeals, the Supreme Court held that a certificate of title
to the land is void when it is obtained through fraud. Therefore, Rod did not acquire
title to the land because he obtained it by fraud by forging the signature of Cesar in the
deed of sale.

b) Discuss the rights of Don, if any, over the property. (2%)

SUGGESTED ANSWER:
It is a well-known rule in this jurisdiction that persons dealing with registered
land have the legal right to rely on the face of the Torrens Certificate of Title and to
dispense with the need to inquire further, except when the party concerned has actual
knowledge of facts and circumstances that would impel a reasonably cautious man to
make such inquiry. (Naawan Community Rural Bank v. Court of Appeals, G.R.
No. 128573, January 13, 2003)

In the given problem, the property was already registered in the name of Rod
when he bought the same from the latter. Thus, Don could be considered as a buyer in
good faith and for value. However, since Rod did not actually sell any property to
him, Don has no right to retain ownership over the property. He has only the right to
recover the purchase price plus damages.
Notes:
In Naawan Community Rural Bank v. CA, the Supreme Court held that persons
dealing with registered land have the legal right to rely on the face of the Torrens
Certificate of Title; they do not need to inquire further. Therefore, Don could be
considered as buyer in good faith and for value.

Notes:
Under the mirror doctrine, Don has the right to retain the property because when
he bought the said property, it was already under the name of Rod. Don cannot be
expected to know deeper than what the title says.

Forgery; Innocent Purchaser; Mirror Principle (1991)


Bruce is the registered owner of a parcel of land with a building thereon and is
in peaceful possession thereof. He pays the real estate taxes and collects the rentals
therefrom. Later, Catalino, the only brother of Bruce, filed a petition where he,
misrepresenting to be the attorney-in-fact of Bruce and falsely alleging that the
certificate of title was lost, succeeded in obtaining a second owner's duplicate
copy of the title and then had the same transferred in his name through a
simulated deed of sale in his favor. Catalino then mortgaged the property to
Desiderio who had the mortgage annotated on the title. Upon learning of the
fraudulent transaction, Bruce filed a complaint against Catalino and Desiderio to
have the title of Catalino and the mortgage in favor of Desiderio declared null
and void. Will the complaint prosper, or will the title of Catalino and the
mortgage to Desiderio be sustained?

SUGGESTED ANSWER:
The complaint for the annulment of Catalino's Title will prosper. In the first
place, the second owner's copy of the title secured by him from the Land
Registration Court is void ab initio, the owner's copy thereof having never been lost,
let alone the fact that said second owner's copy of the title was fraudulently procured
and improvidently issued by the Court.

In the second place, the Transfer Certificate of Title procured by Catalino is


equally null and void, it having been issued on the basis of a simulated or forged
Deed of Sale. A forged deed is an absolute nullity and conveys no title. The
mortgage in favor of Desiderio is likewise null and void because the mortgagor is not
the owner of the mortgaged property.
While it may be true that under the "Mirror Principle" of the Torrens System of
Land Registration, a buyer or mortgagee has the right to rely on what appears on the
Certificate of Title, and in the absence of anything to excite suspicion, is under no
obligation to look beyond the certificate and investigate the mortgagor's title, this rule
does not find application in the case at hand because here. Catalino's title suffers from
two fatal infirmities, namely:
a) The fact that it emanated from a forged deed of a simulated sale;
b) The fact that it was derived from a fraudulently procured or improvidently
issued second owner's copy, the real owner's copy being still intact and in the
possession of the true owner, Bruce. The mortgage to Desiderio should be cancelled
without prejudice to his right to go after Catalino and/or the government for
compensation from the assurance fund.

Notes:
Under the law, a forged deed of sale is void; thus, it does not confer right.
Therefore, the title of Catalino to the land shall be declared void since he
obtained it by forging the signature of Bruce in the deed of sale.

However, the law provides that a person who deals with a parcel of land covered
with title shall have the right to depend upon it unless there is a fact indicating that he
should conduct an inquiry.

Fraud; Procurement of Patent; Effect (2000)


In 1979, Nestor applied for and was granted a Free Patent over a parcel of
agricultural land with an area of 30 hectares, located in General Santos City. He
presented the Free Patent to the Register of Deeds, and he was issued a corresponding
Original Certificate of Title (OCT) No. 375, Subsequently, Nestor sold the land to
Eddie. The deed of sale was submitted to the Register of Deeds and on the basis
thereof, OCT No, 375 was cancelled and Transfer Certificate of Title (TCT) No. 4576
was issued in the name of Eddie. In 1986, the Director of Lands filed a complaint for
annulment of OCT No, 375 and TCT No. 4576 on the ground that Nestor obtained the
Free Patent through fraud. Eddie filed a motion to dismiss on the ground that he
was an innocent purchaser for value and in good faith and as such, he has
acquired a title to the property which is valid, unassailable and indefeasible.
Decide the motion. (5%)

SUGGESTED ANSWER:
The motion of Eddie to dismiss the complaint for annulment of O.C.T. No. 375
and T.C.T. No. 4576 should be denied for the following reasons:
1) Eddie cannot claim protection as an innocent purchaser for value nor can he
interpose the defense of indefeasibility of his title, because his TCT is rooted on a
void title. Under the Public Land Act, statements of material facts in the applications
for public land must be under oath. Moreover, any false statement therein shall be
a cause for the cancellation of the concession. The patent issued to Nestor in this
case is void ab initio because it was obtained by fraud and it covers 30 hectares which
is far beyond the maximum of 24 hectares provided by the free patent law.

2) The government can seek annulment of the original and transfer certificates of
title and the reversion of the land to the state. Eddie's defense is untenable. The
protection afforded by the Torrens System to an innocent purchaser for value can be
availed of only if the land has been titled thru judicial proceedings where the issue of
fraud becomes academic after the lapse of one (1) year from the issuance of the
decree of registration. In public land grants, the action of the government to annul a
title fraudulently obtained does not prescribe such action and will not be barred by
the transfer of the title to an innocent purchaser for value.

Notes:

The motion of Eddie to dismiss the complaint for annulment of O.C.T. No. 375
and T.C.T. No. 4576 should be denied for the following reasons:

1) Under the Public Land Act, if title to the land is obtained through fraud, it is a
cause for cancellation of the concession. Moreover, the right of the government to
cancel the grant does not prescribe. Therefore, Eddie cannot claim a buyer in good
faith because his title to the land rooted from fraud perpetrated by Nestor who is the
transferor of the land to him, so his title to the land can be cancelled.

2) . In public land grants, the action of the government to annul a title


fraudulently obtained does not prescribe such action and will not be barred by the
transfer of the title to an innocent purchaser for value. Moreover, the doctrine of
innocent purchaser for value can be availed of only if the land was titled through
judicial proceedings because here, the issue of fraud becomes academic after the
lapse of 1 year from the issuance of decree of registration. Therefore, Eddie cannot
avail of the defense of innocent purchaser for value.
Homestead Patents; Void Sale (1999)
In 1950, the Bureau of Lands issued a Homestead patent to A. Three years later,
A sold the homestead to B. A died in 1990, and his heirs filed an action to recover the
homestead from B on the ground that its sale by their father to the latter is void under
Section 118 of the Public Land Law. B contends, however, that the heirs of A
cannot recover the homestead from him anymore because their action has
prescribed and that furthermore, A was in pari delicto. Decide. (5%)

SUGGESTED ANSWER:
The sale of the land by A to B 3 years after issuance of the homestead patent,
being in violation of Section 118 of the Public Land Act, is void from its inception.
The action filed by the heirs of B to declare the nullity or inexistence of the contract
and to recover the land should be given due course.

B's defense of prescription is untenable because an action which seeks to declare


the nullity or inexistence of A contract does not prescribe. (Article 1410; Banaga vs.
Soler, 2 8CRA 765) On the other hand, B's defense of pari delicto is equally
untenable. While as a rule, parties who are in pari delicto have no recourse against
each other on the principle that a transgressor cannot profit from his own wrongdoing,
such rule does not apply to violations of Section 118 of the Public Land Act because
of the underlying public policy in the said Act "to conserve the land which a
homesteader has acquired by gratuitous grant from the government for himself
and his family". In keeping with this policy, it has been held that one who purchases
a homestead within the five-year prohibitory period can only recover the price which
he has paid by filing a claim against the estate of the deceased seller (Labrador vs.
Delos Santos 66 Phil. 579) under the principle that no one shall enrich himself at the
expense of another. Applying the pari delicto rule to violation of Section 118 of the
Public Land Act, the Court of Appeals has ruled that "the homesteader suffers the loss
of the fruits realized by the vendee who in turn forfeits the improvement that he has
introduced into the land." (Obot vs. Sandadiuas, 69 OG, April 35, 1966}

Notes:
Under the Civil Code, an action or defense to declare the nullity of a void or
inexistence contract does not prescribe. In this case, the homestead was sold within
the prohibited period, thus, void. Therefore, the heirs of the seller can still recover the
homestead from the buyer.
In Banaga v. Soler, the Supreme Court ruled that the principle of in pari delicto
is not available as a defense against the nullity of a void contract because of public
policy to conserve the land for homesteader who has acquired it by gratuitous grant
from the government for himself and his family. Therefore, the contract of sale
involving a homestead patent within the prohibited period shall be declared void.

ARTICLE 1410. The action or defense for the declaration of the inexistence of a
contract does not prescribe.

FIRST ALTERNATIVE ANSWER:


The action to declare the nullity of the sale did not prescribe (Art. 1410}, such
sale being one expressly prohibited and declared void by the Public Lands Act [Art.
1409, par. (7)]. The prohibition of the law is clearly for the protection of the heirs of A
such that their recovering the property would enhance the public policy regarding
ownership of lands acquired by homestead patent (Art. 1416). The defense of pari
delicto is not applicable either, since the law itself allows the homesteader to reacquire
the land even if it has been sold.

SECOND ALTERNATIVE ANSWER:


Prescription does not arise with respect to actions to declare a void contract a
nullity (Article 1410). Neither is the doctrine of pari delicto applicable because of
public policy. The law is designed for the protection of the plaintiff so as to enhance
the public policy of the Public Land Act to give land to the landless.

If the heirs are not allowed to recover, it could be on the ground of laches
inasmuch as 40 years had elapsed and the owner had not brought any action against B
especially if the latter had improved the land. It would be detrimental to B if the
plaintiff is allowed to recover.

Notes:
Under the Public Land Act, the policy of the state is to give the land or the
homestead to the landless; thus, the right of the seller to declare the contract of sale as
void does not prescribe. Moreover, the defense of pari delicto to uphold the contract
of sale cannot be availed of.

Innocent Purchaser for Value (2001)


Cesar bought a residential condominium unit from High Rise Co. and paid
the price in full. He moved into the unit, but somehow he was not given the
Condominium Certificate of Title covering the property. Unknown to him, High
Rise Co. subsequently mortgaged the entire condominium building to Metrobank as
security for a loan of P500 million. High Rise Co. failed to pay the loan and the bank
foreclosed the mortgage. At the foreclosure sale, the bank acquired the building, being
the highest bidder. When Cesar learned about this, he filed an action to annul the
foreclosure sale insofar as his unit was concerned. The bank put up the defense
that it relied on the condominium certificates of title presented by High Rise Co.,
which were clean. Hence, it was a mortgagee and buyer in good faith. Is this
defense tenable or not? Why? (5%.) function of the banks are imbued with
public interest hence they must exercise diligence of more than a good farther of
a family, doctrine does not apply to banks

SUGGESTED ANSWER:
Metrobank's defense is untenable. As a rule, an innocent purchaser for value
acquires a good and a clean title to the property. However, it is settled that one who
closes his eyes to facts that should put a reasonable man on guard is not an innocent
purchaser for value. In the present problem the bank is expected, as a matter of
standard operating procedure, to have conducted an ocular inspection of the promises
before granting any loan. Apparently, Metrobank did not follow this procedure.
Otherwise, it should have discovered that the condominium unit in question was
occupied by Cesar and that fact should have led it to make further inquiry. Under the
circumstances, Metrobank cannot be considered a mortgagee and buyer in good faith.

Notes:
In a long line of cases, the Supreme Court held that in dealing with lands, banks
shall exercise utmost diligence. In this case, Metrobank failed to observe utmost
diligence by failing to conduct an inquiry as to why Cesar is occupying one of the
rooms of the condominium. Therefore, Metrobank cannot claim that it is a buyer in
good faith because it failed to follow the procedure laid down for banks.

(Innocent purchaser for value means a buyer in good faith).

MIRROR PRINCIPLE

Mirror Principle (1990)


In 1950's, the Government acquired a big landed estate in Central Luzon from
the registered owner for subdivision into small farms and redistribution of bonafide
occupants, F was a former lessee of a parcel of land, five hectares in area. After
completion of the resurvey and subdivision, F applied to buy the said land in
accordance with the guidelines of the implementing agency. Upon full payment of the
price in 1957, the corresponding deed of absolute sale was executed in his favor and
was registered, and in 1961, a new title was issued in his name. In 1963, F sold the
said land to X; and in 1965 X sold it to Y, new titles were successively issued in the
names of the said purchasers.

In 1977, C filed an action to annul the deeds of sale to F, X and Y and their
titles, on the ground that he (C) had been in actual physical possession of the land, and
that the sale to F and the subsequent sales should be set aside on the ground of fraud.
Upon motion of defendants, the trial court dismissed the complaint, upholding
their defenses of their being innocent purchasers for value, prescription and
laches. Plaintiff appealed.

(a) Is the said appeal meritorious? Explain your answer


(b) Suppose the government agency concerned joined C in filing the said
action against the defendants, would that change the result of the litigation?
Explain.

SUGGESTED ANSWER:
(a) The appeal is not meritorious. The trial court ruled correctly in granting
defendant's motion to dismiss for the following reasons: 1. While there is the
possibility that F, a former lessee of the land was aware of the fact that C was the bona
fide occupant thereof and for this reason his transfer certificate of title may be
vulnerable, the transfer of the same land and the issuance of new TCTs to X and Y
who are innocent purchasers for value render the latter's titles indefeasible. A person
dealing with registered land may safely rely on the correctness of the certificate
of title and the law will not in any way oblige him to go behind the certificate to
determine the condition of the property in search for any hidden defect or
inchoate right which may later invalidate or diminish the right to the land. This
is the mirror principle of the Torrens System of land registration.
1. The action to annul the sale was instituted in 1977 or more than (10) years
from the date of execution thereof in 1957, hence, it has long prescribed.
The action to annul the sale was instituted in 1977 or more than (10) years from the
date of execution thereof in 1957, hence, it has long prescribed.
2. Under Sec 45 of Act 496, “the entry of a certificate of title shall be
regarded as an agreement running with the land, and binding upon the applicant and
all his successors in title that the land shall be and always remain registered land. A
title under Act 496 is indefeasible and to preserve that character, the title is cleansed
anew with every transfer for value (De Jesus v City of Manila; 29 Phil. 73; Laperal
v City of Manila, 62 Phil 313; Penullar v PNB 120 S 111).

Notes:
A person dealing with registered land may safely rely on the correctness of the
certificate of title; ;the law will not in any way oblige him to go behind the certificate
to determine the condition of the property in search for any hidden defect which may
later invalidate right to the land. This is the mirror principle of the Torrens System of
land registration.

Go beyond a perfect score:


If your answer is mirror doctrine, it is well and good. If your answer is based on
fraud in acquiring the land in which prescription is not a defense, it is acceptable.

SUGGESTED ANSWER:
(b) Even if the government joins C, this will not alter the outcome of the case so
much because of estoppel as an express provision in Sec 45 of Act 496 and Sec 31 of
PD 1529 that a decree of registration and the certificate of title issued in
pursuance thereof “shall be conclusive upon and against all persons, including
the national government and all branches thereof, whether mentioned by name in
the application or not.”

Notes:
Under PD 1529, a decree of registration and the certificate of title issued in
pursuance thereof shall be conclusive upon and against all persons, including the
national government and all branches thereof. Therefore, even the national
government cannot nullify the sale of the land which was a registered land.

Mirror Principle; Forgery; Innocent Purchaser (1999)


The spouses X and Y mortgaged a piece of registered land to A, delivering as
well the OCT to the latter, but they continued to possess and cultivate the land, giving
1/2 of each harvest to A in partial payment of their loan to the latter. A, however,
without the knowledge of X and Y, forged a deed of sale of the aforesaid land in
favor of himself, got a TCT in his name, and then sold the land to B, who bought
the land relying on A's title, and who thereafter also got a TCT in his name. It was
only then that the spouses X and Y learned that their land had been titled in B's name.
May said spouses file an action for reconveyance of the land in question against
B? Reason. (5%)
SUGGESTED ANSWER:
The action of X and Y against B for reconveyance of the land will not prosper
because B has acquired a clean title to the property being an innocent purchaser for
value.
A forged deed is an absolute nullity and conveys no title. The fact that the forged
deed was registered and a certificate of title was issued in his name, did not operate to
vest upon an ownership over the property of X and Y. The registration of the forged
deed will not cure the infirmity.

However, once the title to the land is registered in the name of the forger and
title to the land thereafter falls into the hands of an innocent purchaser for value,
the latter acquires a clean title thereto. A buyer of a registered land is not
required to explore beyond what the record in the registry indicates on its face in quest
for any hidden defect or inchoate right which may subsequently defeat his right
thereto. This is the "mirror principle' of the Torrens system which makes it possible
for a forged deed to be the root of a good title.
Besides, it appears that spouses X and Y are guilty of contributory negligence
when they delivered this OCT to the mortgagee without annotating the mortgage
thereon. Between them and the innocent purchaser for value, they should bear the loss.

Notes:
Under the mirror doctrine, an innocent purchaser for value who relies on the face
of the title of the land is deemed to be a purchaser in good faith. He is not required to
go beyond the title of the land to determine the defect thereof. Thus, his title to the
land cannot be annulled.

ALTERNATIVE ANSWER:
If the buyer B, who relied on the seller A's title, was not aware of the adverse
possession of the land by the spouses X and Y, then the latter cannot recover the
property from B. B has in his favor the presumption of good faith which can only be
overthrown by adequate proof of bad faith. However, nobody buys land without
seeing the property, hence, B could not have been unaware of such adverse
possession. If after learning of such possession, B simply closed his eyes and did
nothing about it, then the suit for reconveyance will prosper as the buyer's bad
faith will have become evident.

NOTICE OF LIS PENDENS


Notice of Lis Pendens (1995)
Rommel was issued a certificate of title over a parcel of land in Quezon City.
One year later Rachelle, the legitimate owner of the land, discovered the fraudulent
registration obtained by Rommel. She filed a complaint against Rommel for
reconveyance and caused the annotation of a notice of lis pendens on the certificate of
title issued to Rommel. Rommel now invokes the indefeasibility of his title
considering that one year has already elapsed from its issuance. He also seeks the
cancellation of the notice of Lis pendens. May the court cancel the notice of lis
pendens even before final judgment is rendered? Explain.

SUGGESTED ANSWER:
A Notice of Lis Pendens may be canceled even before final Judgment upon
proper showing that the notice is for the purpose of molesting or harassing the
adverse party or that the notice of lis pendens is not necessary to protect the right
of the party who caused it to be registered. (Section 77, P.D. No. 1529) In this case,
it is given that Rachelle is the legitimate owner of the land in question. It can be said,
therefore, that when she filed her notice of lis pendens her purpose was to protect her
interest in the land and not just to molest Rommel. It is necessary to record the Lis
pendens to protect her interest because if she did not do it, there is a possibility that
the land will fall into the hands of an innocent purchaser for value and in that
event, the court loses control over the land making any favorable judgment
thereon moot and academic. For these reasons, the notice of lis pendens may not be
canceled.

Notes:
Under P.D. No. 1529, a Notice of Lis Pendens may be cancelled even before
final Judgment upon proper showing that the notice is for the purpose of molesting or
harassing the adverse party or that the notice of lis pendens is not necessary to
protect the right of the party who caused it to be registered. In this case, the notice of
lis pendens is to protect the right of the owner, Rachelle. Therefore, the notice of lis
pendens may not be cancelled.

The purpose of the notice of lis pendens is to protect the owner of the land from
any transfer or alienation of such land to an innocent purchaser for value.

Notice of Lis Pendens; Transferee Pendente Lite (2002)


Sancho and Pacifico are co-owners of a parcel of land. Sancho sold the property
to Bart. Pacifico sued Sancho and Bart for annulment of the sale and reconveyance of
the property based on the fact that the sale included his one-half pro-indiviso share.
Pacifico had a notice of lis pendens annotated on the title covering the property and
ordered the cancellation of the notice of lis pendens. The notice of lis pendens could
not be cancelled immediately because the title over the property was with a bank to
which the property had been mortgaged by Bart. Pacifico appealed the case. While the
appeal was pending and with the notice of lis pendens still uncancelled, Bart sold the
property to Carlos, who immediately caused the cancellation of the notice of lis
pendens, as well as the issuance of a new title in his name. Is Carlos (a) a purchaser
in good faith, or (b) a transferee pendente lite? If your answer is (a), how can the
right of Pacifico as co-owner be protected? Explain. (5%)

SUGGESTED ANSWER:
A. Carlos is a buyer in bad faith. The notice of lis pendens was still annotated
at the back of the title at the time he bought the land from Bart. The uncancelled
notice of lis pendens operates as constructive notice of its contents as well as interests,
legal or equitable, included therein. All persons are charged with the knowledge of
what it contains. In an earlier case, it was held that a notice of an adverse claim
remains effective and binding notwithstanding the lapse of the 30 days from its
inscription in the registry. This ruling is even more applicable in a lis pendens.

Carlos is a transferee pendente lite insofar as Sancho’s share in the co-


ownership in the land is concerned because the land was transferred to him during
the pendency of the appeal.

B. Pacifico can protect his right as a co-owner by pursuing his appeal; asking
the Court of Appeals to order the re-annotation of the lis pendens on the title of
Carlos; and by invoking his right of redemption of Bart’s share under Articles 1620
of the New Civil Code.

Notes:
Under the Civil Code, right of redemption may be exercised by a co-owner
upon a thing subject of the co-ownership which was sold to a third person by another
co-owner.

ARTICLE 1620. A co-owner of a thing may exercise the right of redemption in


case the shares of all the other co-owners or of any of them, are sold to a third person.
If the price of the alienation is grossly excessive, the redemptioner shall pay only a
reasonable one.
Should two or more co-owners desire to exercise the right of redemption, they
may only do so in proportion to the share they may respectively have in the thing
owned in common.

ALTERNATIVE ANSWER:
A. Carlos is a purchaser in good faith. A possessor in good faith has been
defined as “one who is unaware that there exists a flaw which invalidates his
acquisition of the thing” (Art. 526, NCC). Good faith consists in the possessor’s
belief that the person from whom he received the thing was the owner of the same and
could convey his title. In the case [at bar], in question, while Carlos bought the subject
property from Bart while a notice of lis pendens was still annotated thereon, there was
also an existing court order canceling the same. Hence, Carlos cannot be considered as
being “aware of a flaw which invalidates [their] the acquisition of the thing” since the
alleged flaw, the notice of lis pendens, was already being ordered cancelled at the time
of the purchase. On this ground alone, Carlos can already be considered a buyer in
good faith. (Po Lam v. Court of Appeals, 347 SCRA 86, [2000]).

B. To protect his right over the subject property, Pacifico should have timely
filed an action for reconveyance and reinstated the notice of lis pendens.
Notes:

ARTICLE 526. He is deemed a possessor in good faith who is not aware that
there exists in his title or mode of acquisition any flaw which invalidates it.

He is deemed a possessor in bad faith who possesses in any case contrary to the
foregoing.

Mistake upon a doubtful or difficult question of law may be the basis of good faith.

PRESCRIPTION AND LACHES

Prescription & Laches; Elements of Laches (2000)


In an action brought to collect a sum of money based on a surety agreement, the
defense of laches was raised as the claim was filed more than seven years from the
maturity of the obligation. However, the action was brought within the ten-year
prescriptive period provided by law wherein actions based on written contracts can be
instituted.
a) Will the defense prosper? Reason. (3%)
b) What are the essential elements of laches? (2%)
SUGGESTED ANSWER:
No, the defense will not prosper. The problem did not give facts from which
laches may be inferred. Mere delay in filing an action, standing alone, does not
constitute laches (Agra v. PNB. 309 SCRA 509).

Notes:
In the case of Agra v. PNB, the Supreme Court held that mere delay in the filing
of an action does not constitute laches. Therefore, a mere delay in the filing of an
action for a period of more than 7 years does not constitute laches.

SUGGESTED ANSWER:
b) The four basic elements of laches are; (1) conduct on the part of the defendant
or of one under whom he claims, giving rise to the situation of which complainant
seeks a remedy; (2) delay in asserting the complainant's rights, the complainant having
had knowledge or notice of the defendant's conduct and having been afforded an
opportunity to institute suit; (3) lack of knowledge on the part of the defendant that the
complainant would assert the right on which he bases his suit; and (4) injury or
prejudice to the defendant in the event relief is accorded to the complainant, or the suit
is not held to be barred.

1. Conduct on the part of defendant which made claimant seek remedy


2. Delaying in asserting claimant’s right
3. Lack of knowledge on the part of defendant that claimant would assert
said right
4. Injury to defendant in the event relief is accorder to complainant

Notes:
The elements of laches are:
(1) Knowledge of the claim;
(2) Unreasonable delay in filing the claim; and
(3) Prejudice to the defendant caused by the delay

(These elements are based on American jurisprudence).

Re-engineering:

1) You know that you have a claim;


2) You delay your claim;
3) The delay caused prejudice to the defendant.

Prescription & Laches; Indefeasibility Rule of Torrens Title (2002)


Way back in 1948, Winda’s husband sold in favor of Verde Sports Center
Corp. (Verde) a 10-hectare property belonging to their conjugal partnership. The
sale was made without Winda’s knowledge, much less consent. In 1950, Winda
learned of the sale, when she discovered the deed of sale among the documents in her
husband’s vault after his demise. Soon after, she noticed that the construction of the
sports complex had started. Upon completion of the construction in 1952, she tried but
failed to get free membership privileges in Verde. Winda now files a suit against
Verde for the annulment of the sale on the ground that she did not consent to the sale.
In answer, Verde contends that, in accordance with the Spanish Civil Code which was
then in force, the sale in 1948 of the property did not need her concurrence. Verde
contends that in any case the action has prescribed or is barred by laches. Winda
rejoins that her Torrens title covering the property is indefeasible, and
imprescriptible.

A. Define or explain the term “laches”. (2%)


B. Decide the case, stating your reasons for your decision. (3%)

SUGGESTED ANSWER:
A. LACHES means failure or neglect, for an unreasonable and unexplained
length of time, to do what, by exercising due diligence, could or should have been
done earlier. It is negligence or omission to assert a right within a reasonable time.
(De Vera v. CA, 305 SCRA 624 [1999])

Notes:
In the case of De Vera v. CA, the Supreme court held that laches is negligence
or omission to assert a right within a reasonable time.

B. While Article 1413 of the Spanish Civil Code did not require the consent of
the wife for the validity of the sale, an alienation by the husband in fraud of the wife
is void as held in Uy Coque v. Navas, 45 Phil. 430 (1923). Assuming that the
alienation in 1948 was in fraud of Winda and, therefore, makes the sale to Verde void,
the action to set aside the sale, nonetheless, is already barred by prescription and
laches. More than 52 years have already elapsed from her discovery of the sale in
1950.
Notes:
In Uy Coque v. Navas, the Supreme Court held that alienation of conjugal
property needs the consent of both the husband and the wife; when one of them
alienates it without the consent of the other, it is void and a void contract is
imprescriptible.

it was held that alienation of the conjugal property by the husband in fraud of
the wife is void and when the contract is void, it is imprescriptible.

In this case, therefore, Winda can file a case for the declaration of the sale as
void.

ALTERNATIVE ANSWER:
B. Winda’s claim that her Torrens Title covering the property is indefeasible and
imprescriptible [does not hold water] is not tenable. The rule of indefeasibility of a
Torrens Title means that after one year from the date of issue of the decree of
registration or if the land has fallen into the hands of an innocent purchaser for value,
the title becomes incontestable and incontrovertible.
IMPRESCRIPTIBILITY, on the other hand, means that no title to the land in
derogation of that of the registered owner may be acquired by adverse possession or
acquisitive prescription or that the registered owner does not lose by extinctive
prescription his right to recover ownership and possession of the land.
The action in this case is for annulment of the sale executed by the husband over a
conjugal partnership property covered by a Torrens Title. Action on contracts are
subject to prescription.

Notes:
Under the rule of indefeasibility of a torrens title, the title cannot be annulled
after the lapse of 1 year from the issuance of decree of registration.

ACQUISITIVE PRESCRIPTION

Prescription (1990)
In 1960, an unregistered parcel of land was mortgaged by owner O to M, a
family friend, as collateral for a loan. O acted through his attorney-in-fact, son S, who
was duly authorized by way of a special power of attorney, wherein O declared that he
was the absolute owner of the land, that the tax declarations/receipts were all issued in
his name, and that he has been in open, continuous and adverse possession in the
concept of owner.

As O was unable to pay back the loan plus interest for the past five [5) years, M
had to foreclose the mortgage. At the foreclosure sale, M was the highest bidder.
Upon issuance of the sheriff’s final deed of sale and registration in January, 1966, the
mortgage property was turned over to M's possession and control. M has since then
developed the said property. In 1967, O died, survived by sons S and P.

In 1977, after the tenth (10th) death anniversary of his father O, son P filed
a suit to annul the mortgage deed and subsequent sale of the property, etc., on
the ground of fraud. He asserted that the property in question was conjugal in
nature actually belonging, at the time of the mortgage, to O and his wife, W,
whose conjugal share went to their sons (S and P) and to O.

(a) Is the suit filed by P barred by prescription? Explain your answer. Yes,
10 years

(b) After the issuance of the sheriff's final deed of sale in 1966 in this case,
assuming that M applied for registration under the Torrens System and was
issued a Torrens Title to the said property in question, would that added fact
have any significant effect on your conclusion? State your reason. Rule on
indefeasibility of title, a title cannot be annulled 1 year after the issuance of its
decree of registration. Hence, P is barred now to assail title by reason of
prescription

SUGGESTED ANSWER:
(a) Under Art. 173 of the Civil Code, the action is barred by prescription because
the wife had only ten (10) years from the transaction and during the marriage to file a
suit for the annulment of the mortgage deed.

Notes:
The action is not barred by prescription because under the Civil Code, an action
upon a void contract is imprescriptible. The contract of sale between O and M is void
because the property mortgaged is conjugal and it was mortgaged without the consent
of the wife. Thus, the action is imprescriptible.

Alternative Answers to (a)


first Alternative Answer:
(a) The mortgage contract executed by O, if at all, is only a voidable contract
since it involves a conjugal partnership property. The action to annul the same
instituted in 1977, or eleven years after the execution of the sheriff's final sale, has
obviously prescribed because:
1) An action to annul a contract on the ground of fraud must be brought within
four (4) years from the date of discovery of the fraud. Since this is in essence an action
to recover ownership, it must be reckoned from the date of execution of the
contract or from the registration of the alleged fraudulent document with the
assessor's office for the purpose of transferring the tax declaration, this being
unregistered land, (Bael u. Intermediate Appellate Court G. R. L-74423 Jan.30,
1989 169 SCRA 617).

2) If the action is to be treated as an action to recover ownership of land, it


would have prescribed just the same because more than 10 years have already elapsed
since the date of the execution of the sale.

Notes:
a) Under Art. 124 of the Family Code, any disposition of the conjugal property
by a spouse without the consent of the other is void. It is provided further in the same
code that when a contract is void, the action to nullify such contract is imprescriptible.
Thus, the suit filed by P is not yet barred by prescription.
(b) Under the doctrine of indefeasibility of title, title to the land cannot be
annulled after the lapse of one year from the issuance of decree of registration.
Therefore, the added fact would have significant in my conclusion under the doctrine
of indefeasibility.

Art. 124. The administration and enjoyment of the conjugal partnership shall
belong to both spouses jointly. In case of disagreement, the husband’s decision shall
prevail, subject to recourse to the court by the wife for proper remedy, which must be
availed of within five years from the date of the contract implementing such decision.

In the event that one spouse is incapacitated or otherwise unable to participate in


the administration of the conjugal properties, the other spouse may assume sole
powers of administration. These powers do not include disposition or encumbrance
without authority of the court or the written consent of the other spouse. In the
absence of such authority or consent, the disposition or encumbrance shall be
void. However, the transaction shall be construed as a continuing offer on the part of
the consenting spouse and the third person, and may be perfected as a binding contract
upon the acceptance by the other spouse or authorization by the court before the offer
is withdrawn by either or both offerors.

SECOND ALTERNATIVE ANSWER:


(a) The action to recover has been barred by acquisitive prescription in favor of
M considering that M has possessed the land under a claim of ownership for ten (10)
years with a just title.

(b) If M had secured a Torrens Title to the land, all the more S and P could not
recover because if at all their remedies would be:
1. A Petition to Review the Decree of Registration. This can be availed of within
one (1) year from-the entry thereof, but only upon the basis of "actual fraud." There is
no showing that M committed actual fraud in securing his title to the land; or

2. An action in personam against M for the reconveyance of the title in their


favor. Again, this remedy is available within four years from the date of the discovery
of the fraud but not later than ten (10) years from the date of registration of the title in
the name of M.

Prescription; Real Rights (1992)


A owned a parcel of unregistered land located on the Tarlac side of the boundary
between Tarlac and Pangasinan. His brother B owned the adjoining parcel of
unregistered land on the Pangasinan side.

A sold the Tarlac parcel to X in a deed of sale executed as a public instrument


by A and X. After X paid in full the price of the sale, X took possession of the
Pangasinan parcel in the belief that it was the Tarlac parcel covered by the deed of
sale executed by A and X. There was no mode of acquiring ownership of Pangaisnan
parcel since what was the subject matter of the deed of sale is the Tarlac Parcel. The
second requisite of Acquisitive prescription is wanting. There is no “just title” on the
Pangasinan land

After twelve (12) years, a controversy arose between B and X on the issue of
the ownership of the Pangasinan parcel, B claims a vested right of ownership over the
Pangasinan parcel because B never sold that parcel to X or to anyone else.

SUGGESTED ANSWER:
At this point in time, X cannot claim the right of vested ownership over the
Pangasinan parcel by acquisitive prescription. In addition to the requisites common to
ordinary and extraordinary acquisitive prescription consisting of uninterrupted,
peaceful, public, adverse and actual possession in the concept of owner, ordinary
acquisitive prescription for ten (10) years requires (1) possession in good faith and (2)
just title. "Just title" means that the adverse claimant came into possession of the
property through one of the modes recognized by law for the acquisition of ownership
but the grantor was not the owner or could not transmit any right (Art. 1129. Civil
Code). In this case, there is no "just title" and no "mode" that can be invoked by X for
the acquisition of the Pangasinan parcel. There was no constructive delivery of the
Pangasinan parcel because it was not the subject-matter of the deed of sale. Hence, B
retains ownership of the Pangasinan parcel of land.

Notes:
Under the Civil Code, acquisitive prescription may be ordinary or extraordinary;
ordinary prescription may be invoked if the possessor possesses the property in the
concept of owner; he will acquire a just title to such property after the lapse of 10
years.

On the other hand, extraordinary prescription may be invoked if the possessor


possesses the property for more than 30 years and the possession was actual,
notorious and exclusive.

ARTICLE 1129. For the purposes of prescription, there is just title when the
adverse claimant came into possession of the property through one of the modes
recognized by law for the acquisition of ownership or other real rights, but the
grantor was not the owner or could not transmit any right.

Notes:
Under the law, ordinary acquisitive prescription may be invoked if the possessor
possesses the property in the concept of owner; he has a just title to the property if he
acquires the property by sale, donation, among others.

Primary Entry Book; Acquisitive Prescription; Laches (1998)


In 1965, Renren bought from Robyn a parcel of registered land evidenced by a
duly executed deed of sale. The owner presented the deed of sale and the owner's
certificate of title to the Register of Deeds. The entry was made in the daybook and
corresponding fees were paid as evidenced by official receipt. However, no transfer of
certificate of title was issued to Renren because the original certificate of title in
Robyn's name was temporarily misplaced after fire partly gutted the Office of the
Register of Deeds. Meanwhile, the land had been possessed by Robyn's distant cousin,
Mikaelo, openly, adversely and continuously in the concept of owner since 1960. It
was only in April 1998 that Renren sued Mikaelo to recover possession. Mikaelo
invoked a) acquisitive prescription and b) laches, asking that he be declared
owner of the land. Decide the case by evaluating these defenses, [5%] no, the
entry which was made in the day book and the corresponding payment of fees
constitutes registration and no acquisitive prescription shall prevail over a
registered land

SUGGESTED ANSWER:
a) Renren's action to recover possession of the land will prosper. In 1965, after
buying the land from Robyn, he submitted the Deed of Sale to the Registry of
Deeds for registration together with the owner's duplicate copy of the title, and
paid the corresponding registration fees. Under Section 56 of PD No. 1529, the
Deed of Sale to Renren is considered registered from the time the sale was
entered in the Day Book (now called the Primary Entry Book).
For all legal intents and purposes, Renren is considered the registered
owner of the land. After all, it was not his fault that the Registry of Deeds could not
issue the corresponding transfer certificate of title.
Mikaelo's defense of prescription can not be sustained. A Torrens title is
imprescriptible. No title to registered land in derogation of the title of the
registered owner shall be acquired by prescription or adverse possession. (Section
47, P.D. No, 1529)
The right to recover possession of registered land likewise does not prescribe because
possession is just a necessary incident of ownership.

Notes:

Under P.D. No, 1529, a Torrens title is imprescriptible and a titled land cannot
be acquired by acquisitive prescription. Therefore, Micahelangelo cannot claim
acquisitive prescription over the land.

SUGGESTED ANSWER:
b) Mikaelo's defense of laches, however, appears to be more sustainable. Renren
bought the land and had the sale registered way back in 1965. From the facts, it
appears that it was only in 1998 or after an inexplicable delay of 33 years that he took
the first step asserting his right to the land. It was not even an action to recover
ownership but only possession of the land. By ordinary standards, 33 years of neglect
or inaction is too long and maybe considered unreasonable. As often held by the
Supreme Court, the principle of imprescriptibility sometimes has to yield to the
equitable principle of laches which can convert even a registered land owner's
claim into a stale demand.

Mikaelo's claim of laches, however, is weak insofar as the element of equity is


concerned, there being no showing in the facts how he entered into the ownership and
possession of the land.

Reclamation of Foreshore Lands; Limitations (2000)


Republic Act 1899 authorizes municipalities and chartered cities to reclaim
foreshore lands bordering them and to construct thereon adequate docking and harbor
facilities. Pursuant thereto, the City of Cavite entered into an agreement with the Fil-
Estate Realty Company, authorizing the latter to reclaim 300 hectares of land from
the sea bordering the city, with 30% of the land to be reclaimed to be owned by Fil-
Estate as compensation for its services. The Solicitor General questioned the validity
of the agreement on the ground that it will mean reclaiming land under the sea which
is beyond the commerce of man. The City replies that this is authorized by RA. 1899
because it authorizes the construction of docks and harbors. Who is correct? (3%)

SUGGESTED ANSWER:
The Solicitor General is correct. The authority of the City of Cavite under RA
1899 to reclaim land is limited to foreshore lands. The Act did not authorize it to
reclaim land from the sea. "The reclamation being unauthorized, the City of Cavite
did not acquire ownership over the reclaimed land. Not being the owner, it could not
have conveyed any portion thereof to the contractor.

Notes:
The claim of the Solicitor General is correct because lands under the sea are
beyond the commerce of man. They cannot be appropriated.

ALTERNATIVE ANSWER:
It depends. If the reclamation of the land from the sea is necessary in the
construction of the docks and the harbors, the City of Cavite is correct. Otherwise, it is
not. Since RA 1899 authorized the city to construct docks and harbors, all works that
are necessary for such construction are deemed authorized. Including the reclamation
of land from the sea. The reclamation being authorized, the city is the owner of the
reclaimed land and it may convey a portion thereof as payment for the services of the
contractor.
ANOTHER ALTERNATIVE ANSWER:
On the assumption that the reclamation contract was entered into before RA
1899 was repealed by PD 3-A, the City of Cavite is correct. Lands under the sea are
"beyond the commerce of man" in the sense that they are not susceptible of
private appropriation, ownership or alienation. The contract in question merely
calls for the reclamation of 300 hectares of land within the coastal waters of the city.
Per se, it does not vest, alienate or transfer ownership of land under the sea. The city
merely engaged the services of Fil-Estate to reclaim the land for the city.

Registration; Deed of Mortgage (1994)


How do you register now a deed of mortgage of a parcel of land originally
registered under the Spanish Mortgage Law?

SUGGESTED ANSWER:
a) After the Spanish Mortgage Law was abrogated by P.D. 892 on February 16,
1976, all lands covered by Spanish titles that were not brought under the Torrens
system within six 16] months from the date thereof have been considered as
"unregistered private lands."

Thus, a deed of mortgage affecting land originally registered under the Spanish
Mortgage Law is now governed by the system of registration of transactions or
instruments affecting unregistered land under Section 194 of the Revised
Administrative Code as amended by Act No. 3344. Under this law, the instrument or
transaction affecting unregistered land is entered in a book provided for the purpose
but the registration thereof is purely voluntary and does not adversely affect third
persons who have a better right.

b) By recording and registering with the Register of Deeds of the place


where the land is located, in accordance with Act 3344. However, P.D. 892 required
holders of Spanish title to bring the same under the Torrens System within 6 months
from its effectivity on February 16, 1976.

Notes:
The deed of mortgage of a parcel of land originally registered under the Spanish
Mortgage Law is now registered by recording and registering with the Register of
Deeds of the place where the land is located.
Remedies; Judicial Confirmation; Imperfect Title (1993)
On June 30, 1986, A filed in the RTC of Abra an application for registration of
title to a parcel of land under P. D. No. 1529, claiming that since June 12, 1945, he
has been in open, continuous, exclusive and notorious possession and occupation
of said parcel of land of the public domain which was alienable and disposable,
under a bona fide claim of ownership. After issuance of the notice of initial hearing
and publication, as required by law, the petition was heard on July 29, 1987. On the
day of the hearing nobody but the applicant appeared. Neither was there anyone who
opposed the application. Thereupon, on motion of the applicant, the RTC issued an
order of general default and allowed the applicant to present his evidence. That he
did. On September 30, 1989, the RTC dismissed A's application for lack of sufficient
evidence. A appealed to the Court of Appeals.

The appellant urged that the RTC erred in dismissing his application for
registration and in not ordering registration of his title to the parcel of land in
question despite the fact that there was no opposition filed by anybody to his
application. Did the RTC commit the error attributed to it?

SUGGESTED ANSWER:
No, the RTC did not commit the error attributed to it. In an application for
Judicial confirmation of imperfect or incomplete title to public agricultural land under
Section 48 of the Public Land Act, the lack of opposition and the consequent order of
default against those who did not answer or show up on the date of initial hearing,
does not guarantee the success of the application. It is still incumbent upon the
applicant to prove with well nigh incontrovertible evidence that he has acquired a title
to the land that is fit for registration. Absent such registrable title, it is the clear duty of
the Land Registration Court to dismiss the application and declare the land as public
land.

An application for land registration is a proceeding in rem. Its main objective is


to establish the status of the res whether it is still part of our public domain as
presumed under the Regalian doctrine or has acquired the character of a private
property. It is the duty of the applicant to overcome that presumption with sufficient
evidence.

Notes:
Under the Public Land Act, it is the duty of the applicant to establish that the
land which is the subject of the application for registration is an alienable agricultural
land and that he has been in open, continuous and uninterrupted possession of such
land for a period of time under the claim of ownership. If he failed to introduce
sufficient evidence, the action for judicial confirmation may be dismissed even
though there were no opposition from the public.

Remedies; Judicial Reconstitution of Title (1996)


In 1989, the heirs of Gavino, who died on August 10, 1987, filed a petition for
reconstitution of his lost or destroyed Torrens Title to a parcel of land in Ermita,
Manila. This was opposed by Marilou who claimed ownership of the said land by a
series of sales. She claimed that Gavino had sold the property to Bernardo way back
in 1941 and as evidence thereof, she presented a Tax Declaration in 1948 in the name
of Bernardo, which cancelled the previous Tax Declaration in the name of Gavino.
Then she presented two deeds of sale duly registered with the Register of Deeds, the
first one executed by Bernardo in 1954 selling the same property to Carlos, and the
second one executed by Carlos in 1963, selling the same property to her. She also
claimed that she and her predecessors in interest have been in possession of the
property since 1948. If you were the judge, how will you decide the petition?
Explain.

SUGGESTED ANSWER:
If I were the judge, I will give due course to the petition of the heirs of Gavino
despite the opposition of Marilou for the following reasons:

a) Judicial reconstitution of a certificate of title under RA. No. 26 partakes of a


land registration proceeding and is perforce a proceeding in rem. It denotes restoration
of an existing instrument which has been lost or destroyed in its original form and
condition. The purpose of reconstitution of title or any document is to have the same
reproduced, after proceedings in the same form they were when the loss or
destruction occurred.

b) If the Court goes beyond that purpose, it acts without or in excess of


jurisdiction. Thus, where the Torrens Title sought to be reconstituted is in the name of
Gavino, the court cannot receive evidence proving that Marilou is the owner of the
land. Marilou's dominical claim to the land should be ventilated in a separate civil
action before the Regional Trial Court in its capacity as a court of general jurisdiction.
REFERENCES: Heirs of Pedro Pinate vs. Dulay. 187 SCRA 12-20 (1990); Bunagan
vs. CF1 Cebu Branch VI. 97 SCRA 72 (1980); Republic vs. IAC. 157 SCRA 62,66
(1988); Margolles vs. CA, 230 SCRA 709; Republic us, Feliciano, 148 SCRA 924.
Notes:
In Republic v. Feliciano, the Supreme Court held that judicial reconstitution of a
certificate of title is a restoration of an existing instrument which has been lost or
destroyed. The instrument will be restored to its original form; therefore, courts cannot
determine the issue of ownership in this proceeding.

Remedies; Procedure; Consulta (1994)


What is the procedure of consulta when an instrument is denied
registration?

SUGGESTED ANSWER:
1) The Register of Deeds shall notify the interested party in writing, setting
forth the defects of the instrument or the legal ground relied upon for denying the
registration, and advising that if he is not agreeable to such ruling, he may, without
withdrawing the documents from the Registry, elevate the matter by Consulta to the
Administrator of the Land Registration Authority (LRA).
2) Within five {5) days from receipt of notice of denial, the party-in-interest
shall file his Consulta with the Register of Deeds concerned and pay the consulta fee.
3) After receipt of the Consulta and payment of the corresponding fee the
Register of Deeds makes an annotation of the pending consulta at the back of the
certificate of title.
4) The Register of Deeds then elevates the case to the LRA Administrator with
certified records thereof and a summary of the facts and issues involved.

5) The LRA Administrator then conducts hearings after due notice or may just
require parties to submit their memoranda.
6) After hearing, the LRA Administrator issues an order prescribing the step to
be taken or the memorandum to be made. His resolution in consulta shall be
conclusive and binding upon all Registers of Deeds unless reversed on appeal by the
Court of Appeals or by the Supreme Court. (Section 117, P.D. 1529).

The procedure of consulta is a mode of appeal from denial by the Register of


Deeds of the registration of the instrument to the Commissioner of Land Registration.
Within five days from receipt of the notice of denial, the interested party may elevate
the matter by consulta to the Commissioner of Land Registration who shall enter an
order prescribing the step to be taken or memorandum to be made. Resolution in
consulta shall be binding upon all Registers of Deeds provided that the party in
interest may appeal to the Court of Appeals within the period prescribed (Sec. 117,
P.D. 1529).

Notes:
Under P.D. 1529, the procedure of consulta is a mode of appeal from denial
by the Register of Deeds of the registration of the instrument to the Commissioner of
Land Registration. The law mandates that within 5 days from the receipt of such
denial, the aggrieved party shall elevate the matter by consulta to the Commissioner
of Land Registration. ROD to Commissioner of Land Regis

ACTION FOR RECONVEYANCE

Remedies; Reconveyance vs. Reopening of a Decree; Prescriptive Period (2003)


Louie, before leaving the country to train as a chef in a five-star hotel in New
York, U.S.A., entrusted to his first-degree cousin Dewey an application for
registration, under the Land Registration Act, of a parcel of land located in Bacolod
City. A year later, Louie returned to the Philippines and discovered that Dewey
registered the land and obtained an Original Certificate of Title over the property in
his Dewey’s name. Compounding the matter, Dewey sold the land to Huey, an
innocent purchaser for value. Louie promptly filed an action for reconveyance of
the parcel of land against Huey.

(a) Is the action pursued by Louie the proper remedy?


(b) Assuming that reconveyance is the proper remedy, will the action
prosper if the case was filed beyond one year, but within ten years, from the
entry of the decree of registration? 5%

SUGGESTED ANSWER:
(a) An action for reconveyance against Huey is not the proper remedy, because
Huey is an innocent purchaser for value. The proper recourse is for Louie to go
after Dewey for damages by reason of the fraudulent registration and subsequent sale
of the land. If Dewey is insolvent, Louie may file a claim against the Assurance Fund
(Heirs of Pedro Lopez v. De Castro 324 SCRA 591 [2000] citing Sps. Eduarte v. CA,
323 Phil. 462, 467 [1996]).

(b) Yes, the remedy will prosper because the action prescribes in ten (10) years,
not within one (1) year when a petition for the reopening of the registration decree
may be filed. The action for reconveyance is distinct from the petition to reopen the
decree of registration (Grey Alba v. De la Cruz, 17 Phil. 49 [1910}). There is no need
to reopen the registration proceedings, but the property should just be reconveyed to
the real owner.

The action for reconveyance is based on implied or constructive trust, which


prescribes in ten (10) years from the date of issuance of the original certificate of
title. This rule assumes that the defendant is in possession of the land. Where it is the
plaintiff who is in possession of the land, the action for reconveyance would be in the
nature of a suit for quieting for the title which action is imprescriptible (David v.
Malay, 318 SCRA 711 [1999]).

Notes:
Under the law, an action for reconveyance of a parcel of land cannot prosper
against an innocent purchaser for value.

In the case of David v. Malay, the Supreme Court held that an action for
reconveyance prescribes in 10 years; this action presumes that the land is in the
possession of a defendant; if the land is in the possession of a plaintiff, the action is
quieting of title which is imprescriptible.

Remedies; Reconveyance; Elements (1995)


Rommel was issued a certificate of title over a parcel of land in Quezon City.
One year later Rachelle, the legitimate owner of the land, discovered the fraudulent
registration obtained by Rommel. She filed a complaint against Rommel for
reconveyance and caused the annotation of a notice of lis pendens on the certificate of
title issued to Rommel. Rommel now invokes the indefeasibility of his title
considering that one year has already elapsed from its issuance. He also seeks the
cancellation of the notice of Lis pendens. Will Rachelle's suit for reconveyance
prosper? Explain.

SUGGESTED ANSWER:
Yes, Rachelle's suit will prosper because all elements for an action for
reconveyance are present, namely:
a) Rachelle is claiming dominical rights over the same land.
b) Rommel procured his title to the land by fraud.
c) The action was brought within the statutory period of four (4) years from
discovery of the fraud and not later than ten (10} years from the date of
registration of Rommel's title.
d) Title to the land has not passed into the hands of an innocent purchaser
for value. Rommel can invoke the indefeasibility of his title if Rachelle had filed a
petition to reopen or review the decree of registration. But Rachelle instead filed an
ordinary action in personam for reconveyance. In the latter action, indefeasibility is
not a valid defense because, in filing such action, Rachelle is not seeking to nullify nor
to impugn the indefeasibility of Rommel's title. She is only asking the court to compel
Rommel to reconvey the title to her as the legitimate owner of the land.

Notes:
The elements for an action for reconveyance are:
a) Plaintiff is claiming dominical rights over the same land.
b) Defendant procured his title to the land by fraud.
c) The action was brought within four (4) years from discovery of the fraud and
not later than ten (10} years from the date of registration of the title of the defendant.
d) Title to the land has not passed into the hands of an innocent purchaser for
value.

ALTERNATIVE ANSWER:
Yes. The property registered is deemed to be held in trust for the real owner by
the person in whose name it is registered. The Torrens system was not designed to
shield one who had committed fraud or misrepresentation and thus holds the title in
bad faith. (Walstrom v. Mapa Jr., (G .R 38387, 29 Jan. 1990) as cited in Martinez, D.,
Summary of SC Decisions, January to June, 1990, p. 359],

Notes:
In Walstrom v. Mapa Jr., the Supreme Court held that in case of fraud, an action
for reconveyance of a parcel of land will prosper although it was already titled in the
name of the fraudster because land registration cannot be used to shield fraud.

Remedies; Reconveyance; Prescriptive Period (1997)


On 10 September 1965, Melvin applied for a free patent covering two lots - Lot
A and Lot B - situated in Santiago, Isabela. Upon certification by the Public Land
Inspector that Melvin had been in actual, continuous, open, notorious, exclusive and
adverse possession of the lots since 1925, the Director of Land approved Melvin's
application on 04 June 1967. On 26 December 1967, Original Certificate of Title
(OCT) No. P-2277 was issued in the name of Melvln.

On 7 September 1971, Percival filed a protest alleging that Lot B which he had
been occupying and cultivating since 1947 was included in the Free Patent issued in
the name of Melvin. The Director of Lands ordered the investigation of Percival's
protest. The Special Investigator who conducted the investigation found that Percival
had been in actual cultivation of Lot B since 1947.

On 28 November 1986, the Solicitor General filed in behalf of the Republic of


the Philippines a complaint for cancellation of the free patent and the OCT issued in
the name of Melvin and the reversion of the land to public domain on the ground of
fraud and misrepresentation in obtaining the free patent. On the same date, Percival
sued Martin for the reconveyance of Lot B.

Melvin filed his answers interposing the sole defense in both cases that the
Certificate of Title issued in his name became incontrovertible and indefeasible
upon the lapse of one year from the issuance of the free patent.

Given the circumstances, can the action of the Solicitor General and the
case for reconveyance filed by Percival possibly prosper?

SUGGESTED ANSWER:
"If fraud be discovered in the application which led to the issuance of the patent
and Certificate of Title, this Title becomes ipso facto null and void. Thus, in a case
where a person who obtained a free patent, knowingly made a false statement of
material and essential facts in his application for the same, by stating therein that the
lot in question was part of the public domain not occupied or claimed by any other
person, his title becomes ipso facto canceled and consequently rendered null and
void." "It is to the public interest that one who succeeds in fraudulently acquiring title
to public land should not be allowed to benefit therefrom and the State, through the
Solicitor General, may file the corresponding action for annulment of the patent and
the reversion of the land involved to the public domain" (Dinero us. Director of
Lands; Kayaban vs. Republic L-33307,8-20-73; Director of Lands us. Hon. Pedro
Samson Animas, L-37682, 3-29-74.)

This action does not prescribe. With respect to Percival's action for
reconveyance, it would have prescribed, having been filed more than ten (10) years
after registration and issuance of an O.C.T. in the name of Melvin, were it not for the
inherent infirmity of the latter's title. Under the facts, the statute of limitations will not
apply to Percival because Melvin knew that a part of the land covered by his title
actually belonged to Percival. So, instead of nullifying in toto the title of Melvin,
the court, in the exercise of equity and jurisdiction, may grant prayer for the
reconveyance of Lot B to Percival who has actually possessed the land under a
claim of ownership since 1947. After all, if Melvin's title is declared void ab initio
and the land is reverted to the public domain, Percival would just the same be entitled
to preference right to acquire the land from the government. Besides, well settled is
the rule that once public land has been in open, continuous, exclusive and notorious
possession under a bonafide claim of acquisition of ownership for the period
prescribed by Section 48 of the Public Land Act, the same ipso jure ceases to be
public and in contemplation of law acquired the character of private land. Thus,
reconveyance of the land from Melvin to Percival would be the better procedure,
(Vitale vs. Anore, 90 Phil. 855; Pena, Land Titles and Deeds, 1982, Page 427)

Notes:
In Kayaban v. Republic, the Supreme Court held that the action filed by the
government to cancel a certificate of title on the ground of fraud or
misrepresentation is imprescriptible. Therefore, the doctrine of indefeasibility of
title after the lapse of 1 year from the issuance thereof does not find application.

ALTERNATIVE ANSWER:
The action of the Solicitor General should prosper, considering that the doctrine
of indefeasibility of title does not apply to free patent secured through fraud. A
certificate of title cannot be used as shield to perpetuate fraud. The State is not bound
by the period of prescription stated in Sec. 38 of Act 496. (Director of Lands vs.
Abanilla, 124 SCRA 358) The action for reconveyance filed by Percival may still
prosper provided that the property has not passed to an innocent third party for
value (Dablo us. Court of Appeals. 226 SCRA 618), and provided that the action is
filed within the prescriptive period of ten years (Tale vs. Court of Appeals. 208
SCRA 266). Since the action was filed by Percival 19 years after the issuance of
Melvin's title, it is submitted that the same is already barred by prescription.

ALTERNATIVE ANSWER (to second part of question):


The action for reconveyance filed by Percival will prosper, because the land
has ceased to be public land and has become private land by open, continuous,
public, exclusive possession under a bona fide claim of ownership for more than
thirty years, and Percival is still in possession of the property at present. His action
for reconveyance can be considered as an action to quiet title, which does not
prescribe if the plaintiff is in possession of the property. (Olviga v. CA. GR 1048013.
October 21, 1993)

Remedies; Reopening of a Decree; Elements (1992)


What are the essential requisites or elements for the allowance of the
reopening or review of a decree of registration?

SUGGESTED ANSWER:
The essential elements are: (1) that the petitioner has a real or dominical right;
(2) that he has been deprived thereof through fraud; (3) that the petition is filed within
one (1) year from the issuance of the decree; and (4) that the property has not yet been
transferred to an innocent purchaser {Rublico vs. Orellana 30 SCRA 511; Ubudan
vs. Gil45 SCRA 17).

Notes:
The elements to reopen or review a decree of registration are:
a) The petitioner claims dominical right to the land registered in the name of
respondent.
b) The registration of the land in the name of the respondent was procured by
means of actual fraud.
c) The petition must be filed within one (1) year from the date of the issuance of
the decree.
d) Title to the land has not passed to an Innocent purchaser for value.

OPTIONAL EXTENDED ANSWER:


Petition for review of the Decree of Registration. A remedy expressly provided
in Section 32 of P. D. No. 1529 (formerly Section 38. Act 496), this remedy has the
following elements:
a) The petition must be filed by a person claiming dominical or other real rights
to the land registered in the name of respondent.
b) The registration of the land in the name of respondent was procured by means
of actual, (not just constructive) fraud, which must be extrinsic. Fraud is actual if
the registration was made through deceit or any other intentional act of downright
dishonesty to enrich oneself at the expense of another. It is extrinsic when it is
something that was not raised, litigated and passed upon in the main proceedings.
c) The petition must be filed within one (1) year from the date of the issuance of
the decree.
d) Title to the land has not passed to an Innocent purchaser for value (Libudan
vs. Gil, 45_ SCRA 27, 1972), Rublico vs. Orrelana. 30 SCRA 511, 1969); RP vs. CA,
57 G. R No. 40402. March 16, 1987).

Torrens System vs. Recording of Evidence of Title (1994)


Distinguish the Torrens system of land registration from the system of
recording of evidence of title.

SUGGESTED ANSWER:
a) The TORRENS SYSTEM OF LAND REGISTRATION is a system for the
registration of title to the land. Thus, under this system what is entered in the Registry
of Deeds, is a record of the owner's estate or interest in the land, unlike the system
under the Spanish Mortgage Law or the system under Section 194 of the Revised
Administrative Code as amended by Act 3344 where only the evidence of such title is
recorded. In the latter system, what is recorded is the deed of conveyance from
hence the owner's title emanated—and not the title itself.
b) Torrens system of land registration is that which is prescribed in Act 496
(now PD 1529), which is either Judicial or quasi-judicial. System or recording of
evidence of title is merely the registration of evidence of acquisitions of land with
the Register of Deeds, who annotates the same on the existing title, cancels the old
one and issues a new title based on the document presented for registration.

Notes:
In the torrens system of land registration, the title to the land of the owner is
recorded in the Registry of Deeds.

On the other hand, in recording of evidence of title, deed of conveyance


emanated from the owner is recorded in the registry of deeds.
Unregistered Land (1991)
Maria Enriquez failed to pay the realty taxes on her unregistered agricultural
land located in Magdugo, Toledo City. In 1989, to satisfy the taxes due, the City sold
it at public auction to Juan Miranda, an employee at the Treasurer's Office of said
City, whose bid at P10,000.00 was the highest. In due time, a final bill of sale was
executed in his favor.

Maria refused to turn-over the possession of the property to Juan alleging


that (1) she had been, in the meantime, granted a free patent and on the basis
thereof an Original Certificate of Title was issued to her, and (2) the sale in favor
of Juan is void from the beginning in view of the provision in the Administrative
Code of 1987 which prohibits officers and employees of the government from
purchasing directly or indirectly any property sold by the government for
nonpayment of any tax, fee or other public charge.
(a) Is the sale to Juan valid? If so, what is the effect of the Issuance of the
Certificate of Title to Maria?
(b) If the sale is void, may Juan recover the P10,000.00? If not, why not?
(c) If the sale is void, did it not nevertheless, operate to divert Maria of her
ownership? If it did, who then is the owner of the property?

SUGGESTED ANSWER:
A. The sale of the land to Juan is not valid, being contrary to law. Therefore, no
transfer of ownership of the land was effected from the delinquent taxpayer to him.
The original certificates of title obtained by Maria thru a free patent grant from the
Bureau of Lands under Chapter VII, CA 141 is valid but in view of her delinquency,
the said title is subject to the right of the City Government to sell the land at
public auction. The issuance of the OCT did not exempt the land from the tax sales.
Section 44 of P.O. No. 1529 provides that every registered owner receiving a
Certificate of Title shall hold the same free from an encumbrances, subject to certain
exemptions.

B. Juan may recover because he was not a party to the violation of the law.

C. No, the sale did not divest (deprive) Maria of her title precisely because the
sale is void. It is as good as if no sale ever took place. In tax sales, the owner is
divested of his land initially upon award and issuance of a Certificate of Sale, and
finally after the lapse of the 1 year period from date of registration to redeem,
upon execution by the treasurer of an instrument sufficient in form and effects to
convey the property. Maria remained owner of the land until another tax sale is to be
performed in favor of a qualified buyer.

Notes:
A. Under the Administrative Code of 1987, officers and employees of the
government cannot acquire by purchase directly or indirectly any property sold by
the government for nonpayment of any tax, fee or other charge. Therefore, the sale to
employee of the treasurer’s office of the land sold for nonpayment of tax is void.

B. Juan may recover the money he paid for the land because he was not a party
to the violation of the law.

C. No, the sale did not divest Maria of her title precisely because the sale is
void. It is as good as if no sale ever took place.
Furthermore, the owner may be divested of his right to his property if he cannot
redeem it after the lapse of one year from registration of the sale by the buyer.

CONTRACTS

Consensual vs. Real Contracts; Kinds of Real Contracts (1998)


Distinguish consensual from real contracts and name at least four (4) kinds
of real contracts under the present law. [3%]

SUGGESTED ANSWER:
CONSENSUAL CONTRACTS are those which are perfected by mere consent
(Art. 1315. Civil Code). REAL CONTRACTS are those which are perfected by the
delivery of the object of the obligation. (Art. 1316, Civil Code) Examples of real
contracts are deposit, pledge, commodatum and simple loan (mutuum).

Notes:
Consensual contracts are perfected by mere consent, while real contracts are
perfected by the delivery of the object of the contract.

Examples of real contract are: deposit, pledge, commodatum and simple loan or
mutuum.
Example of consensual contract is sale.

Consideration; Validity (2000)


Lolita was employed in a finance company. Because she could not account for
the funds entrusted to her, she was charged with estafa and ordered arrested. In order
to secure her release from jail, her parents executed a promissory note to pay the
finance company the amount allegedly misappropriated by their daughter. The finance
company then executed an affidavit of desistance which led to the withdrawal of the
information against Lolita and her release from jail. The parents failed to comply
with their promissory note and the finance company sued them for specific
performance. Will the action prosper or not? (3%)

SUGGESTED ANSWER:
The action will prosper. The promissory note executed by Lolita's parents is
valid and binding, the consideration being the extinguishment of Lolita's civil liability
and not the stifling of the criminal prosecution.
ALTERNATIVE ANSWER:
The action will not prosper because the consideration for the promissory note
was the non-prosecution of the criminal case for estafa. This cannot be done
anymore because the information has already been filed in court and to do it is illegal.
That the consideration for the promissory note is the stifling of the criminal
prosecution is evident from the execution by the finance company of the affidavit of
desistance immediately after the execution by Lolita's parents of the promissory note.
The consideration being illegal, the promissory note is invalid and may not be
enforced by court action.

Notes:
Under the law, an action for specific performance will not prosper if the
consideration in a contract is unlawful; further, a promissory note is rendered invalid
if the consideration is illegal. Therefore, a party cannot file an action for specific
performance for the fulfillment of the promise to pay for the settlement of a criminal
action because criminal action cannot be the subject of amicable settlement.

Contract of Option; Elements (2005)


Marvin offered to construct the house of Carlos for a very reasonable price of
P900,000.00, giving the latter 10 days within which to accept or reject the offer. On
the fifth day, before Carlos could make up his mind, Marvin withdrew his offer.

a) What is the effect of the withdrawal of Marvin's offer? (2%)

SUGGESTED ANSWER:
The withdrawal of Marvin's offer will cause the offer to cease in law. Hence,
even if subsequently accepted, there could be no concurrence of the offer and the
acceptance. In the absence of concurrence of offer and acceptance, there can be no
consent. (Laudico v. Arias Rodriguez, G.R. No. 16530, March 31, 1922) Without
consent, there is no perfected contract for the construction of the house of Carlos.
(Salonga v. Farrales, G.R. No. L-47088, July 10, 1981) Article 1318 of the Civil
Code provides that there can be no contract unless the following requisites concur: (1)
consent of the parties; (2) object certain which is the subject matter of the contract;
and (3) cause or consideration of the obligation.
Marvin will not be liable to pay Carlos any damages for withdrawing the offer before
the lapse of the period granted. In this case, no consideration was given by Carlos
for the option given, thus there is no perfected contract of option for lack of cause
of obligation. Marvin cannot be held to have breached the contract. Thus, he cannot be
held liable for damages.
Notes:
In Laudico v. Arias Rodriguez, the Supreme Court held that to have a
perfected contract, there must be an offer and acceptance of such offer.
Therefore, an option contract is not a perfected contract if it is not accepted.

b) Will your answer be the same if Carlos paid Marvin P10,000.00 as


consideration for that option? Explain. (2%)

ALTERNATIVE ANSWER:
My answer will be the same as to the perfection of the contract for the
construction of the house of Carlos. No perfected contract arises because of lack of
consent. With the withdrawal of the offer, there could be no concurrence of offer and
acceptance.

ANOTHER ANSWER:
My answer will not be the same as to damages. Marvin will be liable for
damages for breach of contract of option. With the payment of the consideration for
the option given, and with the consent of the parties and the object of contract being
present, a perfected contract of option was created. (San Miguel, Inc. v. Huang, G.R.
No. 137290, July 31, 2000) Under Article 1170 of the Civil Code, those who in the
performance of their obligation are guilty of contravention thereof, as in this case,
when Marvin did not give Carlos the agreed period of ten days, are liable for damages.

Notes:
In the case of San Miguel, Inc. v. Huang, the Supreme Court held that an option
contract is perfected if it is founded upon consideration; it cannot be withdrawn
without being liable for breach of contract.

ALTERNATIVE ANSWER:
My answer will not be the same if Carlos paid Marvin P10,000.00 because an
option contract was perfected. Thus, if Marvin withdrew the offer prior to the
expiration of the 10-day period, he breached the option contract. (Article 1324, Civil
Code)

c) Supposing that Carlos accepted the offer before Marvin could


communicate his withdrawal thereof? Discuss the legal consequences. (2%)

SUGGESTED ANSWER:
A contract to construct the house of Carlos is perfected. Contracts are perfected
by mere consent manifested by the meeting of the offer and the acceptance upon the
thing and the cause which are to constitute the contract. (Gomez v. Court of Appeals,
G.R. No. 120747, September 21, 2000) Under Article 1315 of the Civil Code, Carlos
and Marvin are bound to fulfill what has been expressly stipulated and all
consequences thereof. Under Article 1167, if Marvin would refuse to construct the
house, Carlos is entitled to have the construction be done by a third person at the
expense of Marvin. Marvin in that case will be liable for damages under Article 1170.

Notes:
A contract is perfected by mere consent; when the offer of a party is accepted by
the other, there is already a meeting of the minds. Thus, the contract is already
perfected.
When a party to the contract withdraws, he is liable for breach of contract. Thus,
he will be liable for damages.

ARTICLE 1315. Contracts are perfected by mere consent, and from that moment
the parties are bound not only to the fulfillment of what has been expressly stipulated
but also to all the consequences which, according to their nature, may be in keeping
with good faith, usage and law.

Notes:
Under the Civil Code, when a contract is perfected, whoever fails to perform his
obligation is liable for damages.

ARTICLE 1167. If a person obliged to do something fails to do it, the same shall
be executed at his cost.

This same rule shall be observed if he does it in contravention of the tenor of the
obligation. Furthermore, it may be decreed that what has been poorly done be undone.

ARTICLE 1170. Those who in the performance of their obligations are guilty of
fraud, negligence, or delay, and those who in any manner contravene the tenor thereof,
are liable for damages.

Notes:
A tricycle driver who did not bring his passenger to the destination agreed upon
is liable for damages because he performed his obligation contrary to the tenor
thereof.

Inexistent Contracts vs. Annullable Contracts (2004)


Distinguish briefly but clearly between Inexistent contracts and annullable
contracts.

SUGGESTED ANSWER:
INEXISTENT CONTRACTS are considered as not having been entered into
and, therefore, void ab initio. They do not create any obligation and cannot be ratified
or validated, as there is no agreement to ratify or validate. On the other hand,
ANNULLABLE or VOIDABLE CONTRACTS are valid until invalidated by the
court but may be ratified. In inexistent contracts, one or more requisites of a valid
contract are absent. In anullable contracts, all the elements of a contract are present
except that the consent of one of the contracting parties was vitiated or one of them
has no capacity to give consent.

Notes:
Inexistence contracts are void contract; they are considered as not having been
entered into. They cannot be ratified. On the other hand, annullable or voidable
contracts are valid contracts until annulled; they can be ratified.

Nature of Contracts; Obligatoriness (1991)


Roland, a basketball star, was under contract for one year to play-for-play
exclusively for Lady Love, Inc. However, even before the basketball season could
open, he was offered a more attractive pay plus fringes benefits by Sweet Taste, Inc.
Roland accepted the offer and transferred to Sweet Taste. Lady Love sues Roland and
Sweet Taste for breach of contract. Defendants claim that the restriction to play for
Lady Love alone is void, hence, unenforceable, as it constitutes an undue interference
with the right of Roland to enter into contracts and the impairment of his freedom to
play and enjoy basketball.
Can Roland be bound by the contract he entered into with Lady Love or
can he disregard the same? Is he liable at all? How about Sweet Taste? Is it liable
to Lady Love?

SUGGESTED ANSWER:
Roland is bound by the contract he entered into with Lady Love and he cannot
disregard the same, under the principles of obligatoriness of contracts. Obligations
arising from contracts have the force of law between the parties.
Notes:
One of the characteristics of a contract is obligatoriness which means that a
contract has the force of law between parties. Thus, Roland is bound by his contract.
SUGGESTED ANSWER:
Yes, Roland is liable under the contract as far as Lady Love is concerned. He is
liable for damages under Article 1170 of the Civil Code since he contravened the
tenor of his obligation. Not being a contracting party, Sweet Taste is not bound by the
contract but it can be held liable under Art. 1314. The basis of its liability is not
prescribed by contract but is founded on quasi-delict, assuming that Sweet Taste knew
of the contract. Article 1314 of the Civil Code provides that any third person who
induces another to violate his contract shall be liable for damages to the other
contracting party.

Notes:
Under the Civil Code, a contract between parties has the force of law.

ARTICLE 1170. If a person is guilty of fraud, negligence, or delay in the


performance of his obligations, he shall be liable for damages; he shall also be liable
for damages if he contravenes the tenor of his obligation. (Reengineered)

ARTICLE 1314. Any third person who induces another to violate his contract
shall be liable for damages to the other contracting party.

ALTERNATIVE ANSWER:
It is assumed that Lady Love knew of the contract. Neither Roland nor Sweet
Taste would be liable, because the restriction in the contract is violative of Article
1306 as being contrary to law morals, good customs, public order or public policy.

Notes:

ARTICLE 1305. A contract is a meeting of minds between two persons


whereby one binds himself, with respect to the other, to give something or to render
some service. (1254a)
ARTICLE 1306. The contracting parties may establish such stipulations,
clauses, terms and conditions as they may deem convenient, provided they are not
contrary to law, morals, good customs, public order, or public policy.

Nature of Contracts; Privity of Contract (1996)


Baldomero leased his house with a telephone to Jose. The lease contract
provided that Jose shall pay for all electricity, water and telephone services in the
leased premises during the period of the lease. Six months later, Jose surreptitiously
vacated the premises. He left behind unpaid telephone bills for overseas telephone
calls amounting to over P20,000.00. Baldomero refused to pay the said bills on the
ground that Jose had already substituted him as the customer of the telephone
company. The latter maintained that Baldomero remained as his customer as far as
their service contract was concerned, notwithstanding the lease contract between
Baldomero and Jose. Who is correct, Baldomero or the telephone company? Explain.

SUGGESTED ANSWER:
The telephone company is correct because as far as it is concerned, the only
person it contracted with was Baldomero. The telephone company has no contract
with Jose. Baldomero cannot substitute Jose in his stead without the consent of the
telephone company (Art. 1293, NCC). Baldomero is, therefore, liable under the
contract.

Notes:
Under the Civil Code, contracts take effect between the contracting parties, their
heirs and assigns only. This is known as privity of contract.

ARTICLE 1293. Novation, which consists in substituting a new debtor in


place of the original debtor, may be made without the knowledge or against the
will of the original debtor; but it cannot be made without the consent of the
creditor.
Payment by the new debtor gives him the rights mentioned in articles 1236 and
1237. (Reengineered)

Nature of Contracts; Relativity of Contracts (2002)


Printado is engaged in the printing business. Suplico supplies printing paper to
Printado pursuant to an order agreement under which Suplico binds himself to deliver
the same volume of paper every month for a period of 18 months, with Printado in
turn agreeing to pay within 60 days after each delivery. Suplico has been faithfully
delivering under the order agreement for 10 months but thereafter stopped doing so,
because Printado has not made any payment at all. Printado has also a standing
contract with publisher Publico for the printing of 10,000 volumes of school
textbooks. Suplico was aware of said printing contract. After printing 1,000 volumes,
Printado also fails to perform under its printing contract with Publico. Suplico sues
Printado for the value of the unpaid deliveries under their order agreement. At the
same time Publico sues Printado for damages for breach of contract with respect to
their own printing agreement. In the suit filed by Suplico, Printado counters that:

(a) Suplico cannot demand payment for deliveries made under their order
agreement until Suplico has completed performance under said contract;

(b) Suplico should pay damages for breach of contract; and

(c) Publico should be liable for Printado’s breach of his contract with
Publico because the order agreement between Suplico and Printado was for the
benefit of Publico.
Are the contentions of Printado tenable? Explain your answers as to each
contention. (5%)

SUGGESTED ANSWER:
No, the contentions of Printado are untenable. Printado having failed to pay for
the printing paper covered by the delivery invoices on time, Suplico has the right to
cease making further delivery. And the latter did not violate the order agreement
(Integrated Packaging Corporation v. Court of Appeals, (333 SCRA 170, G.R. No.
115117, June 8, [2000]). Suplico cannot be held liable for damages, for breach of
contract, as it was not he who violated the order agreement, but Printado. Suplico
cannot be held liable for Printado’s breach of contract with Publico. He is not a party
to the agreement entered into by and between Printado and Publico. Theirs is not
a stipulation pour atrui. [Aforesaid] Such contracts do could not affect third persons
like Suplico because of the basic civil law principle of relativity of contracts which
provides that contracts can only bind the parties who entered into it, and it cannot
favor or prejudice a third person, even if he is aware of such contract and has acted
with knowledge thereof. (Integrated Packaging Corporation v. CA, supra).

Notes:
In the case of Integrated Packaging Corporation v. CA, the Supreme Court held
that under the principle of relativity of contract, contracts bind parties who entered to
them.

ARTICLE 1311. Contracts take effect only between the parties, their assigns
and heirs, except in case where the rights and obligations arising from the contract are
not transmissible by their nature, or by stipulation or by provision of law. The heir is
not liable beyond the value of the property he received from the decedent.

If a contract should contain some stipulation in favor of a third person, he may


demand its fulfillment provided he communicated his acceptance to the obligor before
its revocation. A mere incidental benefit or interest of a person is not sufficient. The
contracting parties must have clearly and deliberately conferred a favor upon a third
person. Stipulation pour autri

Rescission of Contracts; Proper Party (1996)


In December 1985, Salvador and the Star Semiconductor Company (SSC)
executed a Deed of Conditional Sale wherein the former agreed to sell his 2,000
square meter lot in Cainta, Rizal, to the latter for the price of P1,000,000.00, payable
P100,000.00 down, and the balance 60 days after the squatters in the property
have been removed. If the squatters are not removed within six months, the
P100,000.00 down payment shall be returned by the vendor to the vendee, Salvador
filed ejectment suits against the squatters, but in spite of the decisions in his favor, the
squatters still would not leave. In August, 1986, Salvador offered to return the
P100,000.00 down payment to the vendee, on the ground that he is unable to remove
the squatters on the property. SSC refused to accept the money and demanded
that Salvador execute a deed of absolute sale of the property in its favor, at which
time it will pay the balance of the price. Incidentally, the value of the land had
doubled by that time. Salvador consigned the P 100,000.00 in court, and filed an
action for rescission of the deed of conditional sale, plus damages.
Will the action prosper? Explain.

SUGGESTED ANSWER:
No, the action will not prosper. The action for rescission may be brought only by
the aggrieved party to the contract. Since it was Salvador who failed to comply with
his conditional obligation, he is not the aggrieved party who may file the action for
rescission but the Star Semiconductor Company. The company, however, is not opting
to rescind the contract but has chosen to waive Salvador's compliance with the
condition which it can do under Art. 1545, NCC.

Notes:
Under the Civil Code, an action for rescission may be filed by the aggrieved
party only. In this case, it was Salvador who failed to fulfill his obligation by failing to
eject the squatters. Therefore, Salvador cannot file an action for rescission of the
contract because he is not the aggrieved party but the Star Semiconductor Company.

ARTICLE 1545. Where the obligation of either party to a contract of sale is


subject to any condition which is not performed, such party may refuse to proceed
with the contract or he may waive performance of the condition. If the other party has
promised that the condition should happen or be performed, such first mentioned party
may also treat the nonperformance of the condition as a breach of warranty.

Where the ownership in the thing has not passed, the buyer may treat the fulfillment
by the seller of his obligation to deliver the same as described and as warranted
expressly or by implication in the contract of sale as a condition of the obligation of
the buyer to perform his promise to accept and pay for the thing.

ALTERNATIVE ANSWER:
The action for rescission will not prosper. The buyer has not committed any
breach, let alone a substantial or serious one, to warrant the rescission/resolution
sought by the vendor. On the contrary, it is the vendor who appears to have failed to
comply with the condition imposed by the contract the fulfillment of which would
have rendered the obligation to pay the balance of the purchase price demandable.
Further, far from being unable to comply with what is incumbent upon it, ie., pay the
balance of the price the buyer has offered to pay it even without the vendor having
complied with the suspensive condition attached to the payment of the price, thus
waiving such condition as well as the 60-day term in its favor The stipulation that the
P100,000.00 down payment shall be returned by the vendor to the vendee if the
squatters are not removed within six months, is also a covenant for the benefit of the
vendee, which the latter has validly waived by implication when it offered to pay the
balance of the purchase price upon the execution of a deed of absolute sale by the
vendor. (Art. 1545, NCC)
OBLIGATIONS

Aleatory Contracts; Gambling (2004)


A. Mr. ZY lost P100,000 in a card game called Russian poker, but he had no
more cash to pay in full the winner at the time the session ended. He promised to pay
PX, the winner, two weeks thereafter. But he failed to do so despite the lapse of two
months, so PX filed in court a suit to collect the amount of P50,000 that he won but
remained unpaid. Will the collection suit against ZY prosper? Could Mrs. ZY file
in turn a suit against PX to recover the P100,000 that her husband lost? Reason.
(5%)

SUGGESTED ANSWER:
A. 1. The suit by PX to collect the balance of what he won from ZY will not
prosper. Under Article 2014 of the Civil Code, no action can be maintained by the
winner for the collection of what he has won in a game of chance. Although poker
may depend in part on ability, it is fundamentally a game of chance.
2) If the money paid by ZY to PX was conjugal or community property, the wife
of ZY could sue to recover it because Article 117(7) of the Family Code provides that
losses in gambling or betting are borne exclusively by the loser-spouse. Hence,
conjugal or community funds may not be used to pay for such losses. If the money
were exclusive property of ZY, his wife may also sue to recover it under Article 2016
of the Civil Code if she and the family needed the money for support.

ALTERNATIVE ANSWER (2):


A. (2). Mrs. ZY cannot file a suit to recover what her husband lost. Art 2014 of
the Civil Code provides that any loser in a game of chance may recover his loss from
the winner, with legal interest from the time he paid the amount lost. This means that
only he can file the suit. Mrs. ZY cannot recover as a spouse who has interest in the
absolute community property or conjugal partnership of gains, because under Art.
117(7} of the Family Code, losses are borne exclusively by the loser-spouse.
Therefore, these cannot be charged against absolute community property or conjugal
partnership of gains. This being so, Mrs. ZY has no interest in law to prosecute and
recover as she has no legal standing in court to do so.

Notes:
Under the Civil Code, losses in gambling or betting shall exclusively be borne
by the losing-spouse. Therefore, the wife cannot recover the loss of her husband in
gambling.
Conditional Obligations (2000)
Pedro promised to give his grandson a car if the latter will pass the bar
examinations. When his grandson passed the said examinations, Pedro refused to
give the car on the ground that the condition was a purely potestative one. Is he
correct or not? (2%)

SUGGESTED ANSWER:
No, he is not correct. First of all, the condition is not purely potestative, because
it does not depend on the sole will of one of the parties. Secondly, even if it were, it
would be valid because it depends on the sole will of the creditor (the donee) and not
of the debtor (the donor).

Notes:
Under the Civil Code, an obligation which depends upon the happening of an
event shall be valid. Therefore, Pedro shall give his car to his grandson when the latter
passed the bar examinations.

Conditional Obligations (2003)


Are the following obligations valid, why, and if they are valid, when is the
obligation demandable in each case?
a) If the debtor promises to pay as soon as he has the means to pay; valid with a
period, period will be fixed by court
b) If the debtor promises to pay when he likes; invalid, potestative
c) If the debtor promises to pay when he becomes a lawyer; valid, mix
d) If the debtor promises to pay if his son, who is sick with cancer, does not die
within one year. 5% valid

SUGGESTED ANSWER:
(a) The obligation is valid. It is an obligation subject to an indefinite period
because the debtor binds himself to pay when his means permit him to do so (Article
1180, NCC). When the creditor knows that the debtor already has the means to
pay, he must file an action in court to fix the period, and when the definite period
as set by the court arrives, the obligation to pay becomes demandable. (Article
1197, NCC).

Notes:
Under the Civil Code, when a debtor binds himself to pay when his means
permit him to do so, it is deemed to have a period. Thus, the obligation is valid.

ARTICLE 1180. When the debtor binds himself to pay when his means permit
him to do so, the obligation shall be deemed to be one with a period, subject to the
provisions of article 1197.

ARTICLE 1187. The effects of a conditional obligation to give shall retroact to


the day of the constitution of the obligation once the condition has been fulfilled.

Nevertheless, when the obligation imposes reciprocal prestations (performance)


upon the parties, the fruits and interests during the pendency of the condition are
mutually compensated.

If the obligation is unilateral, the debtor shall appropriate the fruits and interests
received, unless from the nature and circumstances of the obligation it should be
inferred that the intention of the person constituting the same was different.

In each case and in obligations to do and not to do, the courts shall determine the
retroactive effect of the condition that has been complied with. (Reengineered)

SUGGESTED ANSWER:
(b) The obligation “to pay when he likes” is a suspensive condition the
fulfillment of which is subject to the sole will of the debtor and, therefore the
conditional obligation is void. (Article 1182, NCC).

Notes:
ARTICLE 1182. When the fulfillment of the condition depends upon the sole
will of the debtor, the conditional obligation shall be void. If it depends upon chance
or upon the will of a third person, the obligation shall take effect in conformity with
the provisions of this Code.

Notes:
When the fulfillment of the condition depends upon the sole will of the debtor,
the conditional obligation shall be void.
If the fulfilment of the condition depends upon chance, the obligation is valid. If
the fulfilment of the obligation depends upon the will of a third person, the obligation
is also valid.

SUGGESTED ANSWER:
(c) The obligation is valid. It is subject to a suspensive condition, i.e. the future
and uncertain event of his becoming a lawyer. The performance (prestation) of
this obligation does not depend solely on the will of the debtor but also on other
factors outside the debtor’s control.

SUGGESTED ANSWER:
(d) The obligation is valid. The death of the son of cancer within one year is
made a negative suspensive condition to his making the payment. The obligation is
demandable if the son does not die within one year (Article 1185, NCC).

Notes:
Under the Civil Code, when an obligation depends upon a condition that some
event will not happen at a determinate time, the obligation shall become demandable
from the moment the time elapsed without the happening of the event.

Ex: The condition which depends upon an event which will not happen like
when the child does not die of cancer within one year is a valid condition. It becomes
demandable when the child does not die within one year.

ARTICLE 1185. The condition that some event will not happen at a
determinate time shall render the obligation effective from the moment the time
indicated has elapsed, or if it has become evident that the event cannot occur
negative suspensive condition.

Notes:
The obligation becomes effective when the time as a condition elapsed and the
event did not happen; it will also become effective if it is evident that the event will
not occur.
Ex: I will give you my car when Duterte does not die until the end of his term. If
Duterte did not die until he finished his term, the obligation to give the car becomes
effective.

Conditional Obligations; Promise (1997)


In two separate documents signed by him, Juan Valentino "obligated" himself
each to Maria and to Perla, thus - 'To Maria, my true love, I obligate myself to give
you my one and only horse when I feel like It." - and -'To Perla, my true sweetheart, I
obligate myself to pay you the P500.00 I owe you when I feel like it." Months passed
but Juan never bothered to make good his promises. Maria and Perla came to
consult you on whether or not they could recover on the basis of the foregoing
settings. What would your legal advice be?

SUGGESTED ANSWER:
I would advise Maria not to bother running after Juan for the latter to make good
his promise. [This is because a promise is not an actionable wrong that allows a party
to recover especially when she has not suffered damages resulting from such promise.
A promise does not create an obligation on the part of Juan because it is not
something which arises from a contract, law, quasi-contracts or quasidelicts (Art,
1157)]. Under Art. 1182, Juan's promise to Maria is void because a conditional
obligation depends upon the sole will of the obligor.

As regards Perla, the document is an express acknowledgment of a debt, and


the promise to pay what he owes her when he feels like it is equivalent to a promise
to pay when his means permits him to do so, and is deemed to be one with an
indefinite period under Art. 1180. Hence the amount is recoverable after Perla
asks the court to set the period as provided by Art. 1197, par. 2.

Notes:
Under the Civil Code, a conditional obligation which depends upon the sole will
of a debtor is void. In other words, a promise which depends upon the sole will of a
debtor does not bind him because a promise does not arise from any of the sources of
obligation.

Ex: a promise which says that I will give you my ring when I like it is a void
obligation because the fulfilment of the condition depends upon the sole will of the
debtor. Here, the obligation is not demandable.

I promise you to pay my debt when I like it is an acknowledgement of debt;


courts may fix the period within which the debtor shall pay his debt.

ARTICLE 1157. Obligations arise from:

(1) Law;
(2) Contracts;

(3) Quasi-contracts;

(4) Acts or omissions punished by law; and

(5) Quasi-delicts.

ARTICLE 1180. When the debtor binds himself to pay when his means permit
him to do so, the obligation shall be deemed to be one with a period, subject to the
provisions of Article 1197 indefinite period

ARTICLE 1182. When the fulfillment of the condition depends upon the sole
will of the debtor, the conditional obligation shall be void. If it depends upon chance
or upon the will of a third person, the obligation shall take effect in conformity with
the provisions of this Code.

ARTICLE 1197. If the obligation does not fix a period, the courts may fix the
duration thereof if from its nature and the circumstances it can be inferred that a
period was intended. (Reengineered)

Conditional Obligations; Resolutory Condition (1999)


In 1997, Manuel bound himself to sell Eva a house and lot which is being
rented by another person, if Eva passes the 1998 bar examinations. Luckily for Eva,
she passed said examinations.

(a) Suppose Manuel had sold the same house and lot to another before Eva
passed the 1998 bar examinations, is such sale valid? Why? (2%) it is valid but
subject to the resolutory condition of Eva passing the bar

(b) Assuming that it is Eva who is entitled to buy said house and lot, is she
entitled to the rentals collected by Manuel before she passed the 1998 bar
examinations? Why? (3%) no. there was not yet sale

SUGGESTED ANSWER:
(a) Yes, the sale to the other person is valid as a sale with a resolutory condition
because what operates as a suspensive condition for Eva operates a resolutory
condition for the buyer.

Notes:
In suspensive condition, the happening of the event will make the obligation
effective, while in resolutory condition, the happening of the event will extinguish the
obligation.

FIRST ALTERNATIVE ANS WER:


Yes, the sale to the other person is valid. However, the buyer acquired the
property subject to a resolutory condition of Eva passing the 1998 Bar Examinations.
Hence, upon Eva's passing the Bar, the rights of the other buyer terminated and Eva
acquired ownership of the property.

SECOND ALTERNATIVE ANSWER:


The sale to another person before Eva could buy it from Manuel is valid, as the
contract between Manuel and Eva is a mere promise to sell and Eva has not acquired a
real right over the land assuming that there is a price stipulated in the contract for the
contract to be considered a sale and there was delivery or tradition of the thing sold.

SUGGESTED ANSWER:
(b) No, she is not entitled to the rentals collected by Manuel because at the time
they accrued and were collected, Eva was not yet the owner of the property.

FIRST ALTERNATIVE ANSWER:


Assuming that Eva is the one entitled to buy the house and lot, she is not entitled
to the rentals collected by Manuel before she passed the bar examinations. Whether it
is a contract of sale or a contract to sell, reciprocal prestations are deemed imposed A
for the seller to deliver the object sold and for the buyer to pay the price. Before the
happening of the condition, the fruits of the thing and the interests on the money are
deemed to have been mutually compensated under Article 1187.

SECOND ALTERNATIVE ANSWER:


Under Art. 1164, there is no obligation on the part of Manuel to deliver the fruits
(rentals) of the thing until the obligation to deliver the thing arises. As the suspensive
condition has not been fulfilled, the obligation to sell does not arise.

Notes:
Under the Civil Code, the creditor has a right to the fruits of the thing from the
time the obligation to deliver it arises.
ARTICLE 1164. The creditor has a right to the fruits of the thing from the time
the obligation to deliver it arises. However, he shall acquire no real right over it until
the same has been delivered to him.

EXTINGUISHMENT OF OBLIGATIONS

Extinguishment; Assignment of Rights (2001)


The sugar cane planters of Batangas entered into a long-term milling contract
with the Central Azucarera de Don Pedro Inc. Ten years later, the Central assigned
its rights to the said milling contract to a Taiwanese group which would take over the
operations of the sugar mill. The planters filed an action to annul the said
assignment on the ground that the Taiwanese group was not registered with the
Board of Investments. Will the action prosper or not? Explain briefly. (5%)

(Note: The question presupposes knowledge and requires the application of the
provisions of the Omnibus Investment Code, which properly belongs to Commercial
law)
SUGGESTED ANSWER:
The action will prosper not on the ground invoked but on the ground that the
farmers have not given their consent to the assignment. The milling contract imposes
reciprocal obligations on the parties. The sugar central has the obligation to mill the
sugar cane of the farmers while the latter have the obligation to deliver their sugar
cane to the sugar central. As to the obligation to mill the sugar cane, the sugar central
is a debtor of the farmers. In assigning its rights under the contract, the sugar central
will also transfer to the Taiwanese its obligation to mill the sugar cane of the farmers.
This will amount to a novation of the contract by substituting the debtor with a third
party. Under Article 1293 of the Civil Code, such substitution cannot take effect
without the consent of the creditor. The formers, who are creditors as far as the
obligation to mill their sugar cane is concerned, may annul such assignment for not
having given their consent thereto.

Notes:
Under the Civil Code, novation which consists in substituting a new debtor in
the place of the original one may not be made without the consent of the creditor.
Therefore, the assignment of the milling contract without the consent of the planters
who are the creditors in this case is not valid.
ARTICLE 1293. Novation which consists in substituting a new debtor in the
place of the original one, may be made even without the knowledge or against the will
of the latter, but not without the consent of the creditor. Payment by the new debtor
gives him the rights mentioned in articles 1236 and 1237.
Novation arises when a new individual assumes an obligation to pay that was
incurred by the original party to the contract.

Under the Civil Code, in case of novation, the creditor should give his consent.
Without his consent, the novation is not valid.

In this case, the sugar planters are the creditors of the Central Azucarera de
Don Pedro Inc. The Central cannot assign its rights to the to a Taiwanese group to
take over the operations of the sugar mill without the consent of the planters.

ALTERNATIVE ANSWER:
The assignment is valid because there is absolute freedom to transfer the credit
and the creditor need not get the consent of the debtor. He only needs to notify him.

Extinguishment; Cause of Action (2004)


TX filed a suit for ejectment against BD for non-payment of condominium
rentals amounting to P150,000. During the pendency of the case, BD offered and TX
accepted the full amount due as rentals from BD, who then filed a motion to dismiss
the ejectment suit on the ground that the action is already extinguished. Is BD’s
contention correct? Why or why not? Reason. (5%)

SUGGESTED ANSWER:
BD's contention is not correct. TX can still maintain the suit for ejectment. The
acceptance by the lessor of the payment by the lessee of the rentals in arrears even
during the pendency of the ejectment case does not constitute a waiver or
abandonment of the ejectment case. (Spouses Clutario v. CA, 216 SCRA 341
[1992]).

Notes:
In Spouses Clutario v. CA, the Supreme Court held that payment of the arrears
in the ejectment case does not extinguish the cause of action. Therefore, the action for
ejectment is not extinguish even though the lessee had paid his arrears to the lessor.
Extinguishment; Compensation (2002)
Stockton is a stockholder of Core Corp. He desires to sell his shares in Core
Corp. In view of a court suit that Core Corp. has filed against him for damages in the
amount of P 10 million, plus attorney’s fees of P 1 million, as a result of statements
published by Stockton which are allegedly defamatory because it was calculated to
injure and damage the corporation’s reputation and goodwill. The articles of
incorporation of Core Corp. provide for a right of first refusal in favor of the
corporation. Accordingly, Stockton gave written notice to the corporation of his offer
to sell his shares of P 10 million. The response of Core corp. was an acceptance of the
offer in the exercise of its rights of first refusal, offering for the purpose payment in
form of compensation or set-off against the amount of damages it is claiming against
him, exclusive of the claim for attorney’s fees. Stockton rejected the offer of the
corporation, arguing that compensation between the value of the shares and the
amount of damages demanded by the corporation cannot legally take effect. Is
Stockton correct? Give reason for your answer. (5%)

SUGGESTED ANSWERS:
Stockton is correct. There is no right of compensation between his price of P10
million and Core Corp.’s unliquidated claim for damages. In order that
compensation may be proper, the two debts must be liquidated and demandable. The
case for the P 10million damages being still pending in court, the corporation has as
yet no claim which is due and demandable against Stockton.

Notes:
Compensation as a mode of extinguishing an obligation may take place when
both debts are due and demandable. In this case, the case for damages against
Stockton is still pending. There was no decision yet awarding P 10 million pesos to
the corporation. Thus, the claim of compensation by the corporation is without merit
since the expected award is not yet demandable.

ANOTHER MAIN ANSWER:


The right of first refusal was not perfected as a right for the reason that there was
a conditional acceptance equivalent to a counter-offer consisting in the amount of
damages as being credited on the purchase price. Therefore, compensation did not
result since there was no valid right of first refusal (Art. 1475 & 1319, NCC)

ARTICLE 1475. The contract of sale is perfected at the moment there is a


meeting of minds upon the thing which is the object of the contract and upon the
price.
From that moment, the parties may reciprocally demand performance, subject to
the provisions of the law governing the form of contracts.

ANOTHER MAIN ANSWER:


Even [if] assuming that there was a perfect right of first refusal, compensation
did not take place because the claim is unliquidated.
Extinguishment; Compensation vs. Payment (1998)
Define compensation as a mode of extinguishing an obligation, and
distinguish it from payment. [2%]

SUGGESTED ANSWER:
COMPENSATION is a mode of extinguishing to the concurrent amount, the
obligations of those persons who in their own right are reciprocally debtors and
creditors of each other (Tolentino, 1991 ed., p. 365, citing 2 Castan 560 and
Francia vs. IAC. 162 SCRA 753). It involves the simultaneous balancing of two
obligations in order to extinguish them to the extent in which the amount of one is
covered by that of the other. (De Leon, 1992 ed., p. 221, citing 8 Manresa 401).

PAYMENT means not only delivery of money but also performance of an


obligation (Article 1232, Civil Code). In payment, capacity to dispose of the thing
paid and capacity to receive payment are required for debtor and creditor,
respectively: in compensation, such capacity is not necessary, because the
compensation operates by law and not by the act of the parties. In payment, the
performance must be complete; while in compensation there may be partial
extinguishment of an obligation (Tolentino, supra)

Notes:
Under the Civil Code, compensation is a mode of extinguishing an obligation;
the obligations of persons who are reciprocally debtors and creditors of each other are
extinguished by compensation.

Extinguishment; Compensation/Set-Off; Banks (1998)


X, who has a savings deposit with Y Bank in the sum of P1,000,000.00 incurs a
loan obligation with the said Bank in the sum of P800.000.00 which has become due.
When X tries to withdraw his deposit, Y Bank allows only P200.000.00 to be
withdrawn, less service charges, claiming that compensation has extinguished its
obligation under the savings account to the concurrent amount of X's debt. X
contends that compensation is improper when one of the debts, as here, arises from a
contract of deposit. Assuming that the promissory note signed by X to evidence the
loan does not provide for compensation between said loan and his savings deposit,
who is correct? [3%]

SUGGESTED ANSWER:
Y bank is correct. Art. 1287, Civil Code, does not apply. All the requisites of
Art. 1279, Civil Code are present. In the case of Gullas vs. PNB [62 Phil. 519), the
Supreme Court held: "The Civil Code contains provisions regarding compensation
(set off) and deposit. These portions of Philippine law provide that compensation
shall take place when two persons are reciprocally creditor and debtor of each other.
In this connection, it has been held that the relation existing between a depositor and a
bank is that of creditor and debtor, x x x As a general rule, a bank has a right of set off
of the deposits in its hands for the payment of any indebtedness to it on the part of a
depositor." Hence, compensation took place between the mutual obligations of X and
Y bank.

Notes:
In Gullas vs. PNB, it was held that compensation shall take place when two
persons are reciprocally creditor and debtor of each other. As a general rule, a bank
has a right of set off of the deposits in its hands for the payment of any indebtedness
to it on the part of a depositor.

ARTICLE 1279. In order that compensation may be proper, it is necessary:

(1) That each one of the obligors is bound principally, and that he is at the same
time a principal creditor of the other;

(2) That both debts consist in a sum of money, or if the things due are
consumable, they are of the same kind, and also of the same quality if the latter has
been stated;

(3) That the two debts are due;

(4) That they are liquidated and demandable;

(5) That over neither of them there be any retention or controversy, commenced
by third persons and communicated in due time to the debtor.

Extinguishment; Condonation (2000)


Arturo borrowed P500,000.00 from his father. After he had paid P300,000.00,
his father died. When the administrator of his father's estate requested payment of the
balance of P200,000.00. Arturo replied that the same had been condoned by his
father as evidenced by a notation at the back of his check payment for the
P300,000.00 reading: "In full payment of the loan". Will this be a valid defense in an
action for collection? (3%)

SUGGESTED ANSWER:
It depends. If the notation "in full payment of the loan" was written by Arturo's
father, there was an implied condonation of the balance that discharges the obligation.
In such case, the notation is an act of the father from which condonation may be
inferred. The condonation being implied, it need not comply with the formalities of a
donation to be effective. The defense of full payment will, therefore, be valid.
When, however, the notation was written by Arturo himself. It merely proves his
intention in making that payment but in no way does it bind his father (Yam v. CA,
G.R No. 104726. 11 February 1999). In such case, the notation was not the act of his
father from which condonation may be inferred. There being no condonation at all the
defense of full payment will not be valid.

Notes:
Under the Civil Code, condonation is gratuitous which requires the acceptance
of the obligor. In this case, the son did not accept the condonation made by his father.
Therefore, the defense of the son that his obligation to pay is extinguished because of
the notation made by his father at the back of the promissory note is not valid since
there was no, in writing, acceptance of the condonation.

ARTICLE 1270. Condonation or remission is essentially gratuitous, and requires


the acceptance by the obligor. It may be made expressly or impliedly.

One and the other kinds shall be subject to the rules which govern inofficious
donations. Express condonation shall, furthermore, comply with the forms of
donation.

ALTERNATIVE ANSWER:
If the notation was written by Arturo's father, it amounted to an express
condonation of the balance which must comply with the formalities of a donation to
be valid under the 2nd paragraph of Article 1270 of the New Civil Code. Since the
amount of the balance is more than 5,000 pesos, the acceptance by Arturo of the
condonation must also be in writing under Article 748. There being no acceptance in
writing by Arturo, the condonation is void and the obligation to pay the balance
subsists. The defense of full payment is, therefore, not valid. In case the notation was
not written by Arturo's father, the answer is the same as the answers above.
Extinguishment; Extraordinary Inflation or Deflation (2001)
On July 1, 1998, Brian leased an office space in a building for a period of five
years at a rental rate of P1,000.00 a month. The contract of lease contained the proviso
that "in case of inflation or devaluation of the Philippine peso, the monthly rental will
automatically be increased or decreased depending on the devaluation or inflation of
the peso to the dollar." Starting March 1, 2001, the lessor increased the rental to
P2,000 a month, on the ground of inflation proven by the fact that the exchange rate of
the Philippine peso to the dollar had increased from P25.00=$1.00 to P50.00=$1.00.
Brian refused to pay the increased rate and an action for unlawful detainer was
filed against him. Will the action prosper? Why? (5%)

SUGGESTED ANSWER:
The unlawful detainer action will not prosper. Extraordinary inflation or
deflation is defined as the sharp decrease in the purchasing power of the peso. It does
not necessarily refer to the exchange rate of the peso to the dollar. Whether or not
there exists an extraordinary inflation or deflation is for the courts to decide. There
being no showing that the purchasing power of the peso had been reduced
tremendously, there could be no inflation that would justify the increase in the amount
of rental to be paid. Hence, Brian could refuse to pay the increased rate.

Notes:
Under the Civil Code, under the rule on obligatoriness of contract, a contract has
the force of law. In this case, the parties stipulated that in case of inflation or
devaluation, the rental will increase. The fact clearly states that there was inflation.
Therefore, the refusal of the lessee to pay an increase is a ground for ejectment.

(This topic was discussed by Atty. Uribe).

ALTERNATIVE ANSWER:
The action will not prosper. The existence of inflation or deflation requires an
official declaration by the Bangko Sentral ng Pilipinas.

ALTERNATIVE ANSWER:
The unlawful detainer action will prosper. It is a given fact in the problem, that
there was inflation, which caused the exchange rate to double. Since the contract itself
authorizes the increase in rental in the event of an inflation or devaluation of the
Philippine peso, the doubling of the monthly rent is reasonable and is therefore a valid
act under the very terms of the contract. Brian's refusal to pay is thus a ground for
ejectment.

Extinguishment; Loss (1994)


Dino sued Ben for damages because the latter had failed to deliver the antique
Marcedes Benz car Dino had purchased from Ben, which was—by agreement—due
for delivery on December 31, 1993. Ben, in his answer to Dino's complaint, said
Dino's claim has no basis for the suit, because as the car was being driven to be
delivered to Dino on January 1, 1994, a reckless truck driver had rammed into the
Mercedes Benz. The trial court dismissed Dino's complaint, saying Ben's obligation
had indeed, been extinguished by force majeure. Is the trial court correct?

SUGGESTED ANSWER:
No. Article 1262, New Civil Code provides, "An obligation which consists in
the delivery of a determinate thing shall be extinguished if it should be lost or
destroyed without the fault of the debtor, and before he has incurred in delay.

b) The judgment of the trial court is incorrect. Loss of the thing due by fortuitous
events or force majeure is a valid defense for a debtor only when the debtor has not
incurred delay. Extinguishment of liability for fortuitous event requires that the debtor
has not yet incurred any delay. In the present case, the debtor was in delay when the
car was destroyed on January 1, 1993 since it was due for delivery on December 31,
1993. (Art. 1262 Civil Code)

c) It depends whether or not Ben the seller, was already in default at the time of
the accident because a demand for him to deliver on due date was not complied with
by him. That fact not having been given in the problem, the trial court erred in
dismissing Dino's complaint. Reason: There is default making him responsible for
fortuitous events including the assumption of risk or loss.

If on the other hand Ben was not in default as no demand has been sent to him
prior to the accident, then we must distinguish whether the price has been paid or not.
If it has been paid, the suit for damages should prosper but only to enable the buyer to
recover the price paid. It should be noted that Ben, the seller, must bear the loss on the
principle of res perit domino. He cannot be held answerable for damages as the loss of
the car was not imputable to his fault or fraud. In any case, he can recover the value of
the car from the party whose negligence caused the accident. If no price has been paid
at all, the trial court acted correctly in dismissing the complaint.
Notes:
Under the Civil Code, an obligation to deliver a determinate thing is
extinguished upon its lost provided the obligor did not incur in delay.

ARTICLE 1262. An obligation to deliver a determinate thing shall be


extinguished if it is lost or destroyed without the fault of the debtor, and before he has
incurred in delay. (Reengineered)

When by law or stipulation, the obligor is liable even for fortuitous events, the
loss of the thing does not extinguish the obligation, and he shall be responsible for
damages. The same rule applies when the nature of the obligation requires the
assumption of risk.

Extinguishment; Loss; Impossible Service (1993)


In 1971, Able Construction, Inc. entered into a contract with Tropical Home
Developers, Inc. whereby the former would build for the latter the houses within its
subdivision. The cost of each house, labor and materials included, was P100,000.00.
Four hundred units were to be constructed within five years. In 1973, Able found that
it could no longer continue with the job due to the increase in the price of oil and its
derivatives and the concomitant worldwide spiraling of prices of all commodities,
including basic raw materials required for the construction of the houses. The cost of
development had risen to unanticipated levels and to such a degree that the conditions
and factors which formed the original basis of the contract had been totally changed.
Able brought suit against Tropical Homes praying that the Court relieve it of its
obligation. Is Able Construction entitled to the relief sought?

SUGGESTED ANSWER:
Yes, the Able Construction. Inc. is entitled to the relief sought under Article
1267, Civil Code. The law provides: "When the service has become so difficult as to
be manifestly beyond the contemplation of the parties, the obligor may also be
released therefrom, in whole or in part."

Notes:
Under the Civil Code, an obligor is released from obligation if the service
becomes so difficult which was beyond the contemplation of the parties.
ARTICLE 1267. When the service has become so difficult as to be manifestly
beyond the contemplation of the parties, the obligor may also be released therefrom,
in whole or in part.

Extinguishment; Novation (1994)


In 1978, Bobby borrowed Pl,000,000.00 from Chito payable in two years. The
loan, which was evidenced by a promissory note, was secured by a mortgage on real
property. No action was filed by Chito to collect the loan or to foreclose the mortgage.
But in 1991, Bobby, without receiving any amount from Chito, executed another
promissory note which was worded exactly as the 1978 promissory note, except for
the date thereof, which was the date of its execution.

1) Can Chito demand payment on the 1991 promissory note in 1994?


2) Can Chito foreclose the real estate mortgage if Bobby fails to make good
his obligation under the 1991 promissory note?

SUGGESTED ANSWER:
1) Yes, Chito can demand payment on the 1991 promissory note in 1994.
Although the 1978 promissory note for P1 million payable two years later or in 1980
became a natural obligation after the lapse of ten (10) years, such natural obligation
can be a valid consideration of a novated promissory note dated in 1991 and payable
two years later, or in 1993. All the elements of an implied real novation are present:
a) an old valid obligation;
b) a new valid obligation;
c) capacity of the parties;
d) animus novandi or intention to novate; and
e) The old and the new obligation should be incompatible with each other on all
material points (Article 1292). The two promissory notes cannot stand together,
hence, the period of prescription of ten (10) years has not yet lapsed.

ARTICLE 1292. In order that an obligation may be extinguished by another


which substitutes the same, it is imperative that it be so declared in unequivocal terms,
or that the old and the new obligations be on every point incompatible with each
other.

Notes:
Novation is the act of substituting a valid existing contract with a replacement
contract, where all concerned parties mutually agree to make the switch.
The two kinds of novation are: (1) it substitutes the party whether the obligor or
the obligee; and (2) it substitutes the obligation like substitution of the older debt
which has become a natural obligation.
SUGGESTED ANSWER:

2) No. The mortgage being an accessory contract prescribed with the loan. The
novation of the loan, however, did not expressly include the mortgage, hence, the
mortgage is extinguished under Article 1296 of the NCC. The contract has been
extinguished by the novation or extinction of the principal obligation insofar as third
parties are concerned.

Notes:
Under the Civil Code, in case of a novation of a principal contract, the accessory
contract is extinguished; however, the accessory contract may subsist in favor of a
third person who did not give his consent to the novation.

ARTICLE 1296. When the principal obligation is extinguished in consequence


of a novation, accessory obligations may subsist only insofar as they may benefit third
persons who did not give their consent.

Extinguishment; Payment (1995)


In 1983, PHILCREDIT extended loans to Rivett-Strom Machineries, Inc.
(RIVETTT-STROM), consisting of US$10 Million for the cost of machineries
imported and directly paid by PHTLCREDIT, and 5 Million in cash payable in
installments over a period of ten (10) years on the basis of the value thereof computed
at the rate of exchange of the U.S. dollar vis-à-vis the Philippine peso at the time of
payment.

RIVETT-STROM made payments on both loans which if based on the rate of


exchange in 1983 would have fully settled the loans.

PHILCREDIT contends that the payments on both loans should be based on the
rate of exchange existing at the time of payment, which rate of exchange has been
consistently increasing, and for which reason there would still be a considerable
balance on each loan. Is the contention of PHILCREDIT correct? Discuss fully.

SUGGESTED ANSWER:
As regards the loan consisting of dollars, the contention of PHILCREDIT is
correct. It has to be paid in Philippine currency computed on the basis of the
exchange rate at the TIME OF PAYMENT of each installment, as held in Kalalo v.
Luz, 34 SCRA 337. As regards the P5 Million loan in Philippine pesos,
PHILCREDIT is wrong. The payment thereof cannot be measured by the peso-dollar
exchange rate. That will be violative of the Uniform Currency Act (RA, 529] which
prohibits the payment of an obligation which, although to be paid in Philippine
currency, is measured by a foreign currency. (Palanca v. CA, 238 SCRA 593).

Notes:
In Kalalo v. Luz, it was held that when a loan is consisting of dollars, it shall be
paid and measured at the rate of exchange at the time of the payment. Thus, the
contention of the debtor is with merit.

However, in Palanca v. CA and under the Uniform Currency Act, payment in


Philippine currency to be measured in foreign currency is prohibited. Thus, the
contention of the debtor is not meritorious.

JOINT AND SOLIDARY OBLIGATION

Liability; Lease; Joint Liability (2001)


Four foreign medical students rented the apartment of Thelma for a period of
one year. After one semester, three of them returned to their home country and the
fourth transferred to a boarding house. Thelma discovered that they left unpaid
telephone bills in the total amount of P80,000.00. The lease contract provided that the
lessees shall pay for the telephone services in the leased premises. Thelma demanded
that the fourth student pay the entire amount of the unpaid telephone bills, but
the latter is willing to pay only one fourth of it. Who is correct? Why? (5%)

SUGGESTED ANSWER:
The fourth student is correct. His liability is only joint, hence, pro rata. There is
solidary liability only when the obligation expressly so states or when the law or
nature of the obligation requires solidarity (Art. 1207, CC). The contract of lease in
the problem does not, in any way, stipulate solidarity.

Notes:
Under the Civil Code, an obligation is solidary when it expressly so states or
when the nature of the obligation is solidary.
ARTICLE 1207. The concurrence of two or more creditors or of two or more
debtors in one and the same obligation does not imply that each one of the former has
a right to demand, or that each one of the latter is bound to render, entire compliance
with the prestation.

There is a solidary liability only when the obligation expressly so states, or when
the law or the nature of the obligation requires solidarity.

Liability; Solidary Liability (1998)


Joey, Jovy and Jojo are solidary debtors under a loan obligation of
P300,000.00 which has fallen due. The creditor has, however, condoned Jojo's entire
share in the debt. Since Jovy has become insolvent, the creditor makes a demand on
Joey to pay the debt.

1) How much, if any, may Joey be compelled to pay? [2%]


2) To what extent, if at all, can Jojo be compelled by Joey to contribute to
such payment? [3%]

SUGGESTED ANSWER:
1. Joey can be compelled to pay only the remaining balance of P200.000, in
view of the remission of Jojo's share by the creditor. (Art. 1219, Civil Code)

2. Jojo can be compelled by Joey to contribute P50.000 Art. 1217. par. 3, Civil
Code provides. "When one of the solidary debtors cannot, because of his
insolvency, reimburse his share to the debtor paying the obligation, such share
shall be borne by all his co-debtors, in proportion to the debt of each."

Since the insolvent debtor's share which Joey paid was P100,000, and there are
only two remaining debtors - namely Joey and Jojo - these two shall share equally the
burden of reimbursement. Jojo may thus be compelled by Joey to contribute
P50.000.00.

Notes:
Under the Civil Code, all co-debtors shall pay the share of a co-debtor who
became insolvent. Such co-debtors shall pay in proportion to their share.
ARTICLE 1217. Payment made by one of the solidary debtors extinguishes the
obligation. If two or more solidary debtors offer to pay, the creditor may choose which
offer to accept.

He who made the payment may claim from his co-debtors only the share which
corresponds to each, with the interest for the payment already made. If the payment is
made before the debt is due, no interest for the intervening period may be demanded.

When one of the solidary debtors cannot pay his share to the solidary debtor who
pays the obligation because of his insolvency, such share shall be borne by all his co-
debtors in proportion to the debt of each.

Liability; Solidary Obligation (1992)


In June 1988, X obtained a loan from A and executed with Y as solidary co-
maker a promissory note in favor of A for the sum of P200,000.00. The loan was
payable at P20,000.00 with interest monthly within the first week of each month
beginning July 1988 until maturity in April 1989. To secure the payment of the loan,
X put up as security a chattel mortgage on his car, a Toyota Corolla sedan. Because of
failure of X and Y to pay the principal amount of the loan, the car was extrajudicially
foreclosed. A acquired the car at A's highest bid of P120,000.00 during the auction
sale.

After several fruitless letters of demand against X and Y, A sued Y alone for the
recovery of P80.000.00 constituting the deficiency. Y resisted the suit raising the
following defenses:

a) That Y should not be liable at all because X was not sued together with
Y.

b) That the obligation has been paid completely by A's acquisition of the car
through "dacion en pago" or payment by cession.
c) That Y should not be held liable for the deficiency of P80,000.00 because
he was not a co-mortgagor in the chattel mortgage of the car which contract was
executed by X alone as owner and mortgagor.
d) That assuming that Y is liable, he should only pay the proportionate sum
of P40,000.00. Decide each defense with reasons.

SUGGESTED ANSWER:
(a) This first defense of Y is untenable. Y is still liable as solidary debtor. The
creditor may proceed against any one of the solidary debtors. The demand against
one does not preclude further demand against the others so long as the debt is not fully
paid.

(b) The second defense of Y is untenable. Y is still liable. The chattel mortgage
is only given as a security and not as payment for the debt in case of failure to pay. Y
as a solidary co-maker is not relieved of further liability on the promissory note as a
result of the foreclosure of the chattel mortgage.

(c) The third defense of Y is untenable. Y is a surety of X and the extrajudicial


demand against the principal debtor is not inconsistent with a judicial demand against
the surety. A suretyship may co-exist with a mortgage.

(d) The fourth defense of Y is untenable. Y is liable for the entire prestation
since Y incurred a solidary obligation with (Arts. 1207, 1216. 1252 and 2047 Civil
Code; Bicol Savings and Loan Associates vs. Guinhawa 188 SCRA 642)

Notes:
Under the Civil Code, a solidary debtor can be sued for the payment of the
whole obligation; he can be sued alone or together with one or all of the solidary
debtors; he can be sued for the deficiency of the payment of other solidary debtors. A
surety is a solidary debtor.

ARTICLE 1207. The concurrence of two or more creditors or of two or more


debtors in one and the same obligation does not imply that each one of the former has
a right to demand, or that each one of the latter is bound to render, entire compliance
with the prestation.

There is a solidary liability only when the obligation expressly so states, or when
the law or the nature of the obligation requires solidarity.

ARTICLE 1216. The creditor may proceed against any one of the solidary
debtors or some or all of them simultaneously. The demand made against one of them
shall not be an obstacle to those which may subsequently be directed against the
others, so long as the debt has not been fully collected.

ARTICLE 1252. He who has various debts of the same kind in favor of one and
the same creditor, may declare at the time of making the payment, to which of them
the same must be applied. Unless the parties so stipulate, or when the application of
payment is made by the party for whose benefit the term has been constituted,
application shall not be made as to debts which are not yet due.
If the debtor accepts from the creditor a receipt in which an application of the payment
is made, the former cannot complain of the same, unless there is a cause for
invalidating the contract.

ARTICLE 2047. By guaranty, a person, called the guarantor, binds himself to


the creditor to fulfill the obligation of the principal debtor in case the latter should fail
to do so.

If a person binds himself solidarily with the principal debtor, the provisions of
Section 4, Chapter 3, Title I of this Book shall be observed. In such case the contract
is called a suretyship.

Liability; Solidary Obligation; Mutual Guaranty (2003)


A,B,C,D, and E made themselves solidarily indebted to X for the amount of
P50,000.00. When X demanded payment from A, the latter refused to pay on the
following grounds.
a) B is only 16 years old.
b) C has already been condoned by X
c) D is insolvent.
d) E was given by X an extension of 6 months without the consent of the other
four co-debtors.

State the effect of each of the above defenses put up by A on his obligation
to pay X, if such defenses are found to be true.

SUGGESTED ANSWERS:
(a) A may avail the minority of B as a defense, but only for B’s share of P
10,000.00. A solidary debtor may avail himself of any defense which personally
belongs to a solidary co-debtor, but only as to the share of that co-debtor.

(b) A may avail of the condonation by X of C’s share of P 10, 000.00. A


solidary debtor may, in actions filed by the creditor, avail himself of all defenses
which are derived from the nature of the obligation and of those which are personal to
him or pertain to his own share. With respect to those which personally belong to
others, he may avail himself thereof only as regards that part of the debt for which the
latter are responsible. (Article 1222, NCC).
(c) A may not interpose the defense of insolvency of D as a defense. Applying
the principle of mutual guaranty among solidary debtors, A guaranteed the payment
of D’s share and of all the other co-debtors. Hence, A cannot avail of the defense of
D’s insolvency.

(d) The extension of six (6) months given by X to E may be availed of by A as a


partial defense but only for the share of E, there is no novation of the obligation but
only an act of liberality granted to E alone.

Notes:
Under the Civil Code, a co-debtor may avail himself of a defense personal to
another co-debtor like minority; he may likewise avail the defense of condonation or
remission if any; in case of extension of the period to pay extended by the creditor to
a co-debtor, he may use it as a partial defense. However, he may not use the
insolvency of a co-debtor as a defense under the rule on mutual guarantee.

ARTICLE 1222. A solidary debtor may, in actions filed by the creditor, avail
himself of all defenses which are derived from the nature of the obligation and of
those which are personal to him, or pertain to his own share. With respect to those
which personally belong to the others, he may avail himself thereof only as regards
that part of the debt for which the latter are responsible.

Loss of the thing due; Force Majeure (2000)


Kristina brought her diamond ring to a jewelry shop for cleaning. The jewelry
shop undertook to return the ring by February 1, 1999." When the said date arrived,
the jewelry shop informed Kristina that the Job was not yet finished. They asked her
to return five days later. On February 6, 1999, Kristina went to the shop to claim the
ring, but she was informed that the same was stolen by a thief who entered the shop
the night before. Kristina filed an action for damages against the jewelry shop which
put up the defense of force majeure. Will the action prosper or not? (5%)

SUGGESTED ANSWER:
The action will prosper. Since the defendant was already in default not having
delivered the ring when delivery was demanded by plaintiff at due date, the defendant
is liable for the loss of the thing and even when the loss was due to force majeure.
Notes:
Under the Civil Code, the obligation to deliver a determinate thing is
extinguished due to loss of such thing provided the obligor did not incur in delay. In
this case, the shop did not finish the work at the agreed date. Therefore, he is liable for
the loss of the jewelry because it incurred in delay.

Non-Payment of Amortizations; Subdivision Buyer; When justified (2005)


Bernie bought on installment a residential subdivision lot from DEVLAND.
After having faithfully paid the installments for 48 months, Bernie discovered that
DEVLAND had failed to develop the subdivision in accordance with the approved
plans and specifications within the time frame in the plan. He thus wrote a letter to
DEVLAND informing it that he was stopping payment. Consequently, DEVLAND
cancelled the sale and wrote Bernie, informing him that his payments are forfeited in
its favor.

a) Was the action of DEVLAND proper? Explain. (2%)

SUGGESTED ANSWER:
No, the action of DEVLAND is not proper. Under Section 23 of Presidential
Decree No. 957, otherwise known as the Subdivision and Condominium Buyer's
Protection Decree, non-payment of amortizations by the buyer is justified if non-
payment is due to the failure of the subdivision owner to develop the subdivision
project according to the approved plans and within the limit for complying. (Eugenio
v. Drilon, G.R. No. 109404, January 22, 1996)

Notes:
According to Subdivision and Condominium Buyer’s Protection Decree, a buyer
of the condominium is justified not to pay the amortization if the nonpayment was due
to the failure of the condominium owner to develop the condominium in accordance
with the approved plans.

b) Discuss the rights of Bernie under the circumstances. (2%)

SUGGESTED ANSWER:
Under P.D. No. 957, a cancellation option is available to Bernie. If Bernie opts
to cancel the contract, DEVLAND must reimburse Bernie the total amount paid and
the amortizations interest, excluding delinquency interest, plus interest at legal rate.
(Eugenio v. Drilon, G.R. No. 109404, January 22, 1996)
Notes:
In Eugenio v. Drilon, it was held that if a buyer of a subdivision wants to cancel
his contract because the developer did not develop the subdivision according to the
approved plans, the later shall reimburse the former what he has paid and the
amortizations interest.

c) Supposing DEVLAND had fully developed the subdivision but Bernie


failed to pay further installments after 4 years due to business reverses. Discuss
the rights and obligations of the parties. (2%)

SUGGESTED ANSWER:
In this case, pursuant to Section 24 of P.D. No. 957, R.A. No. 6552 otherwise
known as the Realty Installment Buyer Protection Act, shall govern. Under Section 3
thereof, Bernie is entitled: 1) to pay without additional interest the unpaid installments
due within a grace period of four (4) months or one month for every year of
installment paid; 2) if the contract is cancelled, Bernie is entitled to the refund of the
cash surrender value equal to 50% of the total payments made.

DEVLAND on the other hand has the right to cancel the contract after 30 days
from receipt by Bernie of notice of cancellation. DEVLAND is however obliged to
refund to Bernie 50% of the total payments made. (Rillo v. Court of Appeals, G.R.
No. 125347, June 19,1997)

Notes:
In Rillo v. Court of Appeals, the Supreme Court held that when the buyer of a
condominium unit failed to pay after 4 years of continuous payment, he may: 1) pay
without additional interest the unpaid installments due within a grace period of four
(4) months or one month for every year of installment paid; 2) if the contract is
cancelled, he is entitled to the refund of the cash surrender value equal to 50% of the
total payments made.

Period; Suspensive Period (1991)


In a deed of sale of a realty, it was stipulated that the buyer would construct a
commercial building on the lot while the seller would construct a private
passageway bordering the lot. The building was eventually finished but the seller
failed to complete the passageway as some of the squatters, who were already known
to be there at the time they entered into the contract, refused to vacate the premises. In
fact, prior to its execution, the seller filed ejectment cases against the squatters. The
buyer now sues the seller for specific performance with damages. The defense is
that the obligation to construct the passageway should be with a period which,
incidentally, had not been fixed by them, hence, the need for fixing a judicial
period.

Will the action for specific performance of the buyer against the seller
prosper?

SUGGESTED ANSWER:
No. the action for specific performance filed by the buyer is premature under
Art. 1197 of the Civil Code. If a period has not been fixed although contemplated by
the parties, the parties themselves should fix that period, failing in which, the Court
may be asked to fix it taking into consideration the probable contemplation of the
parties. Before the period is fixed, an action for specific performance is premature.

Notes:
Under the Civil Code, if the obligation does not fix a period, but from its nature
and the circumstances it can be inferred that a period was intended, the courts may fix
the duration thereof. In this case, the period was intended by the parties but it was not
fixed yet. Therefore, the action for specific performance is premature.

ARTICLE 1197. If the obligation does not fix a period, but from its nature and
the circumstances it can be inferred that a period was intended, the courts may fix the
duration thereof.

The courts shall also fix the duration of the period when it depends upon the will
of the debtor.

In every case, the courts shall determine such period as may under the
circumstances have been probably contemplated by the parties. Once fixed by the
courts, the period cannot be changed by them.

ALTERNATIVE ANSWER:
It has been held in Borromeo vs. CA (47 SCRA 69), that the Supreme Court
allowed the simultaneous filing of action to fix the probable contemplated period of
the parties where none is fixed in the agreement if this would avoid multiplicity of
suits. In addition, technicalities must be subordinated to substantial justice.

ALTERNATIVE ANSWER:
The action for specific performance will not prosper. The filing of the ejectment
suit by the seller was precisely in compliance with his obligations and should not,
therefore, be faulted if no decision has yet been reached by the Court on the matter.

TRUST
In law, a trust is a relationship where property is held by one party for the benefit
of another party; a trust is created by the owner, also called a “settlor”, “grantor” or
“trustor” who transfers property to a trustee . The trustee holds that property for the
trust’s beneficiaries.

Express Trust; Prescription (1997)


On 01 January 1980, Redentor and Remedies entered into an agreement by
virtue of which the former was to register a parcel of land in the name of Remedies
under the explicit covenant to reconvey the land to Remigio, son of Redentor, upon
the son's graduation from college. In 1981, the land was registered in the name of
Remedies.

Redentor died a year later or in 1982. In March 1983, Remigio graduated from
college. In February 1992, Remigio accidentally found a copy of the document so
constituting Remedies as the trustee of the land. In May 1994, Remigio filed a case
against Remedies for the reconveyance of the land to him. Remedies, in her answer,
averred that the action already prescribed. How should the matter be decided?

SUGGESTED ANSWER:
The matter should be decided in favor of Remigio (trustee) because the action
has not prescribed. The case at bar involves an express trust which does not
prescribe as long as they have not been repudiated by the trustee (Diaz vs. Gorricho.
103 Phil, 261).

Notes:
In Diaz vs. Gorricho, the Supreme Court held that it was held that an express
trust does not prescribe for as long as the trustee does not repudiate it.

In this case, an express trust was created between Remegio, the trustee, and
Remedies, the trustor. Thus, the contention of remedies is without merit because an
express trust does not prescribe.
ARTICLE 1441. Trusts are either express or implied. Express trusts are created
by the intention of the trustor or of the parties. Implied trusts come into being by
operation of law.

Implied Trust (1998)


Juan and his sister Juana inherited from their mother two parcels of farmland
with exactly the same areas. For convenience, the Torrens certificates of title covering
both lots were placed in Juan's name alone. In 1996, Juan sold to an innocent
purchaser one parcel in its entirety without the knowledge and consent of Juana, and
wrongfully kept for himself the entire price paid.

1. What rights of action, if any, does Juana have against and/or the buyer? |3%]

2. Since the two lots have the same area, suppose Juana files a complaint to have
herself declared sole owner of the entire remaining second lot, contending that her
brother had forfeited his share thereof by wrongfully disposing of her undivided share
in the first lot. Will the suit prosper? [2%]

SUGGESTED ANSWER:
1. When, for convenience, the Torrens title to the two parcels of land were
placed in Joan's name alone, there was created an implied trust (a resulting trust) for
the benefit of Juana with Juan as trustee of one-half undivided or ideal portion of each
of the two lots. Therefore, Juana can file an action for damages against Juan for
having fraudulently sold one of the two parcels which he partly held in trust for
Juana's benefit. Juana may claim actual or compensatory damage for the loss of her
share in the land; moral damages for the mental anguish, anxiety, moral shock and
wounded feelings she had suffered; exemplary damage by way of example, and
attorney's fees.

2) Juana has no cause of action against the buyer who acquired the land for value
and in good faith, relying on the transfer certificate of title showing that Juan is the
registered owner of the land.

Notes:
Under the rule on mirror doctrine, a buyer may rely on the face of the title; he is
not obliged by law to conduct an inquiry any further concerning the defect in the title.
Therefore, Juana has no cause of action against the buyer of her land because the later
relied on the clean title of Juan.
ARTICLE 476. Whenever there is a cloud on title to real property or any
interest therein, by reason of any instrument, record, claim, encumbrance or
proceeding which is apparently valid or effective but is in truth and in fact invalid,
ineffective, voidable, or unenforceable, and may be prejudicial to said title, an action
may be brought to remove such cloud or to quiet the title.

An action may also be brought to prevent a cloud from being cast upon title to
real property or any interest therein.

ARTICLE 494. No co-owner shall be obliged to remain in the co-ownership.


Each co-owner may demand at any time the partition of the thing owned in common,
insofar as his share is concerned.

Nevertheless, an agreement to keep the thing undivided for a certain period of


time, not exceeding ten years, shall be valid. This term may be extended by a new
agreement.

A donor or testator may prohibit partition for a period which shall not exceed
twenty years.

Neither shall there be any partition when it is prohibited by law.

No prescription shall run in favor of a co-owner or co-heir against his co-owners


or co-heirs so long as he expressly or impliedly recognizes the co-ownership.

ANOTHER ANSWER:
1. Under Article 476 of the Civil Code, Juana can file an action for quieting of
title as there is a cloud in the title to the subject real property. Second, Juana can also
file an action for damages against Juan, because the settled rule is that the proper
recourse of the true owner of the property who was prejudiced and fraudulently
dispossessed of the same is to bring an action for damages against those who caused
or employed the same. Third, since Juana had the right to her share in the property by
way of inheritance, she can demand the partition of the thing owned in common,
under Article 494 of the Civil Code, and ask that the title to the remaining property be
declared as exclusively hers.

However, since the farmland was sold to an innocent purchaser for value, then
Juana has no cause of action against the buyer consistent with the established rule that
the rights of an innocent purchaser for value must be respected and protected
notwithstanding the fraud employed by the seller in securing his title. (Eduarte vs.
CA, 253 SCRA 391)
Notes:
In the case of Eduarte vs. CA, the Supreme Court held that the right of an
innocent buyer for value must be respected and protected. Therefore, Juana cannot
take back her parcel of land from the buyer who merely depended on the clean title of
her brother, Juan.

ADDITIONAL ANSWER:
1. Juana has the right of action to recover (a) her one-half share in the proceeds
of the sale with legal interest thereof, and (b) such damages as she may be able to
prove as having been suffered by her, which may include actual or compensatory
damages as well as moral and exemplary damages due to the breach of trust and bad
faith (Imperial vs. CA, 259 SCRA 65). Of course, if the buyer knew of the co-
ownership over the lot he was buying, Juana can seek (c) reconvenyance of her one-
half share instead but she must implead the buyer as co-defendant and allege his bad
faith in purchasing the entire lot. Finally, consistent with the ruling in Imperial us.
CA. Juana may seek instead (d) a declaration that she is now the sole owner of the
entire remaining lot on the theory that Juan has forfeited his one-half share therein.

ADDITIONAL ANSWER:
1. Juana can file an action for damages against Juan for having fraudulently sold
one of the two parcels which he partly held in trust for Juana's benefit. Juana may
claim actual or compensatory damage for the loss of her share in the land; moral
damages for the mental anguish, anxiety, moral shock and wounded feelings she had
suffered; exemplary damage by way of example for the common good, and attorney's
fees. Juana has no cause of action against the buyer who acquired the land for value
and in good faith, relying on the transfer certificate showing that Juan is the registered
owner of the land.

SUGGESTED ANSWER:
Juana's suit to have herself declared as sole owner of the entire remaining area
will not prosper because while Juan's act in selling the other lot was wrongful. It did
not have the legal effect of forfeiting his share in the remaining lot. However, Juana
can file an action against Juan for partition or termination of the co-ownership with a
prayer that the lot sold be adjudicated to Juan, and the remaining lot be adjudicated
and reconveyed to her.
ANOTHER ANSWER:
2. The suit will prosper, applying the ruling in Imperial vs. CA cited above. Both
law and equity authorize such a result, said the Supreme Court.

Strictly speaking, Juana's contention that her brother had forfeited his share in
the second lot is incorrect. Even if the two lots have the same area, it does not follow
that they have the same value. Since the sale of the first lot on the Torrens title in the
name of Juan was valid, all that Juana may recover is the value of her undivided
interest therein, plus damages. In addition, she can ask for partition or reconveyance
of her undivided interest in the second lot, without prejudice to any agreement
between them that in lieu of the payment of the value of Juana's share in the first lot
and damages, the second lot be reconveyed to her.

ALTERNATIVE ANSWER:
2. The suit will not prosper, since Juan's wrongful act of pocketing the entire
proceeds of the sale of the first lot is not a ground for divesting him of his rights as a
co-owner of the second lot. Indeed, such wrongdoing by Juan does not constitute, for
the benefit of Juana, any of the modes of acquiring ownership under Art. 712, Civil
Code.

Trust; Implied Resulting Trust (1995)


In 1960, Maureen purchased two lots in a plush subdivision registering Lot 1 in
her name and Lot 2 in the name of her brother Walter with the latter's consent. The
idea was to circumvent a subdivision policy against the acquisition of more than one
lot by one buyer. Maureen constructed a house on Lot 1 with an extension on Lot 2 to
serve as a guest house. In 1987, Walter who had suffered serious business losses
demanded that Maureen remove the extension house since the lot on which the
extension was built was his property. In 1992, Maureen sued for the
reconveyance to her of Lot 2 asserting that a resulting trust was created when
she had the lot registered in Walter's name even if she paid the purchase price.
Walter opposed the suit arguing that assuming the existence of a resulting trust,
the action of Maureen has already prescribed since ten years have already
elapsed from the registration of the title in his name. Decide. Discuss fully.

SUGGESTED ANSWER:
This is a case of an implied resulting trust. If Walter claims to have acquired
ownership of the land by prescription or if he anchors his defense on extinctive
prescription, the ten year period must be reckoned from 1987 when he demanded that
Maureen remove the extension house on Lot No. 2 because such demand amounts to
an express repudiation of the trust and it was made known to Maureen. The action
for reconveyance filed in 1992 is not yet barred by prescription. (Spouses Huang v.
Court of Appeals, Sept. 13, 1994).

Notes:
In Spouses Huang v. CA, it was held that an implied trust prescribes in 10
years after the act of repudiation made known to the real owner. In this case, the
repudiation was made in 1978 when Walter told Maureen to remove her house.
Maureen filed a case for reconveyance in 1992 which is after five years from the act
of repudiation. Therefore, the case for reconveyance will prosper because it was filed
within the 10-year prescriptive period.

SALES

A sale is a transaction between two or more parties in which the buyer receives
tangible or intangible goods, services or assets in exchange for money.

Assignment of Credit vs. Subrogation (1993)


Peter Co, a trader from Manila, has dealt business with Allied Commodities in
Hongkong for five years. All through the years, Peter Co accumulated an indebtedness
of P500,000.00 with Allied Commodities. Upon demand by its agent in Manila, Peter
Co paid Allied Commodities by check the amount owed. Upon deposit in the payee's
account in Manila, the check was dishonored for insufficiency of funds. For and in
consideration of P1.00, Allied Commodities assigned the credit to Hadji Butu who
brought suit against Peter Co in the RTC of Manila for recovery of the amount owed.

Peter Co moved to dismiss the complaint against him on the ground that
Hadji Butu was not a real party in interest and, therefore, without legal capacity
to sue and that he had not agreed to a subrogation of creditor.

Will Peter Co's defense of absence of agreement to a subrogation of creditor


prosper?

SUGGESTED ANSWER:
No, Co's defense will not prosper. This is not a case of subrogation, but an
assignment of credit. ASSIGNMENT OF CREDIT is the process of transferring the
right of the assignor to the assignee. The assignment may be done either gratuitously
or onerously, in which case, the assignment has an effect similar to that of a sale
(Nyco Sales Corp.v.BA Finance Corp. G.R No.71694. Aug.16, 1991 200 SCRA
637). As a result of the assignment, the plaintiff acquired all the rights of the assignor
including the right to sue in his own name as the legal assignee. In assignment, the
debtor's consent is not essential for the validity of the assignment (Art. 1624; 1475.
CC; Rodriguez v. CA, et al, G. R No. 84220, March 25. 1992 207 SCRA 553).

Notes:
In the case of Nyco Sales Corp., v. BA Finance Corp., assignment of credit is the
process of transferring the right of the assignor to the assignee; it has an effect similar
to that of a sale because the assignment may be done either gratuitously or onerously.
In assignment of credit, the consent of the debtor is not necessary. Therefore, the
assignee is the real party in interest to sue the debtor.

ALTERNATIVE ANSWER:
No, the defense of Peter Co will not prosper. Hadji Butu validly acquired his
right by an assignment of credit under Article 1624 of the Civil Code. However, the
provisions on the contract of sale (Article 1475 Civil Code) will apply, and the
transaction is covered by the Statute of Frauds. (Art. 1403 par. (2) Civil Code)

Notes:

Under the Civil Code, an assignment of credits shall be perfected in accordance


with the provisions of sales.

ARTICLE 1403. The following contracts are unenforceable, unless they are
ratified:
(1) Those entered into in the name of another person by one who has been given
no authority or legal representation, or who has acted beyond his powers;

(2) Those that do not comply with the Statute of Frauds as set forth in this
number. In the following cases an agreement hereafter made shall be unenforceable by
action, unless the same, or some note or memorandum, thereof, be in writing, and
subscribed by the party charged, or by his agent; evidence, therefore, of the agreement
cannot be received without the writing, or a secondary evidence of its contents:
(a) An agreement that by its terms is not to be performed within a year from
the making thereof;
(b) A special promise to answer for the debt, default, or miscarriage of
another;
(c) An agreement made in consideration of marriage, other than a mutual
promise to marry;
(d) An agreement for the sale of goods, chattels or things in action, at a price not
less than five hundred pesos, unless the buyer accept and receive part of such goods
and chattels, or the evidences, or some of them, of such things in action, or pay at the
time some part of the purchase money; but when a sale is made by auction and entry is
made by the auctioneer in his sales book, at the time of the sale, of the amount and
kind of property sold, terms of sale, price, names of the purchasers and person on
whose account the sale is made, it is a sufficient memorandum;
(e) An agreement for the leasing for a longer period than one year, or for the sale
of real property or of an interest therein;
(f) A representation as to the credit of a third person.

(3) Those where both parties are incapable of giving consent to a contract.

ARTICLE 1475. The contract of sale is perfected at the moment there is a


meeting of minds upon the thing which is the object of the contract and upon the
price.

From that moment, the parties may reciprocally demand performance, subject to
the provisions of the law governing the form of contracts.

ARTICLE 1624. An assignment of credits and other incorporeal rights shall be


perfected in accordance with the provisions of Article 1475.

Conditional Sale vs. Absolute Sale (1997)


Distinguish between a conditional sale, on the one hand, and an absolute
sale, on the other hand.

SUGGESTED ANSWER:
A CONDITIONAL SALE is one where the vendor is granted the right to
unilaterally rescind the contract predicated on the fulfillment or non-fulfillment, as
the case may be, of the prescribed condition. An ABSOLUTE SALE is one where the
title to the property is not reserved to the vendor or if the vendor is not granted the
right to rescind the contract based on the fulfillment or nonfulfillment, as the case may
be, of the prescribed condition.

Notes:
In a contract of absolute sale, a vendor has no unilateral right to rescind the
contract, while in a contract to sell, a vendor has a unilateral right to rescind the
contract upon non-fulfillment of the condition.

Contract of Sale vs. Agency to Sell (1999)

A granted B the exclusive right to sell his brand of Maong pants in Isabela, the
price for his merchandise payable within 60 days from delivery, and promising B a
commission of 20% on all sales. After the delivery of the merchandise to B but before
he could sell any of them, B’s store in Isabela was completely burned without his
fault, together with all of A's pants. Must B pay A for his lost pants? Why? (5%)

SUGGESTED ANSWER:
The contract between A and B is a sale not an agency to sell because the price
is payable by B upon 60 days from delivery even if B is unable to resell it. If B
were an agent, he is not bound to pay the price if he is unable to resell it.

As a buyer, ownership passed to B upon delivery and, under Art. 1504 of the
Civil Code, the thing perishes for the owner. Hence, B must still pay the price.

Notes:
Under the Civil Code, if the buyer is obliged to pay the price even if the goods
were not sold, it is a contract of sale; however, if a party is obliged to return the goods
when they are not sold out, the contract is an agency to sell.

In a contract of sale, risk of loss shall be borne by the buyer.


ARTICLE 1504. Unless otherwise agreed, the goods remain at the seller’s risk
until the ownership therein is transferred to the buyer, but when the ownership
therein is transferred to the buyer the goods are at the buyer’s risk whether actual
delivery has been made or not, except that:

(1) Where delivery of the goods has been made to the buyer or to a bailee for the
buyer, in pursuance of the contract and the ownership in the goods has been retained
by the seller merely to secure performance by the buyer of his obligations under the
contract, the goods are at the buyer’s risk from the time of such delivery;

(2) Where actual delivery has been delayed through the fault of either the buyer
or seller the goods are at the risk of the party in fault.
Contract of Sale; Marital Community Property; Formalities (2006)
Spouses Biong and Linda wanted to sell their house. They found a prospective
buyer, Ray. Linda negotiated with Ray for the sale of the property. They agreed on a
fair price of P2 Million. Ray sent Linda a letter confirming his intention to buy the
property. Later, another couple, Bernie and Elena, offered a similar house at a lower
price of P 1.5 Million. But Ray insisted on buying the house of Biong and Linda for
sentimental reasons. Ray prepared a deed of sale to be signed by the couple and a
manager's check for P2 Million. After receiving the P2 Million, Biong signed the deed
of sale. However, Linda was not able to sign it because she was abroad. On her return,
she refused to sign the document saying she changed her mind. Linda filed suit for
nullification of the deed of sale and for moral and exemplary damages against
Ray.

Will the suit prosper? Explain. (2.5%)

ALTERNATIVE ANSWER:
No, the suit will not prosper. The contract of sale was perfected when Linda and
Ray agreed on the object of the sale and the price [Art. 1475, New Civil Code]. The
consent of Linda has already been given, as shown by her agreement to the price of
the sale. There is therefore consent on her part as the consent need not be given in any
specific form. Hence, her consent may be given by implication, especially since she
was aware of, and participated in the sale of the property (Pelayo v. CA, G.R. No.
141323, June 8, 2005). Her action for moral and exemplary damages will also not
prosper because the case does not fall under any of those mentioned in Art. 2219 and
2232 of the Civil Code.

Notes:
In Pelayo v. CA, it was held that the sale of the conjugal property is valid
although the wife had not signed the deed of sale because her consent was given ahead
of the making of the deed of sale since she participated during the negotiation.

ARTICLE 1475. The contract of sale is perfected at the moment there is a


meeting of minds upon the thing which is the object of the contract and upon the
price.

From that moment, the parties may reciprocally demand performance, subject to
the provisions of the law governing the form of contracts.
Notes:
The contract of sale is perfected when the buyer and the seller had agreed upon
the price and the object of the contract.

ALTERNATIVE ANSWER:
The suit will prosper. Sale of community property requires written consent of
both spouses. The failure or refusal of Linda to affix her signature on the deed of sale,
coupled with her express declaration of opposing the sale negates any valid consent on
her part. The consent of Biong by himself is insufficient to effect a valid sale of
community property (Art. 96, Family Code; Abalos v. Macatangay, G.R. No.
155043, September 30, 2004).

Does Ray have any cause of action against Biong and Linda? Can he also
recover damages from the spouses? Explain. (2.5%)

Suggested Answer:
Considering that the contract has already been perfected and taken out of the
operation of the statute of frauds, Ray can compel Linda and Biong to observe the
form required by law in order for the property to be registered in the name of Ray
which can be filed together with the action for the recovery of house [Art. 1357 New
Civil Code]. In the alternative, he can recover the amount of Two million pesos
(P2,000,000.00) that he paid. Otherwise, it would result in solutio indebiti or unjust
enrichment.
Ray can recover moral damages on the ground that the action filed by Linda is
clearly an unfounded civil suit which falls under malicious prosecution {Ponce v.
Legaspi, G.R. No. 79184, May 6,1992).
Notes:
In Ponce v. Legaspi, it was held that moral damages are recoverable in case of
unfounded civil suit which falls under malicious prosecution.

When a contract is already perfected, it is no longer covered by the statute of


fraud. Thus, a party may compel another to observe the form required by law in order
for the property to be registered.
Notes:
Art. 96. The administration and enjoyment of the community property shall
belong to both spouses jointly. In case of disagreement, the husband’s decision shall
prevail, subject to recourse to the court by the wife for proper remedy, which must be
availed of within five years from the date of the contract implementing such decision.
In the event that one spouse is incapacitated or otherwise unable to participate in
the administration of the common properties, the other spouse may assume sole
powers of administration. These powers do not include disposition or encumbrance
without authority of the court or the written consent of the other spouse. In the
absence of such authority or consent, the disposition or encumbrance shall be void.
However, the transaction shall be construed as a continuing offer on the part of the
consenting spouse and the third person, and may be perfected as a binding contract
upon the acceptance by the other spouse or authorization by the court before the offer
is withdrawn by either or both offerors.

Notes:
The law provides that sale of conjugal property needs the consent of both
spouses.

ARTICLE 1357. Once the contract has been perfected, the contracting parties
may compel each other to observe that form if the law requires that such contract
must be in a special form. (Reengineered)

Contract to Sell (2001)


Arturo gave Richard a receipt which states: Receipt Received from Richard as
down payment for my 1995 Toyota Corolla with plate No. XYZ-1 23..............
P50.000.00

Balance payable: 12/30/01........ P50 000.00

September 15, 2001.


(Sgd.) Arturo
Does this receipt evidence a contract to sell? Why? (5%)

SUGGESTED ANSWER:
It is a contract of sale because the seller did not reserve ownership until he
was fully paid.

Notes:
A contract to sell is one which the seller reserves the ownership of the object of
the contract until he is fully paid.
Contract to Sell v. Contract of Sale (1997)
State the basic difference (only in their legal effects) Between a contract to
sell, on the one hand, and a contract of sale, on the other.

SUGGESTED ANSWER:
In a CONTRACT OF SALE, ownership is transferred to the buyer upon delivery
of the object to him while in a CONTRACT TO SELL, ownership is retained by the
seller until the purchase price is fully paid. In a contract to sell, delivery of the object
does not confer ownership upon the buyer. In a contract of sale, there is only one
contract executed between the seller and the buyer, while in a contract to sell, there
are two contracts, first the contract to sell (which is a conditional or preparatory sale)
and a second, the final deed of sale or the principal contract which is executed after
full payment of the purchase price.

Notes:
In a contract of sale, ownership is transferred to the buyer upon delivery of the
object to him, while in a contract to sell, ownership is retained by the seller until the
purchase price is fully paid.

In a contract of sale, there is only one contract executed between the seller and
the buyer, while in a contract to sell, there are two contracts. The first contract is the
preparatory one and the second contract is the execution of the deed of sale.

Contract to Sell; Acceptance; Right of First Refusal (1991)


A is the lessee of an apartment owned by Y. A allowed his married but
employed daughter B, whose husband works in Kuwait, to occupy it. The relationship
between Y and A soured. Since he has no reason at all to eject A, Y, in connivance
with the City Engineer, secured from the latter an order for the demolition of the
building. A immediately filed an action in the Regional Trial Court to annul the order
and to enjoin its enforcement. Y and A were able to forge a compromise agreement
under which A agreed to a twenty percent (20%) increase in the monthly rentals. They
further agreed that the lease will expire two (2) years later and that in the event
that Y would sell the property, either A or his daughter B shall have the right of
first refusal. The Compromise Agreement was approved by the court. Six (6) months
before the expiration of the lease, A died. Y sold the property to the Visorro Realty
Corp. without notifying B. B then filed an action to rescind the sale in favor of the
corporation and to compel Y to sell the property to her since under the
Compromise Agreement, she was given the right of first refusal which, she
maintains is a stipulation pour atrui under Article 1311 of the Civil Code. Is she
correct?

SUGGESTED ANSWER:
B is not correct. Her action cannot prosper. Article 1311 requires that the third
person intended to be benefited must communicate his acceptance to the obligor
before the revocation. There is no showing that B manifested her acceptance to Y at
any time before the death of A and before the sale. Hence, B cannot enforce any right
under the alleged stipulation pour atrui.

Notes:
A contract pour atrui is one established not for the benefit of the contracting
parties but for a third person. This contract is valid provided the beneficiary shall
communicate his acceptance to the obligor before the its revocation.

The contract was revoked when one of the parties died. Thus, the beneficiary
cannot benefit from the contract if he did not communicate his acceptance.

Therefore, B cannot exercise her right of first refusal because she did not
communicate to A, the obligor, that she accepted the terms of the contract.

ARTICLE 1311. Contracts take effect only between the parties, their assigns
and heirs, except in case where the rights and obligations arising from the contract are
not transmissible by their nature, or by stipulation or by provision of law. The heir is
not liable beyond the value of the property he received from the decedent.

If a contract should contain some stipulation in favor of a third person, he may


demand its fulfillment provided he communicated his acceptance to the obligor
before its revocation. (Reengineered

Notes:
The third paragraph of this section is a stipulation pour atrui. It is a binding
stipulation provided that the third person must accept it.

Double Sales (2001)


On June 15, 1995, Jesus sold a parcel of registered land to Jaime. On June 30,
1995, he sold the same land to Jose. Who has a better right if:

a) the first sale is registered ahead of the second sale, with knowledge of the
latter. Why? (3%)
b) the second sale is registered ahead of the first sale, with knowledge of the
latter? Why? (5%)

SUGGESTED ANSWER:
(a) The first buyer has the better right if his sale was first to be registered, even
though the first buyer knew of the second sale. The fact that he knew of the second
sale at the time of his registration does not make him as acting in bad faith because the
sale to him was ahead in time, hence, has a priority in right. What creates bad faith in
the case of double sale of land is knowledge of a previous sale.

b) The first buyer is still to be preferred, where the second sale is registered
ahead of the first sale but with knowledge of the latter. This is because the second
buyer, who at the time he registered his sale knew that the property had already been
sold to someone else, acted in bad faith. (Article 1544, C.C.)

Notes:
Under the Civil Code, in case of double sale of land, the first buyer has a better
right even though he registered the land with knowledge of the second sale.
However, the second buyer has no better right even though he registered his sale
if he has knowledge of the first sale because he is a buyer in bad faith.

Double Sales (2004)


JV, owner of a parcel of land, sold it to PP. But the deed of sale was not
registered. One year later, JV sold the parcel again to RR, who succeeded to register
the deed and to obtain a transfer certificate of title over the property in his own name.

Who has a better right over the parcel of land, RR or PP? Why? Explain
the legal basis for your answer. (5%)

SUGGESTED ANSWER:
It depends on whether or not RR is an innocent purchaser for value. Under the
Torrens System, a deed or instrument operated only as a contract between the parties
and as evidence of authority to the Register of Deeds to make the registration. It is the
registration of the deed or the instrument that is the operative act that conveys or
affects the land. (Sec. 51, P.D. No. 1529).

In cases of double sale of titled land, it is a well-settled rule that the buyer who
first registers the sale in good faith acquires a better right to the land. (Art. 1544, Civil
Code).

Persons dealing with property covered by Torrens title are not required to
go beyond what appears on its face.

(Orquiola v. CA 386, SCRA 301, [2002]; Domingo v. Races 401 SCRA 197, [2003]).
Thus, absent any showing that RR knew about, or ought to have known the prior sale
of the land to PP or that he acted in bad faith, and being first to register the sale, RR
acquired a good and a clean title to the property as against PP.

Notes:
Under PD 1529, in the sale of a titled land, ownership to it is conveyed to the
buyer when the buyer registers the deed of sale.

In Domingo v. Races, the Supreme Court ruled that in cases of double sale of
titled land, it is a well-settled rule that the buyer who first registers the sale in good
faith acquires a better right to the land.

ARTICLE 1544. If the same thing should have been sold to different vendees,
the ownership shall be transferred to the person who may have first taken possession
thereof in good faith, if it should be movable property.

Should it be immovable property, the ownership shall belong to the person


acquiring it who in good faith first recorded it in the Registry of Property.

Should there be no inscription, the ownership shall pertain to the person who in
good faith was first in the possession; and, in the absence thereof, to the person who
presents the oldest title, provided there is good faith.

Notes:
In case of double sale of titled land, the buyer who first registered the land shall
have a better right; if both buyers did not register, the land should belong to the one
who first possessed the land. If no one possesses the land, it shall belong to the one
who has the oldest title.

Equitable Mortgage (1991)


On 20 December 1970, Juliet, a widow, borrowed from Romeo P4,000.00 and,
as security therefore, she executed a deed of mortgage over one of her two (2)
registered lots which has a market value of P15,000.00. The document and the
certificate of title of the property were delivered to Romeo.

On 2 June 1971, Juliet obtained an additional sum of P3,000 from Romeo. On


this date, however, Romeo caused the preparation of a deed of absolute sale of the
above property, to which Juliet affixed her signature without first reading the
document. The consideration indicated is P7,000.00. She thought that this document
was similar to the first she signed. When she reached home, her son X, after reading
the duplicate copy of the deed, informed her that what she signed was not a mortgage
but a deed of absolute sale. On the following day, 3 June 1971, Juliet, accompanied by
X, went back to Romeo and demanded the reformation it, Romeo prepared and
signed a document wherein, as vendee in the deed of sale above mentioned, he
obligated and bound himself to resell the land to Juliet or her heirs and successors
for the same consideration as reflected in the deed of sale (P7,000) within a period of
two (2) years, or until 3 June 1973. It is further stated therein that should the Vendor
(Juliet) fail to exercise her right to redeem within the said period, the conveyance shall
be deemed absolute and irrevocable. Romeo did not take possession of the
property. He did not pay the taxes thereon.

Juliet died in January I973 without having repurchased the property. Her only
surviving heir, her son X, failed to repurchase the property on or before 3 June 1973.
In 1975, Romeo sold the property to Y for P50,000.00. Upon learning of the sale, X
filed an action for the nullification of the sale and for the recovery of the property on
the ground that the so-called deed of absolute sale executed by his mother was merely
an equitable mortgage, taking into account the inadequacy of the price and the failure
of Romeo to take possession of the property and to pay the taxes thereon. Romeo and
Y maintain that there was a valid absolute sale and that the document signed by the
former on 3 June 1973 was merely a promise to sell.
a) If you were the Judge, would you uphold the theory of X?
b) If you decide in favor of Romeo and Y, would you uphold the validity of
the promise to sell?
SUGGESTED ANSWER:
A. I will not uphold the theory of X for the nullification of the sale and for the
recovery of the property on the ground that the so-called sale was only an equitable
mortgage. An equitable mortgage may arise only if, in truth, the sale was one with
the right of repurchase. The facts of the case state that the right to repurchase was
granted after the absolute deed of sale was executed.

Following the rule in Cruzo vs. Carriaga (174 SCRA 330), a deed of
repurchase executed independently of the deed of sale where the two stipulations are
found in two instruments instead of one document, the right of repurchase would
amount only to one option granted by the buyer to the seller. Since the contract
cannot be upheld as a contract of sale with the right to repurchase, Art. 1602 of the
Civil Code on equitable mortgage will not apply. The rule could have been different if
both deeds were executed on the same occasion or date, in which case, under the
ruling in spouses Claravall v. CA (190 SCRA 439), the contract may still be
sustained as an equitable mortgage, given the circumstances expressed in Art. 1602.
The reserved right to repurchase is then deemed an original intention.

B. If I were to decide in favor of Romeo and Y, I would not uphold the validity
of the promise to sell, so as to enforce it by an action for specific performance. The
promise to sell would only amount to a mere offer and, therefore, it is not enforceable
unless it was sought to be exercised before a withdrawal or denial thereof.

Even assuming the facts given at the end of the case, there would have been no
separate consideration for such promise to sell. The contract would at most amount to
an option which again may not be the basis for an action for specific performance.

Notes:
Under the Civil Code, there is equitable mortgage when the contract of sale
contained a right to repurchase. In this case, Romeo, the buyer, obliged himself to
resell the subject lots to Juliet; clearly, the intention of the parties is mortgage. It is
provided for in the Civil Code that in case of doubt, the contract of sale shall be
deemed to be an equitable mortgage, so the contract entered into between Romeo and
Juliet is an equitable mortgage. Therefore, the theory of X shall be upheld.

ARTICLE 1602. The contract shall be presumed to be an equitable mortgage, in


any of the following cases:
(1) When the price of a sale with right to repurchase is unusually inadequate;
(2) When the vendor remains in possession as lessee or otherwise;
(3) When upon or after the expiration of the right to repurchase another
instrument extending the period of redemption or granting a new period is executed;
(4) When the purchaser retains for himself a part of the purchase price;
(5) When the vendor binds himself to pay the taxes on the thing sold;
(6) In any other case where it may be fairly inferred that the real intention of the
parties is that the transaction shall secure the payment of a debt or the performance of
any other obligation.

In any of the foregoing cases, any money, fruits, or other benefit to be received
by the vendee as rent or otherwise shall be considered as interest which shall be
subject to the usury laws. (n)

ARTICLE 1603. In case of doubt, a contract purporting to be a sale with right to


repurchase shall be construed as an equitable mortgage.

Equitable Mortgage vs. Sale (2005)


On July 14, 2004, Pedro executed in favor of Juan a Deed of Absolute Sale
over a parcel of land covered by TCT No. 6245. It appears in the Deed of Sale that
Pedro received from Juan P120,000.00 as purchase price. However, Pedro retained the
owner's duplicate of said title. Thereafter, Juan, as lessor, and Pedro, as lessee,
executed a contract of lease over the property for a period of one (1) year with a
monthly rental of Pl,000.00. Pedro, as lessee, was also obligated to pay the realty
taxes on the property during the period of lease.

Subsequently, Pedro filed a complaint against Juan for the reformation of the
Deed of Absolute Sale, alleging that the transaction covered by the deed was an
equitable mortgage. In his verified answer to the complaint, Juan alleged that the
property was sold to him under the Deed of Absolute Sale, and interposed
counterclaims to recover possession of the property and to compel Pedro to turn over
to him the owner's duplicate of title. Resolve the case with reasons. (6%)

SUGGESTED ANSWER:
The complaint of Pedro against Juan should be dismissed. The instances when a
contract — regardless of its nomenclature — may be presumed to be an equitable
mortgage are enumerated in Article 1602 of the Civil Code: "Art. 1602. The contract
shall be presumed to be an equitable mortgage, in any of the following cases:
1. When the price of a sale with right to repurchase is unusually inadequate:
2. When the vendor remains in possession as lessee or otherwise;
3. When upon or after the expiration of the right to repurchase another
instrument extending the period of redemption or granting a new period is executed;
4. When the purchaser retains for himself a part of the purchase price;
5. When the vendor binds himself to pay the taxes on the thing sold;
6. In any other case where it may be fairly inferred that the real intention of the
parties is that the transaction shall secure the payment of a debt or the performance of
any other obligation.

Notes:
Under the Civil Code, a contract of sale is deemed to be an equitable mortgage
when the vendor obliges himself to pay taxes or remains in possession of the land as
lessee; in this case, the contract is presumed to be an equitable mortgage because the
vendor remains in possession of the land as lessee and he was the one who paid taxes.

Maceda Law
X sold a parcel of land to Y on 01 January 2002, payment and delivery to be
made on 01 February 2002. It was stipulated that if payment were not to be made by
Y on 01 February 2002, the sale between the parties would automatically be
rescinded. Y failed to pay on 01 February 2002, but offered to pay three days later,
which payment X refused to accept, claiming that their contract of sale had already
been rescinded. Is X’s contention correct? Why? 5%

SUGGESTED ANSWER:
No, X is not correct. In the sale of immovable property, even though it may
have been stipulated, as in this case, that upon failure to pay the price at the time
agreed upon the rescission of the contract shall of right take place, the vendee may
pay, even after the expiration of the period, as long as no demand for rescission of
the contract has been made upon him either judicially or by a notarial act (Article
1592, New Civil code). Since no demand for rescission was made on Y, either
judicially or by a notarial act, X cannot refuse to accept the payment offered by Y
three (3) days after the expiration of the period.

Notes:
If the subject of the contract of sale is immovable property, a vendor cannot
rescind the contract for failure to pay by the buyer at the stipulated time although that
was the agreement because demand for rescission is necessary; the demand for
rescission shall be done by judicial demand or notarial act.
ARTICLE 1592. In the sale of immovable property, even though it may have
been stipulated that upon failure to pay the price at the time agreed upon the rescission
of the contract shall of right take place, the vendee may pay, even after the expiration
of the period, as long as no demand for rescission of the contract has been made upon
him either judicially or by a notarial act. After the demand, the court may not grant
him a new term.

ANOTHER SUGGESTED ANSWER:


This is a contract to sell and not a contract of absolute sale, since as there has
been no delivery of the land. Article 1592 of the New Civil code is not applicable.
Instead, Article 1595 of the New Civil Code applies. The seller has two alternative
remedies: (1) specific performance, or (2) rescission or resolution under Article
1191 of the New Civil Code. In both remedies, damages are due because of default.

Notes:
Under the Civil Code, in a contract of sale, the seller may maintain an action for
the payment of his goods if the buyer refuses or neglects to pay for the goods. This
right is available to the seller even though the ownership is not yet transferred to the
buyer especially if the goods cannot be resold by the seller.

ARTICLE 1595. Under a contract of sale, where the ownership of the goods has
passed to the buyer, and he wrongfully neglects or refuses to pay for the goods
according to the terms of the contract of sale, the seller may maintain an action against
him for the price of the goods.

Under a contract of sale, where the price is payable on a certain day, irrespective
of delivery or of transfer of title, and the buyer wrongfully neglects or refuses to pay
such price, the seller may maintain an action for the price, although the ownership in
the goods has not passed.

But it shall be a defense to such an action that the seller at any time before the
judgment in such action has manifested an inability to perform the contract of sale
on his part or an intention not to perform it.

Although the ownership in the goods has not passed, if they cannot readily be
resold for a reasonable price, and if the provisions of article 1596, fourth paragraph,
are not applicable, the seller may offer to deliver the goods to the buyer, and, if the
buyer refuses to receive them, may notify the buyer that the goods are thereafter held
by the seller as bailee for the buyer. Thereafter the seller may treat the goods as the
buyer’s and may maintain an action for the price.

ARTICLE 1596. Where the buyer wrongfully neglects or refuses to accept and
pay for the goods, the seller may maintain an action against him for damages for
nonacceptance.
The measure of damages is the estimated loss directly and naturally resulting in
the ordinary course of events, from the buyer’s breach of contract.
Where there is an available market for the goods in question, the measure of damages
is, in the absence of special circumstances showing proximate damage of a different
amount, the difference between the contract price and the market or current price at
the time or times when the goods ought to have been accepted, or, if no time was fixed
for acceptance, then at the time of the refusal to accept.
If, while labor or expense of material amount is necessary on the part of the
seller to enable him to fulfill his obligations under the contract of sale, the buyer
repudiates the contract or notifies the seller to proceed no further therewith, the buyer
shall be liable to the seller for labor performed or expenses made before receiving
notice of the buyer’s repudiation or countermand. The profit the seller would have
made if the contract or the sale had been fully performed shall be considered in
awarding the damages.

ALTERNATIVE ANSWER:
Yes, the contract was automatically rescinded upon Y’s failure to pay on 01
February 2002. By the express terms of the contract, there is no need for X to make a
demand in order for rescission to take place. (Article 1191, New Civil Code, Suria v.
IAC 151 SCRA 661 [1987]; U.P. v. de los Angeles 35 SCRA 102 [1970]).
Notes:
Under the Civil Code, if one of the obligors does not comply with his obligation,
the right to rescind such obligation is implied in reciprocal ones.

ARTICLE 1191. The power to rescind obligations is implied in reciprocal ones,


in case one of the obligors should not comply with what is incumbent upon him.

The injured party may choose between the fulfilment and the rescission of the
obligation, with the payment of damages in either case. He may also seek rescission,
even after he has chosen fulfilment, if the latter should become impossible.

The court shall decree the rescission claimed, unless there be just cause
authorizing the fixing of a period.
This is understood to be without prejudice to the rights of third persons who
have acquired the thing, in accordance with articles 1385 and 1388 and the Mortgage
Law.

Maceda Law (2000)


Priscilla purchased a condominium unit in Makati City from the Citiland
Corporation for a price of P10 Million, payable P3 Million down and the balance
with interest thereon at 14% per annum payable in sixty (60) equal monthly
installments of P198,333.33. They executed a Deed of Conditional Sale in which it is
stipulated that should the vendee fail to pay three (3) successive installments, the sale
shall be deemed automatically rescinded without the necessity of judicial action and
all payments made by the vendee shall be forfeited in favor of the vendor by way of
rental for the use and occupancy of the unit and as liquidated damages. For 46 months,
Priscilla paid the monthly installments religiously, but on the 47th and 48th months,
she failed to pay. On the 49th month, she tried to pay the installments due but the
vendor refused to receive the payments tendered by her. The following month, the
vendor sent her a notice that it was rescinding the Deed of Conditional Sale pursuant
to the stipulation for automatic rescission, and demanded that she vacate the premises.
She replied that the contract cannot be rescinded without judicial demand or
notarial act pursuant to Article 1592 of the Civil Code.

a) Is Article 1592 applicable? (3%)


b) Can the vendor rescind the contract? (2%)

SUGGESTED ANSWER:
a) Article 1592 of the Civil Code does not apply to a conditional sale. In
Valarao v. CA, 304 SCRA 155, the Supreme Court held that Article 1592 applies
only to a contract of sale and not to a Deed of Conditional Sale where the seller has
reserved title to the property until full payment of the purchase price. The law
applicable is the Maceda Law.

Notes:
Article 1592 of the Civil Code is applicable in absolute sale, while Maceda Law
is applicable in conditional sale.
In Maceda Law, the seller retains ownership to the property until full payment
thereof, while in Article 1592, the seller does not reserve title to the property.
The Maceda Law and Article 1592 both apply to a contract of sale of real
property.

SUGGESTED ANSWER:

b) No, the vendor cannot rescind the contract under the circumstances. Under
the Maceda Law, which is the law applicable, the seller on installment may not
rescind the contract till after the lapse of the mandatory grace period of 30 days for
every one year of installment payments, and only after 30 days from notice of
cancellation or demand for rescission by a notarial act. In this case, the refusal of the
seller to accept payment from the buyer on the 49th month was not justified because
the buyer was entitled to 60 days grace period and the payment was tendered within
that period. Moreover, the notice of rescission served by the seller on the buyer was
not effective because the notice was not by a notarial act. Besides, the seller may still
pay within 30 days from such notarial notice before rescission may be effected. All
these requirements for a valid rescission were not complied with by the seller. Hence,
the rescission is invalid.

Notes:
Under the Maceda Law, the buyer has grace period for one month for every year
of payment. In this case, the buyer has already paid for three years of his monthly
instalments, so he is allowed for three months grace period. Thus, he can still pay
within the 49th month. Likewise, demand for rescission shall be done by notarial act.

Maceda Law; Recto Law (1999)


What are the so-called "Maceda" and "Recto" laws in connection with
sales on installments? Give the most important features of each law. (5%)

SUGGESTED ANSWER:
The MACEDA LAW (R.A. 655) is applicable to sales of immovable property
on instalments. The most important features are (Rillo v. CA, 247 SCRA 461):

(1) After having paid instalments for at least two years, the buyer is entitled to a
mandatory grace period of one month for every year of instalment payments made, to
pay the unpaid instalments without interest.

If the contract is cancelled, the seller shall refund to the buyer the cash
surrender value equivalent to fifty percent (50%) of the total payments made, and after
five years of installments, an additional five percent (5%) every year but not to exceed
ninety percent (90%) of the total payments made.

(2) In case the installments paid were less than 2 years, the seller shall give the
buyer a grace period of not less than 60 days. If the buyer fails to pay the installments
due at the expiration of the grace period, the seller may cancel the contract after 30
days from receipt by the buyer of the notice of cancellation or demand for rescission
by notarial act.

The RECTO LAW (Art. 1484} refers to sale of movables payable in


installments and limiting the right of seller, in case of default by the buyer, to one of
three remedies: a) exact fulfillment; b) cancel the sale if two or more installments have
not been paid;
c) foreclose the chattel mortgage on the things sold, also in case of default of two
or more installments, with no further action against the purchaser.

Notes:
The difference between the Maceda Law and the Recto Law are:
In Maceda Law, the object of the sale in instalment is immovable property; the
buyer has a grace period of 60 day if had already paid for more than one year of
monthly instalments. Thereafter, he has a grace period of one month for every year of
instalment paid. In case of the cancellation of the sale, the buyer shall be entitled to
50% of the cash surrender value. The sale may be rescinded by notarial act

On the other hand, in Recto Law, the object of the sale in instalment is movable
property; when the buyer fails to pay, the remedies of the seller are: (1) to collect
payment when the buyer fails to pay one instalment; (2) to cancel the contract of sale
when the buyer fails to pay two or more instalments; and (3) to foreclose the chattel
mortgage if the buyer fails to pay two or more instalments. When the seller choses to
foreclose, he cannot use the other remedies.

Option Contract (2002)


Explain the nature of an option contract. (2%)

SUGGESTED ANSWER:
An OPTION CONTRACT is one granting a privilege to buy or sell within an
agreed time and at a determined price. It must be supported by a consideration
distinct from the price. (Art. 1479 and 1482, NCC)
Notes:
An option contract is a contract between two persons where one party promises
or offering to sell something to another; if this contract is supported by a
consideration, it is a binding contract; the offer cannot be cancelled within the agreed
period.

Ex: if A allows B to buy his pen within 10 days, he may withdraw such offer
within such period; but if B gives a consideration whether in cash or in kind, it is
called an option contract. A cannot unilaterally withdraw from the contract because
the contract between him and B is binding upon them.

ARTICLE 1479. A promise to buy and sell a determinate thing for a price
certain is reciprocally demandable.

An accepted unilateral promise to buy or to sell a determinate thing for a price


certain is binding upon the promisor if the promise is supported by a consideration
distinct from the price.

Option Contract; Earnest Money (1993)


LT applied with BPI to purchase a house and lot in Quezon City, one of its
acquired assets. The amount offered was Pl,000,000.00 payable, as follows:
P200,000.00 down payment, the balance of P800,000.00 payable within 90 days from
June 1, 1985. BPI accepted the offer, whereupon LT drew a check for P200,000.00 in
favor of BPI which the latter thereafter deposited in its account. On September 5,
1985, LT wrote BPI requesting extension until October 10, 1985 within which to pay
the balance, to which BPI agreed. On October 5, 1985, due to the expected delay in
the remittance of the needed amount by his financier from the United States, LT wrote
BPI requesting a last extension until October 30, 1985, within which to pay the
balance. BPI denied LTs request because another had offered to buy the same property
for P1,500,000.00. BPI cancelled its agreement with LT and offered to return to
him the amount of P200,000.00 that LT had paid to it. On October 20, 1985, upon
receipt of the amount of P800,000.00 from his US financier, LT offered to pay the
amount by tendering a cashier's check therefor but which BPI refused to accept. LT
then filed a complaint against BPI in the RTC for specific performance and deposited
in court the amount of P800,000.00. Is BPI legally correct in canceling its contract
with LT?
SUGGESTED ANSWER:
BPI is not correct in cancelling the contract with LT. In Lina Topacio v Court
of Appeals and BPI Investment (G. R No. 102606, July 3. 1993, 211 SCRA 291)
the Supreme Court held that the earnest money is part of the purchase price and is
proof of the perfection of the contract. Secondly, notarial or judicial rescission
under Art. 1592 and 1991 of the Civil Code is necessary (Taguba v. de Leon, 132
SCRA 722.)

Notes:
Under Article 1592 of the Civil Code, a contract for the sale of real property
may be rescinded in case one of the obligors fails to fulfill his obligation, but the
rescission shall be by judicial demand or notarial act. Therefore, BPI cannot cancel the
contract of sale without judicial demand or notarial act.

In Lina Topacio v. CA and BPI Investment, it was held that earnest money is
part of the purchase price and is proof of the perfection of the contract. Thus, the
contract in this case can no longer be cancelled.

Earnest money is down payment; it is a portion of the whole amount which is


the price of the object of the contract of sale.

In Taguba v. de Leon, it was held that since the contract involved a parcel of
land, a notarial or judicial decision is necessary before a contract of sale may be
cancelled.

ALTERNATIVE ANSWER:
BPI is correct in cancelling its contract with LT but BPI must do so by way of
judicial rescission under Article 1191 Civil Code. The law requires a judicial action,
and mere notice of rescission is insufficient if it is resisted. The law also provides that
slight breach is not a ground for rescission (Song Fo & Co, vs, Hawaiian Phil Co.,
47 Phils. 821), Delay in the fulfillment of the obligation (Art. 1169, Civil Code) is a
ground to rescind, only if time is of the essence. Otherwise, the court may refuse the
rescission if there is a just cause for the fixing of a period.

Notes:
ARTICLE 1191. The power to rescind obligations is implied in reciprocal ones,
in case one of the obligors should not comply with what is incumbent upon him.

The injured party may choose between the fulfilment and the rescission of the
obligation, with the payment of damages in either case. He may also seek rescission,
even after he has chosen fulfilment, if the latter should become impossible.

The court shall decree the rescission claimed, unless there be just cause
authorizing the fixing of a period.

This is understood to be without prejudice to the rights of third persons who


have acquired the thing, in accordance with articles 1385 and 1388 and the Mortgage
Law.

ARTICLE 1169. An obligor who is obliged to deliver or to do something incurs


in delay from the time the obligee judicially or extrajudicially demands from him the
fulfillment of his obligation.

However, the demand by the creditor shall not be necessary in order that delay
may exist:

(1) When the obligation or the law expressly so declares that he incurs in delay;
or
(2) When it appears that time is designated in the contract when the thing is to be
delivered or the service is to be rendered and it was a controlling motive to establish
the contract; or
(3) When demand is useless, as when the obligor has rendered it beyond his
power to perform.

In reciprocal obligations, no party incurs in delay if the other does not comply or
is not ready to comply in a proper manner with what is incumbent upon him. From the
moment one of the parties fulfills his obligation, delay by the other begins. (1100a)

Notes:
Demand is necessary in order for an obligor to incur in delay. The demand is
either judicial or extra-judicial.
When an obligor incurs in delay, he shall be liable for damages; he is also liable
for the loss of the thing promised to deliver although the loss was due to fortuitous
event or force majeure.
Perfected Sale; Acceptance of Earnest Money (2002)
Bert offers to buy Simeon’s property under the following terms and conditions:
P1 million purchase price, 10% option money, the balance payable in cash upon the
clearance of the property of all illegal occupants. The option money is promptly paid
and Simeon clears the property of illegal occupants in no time at all. However, when
Bert tenders payment of the balance and ask Simeon for the deed for absolute sale,
Simeon suddenly has a change of heart, claiming that the deal is disadvantageous to
him as he has found out that the property can fetch three time the agreed purchase
price. Bert seeks specific performance but Simeon contends that he has merely given
Bert an option to buy and nothing more, and offers to return the option money which
Bert refuses to accept.

B. Will Bert’s action for specific performance prosper? Explain. (4%)

SUGGESTED ANSWER:
Simeon cannot justify his refusal to proceed with the sale by the fact that the
deal is financially disadvantageous to him. Having made a bad bargain is not a legal
ground for pulling out a biding contract of sale, in the absence of some actionable
wrong by the other party (Vales v. Villa, 35 Phil 769 [1916]), and no such wrong has
been committed by Bert.

Notes:
In the case of Vales v. Villa, the Supreme Court held that the contract of sale is
perfected when the offeror receives option money. Therefore, the action for specific
performance filed by Bert will prosper because the contract of sale is already
perfected.
Redemption; Legal; Formalities (2001)
Betty and Lydia were co-owners of a parcel of land. Last January 31, 2001,
when she paid her real estate tax, Betty discovered that Lydia had sold her share to
Emma on November 10, 2000. The following day, Betty offered to redeem her share
from Emma, but the latter replied that Betty's right to redeem has already prescribed.
Is Emma correct or not? Why? (5%)

SUGGESTED ANSWER:
Emma, the buyer, is not correct. Betty can still enforce her right of legal
redemption as a co-owner. Article 1623 of the Civil Code gives a co-owner 30 days
from written notice of the sale by the vendor to exercise his right of legal redemption.
In the present problem, the 30-day period for the exercise by Betty of her right of
redemption had not even begun to run because no notice in writing of the sale appears
to have been given to her by Lydia.

Notes:
Under the Civil Code, a co-owner shall have the right of legal redemption within
30 days from the notice in writing by the vendor who is also a co-owner.
Under Article 1623 of the Civil Code, the right of legal redemption shall not be
exercised except within 30 days from the notice in writing by the vendor.
In this case, there was no written notice given to Betty. Thus, the period to
redeem has not yet prescribed.

ARTICLE 1623. The right of legal pre-emption or redemption shall not be


exercised except within thirty days from the notice in writing by the prospective
vendor, or by the vendor, as the case may be. The deed of sale shall not be recorded in
the Registry of Property, unless accompanied by an affidavit of the vendor that he has
given written notice thereof to all possible redemptioners.

The right of redemption of co-owners excludes that of adjoining owners.

Redemption; Legal; Formalities (2002)


Adela and Beth are co-owners of a parcel of land. Beth sold her undivided share
of the property to Xandro, who promptly notified Adela of the sale and furnished the
latter a copy of the deed of absolute sale. When Xandro presented the deed for
registration, the register of deeds also notified Adela of the sale, enclosing a copy of
the deed with the notice. However, Adela ignored the notices. A year later, Xandro
filed a petition for the partition of the property. Upon receipt of summons, Adela
immediately tendered the requisite amount for the redemption. Xandro contends that
Adela lost her right of redemption after the expiration of 30 days from her receipt of
the notice of the sale given by him.

May Adela still exercise her right of redemption? Explain. (5%)

SUGGESTED ANSWER:
Yes, Adela may still exercise her right of redemption notwithstanding the lapse
of more than 30 days from notice of the sale given to her because Article 1623 of the
New Civil Code requires that the notice in writing of the sale must come from the
prospective vendor or vendor as the case may be. In this case, the notice of the sale
was given by the vendee and the Register of Deeds. The period of 30 days never
tolled. She can still avail of that right.

Notes:
Under the Civil Code, a co-owner shall have the right of legal redemption within
30 days from the notice in writing by the vendor who is also a co-owner. Therefore,
the period of 30 days has not yet started to run because Beth, the vendor, has not
notified in writing Adela.

Under Art. 1623 of the Civil Code, the notice shall be in writing and given by
the vendor who is the co-owner; if the notice does not come from the vendor, the
period to redeem does not prescribe.

ALTERNATIVE ANSWER:
Adela can no longer exercise her right of redemption. As co-owner, she had only
30 days from the time she received written notice of the sale which in this case took
the form of a copy of the deed of sale being given to her (Conejero v. CA, 16 SCRA
775 [1966]). The law does not prescribe any particular form of written notice, nor any
distinctive method for notifying the redemptioner (Etcuban v. CA, 148 SCRA 507
[1987]). So long as the redemptioner was informed in writing, he has no cause to
complain (Distrito v. CA, 197 SCRA 606, 609 [1991]). In fact, in Distrito, a written
notice was held unnecessary where the co-owner had actual knowledge of the sale,
having acted as middleman and being present when the vendor signed the deed of
sale.

Right of First Refusal; Lessee; Effect (1996)


Ubaldo is the owner of a building which has been leased by Remigio for the
past 20 years. Ubaldo has repeatedly assured Remigio that if he should decide to sell
the building, he will give Remigio the right of first refusal. On June 30, 1994,
Ubaldo informed Remigio that he was willing to sell the building for P5 Million. The
following day, Remigio sent a letter to Ubaldo offering to buy the building at P4.5
Million. Ubaldo did not reply. One week later, Remigio received a letter from Santos
informing him that the building has been sold to him by Ubaldo for P5 Million, and
that he will not renew Remigio's lease when it expires. Remigio filed an action against
Ubaldo and Santos for cancellation of the sale, and to compel Ubaldo to execute a
deed of absolute sale in his favor, based on his right of first refusal.
a) Will the action prosper? Explain.

b) If Ubaldo had given Remigio an option to purchase the building instead


of a right of first refusal, will your answer be the same? Explain.

SUGGESTED ANSWER:
No, the action to compel Ubaldo to execute the deed of absolute sale will not
prosper. According to Ang Yu v. Court of Appeals (238 SCRA 602), the right of
first refusal is not based on contract but is predicated on the provisions of human
relations and, therefore, its violation is predicated on quasi-delict. Secondly, the right
of first refusal implies that the offer of the person in whose favor that right was given
must conform with the same terms and conditions as those given to the offeree. In this
case, however, Remigio was offering only P4.5 Million instead of P5 Million.

Notes:
Under the Civil Code, an offer to sell something may be withdrawn anytime
except that if it is founded upon a consideration distinct from the price in which case,
the offeror cannot unilaterally withdraw the offer. Therefore, the right of first refusal
may be disregarded except if it is founded upon a consideration distinct from the
price.

Under Art. 1324 of the New Civil Code, an offerer may withdraw his offer any
time before the offeree accepts the offer except that when the offer is founded upon
a consideration.
In this case, the offer was not founded upon a consideration, so the offerer may
withdraw such offer.

ALTERNATIVE ANSWER:
No, the action will not prosper. The lessee's right of first refusal does not go so
far as to give him the power to dictate on the lessor the price at which the latter should
sell his property. Upon the facts given, the lessor had sufficiently complied with his
commitment to give the lessee a right of first refusal when he offered to sell the
property to the lessee for P5 Million, which was the same price he got in selling it to
Santos. He certainly had the right to treat the lessee's counter-offer of a lesser amount
as a rejection of his offer to sell at P5 Million. Thus, he was free to find another buyer
upon receipt of such unacceptable counter-offer (Art. 1319. NCC).

SUGGESTED ANSWER:
Yes, the answer will be the same. The action will not prosper because an option
must be supported by a consideration separate and distinct from the purchase
price. In this case there is no separate consideration. Therefore, the option may be
withdrawn by Ubaldo at any time. (Art. 1324, NCC)

Notes:
Under the Civil Code, an offer may be withdrawn anytime except if it is founded
upon a consideration distinct from the price. Therefore, a right of first refusal is not a
binding contract if it is not supported by a consideration distinct from the price.

ARTICLE 1324. When the offerer has allowed the offeree a certain period to
accept, the offer may be withdrawn at any time before acceptance by communicating
such withdrawal, except when the option is founded upon a consideration, as
something paid or promised.

Right of First Refusal; Lessee; Effect (1998)


In a 20-year lease contract over a building, the lessee is expressly granted a right
of first refusal should the lessor decide to sell both the land and building. However,
the lessor sold the property to a third person who knew about the lease and in fact
agreed to respect it. Consequently, the lessee brings an action against both the lessor-
seller and the buyer (a) to rescind the sale and (b) to compel specific performance of
his right of first refusal in the sense that the lessor should be ordered to execute a deed
of absolute sale in favor of the lessee at the same price. The defendants contend that
the plaintiff can neither seek rescission of the sale nor compel specific
performance of a "mere" right of first refusal. Decide the case. [5%]

SUGGESTED ANSWER:
The action filed by the lessee, for both rescission of the offending sale and
specific performance of the right of first refusal which was violated, should prosper.
The ruling in Equatorial Realty Development, Inc. vs. Mayfair Theater, Inc. (264
SCRA 483), a case with similar facts, sustains both rights of action because the buyer
in the subsequent sale knew the existence of right of first refusal, hence in bad faith.

ANOTHER ANSWER:
The action to rescind the sale and to compel the right to first refusal will not
prosper. (Ang Yu Asuncion vs. CA, 238 SCRA 602). The Court ruled in a unanimous
en banc decision that the right of first refusal is not founded upon contract but on a
quasi-delictual relationship covered by the principles of human relations and unjust
enrichment (Art. 19, et seq. Civil Code). Hence the only action that will prosper
according to the Supreme Court is an "action for damages in a proper forum for the
purpose."

Notes:
In the case of Ang Yu Asuncion vs. CA, the Supreme Court held that an action
for specific performance will not prosper to enforce the right of first refusal; however,
action for damages may be filed under the provision on quasi-delict.

The right of first refusal is not a binding contract. The owner of the property can
still sell his property to anyone he wishes except when such right is supported by a
consideration which is separate and distinct from the price. In such case, the lessee
and lessors are bound to fulfil their reciprocal obligation.

Right of Repurchase (1993)


On January 2, 1980, A and B entered into a contract whereby A sold to B a
parcel of land for and in consideration of P10.000.00. A reserving to himself the right
to repurchase the same. Because they were friends, no period was agreed upon for
the repurchase of the property.

1) Until when must A exercise his right of repurchase?


2) If A fails to redeem the property within the allowable period, what would
you advise B to do for his better protection?

SUGGESTED ANSWER:
1) A can exercise his right of repurchase within four (4) years from the date of
the contract (Art. 1606, Civil Code).

Notes:
Right to repurchase is conventional redemption which must be exercised within
four years.

ARTICLE 1601. Conventional redemption shall take place when the vendor
reserves the right to repurchase the thing sold, with the obligation to comply with the
provisions of article 1616 and other stipulations which may have been agreed upon.

ARTICLE 1616. The vendor cannot avail himself of the right of repurchase
without returning to the vendee the price of the sale, and in addition:
(1) The expenses of the contract, and any other legitimate payments made by
reason of the sale;
(2) The necessary and useful expenses made on the thing sold. (1518)

SUGGESTED ANSWER:
2} I would advise B to file an action for consolidation of title and obtain a
judicial order of consolidation which must be recorded in the Registry of Property
(Art. 1607. Civil Code).

Notes:
Under the Civil Code, an action for consolidation of title shall be filed if the
vendor fails to exercise his right of repurchase over the property within 4 years, if
period is not agreed upon, from the sale with right of repurchase.

ARTICLE 1607. In case of real property, the consolidation of ownership in the


vendee by virtue of the failure of the vendor to comply with the provisions of article
1616 shall not be recorded in the Registry of Property without a judicial order, after
the vendor has been duly heard.

Notes:
When the vendor fails to redeem his property which is subject to right of
repurchase within the period agreed upon which shall not exceed 10 years or within 4
years as provided for by law, the vendee shall file an action for consolidation of title;
the order of consolidation of title is necessary for the registration of the title of the
vendee in the registry of property.

Transfer of Ownership; Non-Payment of the Price (1991)

Pablo sold his car to Alfonso who issued a postdated check in full payment
therefor. Before the maturity of the check, Alfonso sold the car to Gregorio who later
sold it to Gabriel. When presented for payment, the check issued by Alfonso was
dishonored by the drawee bank for the reason that he, Alfonso, had already closed his
account even before he issued his check. Pablo sued to recover the car from
Gabriel alleging that he (Pablo) had been unlawfully deprived of it by reason of
Alfonso's deception. Will the suit prosper?

SUGGESTED ANSWER:
No. The suit will not prosper because Pablo was not unlawfully deprived of the
car although he was unlawfully deprived of the price. The perfection of the sale and
the delivery of the car was enough to allow Alfonso to have a right of ownership over
the car, which can be lawfully transferred to Gregorio. Art. 559 applies only to a
person who is in possession in good faith of the property, and not to the owner
thereof. Alfonso, in the problem, was the owner, and, hence, Gabriel acquired the title
to the car. Non-payment of the price in a contract of sale does not render ineffective
the obligation to deliver. The obligation to deliver a thing is different from the
obligation to pay its price. EDCA Publishing Co. v. Santos (1990)

Notes:
In the case of EDCA Publishing Co. v. Santos, the Supreme Court held that a
person who is deprived of his property by the use of a bouncing check is not
unlawfully deprived of his property under Article 559 of the Civil Code. Thus, if the
property is sold to a third person, the owner of the property cannot recover it from him
who is considered as a buyer in good faith and for value.

ARTICLE 559. The possession of movable property acquired in good faith is


equivalent to a title. Nevertheless, one who has lost any movable or has been
unlawfully deprived thereof, may recover it from the person in possession of the
same.

If the possessor of a movable lost or which the owner has been unlawfully
deprived, has acquired it in good faith at a public sale, the owner cannot obtain its
return without reimbursing the price paid therefor.

Notes:
The suit will not prosper because Gabriel is a buyer in good faith and for value.
Under the Civil Code, if a person is unlawfully deprived of his property, he may take
it back except when the property falls in the hand of one who is a buyer in good faith
and for value. In the case, Gabriel is a buyer in good faith because he did not know
any defect to the title of the car when it was sold to him. Consequently, the suit will
not prosper.

Transfer of Ownership; Risk of Loss (1990)


D sold a second-hand car to E for P150,000.00 The agreement between D and E
was that half of the purchase price, or P75,000.00, shall be paid upon delivery of the
car to E and the balance of P75,000.00 shall be paid in five equal monthly installments
of P15,000.00 each. The car was delivered to E, and E paid the amount of P75.000.00
to D. Less than one month thereafter, the car was stolen from E's garage with no fault
on E's part and was never recovered. Is E legally bound to pay the said unpaid
balance of P75.000.00? Explain your answer.
SUGGESTED ANSWER:
Yes, E is legally bound to pay the balance of P75,000.00. The ownership of the
car sold was acquired by E from the moment it was delivered to him. Having acquired
ownership, E bears the risk of the loss of the thing under the doctrine of res perit
domino. [Articles 1496. 1497, Civil Code).

Notes:
Under the Civil Code, the ownership of the thing sold is acquired by the buyer
from the time it is delivered to him. In this case, the car was already delivered to E.
Therefore, E will bear the loss, so he has to pay the balance.

ARTICLE 1496. The ownership of the thing sold is acquired by the vendee
from the moment it is delivered to him in any of the ways specified in articles 1497 to
1501, or in any other manner signifying an agreement that the possession is
transferred from the vendor to the vendee.

ARTICLE 1497. The thing sold shall be understood as delivered, when it is


placed in the control and possession of the vendee.

Notes:
Ownership is acquired by delivery of the thing. There is delivery when the thing
is placed in the control and possession of the buyer.

LEASE

Lease is a contractual arrangement calling for the lessee to pay the lessor for the
use of an asset.

Extinguishment; Total Distruction; Leased Property (1993)


A is the owner of a lot on which he constructed a building in the total cost of
P10,000,000.00. Of that amount B contributed P5,000,000.00 provided that the
building as a whole would be leased to him (B) for a period of ten years from January
1. 1985 to December 31, 1995 at a rental of P100,000.00 a year. To such condition, A
agreed. On December 20, 1990, the building was totally burned. Soon thereafter, A's
workers cleared the debris and started construction of a new building. B then served
notice upon A that he would occupy the building being constructed upon completion,
for the unexpired portion of the lease term, explaining that he had spent partly for the
construction of the building that was burned. A rejected B's demand. Did A has a
right in rejecting B's demand?

SUGGESTED ANSWER:
Yes. A was correct in rejecting the demand of B. As a result of the total
destruction of the building by fortuitous event, the lease was extinguished. (Art.
1655, Civil Code.)

Notes:
Under the New Civil Code, If the thing leased is totally destroyed by a
fortuitous event, the lease is extinguished. Therefore, the destruction of the building
by fire extinguishes the contract of lease.

ARTICLE 1655. If the thing leased is totally destroyed by a fortuitous event, the
lease is extinguished. If the destruction is partial, the lessee may choose between a
proportional reduction of the rent and a rescission of the lease.

Implied New Lease (1999)


Under what circumstances would an implied new lease or a tacita
reconduccion arise? (2%)

SUGGESTED ANSWER:
An implied new lease or tacita reconduccion arises if at the end of the contract
the lessee should continue enjoying the thing leased for 15 days with the
acquiescence of the lessor, and unless a notice to the contrary by either parties has
previously been given (Art. 1670). In short, in order that there may be tacita
reconduccion there must be expiration of the contract; there must be continuation of
possession for 15 days or more; and there must be no prior demand to vacate.

Notes:
There is an implied lease or an implied lease if the period of the lease has
already expired and the lessee is allowed to remain in the property leased for 15 days
with the consent of the lessor.

ARTICLE 1670. There is an implied new lease, not for the period of the original
contract, but for the time established in articles 1682 and 1687 if at the end of the
contract, the lessee should continue enjoying the thing leased for fifteen days with the
acquiescence of the lessor, except when a notice to the contrary by either party has
previously been given. The other terms of the original contract shall be revived.
ARTICLE 1682. The lease of a piece of rural land, when its duration has not
been fixed, is understood to have been for all the time necessary for the gathering of
the fruits which the whole estate leased may yield in one year, or which it may yield
once, although two or more years may have to elapse for the purpose.

ARTICLE 1687. If the period for the lease has not been fixed, it is understood to
be from year to year, if the rent agreed upon is annual; from month to month, if it is
monthly; from week to week, if the rent is weekly; and from day to day, if the rent is
to be paid daily. However, even though a monthly rent is paid, and no period for the
lease has been set, the courts may fix a longer term for the lease after the lessee has
occupied the premises for over one year. If the rent is weekly, the courts may likewise
determine a longer period after the lessee has been in possession for over six months.
In case of daily rent, the courts may also fix a longer period after the lessee has stayed
in the place for over one month

Lease of Rural Lands (2000)


In 1995, Mark leased the rice land of Narding in Nueva Ecija for an annual
rental of P1,000.00 per hectare. In 1998, due to the El Nino phenomenon, the rice
harvest fell to only 40% of the average harvest for the previous years. Mark asked
Narding for a reduction of the rental to P500.00 per hectare for that year but the latter
refused. Is Mark legally entitled to such reduction? (2%)
SUGGESTED ANSWER:
No, Mark is not entitled to a reduction. Under Article 1680 of the Civil Code,
the lessee of a rural land is entitled to a reduction of the rent only in case of loss of
more than 1/2 of the fruits through extraordinary and unforeseen fortuitous events.
While the drought brought about by the "El Nino" phenomenon may be classified as
extraordinary, it is not considered as unforeseen.

Notes:
Under the Civil Code, in case of loss of ½ of the fruits is caused by fortuitous
event, the lessee is entitled to reduction of rental. El nino may be considered as
fortuitous event. Therefore, Narding shall be allowed reduction of his rent.

Under Article 1680 of the New Civil Code, reduction of rental is allowed in a
lease of rural land when the loss is more than ½ of the fruits is caused by unforeseen
fortuitous event.
If you think El Nino is an unforeseen fortuitous event, then the lessee may be
allowed reduction of rental.

ALTERNATIVE ANSWER:
Yes, Mark is entitled to a reduction of the rent. His loss was more than 1/2 of the
fruits and the loss was due to an extraordinary and unforeseen fortuitous event. The
"El Nino" phenomenon is extraordinary because it is uncommon; it does not occur
with regularity. And neither could the parties have foreseen its occurrence. The event
should be foreseeable by the parties so that the lessee can change the time for his
planting, or refrain from planting, or take steps to avoid the loss. To be foreseeable,
the time and the place of the occurrence, as well as the magnitude of the adverse
effects of the fortuitous event must be capable of being predicted. Since the exact
place, the exact time, and the exact magnitude of the adverse effects of the "El Nino"
phenomenon are still unpredictable despite the advances in science, the phenomenon
is considered unforeseen.

Notes:
A lessee of the rural land shall have the right to a reduction of the rent if more
than ½ of the fruits is loss and it is caused by the unforeseen fortuitous event.
ARTICLE 1680. The lessee shall have no right to a reduction of the rent on
account of the sterility of the land leased, or by reason of the loss of fruits due to
ordinary fortuitous events; but he shall have such right in case of the loss of more than
one-half of the fruits through extraordinary and unforeseen fortuitous events, save
always when there is a specific stipulation to the contrary.

Leasee & Lessor; Rights and Obligations (1990)

A vacant lot several blocks from the center of the town was leased by its owner
to a young businessman B for a term of fifteen (15) years renewal upon agreement of
the parties. After taking possession of the lot, the lessee built thereon a building of
mixed materials and a store. As the years passed, he expanded his business, earning
more profits. By the tenth (10th) year of his possession, he was able to build a three
(3)-story building worth at least P300,000.00. Before the end of the term of the lease,
B negotiated with the landowner for its renewal, but despite their attempts to do so,
they could not agree on the new conditions for the renewal. Upon the expiration of the
term of the lease, the landowner asked B to vacate the premises and remove his
building and other improvements. B refused unless he was reimbursed for
necessary and useful expenses. B claimed that he was a possessor and builder in
good faith, with right of retention. This issue is now before the court for
resolution in a pending litigation. a) What are the rights of B? b) What are the
rights of the landowner?

SUGGESTED ANSWER:
a) B has the right to remove the building and other improvements unless the
landowner decides to retain the building at the time of the termination of the lease and
pay the lessee one-half of the value of the improvements at that time. The lessee may
remove the building even though the principal thing may suffer damage but B should
not cause any more impairment upon the property leased than is necessary. The claim
of B that he was a possessor and builder in good faith with the right of retention is not
tenable. B is not a builder in good faith because as lessee he does not claim ownership
over the property leased.

b) The landowner/lessor may refuse to reimburse 1/2 of the value of the


improvements and require the lessee to remove the improvements. [Article 1678,
Civil Code),

Notes:
Under the Civil Code, a builder in good faith is the one who possesses the
property as an owner without any knowledge of any defect in his title, so a lessee shall
not be considered a builder in good faith because he knows that he does not own the
property.

ARTICLE 1678. If the lessee makes, in good faith, useful improvements which
are suitable to the use for which the lease is intended, without altering the form or
substance of the property leased, the lessor upon the termination of the lease shall pay
the lessee one-half of the value of the improvements at that time. Should the lessor
refuse to reimburse said amount, the lessee may remove the improvements, even
though the principal thing may suffer damage thereby. He shall not, however, cause
any more impairment upon the property leased than is necessary.

With regard to ornamental expenses, the lessee shall not be entitled to any
reimbursement, but he may remove the ornamental objects, provided no damage is
caused to the principal thing, and the lessor does not choose to retain them by paying
their value at the time the lease is extinguished.

Lessee; Death Thereof; Effects (1997)


Stating briefly the thesis to support your answer to each of the following
cases, will the death - a) of the lessee extinguish the lease agreement?

SUGGESTED ANSWER:

No. The death of the lessee will not extinguish the lease agreement, since lease
is not personal in character and the right is transmissible to the heirs. (Heirs of
Dimaculangan vs. IAC, 170 SCRA 393).

Notes:
In Heirs of Dimaculangan vs. IAC, it was held that the death of the lessee does
not extinguish the lease contract because a lease contract is not personal in character
and the right is transmissible to the heirs.
Option to Buy; Expired (2001)
On January 1, 1980, Nestor leased the fishpond of Mario for a period of three
years at a monthly rental of P1,000.00, with an option to purchase the same during the
period of the lease for the price of P500,000.00. After the expiration of the three-year
period, Mario allowed Nestor to remain in the leased premises at the same rental rate.
On June 15, 1983, Nestor tendered the amount of P500,000.00 to Mario and
demanded that the latter execute a deed of absolute sale of the fishpond in his favor.
Mario refused, on the ground that Nestor no longer had an option to buy the
fishpond. Nestor filed an action for specific performance. Will the action prosper
or not? Why? (5%)

SUGGESTED ANSWER:
No, the action will not prosper. The implied renewal of the lease on a month-to-
month basis did not have the effect of extending the life of the option to purchase
which expired at the end of the original lease period. The lessor is correct in refusing
to sell on the ground that the option had expired.

Notes:
Under the Civil Code, an offer may be withdrawn anytime unless it is supported
by a consideration distinct from the price. Therefore, the option to buy is not a binding
contract between the lessee and the lessor since it was not supported by a
consideration distinct from the price, so the action for specific performance will fail.
An option to buy the leased property expires with the period agreed upon. When
the lessee is allowed to continuously use the property despite the expiration of the
period, the option to buy is no longer effective.

Moreover, an option to purchase is only an offer; if it is not founded upon a


consideration, the lessor may withdraw it any time. There is no perfected contract in
this case. Therefore, the action will not prosper.

Sublease vs. Assignment of Lease; Rescission of Contract (2005)


Under a written contract dated December 1, 1989, Victor leased his land to Joel
for a period of five (5) years at a monthly rental of Pl,000.00, to be increased to
Pl,200.00 and Pl,500.00 on the third and fifth year, respectively. On January 1, 1991,
Joel subleased the land to Conrad for a period of two (2) years at a monthly rental of
Pl,500.00.

On December 31, 1992, Joel assigned the lease to his compadre, Ernie, who
acted on the belief that Joel was the rightful owner and possessor of the said lot. Joel
has been faithfully paying the stipulated rentals to Victor. When Victor learned on
May 18, 1992 about the sublease and assignment, he sued Joel, Conrad and Ernie for
rescission of the contract of lease and for damages.

a) Will the action prosper? If so, against whom? Explain. (2%)

SUGGESTED ANSWER:
Yes, the action of for rescission of the contract of lease and for damages will
prosper. Under Article 1659 of the Civil Code, "if the lessor or the lessee should not
comply with the obligations set forth in Articles 1654 and 1657, the aggrieved party
may ask for rescission of the contract and indemnification for damages, or only the
latter, allowing the contract to remain in force." Article 1649 of the same Code
provides that "the lessee cannot assign the lease without the consent of the lessor,
unless there is a stipulation to the contrary." Consent is necessary because assignment
would cause novation by the substitution of one of the parties. (Bangayan v. Court of
Appeals, G.R. No. 123581, August 29, 1997)

However, the rule is different in the case of subleasing. When there is no


express prohibition in the Contract of Lease, the lessee may sublet the thing leased.
(Art. 1650, Civil Code) In the given case, when Joel assigned the lease to Ernie, the
same was done without the consent of Victor. The assignment is void. However, there
is no indication that in the written contract of lease between Victor and Joel, that
subleasing the premises is prohibited. Hence, the sublease of Joel with Conrad is
valid. In view of the foregoing, Victor can file the case of rescission and damages only
against Joel and Ernie but he cannot include Conrad.

Notes:
Under the Civil Code, a lessee cannot assign the thing leased to other person
without the consent of the lessor; but a lessee may sublet the thing leased to other
person except when there is a prohibition to do so.

Under New Civil Code, assignment of the leased property is not allowed except
when the lessor allows the lessee to do so.
Under New Civil Code, Subleasing is allowed except when the lessor prohibited
the subleasing.

b) In case of rescission, discuss the rights and obligations of the parties.


(2%)

SUGGESTED ANSWER:
Rescission of the lease necessarily requires the return of the thing to the lessor.
Hence, the judgment granting rescission of the contract should also order the lessee to
vacate and return the leased premises to the lessor. However, since the sublessee can
invoke no right superior to that of his sublessor, the moment the sublessor is duly
ousted from the premises, the sublessee has no leg to stand on. The sublessee's right, if
any, is to demand reparation for damages from his sublessor, should the latter be at
fault.

Notes:
Under the Civil Code, when a lease contract is nullified, the sub-lessee shall be
ousted from the property leased because by the ouster of the sub-lessor, he has no leg
to stand on.

Notes:
In Heirs of Sevilla v. CA, it was held that when the contract of lease is
rescinded, the thing leased should be returned to the lessor. When the sub-lessor is
ousted from the premises, the sub-lessee has no leg to stand on, so he is also ousted.
The sub-lessee may ask for damages if the sub-lessor is at fault.
Sublease; Delay in Payment of Rentals (1994)
In January 1993, Four-Gives Corporation leased the entire twelve floors of the
GQS Towers Complex, for a period of ten years at a monthly rental of
P3,000,000.00. There is a provision in the contract that the monthly rentals should be
paid within the first five days of the month. For the month of March, May, June,
October and December 1993, the rentals were not paid on time with some rentals
being delayed up to ten days. The delay was due to the heavy paper work involved in
processing the checks.
Four-Gives Corporation also subleased five of the twelve floors to wholly-owned
subsidiaries. The lease contract expressly prohibits the assignment of the lease
contract or any portion thereof. The rental value of the building has increased by 50%
since its lease to Four-Gives Corporation.

1) Can the building owner eject Four-Gives Corporation on grounds of the


repeated delays in the payment of the rent?

2} Can the building owner ask for the cancellation of the contract for
violation of the provision against assignment?

SUGGESTED ANSWERS:
1) a) The "repeated delays" in the payment of rentals would, at best, be a
slight or casual breach which does not furnish a ground for ejectment especially
because the delays were only due to heavy paper work. Note that there was not even a
demand for payment obviously because the delay lasted for only a few days (10 days
being the longest), at the end of which time payments were presumably made and
were accepted. There was, therefore, no default. Note also that there was no demand
made upon the lessee to vacate the premises for non-payment of the monthly rent.
There is, therefore, no cause of action for ejectment arising from the "repeated
delays".

b) The building owner cannot eject Four-Gives Corporation on the ground of


repeated delays in the payment of rentals. The delay in the payment of the rentals is
minimal and cannot be made the basis of an ejectment suit. The delay was due to the
heavy paperwork involved in processing the checks. It would be otherwise if the lease
contract stated that in the payment of rentals within the first five days of the month,
time is of the essence or that the lessee will be in delay if he falls to pay within the
agreed period without need of demand. In this case he can judicially eject the tenant
on the ground of lack of payment of the price stipulated after a demand to vacate,
(Article 1673(2), New Civil Code),
c) No. Rescission of a contract will not be permitted for a slight or casual
breach, but only for such substantial and fundamental breach as would defeat the very
object of the parties in making the agreement.(Zepeda v. CA, 216 SCRA 293]. The
delay of ten (10)) days is not such a substantial and fundamental breach to warrant the
resolution of the contract of lease specially so when the delay was due to the heavy
paperwork in processing the checks.

Notes:
In Zepeda v. CA, it was held that a contract of lease may be rescinded on the
ground of substantial breach; a casual breach shall not be a ground for rescission;
repeated delays in paying the rental are a casual breach. Therefore, the contract of
lease may not be rescinded.

The New Civil Code provides that subleasing is allowed except when it is
prohibited in the agreement.

ARTICLE 1673. The lessor may judicially eject the lessee for any of the
following causes:
(1) When the period agreed upon has expired;
(2) When payment is less than what was agreed upon;
(3) When a condition is violated;
(4) When the thing leased is used other than stipulated. (Reengineered)

SUGGESTED ANSWER:
2) a) No. Sublease is different from assignment of lease. Sublease, not being
prohibited by the contract of lease is therefore allowed and cannot be invoked as a
ground to cancel the lease,
b) No, the lessor cannot have the lease cancelled for alleged violation of the
provision against assignment. The lessee did not assign the lease, or any portion
thereof, to the subsidiaries. It merely subleased some floors to its subsidiaries. Since
the problem does not state that the contract of lease contains a prohibition against
sublease, the sublease is lawful, the rule being that in the absence of an express
prohibition a lessee may sublet the thing leased, in whole or in part, without prejudice
to his/its responsibility to the lessor for the performance of the contract.

Sublease; Sublessee; Liability (1999)


May a lessee sublease the property leased without the consent of the lessor,
and what are the respective liabilities of the lessee and sub-lessee to the lessor in
case of such sublease? (3%)

SUGGESTED ANSWER:
Yes, provided that there is no express prohibition against subleasing. Under the
law, when in the contract of lease of things there is no express prohibition, the lessee
may sublet the thing leased without prejudice to his responsibility for the performance
of the contract toward the lessor. [Art. 1650)

In case there is a sublease of the premises being leased, the sublessee is bound
to the lessor for all the acts which refer to the use and preservation of the thing leased
in the manner stipulated between the lessor and the lessee. (Art. 1651} The sublessee
is subsidiarily liable to the lessor for any rent due from the lessee.

However, the sublessee shall not be responsible beyond the amount of the rent
due from him. (Art. 1652) As to the lessee, the latter shall still be responsible to the
lessor for the rents; bring to the knowledge of the lessor every usurpation or untoward
act which any third person may have committed or may be openly preparing to carry
out upon the thing leased; advise the owner the need for all repairs; to return the thing
leased upon the termination of the lease just as he received it, save what has been lost
or impaired by the lapse of time or by ordinary wear and tear or from an inevitable
cause; responsible for the deterioration or loss of the thing leased, unless he proves
that it took place without his fault.

Notes:
Yes, the lessee may sublet the thing leased unless there is a prohibition in the
contract of lease; the lessee cannot assign the thing leased unless he is allowed to do
so.

The sub-lessee is bound by the stipulation between the sublessor and the lessor
which pertains to the use and preservation of the thing leased. The sublessee is liable
for the rent stipulated in the sublease contract.

ARTICLE 1649. The lessee cannot assign the lease without the consent of the
lessor, unless there is a stipulation to the contrary. (n)

ARTICLE 1650. When in the contract of lease of things there is no express


prohibition, the lessee may sublet the thing leased, in whole or in part, without
prejudice to his responsibility for the performance of the contract toward the lessor.
(1550)

ARTICLE 1651. Without prejudice to his obligation toward the sublessor, the
sublessee is bound to the lessor for all acts which refer to the use and preservation of
the thing leased in the manner stipulated between the lessor and the lessee.

ARTICLE 1652. The sublessee is subsidiarily liable to the lessor for any rent
due from the lessee. However, the sublessee shall not be responsible beyond the
amount of rent due from him, in accordance with the terms of the sublease, at the time
of the extra-judicial demand by the lessor.
Payments of rent in advance by the sublessee shall be deemed not to have been
made, so far as the lessor’s claim is concerned, unless said payments were effected in
virtue of the custom of the place.

Sublease; Sublessee; Liability (2000)


A leased his house to B with a condition that the leased premises shall be used
for residential purposes only. B subleased the house to C who used it as a warehouse
for fabrics. Upon learning this, A demanded that C stop using the house as a
warehouse, but C ignored the demand, A then filed an action for ejectment against C,
who raised the defense that there is no privity of contract between him and A, and
that he has not been remiss in the payment of rent.

Will the action prosper? (3%)

SUGGESTED ANSWER:

Yes, the action will prosper. Under Article 1651 of the Civil Code, the sublessee
is bound to the lessor for all acts which refer to the use and preservation of the thing
leased in the manner stipulated between the lessor and the lessee.

Notes:
Under the Civil Code, the sub-lessee is bound to the contract between the sub-
lessor and the lessor concerning the use of the thing leased.
ARTICLE 1651. Without prejudice to his obligation toward the sublessor, the
sublessee is bound to the lessor for all acts which refer to the use and preservation of
the thing leased in the manner stipulated between the lessor and the lessee.

Sublease; Validity; Assignment of Sublease (1990)


A leased a parcel of land to B for a period of two years. The lease contract did
not contain any express prohibition against the assignment of the leasehold or the
subleasing of the leased premises. During the third year of the lease, B subleased the
land to C. In turn, C, without A's consent, assigned the sublease to D. A then filed an
action for the rescission of the contract of lease on the ground that B has violated the
terms and conditions of the lease agreement. If you were the judge, how would you
decide the case, particularly with respect to the validity of:

(a) B's sublease to C? and (b) C's assignment of the sublease to D?

SUGGESTED ANSWER:
(a) B's sublease to C is valid. Although the original period of two years for the
lease contract has expired, the lease continued with the acquiescence of the lessor
during the third year. Hence, there has been an implied renewal of the contract of
lease. Under Art. 1650 of the Civil Code, the lessee may sublet the thing leased, in
whole or in part, when the contract of lease does not contain any express prohibition.
[Articles 1650, 1670 Civil Code). A's action for rescission should not prosper on
this ground.

SUGGESTED ANSWER:
(b) C's assignment of the sublease to D is not valid. Under Art. 1649, of the Civil
Code, the lessee cannot assign the lease without the consent of the lessor, unless there
is a stipulation to the contrary. There is no such stipulation in the contract. If the law
prohibits assignment of the lease without the consent of the lessor, all the more would
the assignment of a sublease be prohibited without such consent. This is a violation of
the contract and is a valid ground for rescission by A.

COMMON CARRIERS

ARTICLE 1732. Common carriers are persons, corporations, firms or


associations engaged in the business of carrying or transporting passengers or goods
or both, by land, water, or air, for compensation, offering their services to the
public.

Extraordinary Diligence (2000)


Despite a warning from the police that an attempt to hijack a PAL plane will be
made in the following week, the airline did not take extra precautions, such as
frisking of passengers, for fear of being accused of violating human rights. Two days
later, an armed hijacker did attempt to hijack a PAL flight to Cebu. Although he was
subdued by the other passengers, he managed to fire a shot which hit and killed a
female passenger. The victim's parents sued the airline for breach of contract,
and the airline raised the defense of force majeure. Is the airline liable or not?
(2%)

SUGGESTED ANSWER:
The airline is liable. In case of death of a passenger, common carriers are
presumed to have been at fault or to have acted negligently, unless they prove that
they observed extraordinary diligence (Article 1756, Civil Code). The failure of the
airline to take extra precautions despite a police warning that an attempt to
hijack the plane would be made, was negligence on the part of the airline. Being
negligent, it is liable for the death of the passenger. The defense of force majeure is
not tenable since the shooting incident would not have happened had the airline taken
steps that could have prevented the hijacker from boarding the plane.

Notes:
Under the Civil Code, in case of death of a passenger, a common carriers are
presumed to have been at fault unless they prove that they observed extraordinary
diligence. In this case, the PAL plane was negligent because they did not frisk the
passengers despite of the police warning. Therefore, it is liable for the death of its
passenger.

ARTICLE 1756. In case of death of or injuries to passengers, common carriers


are presumed to have been at fault or to have acted negligently, unless they prove that
they observed extraordinary diligence as prescribed in Articles 1733 and 1755.

ALTERNATIVE ANSWER:
Under Article 1763 of the Civil Code, the common carrier is not required to
observe extraordinary diligence in preventing injury to its passengers on account of
the willful acts or negligence of other passengers or of strangers. The common carrier,
in that case, is required to exercise only the diligence of a good father of a family;
hence, the failure of the airline to take EXTRA precautions in frisking the passengers
and by leaving that matter to the security personnel of the airport, does not constitute a
breach of that duty so as to make the airline liable. Besides, the use of irresistible force
by the hijackers was farce majeure that could not have been prevented even by the
observance of extraordinary diligence.

Notes:

ARTICLE 1733. Common carriers, from the nature of their business and for
reasons of public policy, are bound to observe extraordinary diligence in the
vigilance over the goods and for the safety of the passengers transported by them,
according to all the circumstances of each case.

ARTICLE 1755. A common carrier is bound to carry the passengers safely


as far as human care and foresight can provide, using the utmost diligence of very
cautious persons, with a due regard for all the circumstances.

ARTICLE 1756. In case of death of or injuries to passengers, common carriers


are presumed to have been at fault or to have acted negligently, unless they prove that
they observed extraordinary diligence as prescribed in articles 1733 and 1755.

The airline is liable because its employees failed to stop the wilful act which caused
the death of the female passenger.

Under the Civil Code, a common carrier may be held liable for injuries
suffered by its passenger because of the wilful acts or negligence of the other
passenger if its employees could have prevented or stopped the act or omission
but failed in doing so.

In this case, the employees of the airline did not frisk the passengers who
were availing its service. The airline should have exercised extraordinary
diligence in carrying its passengers to their destination. The facts show that it did
not frisk its passengers which is an indicia that it did not observe extraordinary
diligence. Thus, the airline should be held liable for the injury of its passenger.

ARTICLE 1763. A common carrier is responsible for injuries suffered by a


passenger on account of the wilful acts or negligence of other passengers or of
strangers, if the common carrier’s employees through the exercise of the diligence of a
good father of a family could have prevented or stopped the act or omission.

Notes:
Common carriers are liable for the injury of a passenger caused by another
passenger if it could have prevented the injury when it observed diligence of a good
father of a family.

AGENCY

ARTICLE 1868. By the contract of agency, a person binds himself to render


some service or to do something in representation or on behalf of another, with the
consent or authority of the latter.

Agency (2003)
Jo-Ann asked her close friend, Aissa, to buy some groceries for her in the
supermarket. Was there a nominate contract entered into between Jo-Ann and
Aissa? In the affirmative, what was it? Explain. 5%

SUGGESTED ANSWER:
Yes, there was a nominate contract. On the assumption that Aissa accepted the
request of her close friend Jo-Ann to but some groceries for her in the supermarket,
what they entered into was a nominate contract of Agency. Article 1868 of the New
Civil Code provides that by the contract of agency a person binds himself to render
some service or to do something in representation or on behalf of another, with the
consent or authority of the latter.

Notes:
A contract of agency is entered into when a person binds himself to render some
service or to do something in behalf of another with the consent of the latter.
The typical example of a contract of agency is one which is the subject of SPA.
ARTICLE 1868. By the contract of agency a person binds himself to render
some service or to do something in representation or on behalf of another, with the
consent or authority of the latter.

ALTERNATIVE ANSWER:
Yes, they entered into a nominate contract of lease of service in the absence of a
relation of principal and agent between them (Article 1644, New Civil Code).
ARTICLE 1644. In the lease of work or service, one of the parties binds himself
to execute a piece of work or to render to the other some service for a price certain,
but the relation of principal and agent does not exist between them.

Notes:
Contract of lease of service is entered into when a person binds himself to render
some service for a price.

Agency v. Sale (2000)


A foreign manufacturer of computers and a Philippine distributor entered
into a contract whereby the distributor agreed to order 1,000 units of the
manufacturer's computers every month and to resell them in the Philippines at the
manufacturer's suggested prices plus 10%. All unsold units at the end of the year shall
be bought back by the manufacturer at the same price they were ordered. The
manufacturer shall hold the distributor free and harmless from any claim for defects in
the units. Is the agreement one for sale or agency? (5%)

SUGGESTED ANSWER:
The contract is one of agency, not sale. The notion of sale is negated by the
following indicia: (1) the price is fixed by the manufacturer with the 10% mark-up
constituting the commission; (2) the manufacturer reacquires the unsold units at
exactly the same price; and (3) warranty for the units was borne by the manufacturer.
The foregoing indicia negate sale because they indicate that ownership over the
units was never intended to transfer to the distributor.

Notes:
Under the Civil Code, a contract of sale will transfer ownership of the goods to
the buyer; in this case, it is clear that the ownership was not transferred to the
distributor. Therefore, the contract is one of agency.

Agency; coupled with an interest (2001)


Richard sold a large parcel of land in Cebu to Leo for P100 million payable in
annual installments over a period of ten years, but title will remain with Richard until
the purchase price is fully paid. To enable Leo to pay the price, Richard gave him a
power-of-attorney authorizing him to subdivide the land, sell the individual lots, and
deliver the proceeds to Richard, to be applied to the purchase price.
Five years later, Richard revoked the power of attorney and took over the
sale of the subdivision lots himself. Is the revocation valid or not? Why? (5%)

SUGGESTED ANSWER:
The revocation is not valid. The power of attorney given to the buyer is
irrevocable because it is coupled with an interest: the agency is the means of fulfilling
the obligation of the buyer to pay the price of the land (Article 1927, CC). In other
words, a bilateral contract (contract to buy and sell the land) is dependent on the
agency.

Notes:
Under the Civil Code, an agency cannot be revoked when a bilateral contract
depends upon it. In this case, a contract to buy and sell is existing which is a bilateral
contract. Therefore, this contract of agency cannot be revoked.

ARTICLE 1927. An agency cannot be revoked if a bilateral contract


depends upon it, or if it is the means of fulfilling an obligation already contracted, or
if a partner is appointed manager of a partnership in the contract of partnership and his
removal from the management is unjustifiable.

Agency; Guarantee Commission (2004)


As an agent, AL was given a guarantee commission, in addition to his regular
commission, after he sold 20 units of refrigerators to a customer, HT Hotel. The
customer, however, failed to pay for the units sold. AL’s principal, DRBI,
demanded from AL payment for the customer’s accountability. AL objected, on
the ground that his job was only to sell and not to collect payment for units bought by
the customer. Is AL’s objection valid? Can DRBI collect from him or not?
Reason. (5%)

SUGGESTED ANSWER:
No, AL's objection is not valid and DRBI can collect from AL. Since AL
accepted a guarantee commission, in addition to his regular commission, he agreed to
bear the risk of collection and to pay the principal the proceeds of the sale on the same
terms agreed upon with the purchaser (Article 1907, Civil Code)

Notes:
Under the Civil Code, if a commission agent receives a guarantee commission,
shall bear the risk of collection and shall pay the proceeds of the sale on the same
terms agreed upon with the purchaser.

ARTICLE 1907. Should the commission agent receive on a sale, in addition to


the ordinary commission, another called a guarantee commission, he shall bear the
risk of collection and shall pay the principal the proceeds of the sale on the same terms
agreed upon with the purchaser.

Agency; Real Estate Mortgage (2004)


CX executed a special power of attorney authorizing DY to secure a loan from
any bank and to mortgage his property covered by the owner’s certificate of title. In
securing a loan from MBank, DY did not specify that he was acting for CX in the
transaction with said bank. Is CX liable for the bank loan? Why or why not?
Justify your answer. (5%)

SUGGESTED ANSWER:
CX is liable for the bank loan because he authorized the mortgage on his
property to secure the loan contracted by DY. If DY later defaults and fails to pay the
loan, CX is liable to pay. However, his liability is limited to the extent of the value of
the said property.

ALTERNATIVE ANSWER:
CX is not personally liable to the bank loan because it was contracted by DY in
his personal capacity. Only the property of CX is liable. Hence, while CX has
authorized the mortgage on his property to secure the loan of DY, the bank cannot sue
CX to collect the loan in case DY defaults thereon. The bank can only foreclose the
property of CX. And if the proceeds of the foreclosure are not sufficient to pay the
loan in full, the bank cannot run after CX for the deficiency.

ALTERNATIVE ANSWER:
While as a general rule the principal is not liable for the contract entered into by
his agent in case the agent acted in his own name without disclosing his principal,
such rule does not apply if the contract involves a thing belonging to the principal.
In such case, the principal is liable under Article 1883 of the Civil Code. The contract
is deemed made on his behalf (Sy-juco v. Sy-juco 40 Phil. 634 [1920]).

Notes:
In Sy-juco v. Sy-juco, the Supreme Court held that the principal shall not be held
liable for the contract entered into by the agent in his own name and if the latter does
not disclose his principal; but the principal is still liable if the contract involves his
property.

ARTICLE 1883. If an agent acts in his own name, the principal has no right of
action against the persons with whom the agent has contracted; neither have such
persons against the principal.

In such case the agent is the one directly bound in favor of the person with
whom he has contracted, as if the transaction were his own, except when the contract
involves things belonging to the principal.

The provisions of this article shall be understood to be without prejudice to the


actions between the principal and agent.

ALTERNATIVE ANSWER:
CX would not be liable for the bank loan. CX's property would also not be liable
on the mortgage. Since DY did not specify that he was acting for CX in the
transaction with the bank, DY in effect acted in his own name. In the case of Rural
Bank of Bombon v. CA, 212 SCRA, (1992), the Supreme Court, under the same
facts, ruled that "in order to bind the principal by a mortgage on real property
executed by an agent, it must upon its face purport to be made, signed and sealed in
the name of the principal, otherwise, it will bind the agent only. It is not enough
merely that the agent was in fact authorized to make the mortgage, if he, has not acted
in the name of the principal. Neither is it ordinarily sufficient that in the mortgage the
agent describes himself as acting by virtue of a power of attorney, if in fact the agent
has acted in his own name and has set his own hand and seal to the mortgage. There is
no principle of law by which a person can become liable on a real estate mortgage
which she never executed in person or by attorney in fact".

Appointment of Sub-Agent (1999)


X appoints Y as his agent to sell his products in Cebu City. Can Y appoint a
sub-agent and if he does, what are the effects of such appointment? (5%)

SUGGESTED ANSWER:
Yes, the agent may appoint a substitute or sub-agent if the principal has not
prohibited him from doing so, but he shall be responsible for the acts of the
substitute:
(1) when he was not given the power to appoint one;
(2) when he was given such power, but without designating the person, and the
person appointed was notoriously incompetent or insolvent.

Notes:
An agent is allowed to appoint a substitute or a sub-agent if the principal did not
prohibit him from doing so.

However, he shall be liable for the act of the substitute if: (1) he is not
empowered to appoint one; and (2) he is empowered but the substitute is not
designated, and the substitute he appointed is notoriously incompetent or insolvent.

ARTICLE 1892. The agent may appoint a substitute if the principal has not
prohibited him from doing so; but he shall be responsible for the acts of the substitute:

(1) When he was not given the power to appoint one;

(2) When he was given such power, but without designating the person, and the
person appointed was notoriously incompetent or insolvent.

All acts of the substitute appointed against the prohibition of the principal shall be
void.

General Agency vs. Special Agency (1992)


A as principal appointed B as his agent granting him general and unlimited
management over A's properties, stating that A withholds no power from B and that
the agent may execute such acts as he may consider appropriate.

Accordingly, B leased A's parcel of land in Manila to C for four (4) years at
P60,000.00 per year, payable annually in advance. B leased another parcel of land of
A in Caloocan City to D without a fixed term at P3,000.00 per month payable
monthly. B sold to E a third parcel of land belonging to A located in Quezon City for
three (3) times the price that was listed in the inventory by A to B. All those contracts
were executed by B while A was confined due to illness in the Makati Medical
Center. Rule on the validity and binding effect of each of the above contracts
upon A the principal. Explain your answers.

SUGGESTED ANSWER:
The agency couched in general terms comprised only acts of administration (Art.
1877, Civil Code). The lease contract on the Manila parcel is not valid, not
enforceable and not binding upon A. For B to lease the property to C, for more than
one (1) year, A must provide B with a special power of attorney (Art. 1878. Civil
Code).
The lease of the Caloocan City property to D is valid and binding upon A. Since
the lease is without a fixed term, it is understood to be from month to month, since the
rental is payable monthly (Art. 1687, Civil Code).
The sale of the Quezon City parcel to E is not valid and not binding upon A. B
needed a special power of attorney to validly sell the land (Arts. 1877 and 1878, Civil
Code). The sale of the land at a very good price does not cure the defect of the
contract arising from lack of authority

Notes:
Under the Civil Code, when an agency is couched in general terms, it concerns
only acts of administration. Leasing the property of the principal for more than a year
is not an act of administration. Therefore, the principal is not bound by the contract of
lease.

However, lease of the property of the principal for a month does not need SPA
to be valid.

ARTICLE 1877. An agency couched in general terms comprises only acts of


administration, even if the principal should state that he withholds no power or that
the agent may execute such acts as he may consider appropriate, or even though the
agency should authorize a general and unlimited management. (n)

ARTICLE 1878. Special powers of attorney are necessary in the following


cases:

(1) To make such payments as are not usually considered as acts of


administration;

(2) To effect novations which put an end to obligations already in existence at


the time the agency was constituted;

(3) To compromise, to submit questions to arbitration, to renounce the right to


appeal from a judgment, to waive objections to the venue of an action or to abandon a
prescription already acquired;
(4) To waive any obligation gratuitously;

(5) To enter into contract that transfer ownership of an immovable by


consideration or by gratuitous title.

(6) To make gifts, except customary ones for charity or those made to
employees in the business managed by the agent;

(7) To loan or borrow money, unless the latter act be urgent and indispensable
for the preservation of the things which are under administration;

(8) To lease any real property to another person for more than one year;

(9) To bind the principal to render some service without compensation;

(10) To bind the principal in a contract of partnership;

(11) To obligate the principal as a guarantor or surety;

(12) To create or convey real rights over immovable property;

(13) To accept or repudiate an inheritance;

(14) To ratify or recognize obligations contracted before the agency;

(15) Any other act of strict dominion.

ARTICLE 1687. If the period for the lease has not been fixed, it is understood to
be from year to year, if the rent agreed upon is annual; from month to month, if it is
monthly; from week to week, if the rent is weekly; and from day to day, if the rent is
to be paid daily. However, even though a monthly rent is paid, and no period for the
lease has been set, the courts may fix a longer term for the lease after the lessee has
occupied the premises for over one year. If the rent is weekly, the courts may likewise
determine a longer period after the lessee has been in possession for over six months.
In case of daily rent, the courts may also fix a longer period after the lessee has stayed
in the place for over one month.

Notes:
Under the Civil Code, any act of ownership shall need SPA; to lease the
property for more than a year is an act of ownership; to sale the property is an act of
ownership. But leasing the property for one month is an act of administration which
does not need SPA.
Powers of the Agent (1994)
Prime Realty Corporation appointed Nestor the exclusive agent in the sale of
lots of its newly developed subdivision. Prime Realty told Nestor that he could not
collect or receive payments from the buyers. Nestor was able to sell ten lots to Jesus
and to collect the down payments for said lots. He did not turn over the collections to
Prime Realty. Who shall bear the loss for Nestor's defalcation, Prime Realty or
Jesus?

SUGGESTED ANSWER:
a) The general rule is that a person dealing with an agent must inquire into the
authority of that agent. In the present case, if Jesus did not inquire into that authority,
he is liable for the loss due to Nestor's defalcation unless Article 1900, Civil Code
governs, in which case the developer corporation bears the loss. Art. 1900 Civil Code
provides: "So far as third persons are concerned, an act is deemed to have been
performed within the scope of the agent's authority, if such act is within the terms of
the power of attorney, as written, even if the agent has in fact exceeded the limits of
his authority according to an understanding between the principal and the agent.

However, if Jesus made due inquiry and he was not informed by the principal
Prime Realty of the limits of Nestor's authority. Prime Realty shall bear the loss.

b) Considering that Prime Realty Corporation only "told" Nestor that he could
not receive or collect payments, it appears that the limitation does not appear in his
written authority or power of attorney. In this case, insofar as Jesus, who is a third
person is concerned, Nestor's acts of collecting payments is deemed to have been
performed within the scope of his authority {Article 1900. Civil Code). Hence, the
principal is liable.
However, if Jesus was aware of the limitation of Nestor's power as an agent, and
Prime Realty Corporation does not ratify the sale contract, then Jesus shall be liable
(Article 1898. Civil Code).

Notes:
Under the Civil Code, a person dealing with an agent may inquire the scope of
his power. If he does not inquire, he is liable for the act of the agent which is outside
of his authority. In this case, Jesus did not inquire the scope of the authority of Nestor.
Therefore, Jesus shall be liable for the defalcation of Nestor.
ARTICLE 1898. If the agent contracts in the name of the principal, exceeding
the scope of his authority, and the principal does not ratify the contract, it shall be
void if the party with whom the agent contracted is aware of the limits of the powers
granted by the principal. In this case, however, the agent is liable if he undertook to
secure the principal’s ratification.

ARTICLE 1902. A third person with whom the agent wishes to contract on
behalf of the principal may require the presentation of the power of attorney, or the
instructions as regards the agency. Private or secret orders and instructions of the
principal do not prejudice third persons who have relied upon the power of attorney or
instructions shown them.

Termination; Effect of Death of Agent (1997)


Stating briefly the thesis to support your answer to each of the following
cases, will the death - (c) of an agent end an agency?

SUGGESTED ANSWER:
Yes. The death of an agent extinguishes the agency, by express provision of
par. 3, Art 1919 of the Civil Code.

Notes:

ARTICLE 1919. Agency is extinguished:

(1) When it is revoked;

(2) When the agent withdraws;

(3) By the death, civil interdiction, insanity or insolvency of the principal or of


the agent;

(4) When the firm which accepts the agency is dissolved;

(5) When the purpose of the agency is accomplished;

(6) When the period for which the agency was constituted expires;

PARTNERSHIP
ARTICLE 1767. By the contract of partnership, two or more persons bind
themselves to contribute money, property, or industry to a common fund, with the
intention of dividing the profits among themselves.

Two or more persons may also form a partnership for the exercise of a
profession.

Composition of Partnerships; Spouses; Corporations (1994)


1) Can a husband and wife form a limited partnership to engage in real estate
business, with the wife being a limited partner?
2) Can two corporations organize a general partnership under the Civil Code
of the Philippines?
3) Can a corporation and an individual form a general partnership?

SUGGESTED ANSWER:
1) a) Yes. The Civil Code prohibits a husband and wife from constituting a
universal partnership, and not limited. Since a limited partnership is not a universal
partnership, a husband and wife may validly form one.
b) Yes. While spouses cannot enter into a universal partnership, they can enter
into a limited partnership or be members thereof (CIR v. Suter, et al. 27 SCRA 152).

Notes:
Under the Civil Code, a husband and wife are prohibited from constituting a
universal partnership; but they can form a limited one.

SUGGESTED ANSWER:
2) a) No, two corporations cannot organize as a general partnership. A
corporation is managed by its board of directors. If the corporation were to become
a partner, co-partners would have the power to make the corporation party to
transactions in an irregular manner since the partners are not agents subject to the
control of the Board of Directors. But a corporation may enter into a joint venture
with another corporation as long as the nature of the venture is in line with the
business authorized by its charter. (Tuason & Co., Inc. v. Bolano, 95 Phil. 106).

b) As a general rule a corporation may not form a general partnership with


another corporation or an individual because a corporation may not be bound by
persons who are neither directors nor officers of the corporation.

However, a corporation may form a general partnership with another corporation


or an individual provided the following conditions are met:
1) The Articles of Incorporation of the corporation expressly allows the
corporation to enter into partnerships;
2) The Articles of Partnership must provide that all partners will manage the
partnership, and they shall be jointly and severally liable; and
3) In case of a foreign corporation, it must be licensed to do business in the
Philippines.
c) No. A corporation may not be a general partner because the principle of
mutual agency in general partnership allowing the other general partner to bind the
corporation will violate the corporation law principle that only the board of directors
may bind the corporation.

SUGGESTED ANSWER:
3) No, for the same reasons given in the Answer to Number 2 above.

DISSOLUTION OF PARTNERSHIP

Conveyance of a Partner’s Share Dissolution (1998)


Dielle, Karlo and Una are general partners in a merchandising firm. Having
contributed equal amounts to the capital, they also agree on equal distribution of
whatever net profit is realized per fiscal period. After two years of operation, however,
Una conveys her whole interest in the partnership to Justine, without the knowledge
and consent of Dielle and Karlo.
1. Is the partnership dissolved? 12%]
2. What are the rights of Justine, if any, should she desire to participate in the
management of the partnership and in the distribution of a net profit of P360.000.00
which was realized after her purchase of Una's interest? [3%]

SUGGESTED ANSWER:
1. No, a conveyance by a partner of his whole interest in a partnership does not
of itself dissolve the partnership in the absence of an agreement. (Art. 1813. Civil
Code)

Notes:
Under the New Civil Code, a partnership is not dissolved by the conveyance of
a partner of his whole interest in the partnership.
ARTICLE 1813. A conveyance by a partner of his whole interest in the
partnership does not dissolve the partnership; it does not entitle the assignee to
interfere in the management or administration of the partnership business or
affairs, or to inspect the partnership books, to require any information or account of
partnership transactions; but it merely entitles the assignee to receive in accordance
with his contract the profits to which the assigning partner would otherwise be
entitled., or, as against the other partners in the absence of agreement, during the
continuance of the partnership,
However, in case of fraud in the management of the partnership, the assignee
may avail himself of the usual remedies.

In case of a dissolution of the partnership, the assignee is entitled to receive his


assignor’s interest and may require an account from the date only of the last account
agreed to by all the partners. (Reengineered)

SUGGESTED ANSWER:
2. Justine cannot interfere or participate in the management or administration of
the partnership business or affairs. She may, however, receive the net profits to which
Una would have otherwise been entitled. In this case, P120.000 (Art. 1813, Civil
Code)

Notes:
A person to whom the interest of a partner is conveyed cannot participate in the
management of the partnership.

However, he is entitled to the profits to which the assignor would have received;
In case of a dissolution of the partnership, the assignee is entitled to receive the
interest of the assignor.
Dissolution of Partnership (1995)
Pauline, Patricia and Priscilla formed a business partnership for the purpose of
engaging in neon advertising for a term of five (5) years. Pauline subsequently
assigned to Philip her interest in the partnership. When Patricia and Priscilla learned
of the assignment, they decided to dissolve the partnership before the expiration of its
term as they had an unproductive business relationship with Philip in the past. On the
other hand, unaware of the move of Patricia and Priscilla but sensing their negative
reaction to his acquisition of Pauline's interest, Philip simultaneously petitioned for
the dissolution of the partnership.
1. Is the dissolution done by Patricia and Priscilla without the consent of
Pauline or Philip valid? Explain.
2. Does Philip have any right to petition for the dissolution of the
partnership before the expiration of its specified term? Explain.

SUGGESTED ANSWER:
1, Under Art. 1830 (1) (c) of the NCC, the dissolution by Patricia and Priscilla is
valid and did not violate the contract of partnership even though Pauline and Philip
did not consent thereto. The consent of Pauline is not necessary because she had
already assigned her interest to Philip. The consent of Philip is not also necessary
because the assignment to him of Pauline's interest did not make him a partner, under
Art. 1813 of the NCC.

Notes:
Under the New Civil Code, a partner or all of the partners may decide for the
dissolution of partnership.

ARTICLE 1830. Dissolution is caused:


(1) Without violation of the agreement between the partners:
(a) When the definite term or particular undertaking in the agreement is
terminated;
(b) When a partner decides for its dissolution, who must act in good faith, when
no definite term or particular undertaking is specified;
(c) When all the partners who have not assigned their interests decide for its
dissolution;
(d) By the expulsion of any partner from the business bona fide in accordance
with such a power conferred by the agreement between the partners;

(2) In contravention of the agreement between the partners, where the


circumstances do not permit a dissolution under any other provision of this article, by
the express will of any partner at any time;
(3) By any event which makes it unlawful for the business of the partnership to
be carried on or for the members to carry it on in partnership;
(4) When a specific thing, which a partner had promised to contribute to the
partnership, perishes before the delivery; in any case by the loss of the thing, when the
partner who contributed it having reserved the ownership thereof, has only transferred
to the partnership the use or enjoyment of the same; but the partnership shall not be
dissolved by the loss of the thing when it occurs after the partnership has acquired the
ownership thereof;
(5) By the death of any partner;
(6) By the insolvency of any partner or of the partnership;
(7) By the civil interdiction of any partner;
(8) By decree of court under the following article. (Reengineered)

ALTERNATIVE ANSWER:
Interpreting Art. 1830 (1) (c) to mean that if one of the partners had assigned his
interest on the partnership to another the remaining partners may not dissolve the
partnership, the dissolution by Patricia and Priscilla without the consent of Pauline or
Philip is not valid.

SUGGESTED ANSWER:
2. No, Philip has no right to petition for dissolution because he does not have
the standing of a partner (Art. 1813 NCC).

Notes:
The assignee of the interest of a partner does not become a partner, so he has no
standing to ask for the dissolution of the partnership.

ARTICLE 1813. A conveyance by a partner of his whole interest in the


partnership does not of itself dissolve the partnership, or, as against the other
partners in the absence of agreement, entitle the assignee, during the continuance of
the partnership, to interfere in the management or administration of the partnership
business or affairs, or to require any information or account of partnership
transactions, or to inspect the partnership books; but it merely entitles the assignee to
receive in accordance with his contract the profits to which the assigning partner
would otherwise be entitled. However, in case of fraud in the management of the
partnership, the assignee may avail himself of the usual remedies.

In case of a dissolution of the partnership, the assignee is entitled to receive his


assignor’s interest and may require an account from the date only of the last account
agreed to by all the partners.

Dissolution of Partnership; Termination (1993)


A, B and C formed a partnership for the purpose of contracting with the
Government in the construction of one of its bridges. On June 30, 1992, after
completion of the project, the bridge was turned over by the partners to the
Government. On August 30, 1992, D, a supplier of materials used in the project sued
A for collection of the indebtedness to him. A moved to dismiss the complaint against
him on the ground that it was the ABC partnership that is liable for the debt. D replied
that ABC partnership was dissolved upon completion of the project for which purpose
the partnership was formed. Will you dismiss the complaint against A If you were
the Judge?

SUGGESTED ANSWER:
As Judge, I would not dismiss the complaint against A because A is still liable
as a general partner for his pro rata share of 1/3 (Art. 1816, C. C.J. Dissolution of
a partnership caused by the termination of the particular undertaking specified in the
agreement does not extinguish obligations, which must be liquidated during the
"winding up" of the partnership affairs (Articles 1829 and 1830. par. 1-a, Civil Code).

Notes:
Under the Civil Code, a partnership is dissolved upon the termination of the
undertaking in which it was established. However, it continues to exist to settle its
own obligation during winding up.

ARTICLE 1816. All partners, including industrial ones, shall be liable pro rata
with all their property and after all the partnership assets have been exhausted, for the
contracts which may be entered into in the name and for the account of the
partnership, under its signature and by a person authorized to act for the partnership.
However, any partner may enter into a separate obligation to perform a partnership
contract.

ARTICLE 1829. On dissolution, the partnership is not terminated, but


continues until the winding up of partnership affairs is completed.

ARTICLE 1830. Dissolution is caused:

(1) Without violation of the agreement between the partners:

(a) By the termination of the definite term or particular undertaking specified in


the agreement;

(b) By the express will of any partner, who must act in good faith, when no
definite term or particular undertaking is specified;
(c) By the express will of all the partners who have not assigned their interests or
suffered them to be charged for their separate debts, either before or after the
termination of any specified term or particular undertaking;

(d) By the expulsion of any partner from the business bona fide in accordance
with such a power conferred by the agreement between the partners;

(2) In contravention of the agreement between the partners, where the


circumstances do not permit a dissolution under any other provision of this article, by
the express will of any partner at any time;

(3) By any event which makes it unlawful for the business of the partnership to
be carried on or for the members to carry it on in partnership;

(4) When a specific thing, which a partner had promised to contribute to the
partnership, perishes before the delivery; in any case by the loss of the thing, when the
partner who contributed it having reserved the ownership thereof, has only transferred
to the partnership the use or enjoyment of the same; but the partnership shall not be
dissolved by the loss of the thing when it occurs after the partnership has acquired the
ownership thereof;

(5) By the death of any partner;

(6) By the insolvency of any partner or of the partnership;

(7) By the civil interdiction of any partner;

(8) By decree of court under the following article. (1700a and 1701a)

Notes:
Under the Civil Code, dissolution alone does not terminate a partnership. Its
termination becomes possible only after winding up.

Effect of Death of Partner (1997)


Stating briefly the thesis to support your answer to each of the following cases,
will the death - of a partner terminate the partnership?

SUGGESTED ANSWER:
Yes. The death of a partner will terminate the partnership, by express provision
of par. 5, Art. 1830 of the Civil Code.
Obligations of a Partner (1992)
W, X, Y and Z organized a general partnership with W and X as industrial
partners and Y and Z as capitalist partners. Y contributed P50,000.00 and Z
contributed P20,000.00 to the common fund. By a unanimous vote of the partners, W
and X were appointed managing partners, without any specification of their
respective powers and duties.
A applied for the position of Secretary and B applied for the position of
Accountant of the partnership.

The hiring of A was decided upon by W and X, but was opposed by Y and Z.
The hiring of B was decided upon by W and Z, but was opposed by X and Y.

Who of the applicants should be hired by the partnership? Explain and give
your reasons.

SUGGESTED ANSWER:
A should be hired as Secretary. The decision for the hiring of A prevails
because it is an act of administration which can be performed by the duly
appointed managing partners, W and X. B cannot be hired, because in case of a tie
in the decision of the managing partners, the deadlock must be decided by the partners
owning the controlling interest. In this case, the opposition of X and Y prevails
because Y owns the controlling Interest (Art. 1801, Civil Code).

Notes:
Under the Civil Code, if two or more partners are designated to manage the
partnership and it is not stated that each partner cannot act without the consent of the
others, each partner may execute any act of administration; but if anyone of them
opposes the other, the majority shall prevail. In case of a tie, the deadlock shall be
resolved by the partner who has the controlling interest.

ARTICLE 1801. If two or more partners have been entrusted with the
management of the partnership without specification of their respective duties, or
without a stipulation that one of them shall not act without the consent of all the
others, each one may separately execute all acts of administration, but if any of them
should oppose the acts of the others, the decision of the majority shall prevail. In case
of a tie, the matter shall be decided by the partners owning the controlling interest.

Obligations of a Partner; Industrial Partner (2001)


Joe and Rudy formed a partnership to operate a car repair shop in Quezon
City. Joe provided the capital while Rudy contributed his labor and industry. On
one side of their shop, Joe opened and operated a coffee shop, while on the other side,
Rudy put up a car accessories store. May they engage in such separate
businesses? Why? [5%]

SUGGESTED ANSWER:
Joe, the capitalist partner, may engage in the restaurant business because it is not
the same kind of business the partnership is engaged in. On the other hand, Rudy may
not engage in any other business unless their partnership expressly permits him to do
so because as an industrial partner he has to devote his full time to the business
of the partnership [Art. 1789, CC).

Notes:
Under the Civil Code, a capitalist partner may engage in business provided it is
not the same business as the partnership.

On the other hand, an industrial partner cannot engage in business because he


has to devote his full time to the business of the partnership. But if the partnership
allows him do so, he may engage in business.

ARTICLE 1789. An industrial partner cannot engage in business for himself,


unless the partnership expressly permits him to do so; if he engages in business
without the permission, the capitalist partners may exclude him from the firm or ask
for damages.

Commodatum & Mutuum

Commodatum (1993)
A, upon request, loaned his passenger Jeepney to B to enable B to bring his sick
wife from Paniqui, Tarlac to the Philippine General Hospital in Manila for treatment.
On the way back to Paniqui, after leaving his wife at the hospital, people stopped the
passenger Jeepney. B stopped for them and allowed them to ride on board, accepting
payment from them just as in the case of ordinary passenger Jeepneys plying their
route. As B was crossing Bamban, there was an onrush of Lahar from Mt Pinatubo,
the Jeep that was loaned to him was wrecked.
1) What do you call the contract that was entered into by A and B with
respect to the passenger Jeepney that was loaned by A to B to transport the
latter's sick wife to Manila?
2) Is B obliged to pay A for the use of the passenger
jeepney?
3) Is B liable to A for the loss of the Jeepney?

SUGGESTED ANSWER:
1) The contract is called "commodatum". [Art. 1933. Civil Code).
COMMODATUM is a contract by which one of the parties (bailor) delivers to another
(bailee) something not consumable so that the latter may use it for a certain time and
return it.
2) No, B is not obliged to pay A for the use of the passenger Jeepney because
commodatum is essentially gratuitous. (Art. 1933. Civil Code]
3) Yes, because B devoted the thing to a purpose different from that for which
it has been loaned (Art. 1942, par. 2, Civil Code)

Notes:
Under the Civil Code, a contract of commodatum is one which a bailor delivers
something not consumable to the bailee so that the latter may use it for a certain time
and return it. This contract is gratuitous, so the bailee is not obliged to pay a rent.
However, the thing loaned must be used according to the purpose for which the
thing has been loaned; otherwise, the bailee will be held liable for any damage to the
thing event though it was caused by fortuitous event.

ARTICLE 1933. By the contract of loan, one of the parties delivers to another
something not consumable so that the latter may use it for a certain time and return
the same; the contract is called a commodatum;

If what was delivered is money or other consumable thing, on the condition that
the same amount of the same kind and quality shall be paid; the contract is called a
loan or mutuum.

Commodatum is gratuitous.

Simple loan may be gratuitous or with a stipulation to pay interest.


In commodatum, the bailor retains the ownership of the thing loaned, while in
simple loan, ownership passes to the borrower.

ALTERNATIVE ANSWER:
No, because an obligation which consists in the delivery of a determinate thing
shall be extinguished if it should be lost or destroyed without the fault of the debtor,
and before he has incurred in delay. (Art. 1262. Civil Code)

Notes:
Under the Civil Code, an obligation to deliver a determinate thing shall be
extinguished if it is lost or destroyed without the fault of the debtor and before the
debtor incurs in delay.

Notes:
ARTICLE 1262. An obligation which consists in the delivery of a determinate
thing shall be extinguished if it should be lost or destroyed without the fault of the
debtor, and before he has incurred in delay.

When by law or stipulation, the obligor is liable even for fortuitous events, the
loss of the thing does not extinguish the obligation, and he shall be responsible for
damages. The same rule applies when the nature of the obligation requires the
assumption of risk.

Commodatum (2005)
Before he left for Riyadh to work as a mechanic, Pedro left his Adventure van
with Tito, with the understanding that the latter could use it for one year for his
personal or family use while Pedro works in Riyadh. He did not tell Tito that the
brakes of the van were faulty. Tito had the van tuned up and the brakes repaired. He
spent a total amount of P15,000.00. After using the vehicle for two weeks, Tito
discovered that it consumed too much fuel. To make up for the expenses, he leased
it to Annabelle.

Two months later, Pedro returned to the Philippines and asked Tito to return the
van. Unfortunately, while being driven by Tito, the van was accidentally damaged by
a cargo truck without his fault.

a) Who shall bear the P15,000.00 spent for the repair of the van? Explain.
(2%)
ALTERNATIVE ANSWER:
Tito must bear the P15,000.00 expenses for the van. Generally, extraordinary
expenses for the preservation of the thing loaned are paid by the bailor, he being the
owner of the thing loaned. In this case however, Tito should bear the expenses
because he incurred the expenses without first informing Pedro about it. Neither
was the repair shown to be urgent. Under Article 1949 of the Civil Code, bailor
generally bears the extraordinary expenses for the preservation of the thing and should
refund the said expenses if made by the bailee; Provided, the bailee brings the same to
the attention of the bailor before incurring them, except only if the repair is urgent that
reply cannot be awaited.

Notes:

Under the Civil Code, the owner of the thing loaned shall bear the extraordinary
expenses provided the bailee should inform the bailor before incurring them. In this
case, the bailee did not inform the bailor before he incurred the extraordinary
expenses for the fixing of the faulty break of the vehicle. Therefore, the bailee shall
bear the expenses.

ARTICLE 1949. The bailor shall refund the extraordinary expenses during the
contract for the preservation of the thing loaned, provided the bailee brings the same
to the knowledge of the bailor before incurring them, except when they are so urgent
that the reply to the notification cannot be awaited without danger.

If the extraordinary expenses arise on the occasion of the actual use of the thing
by the bailee, even though he acted without fault, they shall be borne equally by both
the bailor and the bailee, unless there is a stipulation to the contrary.

ALTERNATIVE ANSWER:
The P15,000.00 spent for the repair of the van should be borne by Pedro. Where
the bailor delivers to the bailee a non-consummable thing so that the latter may use it
for a certain time and return the identical thing, the contract perfected is a Contract of
Commodatum. (Art. 1933, Civil Code) The bailor shall refund the extraordinary
expenses during the contract for the preservation of the thing loaned provided the
bailee brings the same to the knowledge of the bailor before incurring the same,
except when they are so urgent that the reply to the notification cannot be awaited
without danger. (Art. 1949 of the Civil Code)
In the given problem, Pedro left his Adventure van with Tito so that the latter
could use it for one year while he was in Riyadh. There was no mention of a
consideration. Thus, the contract perfected was commodatum. The amount of
P15,000.00 was spent by Tito to tune up the van and to repair its brakes. Such
expenses are extra-ordinary expenses because they are necessary for the preservation
of the van Thus, the same should be borne by the bailor, Pedro.

The amount of P15,000.00 was spent by Tito to tune up the van and to repair its
brakes. Such expenses are extra-ordinary expenses because they are necessary for
the preservation of the van. Thus, the same should be borne by the bailor, Pedro.
b) Who shall bear the costs for the van's fuel, oil and other materials while
it was with Tito? Explain. (2%)

SUGGESTED ANSWER:
Tito must also pay for the ordinary expenses for the use and preservation of the
thing loaned. He must pay for the gasoline, oil, greasing and spraying. He cannot ask
for reimbursement because he has the obligation to return the identical thing to the
bailor. Under Article 1941 of the Civil Code, the bailee is obliged to pay for the
ordinary expenses for the use and preservation of the thing loaned.

Notes:
Under the Civil Code, a bailee is obliged to pay for the ordinary expenses for
gasoline, oil, greasing, among others.

ARTICLE 1941. The bailee is obliged to pay for the ordinary expenses for the
use and preservation of the thing loaned.

c) Does Pedro have the right to retrieve the van even before the lapse of
one year? Explain. (2%)
ALTERNATIVE ANSWER:
No, Pedro does not have the right to retrieve the van before the lapse of one
year. The parties are mutually bound by the terms of the contract. Under the Civil
Code, there are only 3 instances when the bailor could validly ask for the return of the
thing loaned even before the expiration of the period. These are when: (1) a
precarium contract was entered (Article 1947); (2) if the bailor urgently needs the
thing (Article 1946); and (3) if the bailee commits acts of ingratitude (Article 1948).
Not one of the situations is present in this case.
The fact that Tito had leased the thing loaned to Annabelle would not justify the
demand for the return of the thing loaned before expiration of the period. Under
Article 1942 of the Civil Code, leasing of the thing loaned to a third person not
member of the household of the bailee, will only entitle bailor to hold bailee liable for
the loss of the thing loaned.

Notes:
Under the Civil Code, a bailor may demand the return of the thing loaned if: 1)
he urgently needs it; or 2) the contractual relation between him and the bailee is called
precarium; 3) the bailee commits an act of ingratitude.

ARTICLE 1946. The bailor cannot demand the return of the thing loaned till
after the expiration of the period stipulated, or after the accomplishment of the use for
which the commodatum has been constituted. However, if in the meantime, he should
have urgent need of the thing, he may demand its return or temporary use.

In case of temporary use by the bailor, the contract of commodatum is


suspended while the thing is in the possession of the bailor. (1749a)

ARTICLE 1947. The bailor may demand the thing at will, and the contractual
relation is called a precarium, in the following cases:

(1) If neither the duration of the contract nor the use to which the thing loaned
should be devoted, has been stipulated; or

(2) If the use of the thing is merely tolerated by the owner.

ARTICLE 1948. The bailor may demand the immediate return of the thing if the
bailee commits any act of ingratitude specified in article 765.

ARTICLE 765. The donation may also be revoked at the instance of the donor,
by reason of ingratitude in the following cases:

(1) If the donee should commit some offense against the person, the honor or
the property of the donor, or of his wife or children under his parental authority;

(2) If the donee imputes to the donor any criminal offense, or any act involving
moral turpitude, even though he should prove it, unless the crime or the act has been
committed against the donee himself, his wife or children under his authority;
(3) If he unduly refuses him support when the donee is legally or morally bound
to give support to the donor.

ALTERNATIVE ANSWER:
As a rule, Pedro does not have the right to retrieve the van before the lapse of
one year. Article 1946 of the Code provides that "the bailor cannot demand the return
of the thing loaned till after the expiration of the period stipulated, or after the
accomplishment of the use for which the commodatum has been constituted.
However, if in the meantime, he should have urgent need of the thing, he may demand
its return or temporary use." In the given problem, Pedro allowed Tito to use the van
for one year. Thus, he should be bound by the said agreement and he cannot ask for
the return of the car before the expiration of the one year period. However, if Pedro
has urgent need of the van, he may demand for its return or temporary use.

Notes:
ARTICLE 1946. The bailor cannot demand the return of the thing loaned till
after the expiration of the period stipulated, or after the purpose of commodatum is
accomplished. However, if in the meantime, he urgently needs the thing, he may
demand its return or temporary use.

In case of temporary use by the bailor, the contract of commodatum is


suspended while the thing is in the possession of the bailor.

d) Who shall bear the expenses for the accidental damage caused by the
cargo truck, granting that the truck driver and truck owner are insolvent?
Explain. (2%)

SUGGESTED ANSWER:
Generally, extraordinary expenses arising on the occasion of the actual use of the
thing loaned by the bailee, even if incurred without fault of the bailee, shall be
shouldered equally by the bailor and the bailee. (Art. 1949 of the Civil Code).
However, if Pedro had an urgent need for the vehicle, Tito would be in delay for
failure to immediately return the same, then Tito would be held liable for the
extraordinary expenses.

Notes:
Under the Civil Code, if the extraordinary expenses are incurred due to a
damage sustained during the use of the thing by the bailee, the bailor and the bailee
shall equally bear such expenses even though the bailee was not at fault when the
incident happened.

ARTICLE 1949. The bailor shall refund the extraordinary expenses during the
contract for the preservation of the thing loaned, provided the bailee brings the same
to the knowledge of the bailor before incurring them, except when they are so urgent
that the reply to the notification cannot be awaited without danger.

If the extraordinary expenses arise on the occasion of the actual use of the
thing by the bailee, even though he acted without fault, they shall be borne equally by
both the bailor and the bailee, unless there is a stipulation to the contrary.

Commodatum vs. Usufruct (1998)


Distinguish usufruct from commodatum and state whether these may be
constituted over consumable goods. [2%]

SUGGESTED ANSWER:
1. USUFRUCT is a right given to a person (usufructuary) to enjoy the
property of another with the obligation of preserving its form and substance. (Art.
562. Civil Code)

On the other hand, COMMODATUM is a contract by which one of the parties


(bailor) delivers to another (bailee) something not consumable so that the latter may
use it for a certain time and return it.

In usufruct, the usufructuary gets the right to the use and to the fruits of the
same, while in commodatum, the bailee only acquires the use of the thing loaned but
not its fruits.

Usufruct may be constituted on the whole or a part of the fruits of the thing.
(Art. 564. Civil Code). It may even be constituted over consumables like money
(Alunan v. Veloso, 52 Phil. 545).

On the other hand, in commodatum, consumable goods may be subject thereof


only when the purpose of the contract is not the consumption of the object, as when it
is merely for exhibition. (Art. 1936, Civil Code)

Notes:
Under the Civil Code, commodatum is a contract whereby the bailor delivers to
the baillee a thing not consumable so that the latter may use it and return the same.

On the other hand, usufruct is the right of the usufructuary to the fruits of the
property.
In the case of Alunan v. Veloso, the Supreme Court held that usufruct may be
constituted on money. Likewise, under the Civil Code, commodatum may be
constituted on money when it is merely used for exhibition.

ANOTHER ANSWER:
1. There are several points of distinction between usufruct and commodatum.
Usufruct is constituted by law, by contract, by testamentary succession, or by
prescription (Art. 1933. Civil Code). Usufruct creates a real right to the fruits of
another's property, while commodatum creates only a purely personal right to use
another's property, and requires a stipulation to enable the bailee to "make use" of the
fruits (Arts. 1939& 1940, Civil Code). Usufruct maybe onerous while commodatum is
always or essentially gratuitous (Arts. 1933 & 1935, Civil Code). The contract
constituting usufruct is consensual, while commodatum is a real contract (perfected
only by delivery of the subject matter thereof).

However, both involve the enjoyment by a person of the property of another,


differing only as to the extent and scope of such enjoyment [jus fruendi in one and Jus
utendi in the other); both may have as subject matter either an immovable or a
movable; and, both maybe constituted over consumable goods (Arts. 574 & 1936,
Civil Code). A consumable thing may be the subject-matter of an abnormal usufruct
but in a normal usufruct, the subject-matter may be used only for exhibition. A
commodatum of a consumable thing may be only for the purpose of exhibiting, not
consuming it.

Mutuum vs. Commodatum (2004)


Distinguish briefly but clearly between Mutuum and commodatum.

SUGGESTED ANSWER:
In MUTUUM, the object borrowed must be a consumable thing the ownership
of which is transferred to the borrower who incurs the obligation to return the same
consumable to the lender in an equal amount, and of the same kind and quality. In
COMMODATUM, the object borrowed is usually a non-consumable thing the
ownership of which is not transferred to the borrower who incurs the obligation to
return the very thing to the lender.

Notes:
Under the Civil Code, mutuum is constituted upon consumable thing, while
commodatum is constituted upon a non-consumable thing. The former may be
onerous, while the latter is always gratuitous. In the former, ownership to the thing is
transferred, while in the latter, ownership is not transferred.

Mutuum; Interests (2001)


Samuel borrowed P300,000.00 housing loan from the bank at 18% per annum
interest. However, the promissory note contained a proviso that the bank "reserves the
right to increase interest within the limits allowed by law," By virtue of such proviso,
over the objections of Samuel, the bank increased the interest rate periodically until it
reached 48% per annum. Finally, Samuel filed an action questioning the right of the
bank to increase the interest rate up to 48%. The bank raised the defense that the
Central Bank of the Philippines had already suspended the Usury Law. Will the
action prosper or not? Why? (5%)

SUGGESTED ANSWER:
The action will prosper. While it is true that the interest ceilings set by the Usury
Law are no longer in force, it has been held that PD No. 1684 and CB Circular No.
905 merely allow contracting parties to stipulate freely on any adjustment in the
interest rate on a loan or forbearance of money but do not authorize a unilateral
increase of the interest rate by one party without the other's consent (PNB v. CA, 238
SCRA 2O [1994]]). To say otherwise will violate the principle of mutuality of
contracts under Article 1308 of the Civil Code. To be valid, therefore, any change of
interest must be mutually agreed upon by the parties (Dizon v, Magsaysay, 57
SCRA 25O [1974]). In the present problem, the debtor not having given his consent to
the increase in interest, the increase is void.

Notes:
In Dizon v. Magsaysay, it was held that unilateral increase of interest is not
allowed because it violates the principle of mutuality of contract.

Moreover, in one case the Supreme Court held that although the interest ceiling
is suspended by Usury Law, parties cannot agree on interest that is unreasonable or
unconscionable.
Mutuum; Interests (2002)
Carlos sues Dino for (a) collection on a promissory note for a loan, with no
agreement on interest, on which Dino defaulted, and (b) damages caused by Dino on
his (Carlos’) priceless Michaelangelo painting on which Dino is liable on the
promissory note and awards damages to Carlos for the damaged painting, with
interests for both awards. What rates of interest may the court impose with respect
to both awards? Explain. (5%)

SUGGESTED ANSWER:
With respect to the collection of money or promissory note, it being a
forbearance of money, the legal rate of interest for having defaulted on the payment
of 12% will apply. With respect to the damages to the painting, it is 6% from the time
of the final demand up to the time of finality of judgment until judgment credit is fully
paid. The court considers the latter as a forbearance of money. (Eastern Shipping
Lines, Inc. v. CA, 234 SCRA 78 [1994]; Art 2210 and 2211, CC)

Notes:
In Eastern Shipping Lines, Inc. v. CA, it was held that the rate of interest for the
loan of money is 12% per annum if the interest is not stipulated by the parties to the
contract. The rate of interest of 6% is allowed on the award of damages.

Mutuum; Interests (2004)


The parties in a contract of loan of money agreed that the yearly interest rate is
12% and it can be increased if there is a law that would authorize the increase of
interest rates. Suppose OB, the lender, would increase by 5% the rate of interest to be
paid by TY, the borrower, without a law authorizing such increase, would OB’s
action be just and valid? Why? Has TY a remedy against the imposition of the
rate increase? Explain. (5%)

SUGGESTED ANSWER:
OB's action is not just and valid. The debtor cannot be required to pay the
increase in interest there being no law authorizing it, as stipulated in the contract.
Increasing the rate in the absence of such law violates the principle of mutuality of
contracts.

Notes:
Under the principle of mutuality of contract, parties may agree on stipulation
provided it is not contrary to law. In this case, they agree that the interest may be
increased if there is a law allowing such increase. Since there is no which allows the
increase of the interest, one of the parties to this contract cannot increase the interest
because it violates the principle of mutuality of contract.

ALTERNATIVE ANSWER:
Even if there was a law authorizing the increase in interest rate, the stipulation is
still void because there is no corresponding stipulation to decrease the interest due
when the law reduces the rate of interest.

DEPOSIT

Compensation; Bank Loan (1997)


In order to secure a bank loan, XYZ Corporation surrendered its deposit
certificate, with a maturity date of 01 September 1997 to the bank. The corporation
defaulted on the due repayment of the loan, prompting the bank to encash the deposit
certificate. XYZ Corporation questioned the above action taken by the bank as being a
case of pactum commissorium. The bank disagrees. What is your opinion?

SUGGESTED ANSWER:
We submit that there is no pactum commissorium here. Deposits of money in
banks and similar institutions are governed by the provisions on simple loans (Art.
1980. Civil Code). The relationship between the depositor and a bank is one of
creditor and debtor. Basically this is a matter of compensation as all the elements of
compensation are present in this case (BPI vs. CA, 232 SCRA 302).

Notes:
Under the Civil Code, there is compensation when persons are mutually debtors
and creditors of each other and both debts are due and demandable.

On the other hand, a pactum commissorium is the automatic appropriation of a


creditor of the property of a debtor; this is unlawful.

ADDITIONAL ANSWER:
Where the security for the debt is also money deposited in a bank, it is not
illegal for the creditor to encash the time deposit certificates to pay the debtor's
overdue obligation.
(Chu v. CA, et al., G.R 78519, September 26, 1989).

Notes:
In Chu v. CA, it was held that the time deposit certificate which is held as a
security for the loan of money may be encashed by the creditor if the debtor fails to
pay his obligation on time. That act is not prohibited by the doctrine of pactum
commissiorium.

Deposit; Exchange (1992)


X and Y staged a daring bank robbery in Manila at 10:30 AM in the morning of
a regular business day, and escaped with their loot of two (2) bags, each bag
containing P50,000,00. During their flight to elude the police, X and Y entered the
nearby locked house of A, then working in his Quezon City office. From A's house, X
and Y stole a box containing cash totaling P50,000.00 which box A had been keeping
in deposit for his friend B.
In their hurry, X and Y left in A's bedroom one (1) of the bags which they had
taken from the bank.

With X and Y now at large and nowhere to be found, the bag containing
P50.000.00 is now claimed by B, by the Mayor of Manila, and by the bank.

B claims that the depositary A, by force majeure had obtained the bag of
money in place of the box of money deposited by B.

The Mayor of Manila, on the other hand, claims that the bag of money should
be deposited with the Office of the Mayor as required of the finder by the provisions
of the Civil Code.
The bank resists the claims of B and the Mayor of Manila.
To whom should a deliver the bag of money? Decide with reasons.

SUGGESTED ANSWER:
B would have no right to claim the money. Article 1990 of the Civil Code is not
applicable. The law refers to another thing received in substitution of the object
deposited and is predicated upon something exchanged.

The Mayor of Manila cannot invoke Article 719 of the Civil Code which
requires the finder to deposit the thing with the Mayor only when the previous
possessor is unknown. In this case, A must return the bag of money to the bank as the
previous possessor and known owner (Arts. 719 and 1990. Civil Code.)

Notes:
The bank is entitled to the bag of money which was stolen from it because one
who is unlawfully deprived of his personal property may take it back from the
possessor thereof.

B cannot claim the bag because A is not a depositary. Thus, the provision in the
Civil Code regarding the replacement of the thing lost by another is not applicable.

The mayor cannot claim the bag because it was not found and the owner is
unknown. Here the owner is known.

ARTICLE 1990. If the depositary by force majeure or government order loses


the thing and receives money or another thing in its place, he shall deliver the sum or
other thing to the depositor.

ARTICLE 719. Whoever finds a movable, which is not treasure, must return it
to its previous possessor. If the latter is unknown, the finder shall immediately
deposit it with the mayor of the city or municipality where the finding has taken
place.

The finding shall be publicly announced by the mayor for two consecutive
weeks in the way he deems best.

If the movable cannot be kept without deterioration, or without expenses which


considerably diminish its value, it shall be sold at public auction eight days after the
publication.

Six months from the publication having elapsed without the owner having
appeared, the thing found, or its value, shall be awarded to the finder. The finder and
the owner shall be obliged, as the case may be, to reimburse the expenses.

SURETY

Recovery of Deficiency (1997)


AB sold to CD a motor vehicle for and in consideration of P120,000.00 to be
paid in twelve monthly equal installments of P10,000,00, each installment being due
and payable on the 15th day of each month starting January 1997.
To secure the promissory note, CD (a) executed a chattel mortgage on the
subject motor vehicle, and (b) furnished a surety bond issued by Philam life, CD
failed to pay more than two (2) installments, AB went after the surety but he was
only able to obtain three-fourths (3/4) of the total amount still due and owing from
CD. AB seeks your advice on how he might, if at all, recover the deficiency. How
would you counsel AB?

SUGGESTED ANSWER:
Yes, he can recover the deficiency. The action of AB to go after the surety bond
cannot be taken to mean a waiver of his right to demand payment for the whole debt,
the amount received from the surety is only payment pro tanto, and an action may be
maintained for a deficiency debt.

Notes:
Under the Recto Law, if a seller foreclosed a chattel mortgage constituted upon a
movable which is the object of sale on installment, he can no longer exact fulfillment
of the obligation or cancel the contract of sale. In this case, there was no foreclosure of
the chattel mortgage; the seller went after the surety bond. Therefore, he can still
recover the deficiency because collection on the surety bond is only payment pro
tanto.

Recto Law is otherwise known as the Installment Sales Law.

ARTICLE 1484. In a contract of sale of personal property the price of which is


payable in installments, the vendor may exercise any of the following remedies:

(1) Exact fulfillment of the obligation, should the vendee fail to pay;

(2) Cancel the sale, should the vendee’s failure to pay cover two or more
installments;

(3) Foreclose the chattel mortgage on the thing sold, if one has been constituted,
should the vendee’s failure to pay cover two or more installments. In this case, he
shall have no further action against the purchaser to recover any unpaid balance
of the price. Any agreement to the contrary shall be void.

ARTICLE 1485. The preceding article shall be applied to contracts purporting to


be leases of personal property with option to buy, when the lessor has deprived the
lessee of the possession or enjoyment of the thing.
ARTICLE 1486. In the cases referred to in the two preceding articles, a
stipulation that the installments or rents paid shall not be returned to the vendee or
lessee shall be valid insofar as the same may not be unconscionable under the
circumstances.

ANTICHRESIS
Antichresis (1995)
Olivia owns a vast mango plantation which she can no longer properly manage
due to a lingering illness. Since she is indebted to Peter in the amount of P500.000.00,
she asks Peter to manage the plantation and apply the harvest to the payment of her
obligation to him, principal and interest, until her indebtedness shall have been fully
paid. Peter agrees.

1) What kind of contract is entered into between Olivia and Peter? Explain.
2) What specific obligations are imposed by law on Peter as a consequence of
their contract?
3) Does the law require any specific form for the validity of their contract?
Explain
4) May Olivia re-acquire the plantation before her entire indebtedness shall have
been fully paid? Explain.

SUGGESTED ANSWER:
1. A contract of antichresis was entered into between Olivia and Peter. Under
Article 2132 of the New Civil Code, by a contract of antichresis the creditor acquires
the right to receive the fruits of an immovable of his debtor, with the obligation to
apply them to the payment of the interest, and thereafter to the principal of his
credit.

Notes:
Under the Civil Code, a contract of antichresis is one constituted upon an
immovable whereby a creditor has the right to the fruits of such immovable for the
payment of the interest, and thereafter to the principal of his credit.

ARTICLE 2132. By the contract of antichresis the creditor acquires the right to
receive the fruits of an immovable of his debtor, with the obligation to apply them to
the payment of the interest, if owing, and thereafter to the principal of his credit.
SUGGESTED ANSWER:
2. Peter must pay taxes and charges upon the land and bear the necessary
expenses for preservation and repair which he may deduct from the fruits. (Art,
2135, NCC)

Notes:
ARTICLE 2135. The creditor, unless there is a stipulation to the contrary, is
obliged to pay the taxes and charges upon the estate.
He is also bound to bear the expenses necessary for its preservation and repair.
The sums spent for the purposes stated in this article shall be deducted from the fruits.
SUGGESTED ANSWER:
3. The amount of the principal and interest must be specified in writing,
otherwise the antichresis will be void. (Art. 2134, NCC)

ARTICLE 2134. The amount of the principal and of the interest shall be
specified in writing; otherwise, the contract of antichresis shall be void.

Notes:
Antichresis is a contract upon an immovable which the creditor has the right to
the fruits of the immovable; the creditor is obliged to apply the proceeds of the fruits
to the payment of interest and thereafter to the principal.

2. The creditor must pay taxes and and necessary expenses for preservation of
the land which he may deduct from the fruits.

3. The contract of antichresis shall be in writing; otherwise, it is void and the


agreement on interest is also void.

ARTICLE 2134. The amount of the principal and of the interest shall be
specified in writing; otherwise, the contract of antichresis shall be void. (n)

ARTICLE 2135. The creditor, unless there is a stipulation to the contrary, is


obliged to pay the taxes and charges upon the estate. He is also bound to bear the
expenses necessary for its preservation and repair. The sums spent for the purposes
stated in this article shall be deducted from the fruits.

SUGGESTED ANSWER:
4. No. Art. 2136 specifically provides that the debtor cannot re-acquire the
enjoyment of the immovable without first having totally paid what he owes the
creditor. However, it is potestative on the part of the creditor to do so in order to
exempt him from his obligation under Art. 2135, NCC, the debtor cannot re-acquire
the enjoyment unless Peter compels Olivia to enter again the enjoyment of the
property.

Notes:
ARTICLE 2136. The debtor cannot reacquire the enjoyment of the immovable
without first having totally paid what he owes the creditor.

But the latter, in order to exempt himself from the obligations imposed upon him
by the preceding article, may always compel the debtor to enter again upon the
enjoyment of the property, except when there is a stipulation to the contrary.

PLEDGE

Pledge (1994)
In 1982, Steve borrowed P400.000.00 from Danny, collateralized by a pledge
of shares of stock of Concepcion Corporation worth P800,000,00. In 1983, because
of the economic crisis, the value of the shares pledged fell to only P100,000.00. Can
Danny demand that Steve surrender the other shares worth P700,000.00?

SUGGESTED ANSWER:
a) No. Bilateral contracts cannot be changed unilaterally. A pledge is only a
subsidiary contract, and Steve is still indebted to Danny for the amount of
P400,000.00 despite the fall in the value of the stocks pledged.

b) No. Danny's right as pledgee is to sell the pledged shares at a public sale
and keep the proceeds as collateral for the loan. There is no showing that the fall in
the value of the pledged property was attributable to the pledger's fault or fraud. On
the contrary, the economic crisis was the culprit. Had the pledgee been deceived as to
the substance or quality of the pledged shares of stock, he would have had the right to
claim another thing in their place or to the immediate payment of the obligation. This
is not the case here.
Notes:

Under the Civil Code, the thing pledged may be sold at public sale if it is in
danger of destruction, impairment or diminution in value. In this case, the share
pledged fell in value without the fault of the pledger. Therefore, the right of the
pledgee is limited to selling the securities, but he cannot demand that another security
be given.

ARTICLE 2108. If, without the fault of the pledgee, there is danger of
destruction, impairment, or diminution in value of the thing pledged, he may cause
the same to be sold at a public sale. The proceeds of the auction shall be a security for
the principal obligation in the same manner as the thing originally pledged.

Pledge (2004)
ABC loaned to MNO P40,000 for which the latter pledged 400 shares of stock
in XYZ Inc. It was agreed that if the pledgor failed to pay the loan with 10% yearly
interest within four years, the pledgee is authorized to foreclose on the shares of stock.
As required, MNO delivered possession of the shares to ABC with the understanding
that the shares would be returned to MNO upon the payment of the loan. However,
the loan was not paid on time.

A month after 4 years, may the shares of stock pledged be deemed owned
by ABC or not? Reason. (5%)

SUGGESTED ANSWER:
The shares of stock cannot be deemed owned by ABC upon default of MNO.
They have to be foreclosed. Under Article 2088 of the Civil Code, the creditor
cannot appropriate the things given by way of pledge. And even if the parties have
stipulated that ABC becomes the owner of the shares in case MNO defaults on the
loan, such stipulation is void for being a pactum commissorium.

Pledge; Mortgage; Antichresis (1996)


In the province, a farmer couple borrowed money from the local merchant. To
guarantee payment, they left the Torrens Title of their land with the merchant, for him
to hold until they pay the loan. Is there a - a) contract of pledge, b) contract of
mortgage, c) contract of antichresis, or d) none of the above? Explain.

SUGGESTED ANSWER:
None of the above. There is no pledge because only movable property may be
pledged (Art. 2094. NCC). If at all, there was a pledge of the paper or document
constituting the Torrens Title, as a movable by itself, but not of the land which the
title represents.

There is no mortgage because no deed or contract was executed in the manner


required by law for a mortgage (Arts. 2085 to 2092, NCC; 2124 to 2131, NCC).

There is no contract of antichresis because no right to the fruits of the property


was given to the creditor (Art. 2132 NCC).

A contract of simple loan was entered into with security arrangement agreed
upon by the parties which is not one of those mentioned above.

ALTERNATIVE ANSWER:
There is a contract of mortgage constituted over the land. There is no particular
form required for the validity of a mortgage of real property. It is not covered by the
statute of frauds in Art. 1403, NCC and even assuming that it is covered, the delivery
of the title to the creditor has taken it out of the coverage thereof. A contract of
mortgage of real property is consensual and is binding on the parties despite absence
of writing. However, third parties are not bound because of the absence of a
written instrument evidencing the mortgage and, therefore the absence of
registration. But this does not affect the validity of the mortgage between the parties
(Art. 2125, NCC), The creditor may compel the debtor to execute the mortgage in a
public document in order to allow its registration (Art. 1357.NCC in relation to Art.
1358. NCC).
Notes:
A contract of mortgage over a real property shall be in public instrument to be
valid. If it is not in writing, it cannot be registered. Thus, it cannot bind third party.

QUASI-CONTRACT
Quasi-Contract is a legal contract; it is independent of the agreement of the
parties; it is governed by law.

Negotiorum gestio is the management of business or affairs of another without


authority.
Quasi-Contracts; Negotiorium Gestio (1992)
In fear of reprisals from lawless elements besieging his barangay, X abandoned
his fishpond, fled to Manila and left for Europe. Seeing that the fish in the fishpond
were ready for harvest, Y, who is in the business of managing fishponds on a
commission basis, took possession of the property, harvested the fish and sold the
entire harvest to Z. Thereafter, Y borrowed money from W and used the money to
buy new supplies of fish fry and to prepare the fishpond for the next crop.

a) What is the Juridical relation between X and Y during X's absence?

b) Upon the return of X to the barangay, what are the obligations of Y to X


as regards the contract with Z?

c) Upon X's return, what are the obligations of X as regards Y's contract
with W? d) What legal effects will result if X expressly ratifies Y's management
and what would be the obligations of X in favor of Y? Explain all your answers.

SUGGESTED ANSWER:
(a) The juridical relation is that of the quasi-contract of "negotiorum gestio". Y
is the "gestor" or "officious manager" and X is the "owner" (Art. 2144, Civil Code).

(b) Y must render an account of his operations and deliver to X the price he
received for the sale of the harvested fish (Art, 2145, Civil Code).

(c) X must pay the loan obtained by Y from W because X must answer for
obligations contracted with third persons in the interest of the owner (Art. 2150,
Civil Code),

(d) Express ratification by X provides the effects of an express agency and X is


liable to pay the commissions habitually received by the gestor as manager (Art. 2149,
Civil Code).

Notes:
(a) The juridical relation between the gestor or the officious manager and the
owner of the property is governed by quasi-contract under the New Civil Code.
Negotiorum gestio is the management of business or affairs of another without
authority.
(b) A gestor is obliged to render an account of his operations and deliver to the
owner the proceeds from his operation.
(c) The owner must answer for obligations contracted with third persons for his
interest.

(d) When the owner ratifies the management of the gestor, the former and the
latter enter into a contract of agency.

Quasi-Contracts; Negotiorium Gestio (1993)


In September, 1972, upon declaration of martial rule in the Philippines. A,
together with his wife and children disappeared from his residence along A Mabini
Street, Ermita, Manila. B, his immediate neighbor, noticing that mysterious
disappearance of A and his family, closed the doors and windows of his house to
prevent it from being burglarized. Years passed without B hearing from A and his
family, B continued taking care of A's house, even causing minor repairs to be done at
his house to preserve it. In 1976, when business began to perk up in the area, an
enterprising man, C, approached B and proposed that they build stores at the ground
floor of the house and convert its second floor into a pension house. B agreed to Cs
proposal and together they spent for the construction of stores at the ground floor and
the conversion of the second floor into a pension house. While construction was going
on, fire occurred at a nearby house. The houses at the entire block, including A's were
burned. After the EDSA revolution in February 1986, A and his family returned from
the United States where they took refuge in 1972. Upon learning of what happened to
his house. A sued B for damages, B pleaded as a defense that he merely took charge
of his house under the principle of negotiorum gestio. He was not liable as the burning
of the house is a fortuitous event. Is B liable to A for damages under the foregoing
circumstances?

SUGGESTED ANSWER:
No. B is not liable for damages, because he is a gestor in negotiorum gestio (Art.
2144, Civil Code) Furthermore, B is not liable to A because Article 2147 of the Civil
Code is not applicable.

B did not undertake risky operations which the owner was not accustomed to
embark upon: a) he has not preferred his own interest to that of the owner;
b) he has not failed to return the property or business after demand by the
owner; and
c) he has not assumed the management in bad faith.
Notes:
Under the Civil Code, a gestor is liable for fortuitous event if he undertakes a
risky operation which the owner was not accustomed to it.

ARTICLE 2144. Whoever voluntarily takes charge of the property or business


of another without any authority from the latter is obliged to continue the management
until the affair is terminated; he may require the owner to substitute him if he is in a
position to do so.
This juridical relation does not arise in any of the following;
(1) When the property or business is not neglected or abandoned;
(2) If the manager is authorized by the owner.(Reengineered)

ARTICLE 2147. The officious manager shall be liable for any fortuitous
event:

(1) If he undertakes risky operations which the owner was not accustomed to
embark upon;
(2) If he has preferred his own interest to that of the owner;
(3) If he fails to return the property or business after demand by the owner;
(4) If he assumed the management in bad faith.

ALTERNATIVE ANSWER:
He would be liable under Art. 2147 (1) of the Civil Code, because he used the
property for an operation which the operator is not accustomed to, and in so doing, he
exposed the house to increased risk, namely the operation of a pension house on the
second floor and stores on the first floor

Quasi-Contracts; Negotiorium Gestio (1995)


Armando owns a row of residential apartments in San Juan, Metro Manila,
which he rents out to tenants. On 1 April 1991 he left for the United States without
appointing any administrator to manage his apartments such that uncollected rentals
accumulated for three (3) years. Amparo, a niece of Armando, concerned with the
interest of her uncle, took it upon herself to administer the property. As a
consequence, she incurred expenses in collecting the rents and in some instances even
spent for necessary repairs to preserve the property.
1. What Juridical relation between Amparo and Armando, if any, has
resulted from Amparo's unilateral act of assuming the administration of
Armando's apartments? Explain.
2. What rights and obligations, if any, does Amparo have under the
circumstances? Explain.

SUGGESTED ANSWER:
1. Negotiorum gestio existed between Amparo and Armando, She voluntarily
took charge of the agency or management of the business or property of her uncle
without any power from her uncle whose property was neglected. She is called the
gestor negotiorum or officious manager, (Art. 2144, NCC)
2. It is recommended by the Committee that an enumeration of any two (2)
obligations and two (2) rights as enumerated in Arts. 2145 to 2152, NCC, would
entitle the examinee to full credit.

Art. 2145. The officious manager shall perform his duties with all the diligence
of a good father of a family, and pay the damages which through his fault or
negligence may be suffered by the owner of the property or business under
management.

The courts may, however, increase or moderate the indemnity according to the
circumstances of each case.

Art. 2146. If the officious manager delegates to another person all or some of his
duties, he shall be liable for the acts of the delegate, without prejudice to the direct
obligation of the latter toward the owner of the business.

The responsibility of two or more officious managers shall be solidary, unless


management was assumed to save the thing or business from imminent danger.
Art. 2147. The officious manager shall be liable for any fortuitous event:

(1) If he undertakes risky operations which the owner was not accustomed to
embark upon;
(2) If he has preferred his own interest to that of the owner;
(3) If he fails to return the property or business after demand by the owner,
(4) If he assumed the management in bad faith.
Art. 2148. Except when the management was assumed to save the property or
business from imminent danger, the officious manager shall be liable for fortuitous
events:
(1) If he is manifestly unfit to carry on the management;
(2) If by his Intervention h e prevented a more competent person from taking up
the management.

Art. 2149. The ratification of the management by the owner of the business
produces the effects of an express agency, even if the business may not have been
successful.

Art. 2150, Although the officious management may not have been expressly
ratified, the owner of the property or business who enjoys the advantages of the same
shall be liable for obligations incurred in his interest, and shall reimburse the officious
manager for the necessary and useful expenses and for the damages which the latter
may have suffered in the performance of his duties.

The same obligation shall be incumbent upon him when the management had for
its purpose the prevention of an imminent and manifest loss, although no benefit may
have been derived.

Art. 2151. Even though the owner did not derive any benefit and there has been
no imminent and manifest danger to the property or business, the owner is liable as
under the first paragraph of the preceding article, provided:
(1) The officious manager has acted in good faith, and
(2) The property or business is intact, ready to be returned to the owner.

Art. 2152. The officious manager is personally liable for contracts which he has
entered into with third persons, even though he acted in the name of the owner, and
there shall be no right of action between the owner and third persons. These
provisions shall not apply: (1) If the owner has expressly or tacitly ratified the
management, or(2) When the contract refers to things pertaining to the owner of the
business,

(NOTE: It is recommended by the Committee that an enumeration of any two


(2) obligations and any two (2) rights as enumerated la Arts. 2145 to 2152, NCC
would entitle the examinee to full credit.)

Notes:
Under the Civil Code, the officious manager or the gestor shall observe
diligence of a good father of a family in the management of the property; he shall
not undertake a risky operation; he shall not engage in business in which the owner
of the property is not accustomed to.

Under the Civil Code, the owner of the property under management shall pay
the obligation incurred in his interest; he shall reimburse the officious manager for
the necessary and useful expenses; he shall pay damages which the officious manager
may have suffered in the performance of his duties.

Quasi-Contracts; Solutio Indebiti (2004)


DPO went to a store to buy a pack of cigarettes worth P225.00 only. He gave the
vendor, RRA, a P500-peso bill. The vendor gave him the pack plus P375.00 change.
Was there a discount, an oversight, or an error in the amount given? What
would be DPO’s duty, if any, in case of an excess in the amount of change given
by the vendor? How is this situational relationship between DPO and RRA
denominated? Explain. (5%)

SUGGESTED ANSWER:
There was error in the amount of change given by RRA. This is a case of solutio
indebiti in that DPO received something that is not due him. He has the obligation to
return the P100.00; otherwise, he will unjustly enrich himself at the expense of
RRA. (Art. 2154, Civil Code)

Notes:
Under the Civil Code, a person who receives something which he has no right
thereto shall return the same according to the principle of unjust enrichment.

ARTICLE 2154. If something is received when there is no right to demand it,


and it was unduly delivered through mistake, the obligation to return it arises.

ALTERNATIVE ANSWER:
DPO has the duty to return to RRA the excess P100 as trustee under Article
1456 of the Civil Code which provides: If property is acquired through mistake or
fraud, the person obtaining it is, by force of law, considered a trustee of an implied
trust for the benefit of the person from whom the property comes. There is, in this
case, an implied or constructive trust in favor of RRA.

Notes:
Under the Civil Code, a person becomes a trustee of the owner of the property
which he acquired by mistake or fraud.

ARTICLE 1456. If property is acquired through mistake or fraud, the person


obtaining it is, by force of law, considered a trustee of an implied trust for the benefit
of the person from whom the property comes.

TORTS & DAMAGES


A tort is a legal wrong which one person or entity which is a tortfeasor commits
against another person or entity and for which the usual remedy is an award of
damages.

Collapse of Structures; Last Clear Chance (1990)


Mr and Mrs R own a burned-out building, the firewall of which collapsed and
destroyed the shop occupied by the family of Mr and Mrs S, which resulted in injuries
to said couple and the death of their daughter. Mr and Mrs S had been warned by Mr
& Mrs R to vacate the shop in view of its proximity to the weakened wall but the
former failed to do so. Mr & Mrs S filed against Mr and Mrs R an action for recovery
of damages the former suffered as a result of the collapse of the firewall. In defense,
Mr and Mrs R rely on the doctrine of last clear chance alleging that Mr and Mrs S had
the last clear chance to avoid the accident if only they heeded the former’s warning to
vacate the shop, and therefore Mr and Mrs R’s prior negligence should be disregarded.
If you were the judge, how would you decide the case? State your reasons.

SUGGESTED ANSWER:
I would decide in favor of Mr & Mrs S. The proprietor of a building or structure
is responsible for the damages resulting from its total or partial collapse, if it should be
due to the lack of necessary repairs. (Art 2190 Civil Code)

As regards the defense of “last clear chance,” the same is not tenable because
according to the SC in one case (De Roy v CA L-80718, Jan 29, 1988, 157 S 757)
the doctrine of last clear chance is not applicable to instances covered by Art 2190 of
the Civil Code.
Further, in Phoenix Construction, Inc. v. Intermediate Appellate Court (G.R.
L-65295, March 10, 1987. 148 SCRA 353) the Supreme Court held that the role of the
common law "last clear chance" doctrine in relation to Article 2179 of the Civil Code
is merely to mitigate damages within the context of contributory negligence.

Notes:
In De Roy v CA, it was held that the defense of last clear chance cannot be
invoked by the proprietor of a building which collapsed and caused injury to others.

Under the Civil Code, a proprietor of a building shall be liable for damages due
to the collapse of a building if the collapse was because of lack of necessary repair.

ARTICLE 2190. The proprietor of a building or structure is responsible for the


damages resulting from its total or partial collapse, if it should be due to the lack of
necessary repairs.

Under the Civil Code, a plaintiff cannot recover damages if the injury he
sustained was the proximate cause of his own act. But he may recover damages if the
proximate cause of his injury was due to the lack of care of the defendant and his
negligence was only contributory.

ARTICLE 2179. When the plaintiff’s own negligence was the immediate and
proximate cause of his injury, he cannot recover damages. But if his negligence
was only contributory, the immediate and proximate cause of the injury being the
defendant’s lack of due care, the plaintiff may recover damages, but the courts shall
mitigate the damages to be awarded.

Damages (1994)

On January 5, 1992, Nonoy obtained a loan of Pl,000,000.00 from his friend


Raffy. The promissory note did not stipulate any payment for interest. The note was
due on January 5, 1993 but before this date the two became political enemies. Nonoy,
out of spite, deliberately defaulted in paying the note, thus forcing Raffy to sue him.

1) What actual damages can Raffy recover?


2) Can Raffy ask for moral damages from Nonoy?
3) Can Raffy ask for nominal damages?
4) Can Raffy ask for temperate damages?
5) Can Raffy ask for attorney's fees?
SUGGESTED ANSWER:
1) Raffy may recover the amount of the promissory note of P1 million, together
with interest at the legal rate from the date of judicial or extrajudicial demand. In
addition, however, inasmuch as the debtor is in bad faith, he is liable for all damages
which may be reasonably attributed to the non-performance of the obligation. (Art.
2201(2). NCC).

2) Yes, under Article 2220, NCC moral damages are recoverable in case of
breach of contract where the defendant acted fraudulently or in bad faith.

3) Nominal damages may not be recoverable in this case because Raffy may
already be indemnified of his losses with the award of actual and compensatory
damages. NOMINAL DAMAGES are adjudicated only in order that a right of the
plaintiff, which has been violated or invaded by the defendant may be vindicated or
recognized, and not for the purpose of indemnifying the plaintiff for any loss suffered
by him. (Article 2231. Civil Code)

4) Raffy may ask for, but would most likely not be awarded temperate
damages, for the reason that his actual damages may already be compensated upon
proof thereof with the promissory note. TEMPERATE DAMAGES may be awarded
only when the court finds that some pecuniary loss has been suffered but its amount
cannot, from the nature of the case, be proved with certainty. (Article 2224, Civil
Code)

5) Yes, under paragraph 2, Article 2208 of the Civil Code, considering that
Nonoy's act or omission has compelled Raffy to litigate to protect his interests.
Furthermore, attorneys' fees may be awarded by the court when it is just and
equitable. (Article 2208(110) Civil Code).

Notes:
1) In case of failure to pay, a creditor may recover the amount of the promissory
note and the interest at the legal rate from the date of judicial or extrajudicial
demand. In addition, however, inasmuch as the debtor is in bad faith, he is liable for
all damages which may be reasonably attributed to the non-performance of the
obligation.
2) Under the Civil Code, moral damages are recoverable in case of breach of
contract if the defendant acted fraudulently or in bad faith.
3) Under the Civil Code, nominal damages may no longer be recovered when
actual and compensatory damages are already awarded.

4) Under the Civil Code, Temperate damages may be awarded only when the
court finds that some pecuniary loss has been suffered but its amount cannot, from the
nature of the case, be proved with certainty.

5) Under the Civil Code, attorney’s fees may be awarded by the court when the
plaintiff was compelled to litigate to protect his interest.
Damages arising from Death of Unborn Child (1991)
On her third month of pregnancy, Rosemarie, married to Boy, for reasons
known only to her, and without informing Boy, went to the clinic of X, a known
abortionist, who for a fee, removed and expelled the fetus from her womb, Boy
learned of the abortion six (6) months later. Availing of that portion of Section 12 of
Article II of the1987 Constitution which reads; The State x xx shall equally protect the
life of the mother and the life of the unborn from conception, "xxx" which he claims
confers a civil personality on the unborn from the moment of conception. Boy filed a
case for damages against the abortionist, praying therein that the latter be ordered to
pay him: (a) P30,000.00 as indemnity for the death of the fetus, (b) P100.000.00 as
moral damages for the mental anguish and anxiety he suffered, (c) P50,000.00 as
exemplary damages, (d) P20,000.00 as nominal damages, and (e) P25,000.00 as
attorney's fees.

May actual damages be also recovered? If so, what facts should be alleged and
proved?

SUGGESTED ANSWER:
Yes, provided that the pecuniary loss suffered should be substantiated and duly
proved.

Damages arising from Death of Unborn Child (2003)


If a pregnant woman passenger of a bus were to suffer an abortion following a
vehicular accident due to the gross negligence of the bus driver, may she and her
husband claim damages from the bus company for the death of their unborn
child? Explain. 5%

SUGGESTED ANSWER:
No, the spouses cannot recover actual damages in the form of indemnity for the
loss of life of the unborn child. This is because the unborn child is not yet considered
a person and the law allows indemnity only for loss of life of person. The mother,
however may recover damages for the bodily injury she suffered from the loss of the
fetus which is considered part of her internal organ. The parents may also recover
damages for injuries that are inflicted directly upon them, e.g., moral damages for
mental anguish that attended the loss of the unborn child. Since there is gross
negligence, exemplary damages can also be recovered. (Gelus v. CA, 2 SCRA 801
[1961])

Notes:
In Gelus v. CA, the Supreme Court held that damages from loss of life may be
recovered only in case of a person. In this case, a fetus is not considered a person.
Therefore, damages may not be recovered with respect to the death of a fetus.

Further, it was held that damages for loss of life of a person is not allowed in
case of fetus extraction because fetus is not a person.

However, moral damages may be recovered due to anxiety, sleepless night,


among others.

Death Indemnity (1994)


Johnny Maton's conviction for homicide was affirmed by the Court of Appeals
and in addition, although the prosecution had not appealed at all, the appellate court
increased the indemnity for death from P30,000.00 to P50,000.00. On his appeal to
the Supreme Court, among the other things Johnny Maton brought to the high
court's attention, was the increase of indemnity imposed by the Court of Appeals
despite the clear fact that the People had not appealed from the appellate court's
judgment. Is Johnny Maton correct?

SUGGESTED ANSWER:
a) In Abejam v. Court of Appeals, the Supreme Court said that even if the
issue of damages were not raised by the appellant in the Court of Appeals but the
Court of Appeals in its findings increased the damages, the Supreme Court will not
disturb the findings of the Court of Appeals.

b) No, the contention of the accused is not correct because upon appeal to the
Appellate Court, the court acquired jurisdiction over the entire case, criminal as well
as civil. Since the conviction of homicide had been appealed, there is no finality in the
amount of indemnity because the civil liability arising from the crime and the
judgment on the crime has not yet become final.
c) Yes. Since the civil indemnity is an award in the civil action arising from the
criminal offense, the rule that a party cannot be granted affirmative relief unless he
himself has appealed should apply. Therefore, it was error for the Court of Appeals to
have expanded the indemnity since the judgment on the civil liability had become
final.

d) No. Courts can review matters not assigned as errors. (Hydro Resource
vs. CA . 204 SCRA 309).

Notes:
In Abejam v. Court of Appeals, the Supreme Court held that the finding of the
Court of Appeals to increase the indemnity for the death of the victim will not be
disturbed by it although the matter to increase the indemnity was not assigned as error
in the Court of Appeals because it has the jurisdiction to review the entire case.

Defense; Due Diligence in Selection (2003)


As a result of a collision between the taxicab owned by A and another taxicab
owned by B, X, a passenger of the first taxicab, was seriously injured. X later filed a
criminal action against both drivers.

May both taxicab owners raise the defense of due diligence in the selection
and supervision of their drivers to be absolved from liability for damages to X?
Reason. 5%
SUGGESTED ANSWER:
It depends. If the civil action is based on a quasi-delict the taxicab owners may
raise the defense of diligence of a good father of a family in the selection and
supervision of the driver; if the action against them is based on culpa contractual or
civil liability arising from a crime, they cannot raise the defense.

Notes:
Under the Civil Code, if the action is based on breach of contract, the owner of
the taxicab involved in a collision cannot avail of the defense of due diligence in the
selection and supervision of his employee.
However, this defense is available in an action based on quasi-delict.

Filing of Separate Civil Action; Need for Reservation (2003)


As a result of a collision between the taxicab owned by A and another taxicab
owned by B, X, a passenger of the first taxicab, was seriously injured. X later filed a
criminal action against both drivers.

Is it necessary for X to reserve his right to institute a civil action for


damages against both taxicab owners before he can file a civil action for damages
against them? Why

SUGGESTED ANSWER:
It depends. If the separate civil action is to recover damages arising from the
criminal act, reservation is necessary. If the civil action against the taxicab owners is
based on culpa contractual, or on quasi-delict, there is no need for reservation.

Notes:
Under the Civil Code, reservation to file a civil action is not necessary when the
case is covered by an independent civil action like in the case of quasi-delict and
breach of contract.

Moreover, only civil action arises from criminal aspect needs reservation.

ALTERNATIVE ANSWER:
No, such reservation is not necessary. Under Section 1 of Rule 111 of the 2000
Rules on Criminal Procedure, what is “deemed instituted” with the criminal action is
only the action to recover civil liability arising from the crime or ex delicto. All the
other civil actions under Articles 32, 33, 34 and 2176 of the New Civil Code are no
longer “deemed instituted”, and may be filed separately and prosecuted independently
even without any reservation in the criminal action (Section 3, Rule 111, Ibid). The
failure to make a reservation in the criminal action is not a waiver of the right to file a
separate and independent civil action based on these articles of the New Civil Code
(Casupanan v. Laroya GR No. 145391, August 26, 2002).

Notes:
Under the Rules on Criminal Procedure, what is deemed instituted with the
criminal action is the civil action arising from the crime. Thus, reservation is
necessary in this case.

In Casupanan v. Laroya, it was held that a civil action is deemed instituted only
when it arises from a crime. All other civil actions based on quasi-delict need not be
reserved.
Fortuitous Event; Mechanical Defects (2002)
A van owned by Orlando and driven by Diego, while negotiating a downhill
slope of a city road, suddenly gained speed, obviously beyond the authorized limit in
the area, and bumped a car in front of it, causing severed damage to the car and
serious injuries to its passengers. Orlando was not in the car at the time of the incident.
The car owner and the injured passengers sued Orlando and Diego for damages
caused by Diego’s negligence. In their defense, Diego claims that the downhill slope
caused the van to gain speed and that, as he stepped on the brakes to check the
acceleration, the brakes locked, causing the van to go even faster and eventually to
hit the car in front of it. Orlando and Diego contend that the sudden malfunction
of the van’s brake system is a fortuitous even and that, therefore, they are
exempt from any liability. Is this contention tenable? Explain. (2%)

SUGGESTED ANSWER:
No. Mechanical defects of a motor vehicle do not constitute fortuitous event,
since the presence of such defects would have been readily detected by diligent
maintenance check. The failure to maintain the vehicle in safe running condition
constitutes negligence.

Notes:
Malfunction of the brake of a motor vehicle is not a fortuitous event. Thus, the,
driver can be held liable for injury caused by the malfunction of the brake.

Liability; Airline Company; Non-Performance of an Obligation (2004)


DT and MT were prominent members of the frequent travelers’ club of FX
Airlines. In Hongkong, the couple were assigned seats in Business Class for which
they had bought tickets. On checking in, however, they were told they were upgraded
by computer to First Class for the flight to Manila because the Business Section was
overbooked. Both refused to transfer despite better seats, food, beverage and other
services in First Class. They said they had guests in Business Class they should attend
to. They felt humiliated, embarrassed and vexed, however, when the stewardess
allegedly threatened to offload them if they did not avail of the upgrade. Thus they
gave in, but during the transfer of luggage DT suffered pain in his arm and wrist. After
arrival in Manila, they demanded an apology from FX’s management as well as
indemnity payment. When none was forthcoming, they sued the airline for a million
pesos in damages. Is the airline liable for actual and moral damages? Why or
why not? Explain briefly. (5%)
SUGGESTED ANSWER:
FX Airlines committed breach of contract when it upgraded DT and MT, over
their objections, to First Class because they had contracted for Business Class
passage. However, although there is a breach of contract, DT and MT are entitled to
actual damages only for such pecuniary losses suffered by them as a result of such
breach. There seems to be no showing that they incurred such pecuniary loss. There is
no showing that the pain in DT's arm and wrist resulted directly from the carrier's acts
complained of. Hence, they are not entitled to actual damages. Moreover, DT could
have avoided the alleged injury by requesting the airline staff to do the luggage
transfer as a matter of duty on their part. There is also no basis to award moral
damages for such breach of contract because the facts of the problem do not show
bad faith or fraud on the part of the airline. (Cathay Pacific v. Vazquez, 399 SCRA
207 [2003]). However, they may recover moral damages if the cause of action is
based on Article 21 of the Civil Code for the humiliation and embarrassment they felt
when the stewardess threatened to offload them if they did not avail of the upgrade.

Notes:
In the case of Cathay Pacific v. Vazquez, the Supreme Court held that airlines
company committed breach of contract when it upgraded it passengers, over their
objection, to first class because they had contracted for business class passage.
However, Actual damages may be awarded only when they suffered pecuniary loss as
the result of the upgrading.

ALTERNATIVE ANSWER:
If it can be proved that DT's pain in his arm and wrist occasioned by the
transfer of luggage was caused by fault or negligence on the part of the airline's
stewardess, actual damages may be recovered.

The airline may be liable for moral damages pursuant to Art. 2219 (10) if the
cause of action is based on Article 21 or an act contrary to morals in view of the
humiliation suffered by DT and MT when they were separated from their guests and
were threatened to be offloaded.

Liability; Airline Company; Non-Performance of an Obligation (2005)


Dr. and Mrs. Almeda are prominent citizens of the country and are frequent
travelers abroad. In 1996, they booked round-trip business class tickets for the Manila-
Hong Kong-Manila route of the Pinoy Airlines, where they are holders of Gold
Mabalos Class Frequent Flier cards. On their return flight, Pinoy Airlines upgraded
their tickets to first class without their consent and, inspite of their protestations to be
allowed to remain in the business class so that they could be with their friends, they
were told that the business class was already fully booked, and that they were given
priority in upgrading because they are elite members/holders of Gold Mabalos Class
cards. Since they were embarrassed at the discussions with the flight attendants, they
were forced to take the flight at the first class section apart from their friends who
were in the business class. Upon their return to Manila, they demanded a written
apology from Pinoy Airlines. When it went unheeded, the couple sued Pinoy Airlines
for breach of contract claiming moral and exemplary damages, as well as
attorney's fees. Will the action prosper? Give reasons. (5%)

ALTERNATIVE ANSWER:
Yes, the action will prosper. Article 2201 of the Civil Code entitles the person to
recover damages which may be attributed to non-performance of an obligation. In
Alitalia Airways v. Court of Appeals (G.R. No. 77011, July 24, 1990), when an
airline issues ticket to a passenger confirmed on a particular flight, a contract of
carriage arises and the passenger expects that he would fly on that day. When the
airline deliberately overbooked, it took the risk of having to deprive some passengers
of their seat in case all of them would show up. For the indignity and inconvenience of
being refused the confirmed seat, said passenger is entitled to moral damages.

In the given problem, spouses Almeda had a booked roundtrip business class
ticket with Pinoy Airlines. When their tickets were upgraded to first class without
their consent, Pinoy Airlines breached the contract. As ruled in Zulueta v. Pan
American (G.R. No. L-28589, January 8, 1973), in case of overbooking, airline is in
bad faith. Therefore, spouses Almeda are entitled to damages.

Notes:
In Zulueta v. Pan American, it was held that in case of overbooking, the airline
company is acting in bad faith. Therefore, damages may be awarded because of
overbooking.

ALTERNATIVE ANSWER:
The action may or may not prosper. Moral damages include physical
suffering, mental anguish, fright, serious anxiety, besmirched reputation,
wounded feelings, moral shock, social humiliation, and similar injury. Although
incapable of pecuniary computation, moral damages may be recovered if they are the
proximate result of the defendant's wrongful act or omission. Moral damages
predicated upon a breach of contract of carriage are recoverable only in instances
where the carrier is guilty of fraud or bad faith or where the mishap resulted in the
death of a passenger. (Cathay Pacific Airways, Ltd. v. Court of Appeals, G.R. No.
60501, March 5, 1993) Where there is no showing that the airline acted fraudulently
or in bad faith, liability for damages is limited to the natural and probable
consequences of the breach of the contract of carriage which the parties had foreseen
or could have reasonably foreseen. In such a case the liability does not include moral
and exemplary damages.

In the instant case, if the involuntary upgrading of the Almedas' seat


accommodation was not attended by fraud or bad faith, the award of moral
damages has no leg to stand on.

Thus, spouses would not also be entitled to exemplary damages. It is a requisite


in the grant of exemplary damages that the act of the offender must be accompanied
by bad faith or done in wanton, fraudulent or malevolent manner.

(Morris v. Court of Appeals, G.R. No. 127957, February 21, 2001)


Moreover, to be entitled thereto, the claimant must first establish his right to moral,
temperate, or compensatory damages. (Art. 2234, Civil Code) Since the Almedas are
not entitled to any of these damages, the award for exemplary damages has no legal
basis. Where the awards for moral and exemplary damages are eliminated, so must the
award for attorney's fees be eliminated. (Orosa v. Court of Appeals, G.R. No.
111080, April 5, 2000; Morris v. Court of Appeals, G.R. No. 127957, February
21, 2001) The most that can be adjudged in their favor for Pinoy Airlines' breach of
contract is an award for nominal damages under Article 2221 of the Civil Code.
(Cathay Pacific Airways v. Sps. Daniel & Maria Luisa Vasquez, G.R. No. 150843,
March 14, 2003)
However, if spouses Almeda could prove that there was bad faith on the
part of Pinoy Airlines when it breached the contract of carriage, it could be liable
for moral, exemplary as well as attorney's fees.

Liability; Employer; Damage caused by Employees (1997)

a) When would an employer's liability for damage, caused by an employee in


the performance of his assigned tasks, be primary and when would it be subsidiary
in nature?
b) Would the defense of due diligence in the selection and supervision of the
employee be available to the employer in both instances?
SUGGESTED ANSWER:
(a) The employer's liability for damage based on culpa aquiliana under Art,
2176 and 2180 of the Civil Code is primary; while that under Art. 103 of the Revised
Penal Code is subsidiary.

(b) The defense of diligence in the selection and supervision of the employee
under Article 2180 of the Civil Code is available only to those primarily liable
thereunder, but not to those subsidiarily liable under Article 103 of the Revised Penal
Code (Yumul vs. Juliano, 72 Phil. 94).

Notes:
The liability of the employer under quasi-delict is primary, while his liability
under Article 103 of the Revised Penal Code is subsidiary.

LIABILITY OF THE OWNER WHO WAS IN THE VEHICLE AT THE TIME


OF THE MISHAP

Liability; owner who was in the vehicle (1996)


Marcial, who does not know how to drive, has always been driven by Ben, his
driver of ten years whom he had chosen carefully and has never figured in a vehicular
mishap. One day, Marcial was riding at the back seat of his Mercedes Benz being
driven along EDSA by Ben. Absorbed in reading a book, Marcial did not notice that
they were approaching the corner of Quezon Avenue, when the traffic light had just
turned yellow. Ben suddenly stepped on the gas to cross the intersection before the
traffic light could turn red. But, too late. Midway in the intersection, the traffic light
changed, and a Jeepney full of passengers suddenly crossed the car's path. A collision
between the two vehicles was inevitable. As a result, several jeepney passengers were
seriously injured. A suit for damages based on culpa aquiliana was filed against
Marcial and Ben, seeking to hold them jointly and severally liable for such injuries.
May Marcial be held liable? Explain.

SUGGESTED ANSWER:
Marcial may not be liable because under Art. 2184, NCC, the owner who is in
the vehicle is not liable with the driver if by the exercise of due diligence he could
have prevented the injury. The law does not require the owner to supervise the driver
every minute that he was driving. Only when through his negligence, the owner has
lost an opportunity to prevent the accident would he be liable (Caedo v. Ytt Khe
Thai, 26 SCRA 410 citing Chapman v. Underwood and Manlangit v. Mauler, 250
SCRA 560). In this case, the fact that the owner was absorbed in reading a book does
not conclusively show that he lost the opportunity to prevent the accident through his
negligence.

Notes:
Under the Civil Code, in a motor vehicle accident, the owner is solidarily liable
with his driver if he was in the vehicle and could have prevented the misfortune by the
use of due diligence. In this case, the owner was reading a book when the mishap
transpired. Therefore, he could be held solidarily liable with his driver because he was
reading a book when the incident occurred and he could have prevented the accident.

In Manlangit v. Mauler, it was held that the owner who was absorbed in reading
a book does not make him liable because of the negligence of his driver. The facts
show that the owner, by the exercise of due diligence, is not expected to prevent the
mishap.

ARTICLE 2184. In motor vehicle mishaps, the owner is solidarily liable with
his driver if the former was in the vehicle and could have prevented the misfortune by
the use of the due diligence. It is disputably presumed that a driver was negligent, if he
had been found guilty of reckless driving or violating traffic regulations at least twice
within the next preceding two months.

If the owner was not in the motor vehicle, the provisions of Article 2180 are
applicable.
ALTERNATIVE ANSWER:
Yes, Marcial should be held liable. Art. 2164. NCC makes an owner of a motor
vehicle solidarily liable with the driver if, being in the vehicle at the time of the
mishap, he could have prevented it by the exercise of due diligence. The traffic
conditions along EDSA at any time of day or night are such as to require the
observance of utmost care and total alertness in view of the large number of
vehicles running at great speed. Marcial was negligent in that he rendered himself
oblivious to the traffic hazards by reading a book instead of focusing his attention on
the road and supervising the manner in which his car was being driven. Thus he failed
to prevent his driver from attempting to beat the traffic light at the junction of Quezon
Avenue and EDSA, which Marcial, without being a driver himself could have easily
perceived as a reckless course of conduct.
Liability; owner who was in the vehicle (1998)
A Gallant driven by John and owned by Art, and a Corolla driven by its owner,
Gina, collided somewhere along Adriatico Street. As a result of the accident, Gina
had a concussion. Subsequently, Gina brought an action for damages against John and
Art. There is no doubt that the collision is due to John's negligence. Can Art, who
was in the vehicle at the time of the accident, be held solidarily liable with his
driver, John? (5%)

SUGGESTED ANSWER:
Yes. Art may be held solidary liable with John, if it was proven that the former
could have prevented the misfortune with the use of due diligence. Article 2184 of the
Civil Code states: "In motor mishaps, the owner is solidary liable with his driver, if
the former, who was in the vehicle, could have, by the use of due diligence, prevented
the misfortune, x x x"

ALTERNATIVE ANSWER:
1. It depends. The Supreme Court in Chapman vs, Underwood (27 Phil 374),
held: "An owner who sits in his automobile, or other vehicle, and permits his driver to
continue in a violation of law by the performance of negligent acts, after he has had a
reasonable opportunity to observe them and to direct that the driver cease therefrom,
becomes himself responsible for such acts, x x x On the other hand, if the driver, by a
sudden act of negligence, and without the owner having a reasonable opportunity to
prevent the act or its continuance, injures a person or violates the criminal law, the
owner of the automobile, although present therein at the time the act was committed is
not responsible, either civilly or criminally. Therefor, the act complained of must be
continued in the presence of the owner for such a length of time that the owner, by his
acquiescence, makes his driver's act his own."

Notes:
Under the Civil Code, an owner of a vehicle is solidarily liable with his driver if
he is in the vehicle at the time of the mishap and could have prevented the incident by
the use of proper diligence.

Liability; owner who was in the vehicle (2002)


Does the presence of the owner inside the vehicle causing damage to a third
party affect his liability for his driver’s negligence? Explain (2%)
SUGGESTED ANSWER:
In motor vehicle mishaps, the owner is made solidarily liable with his driver if
he (the owner) was in the vehicle and could have, by the use of due diligence,
prevented the mishap. (Caedo v. Yu Khe Thai, 26 SCRA 410 [1968]).

Notes:
Under the Civil Code, an owner of a vehicle is solidarily liable with his driver if
he is in the vehicle at the time of the mishap and could have prevented the incident by
the use of proper diligence.

Moral Damages & Atty Fees (2002)


Ortillo contracts Fabricato, Inc. to supply and install tile materials in a building
he is donating to his province. Ortillo pays 50% of the contract price as per agreement.
It is also agreed that the balance would be payable periodically after every 10%
performance until completed. After performing about 93% of the contract, for which it
has been paid an additional 40% as per agreement, Fabricato, Inc. did not complete
the project due to its sudden cessation of operations. Instead, Fabricato, Inc. demands
payment of the last 10% of the contract despite its non-completion of the project.
Ortillo refuses to pay, invoking the stipulation that payment of the last amount 10%
shall be upon completion. Fabricato, Inc. brings suit for the entire 10% plus damages.
Ortillo counters with claims for (a) moral damages for Fabricato, Inc.’s
unfounded suit which has damaged his reputation as a philanthropist and respect
businessman in his community, and (b) attorney’s fees.

A. Does Ortillo have a legal basis for his claim for moral damages? (2%)
B. How about his claim for attorney’s fees, having hired a lawyer to defend
him? (3%)

SUGGESTED ANSWER:
A. There is no legal basis to Ortillo’s claim for moral damages. It does not fall
under the coverage of Article 2219 of the New Civil Code.

B. Ortillo is entitled to attorney’s fees because Fabricato’s complaint is a case of


malicious prosecution or a clearly unfounded civil action. (Art. 2208 [4] and [11],
NCC).

Notes:
A. Under the Civil Code, moral damages may be awarded if the plaintiff suffers
wounded feeling, sleepless night, among others. In this case, the fact did not state that
the plaintiff suffered any of the conditions which allows him to recover moral
damages. Therefore, Ortillo’s claim for moral damages is without merit.
B. Under the Civil Code, attorney’s fees may be awarded in case of malicious
prosecution or clearly unfounded civil action.

ARTICLE 2208. In the absence of stipulation, attorney’s fees and expenses of


litigation, other than judicial costs, cannot be recovered, except:

(1) When exemplary damages are awarded;


(2) When the defendant’s act or omission has compelled the plaintiff to litigate
with third persons or to incur expenses to protect his interest;
(3) In criminal cases of malicious prosecution against the plaintiff;
(4) In case of a clearly unfounded civil action or proceeding against the
plaintiff;
(5) Where the defendant acted in gross and evident bad faith in refusing to
satisfy the plaintiff’s plainly valid, just and demandable claim;
(6) In actions for legal support;
(7) In actions for the recovery of wages of household helpers, laborers and
skilled workers;
(8) In actions for indemnity under workmen’s compensation and employer’s
liability laws;
(9) In a separate civil action to recover civil liability arising from a crime;
(10) When at least double judicial costs are awarded;
(11) In any other case where the court deems it just and equitable that attorney’s
fees and expenses of litigation should be recovered.

In all cases, the attorney’s fees and expenses of litigation must be reasonable.

ARTICLE 2219. Moral damages may be recovered in the following and


analogous cases:

(1) A criminal offense resulting in physical injuries;


(2) Quasi-delicts causing physical injuries;
(3) Seduction, abduction, rape, or other lascivious acts;
(4) Adultery or concubinage;
(5) Illegal or arbitrary detention or arrest;
(6) Illegal search;
(7) Libel, slander or any other form of defamation;
(8) Malicious prosecution;
(9) Acts mentioned in article 309;
(10) Acts and actions referred to in articles 21, 26, 27, 28, 29, 30, 32, 34, and 35.

The parents of the female seduced, abducted, raped, or abused, referred to in No.
3 of this article, may also recover moral damages.

The spouse, descendants, ascendants, and brothers and sisters may bring the
action mentioned in No. 9 of this article, in the order named.

Moral Damages; Non-Recovery Thereof (2006)


Under Article 2219 of the Civil Code, moral damages may be recovered in the
cases specified therein several of which are enumerated below. Choose the case
wherein you cannot recover moral damages. Explain. (2.5%)

a) A criminal offense resulting in physical injuries


b) Quasi-delicts causing physical injuries
c) Immorality or dishonesty
d) Illegal search
e) Malicious prosecution

SUGGESTED ANSWER: Immorality and dishonesty, per se, are not among
those cases enumerated in Article 2219 which can be the basis of an action for moral
damages. The law specifically mentions adultery or concubinage, etc. but not any and
every immoral act.

Notes:
ARTICLE 2219. Moral damages may be recovered in the following and
analogous cases:

(1) A criminal offense resulting in physical injuries;


(2) Quasi-delicts causing physical injuries;
(3) Seduction, abduction, rape, or other lascivious acts;
(4) Adultery or concubinage;
(5) Illegal or arbitrary detention or arrest;
(6) Illegal search;
(7) Libel, slander or any other form of defamation;
(8) Malicious prosecution;
(9) Acts mentioned in article 309;
(10) Acts and actions referred to in articles 21, 26, 27, 28, 29, 30, 32, 34, and 35.

The parents of the female seduced, abducted, raped, or abused, referred to in No.
3 of this article, may also recover moral damages.

The spouse, descendants, ascendants, and brothers and sisters may bring the
action mentioned in No. 9 of this article, in the order named.

Quasi-Delict (1992)
As the result of a collision between a public service passenger bus and a cargo
truck owned by D, X sustained physical injuries and Y died. Both X and Y were
passengers of the bus. Both drivers were at fault, and so X and Z, the only heir and
legitimate child of the deceased Y, sued the owners of both vehicles.
a) May the owner of the bus raise the defense of having exercised the
diligence of a good father of a family?
b) May D raise the same defense?
c) May X claim moral damages from both defendants?
d) May Z claim moral damages from both defendants? Give reasons for all
your answers

SUGGESTED ANSWER:
(a) No. The owner of the bus cannot raise the defense because the carrier's
liability is based on breach of contract
(b) Yes. D can raise the defense because his liability is based on a quasi-delict.
(c) Because X suffered physical injuries, X can claim moral damages against
D, but as against the owner of the bus. X can claim moral damages only if X proves
reckless negligence of the carrier amounting to fraud.
(d) Z can claim moral damages against both defendants because the rules on
damages arising from death due to a quasi-delict are also applicable to death of a
passenger caused by breach of contract by a common carrier (Arts. 1755. 1756,
1764, 2206 and 2219. Civil Code).

Notes:
(a) Under the Civil Code, when the liability is based on breach of contract, an
owner of the common carrier cannot raise the defense of due diligence in the
selection and supervision of his driver.
(b) However, under the Civil Code, when the liability of an owner of a common
carrier is based upon quasi-delict, he can raise the defense of due diligence in the
selection and supervision of his driver.
(c) Under the Civil Code, moral damages may be recovered from breach of
contract when the act of reckless negligence of the owner is amounting to fraud.

(d) Under the Civil Code, moral damage is recoverable in case of death
resulting from breach of contract; the rule on death resulting from quasi-delict is
applicable in this case.

Quasi-Delict (2005)
Under the law on quasi-delict, aside from the persons who caused injury to
persons, who else are liable under the following circumstances:
a) When a 7-year old boy injures his playmate while playing with his
father's rifle. Explain. (2%)

SUGGESTED ANSWER:
The parents of the 7-year old boy who caused injury to his playmate are liable
under Article 219 of the Family Code, in relation to Article 2180 of the Civil Code
since they exercise parental authority over the person of the boy. (Tamargo v. Court of
Appeals, G.R. No. 85044, June 3, 1992; Elcano v. Hill, G.R. No. L-24803, May 26,
1977)

Notes:
In Tamargo v. Court of Appeals, it was held that the parents of the 7-year old
boy who caused injury to his playmate are liable since they exercise parental authority
over the person of the boy.

Art. 218. The school, its administrators and teachers, or the individual, entity or
institution engaged in child are shall have special parental authority and responsibility
over the minor child while under their supervision, instruction or custody.

Authority and responsibility shall apply to all authorized activities whether


inside or outside the premises of the school, entity or institution. (349a)

Art. 219. Those given the authority and responsibility under the preceding
Article shall be principally and solidarily liable for damages caused by the acts or
omissions of the unemancipated minor. The parents, judicial guardians or the persons
exercising substitute parental authority over said minor shall be subsidiarily liable.

The respective liabilities of those referred to in the preceding paragraph shall not
apply if it is proved that they exercised the proper diligence required under the
particular circumstances.

All other cases not covered by this and the preceding articles shall be governed
by the provisions of the Civil Code on quasi-delicts.

ARTICLE 2176. Whoever by act or omission causes damage to another,


there being fault or negligence, is obliged to pay for the damage done. Such fault
or negligence, if there is no pre-existing contractual relation between the parties, is
called a quasi-delict and is governed by the provisions of this Chapter.

Under the Civil Code, whoever causes damage to another by act or by omission
through fault or negligence is obliged to pay for the damage done.

ARTICLE 2180. The obligation imposed by article 2176 is demandable not


only for one’s own acts or omissions, but also for those of persons for whom one is
responsible.

The father and, in case of his death or incapacity, the mother, are responsible
for the damages caused by the minor children who live in their company.

Guardians are liable for damages caused by the minors or incapacitated persons
who are under their authority and live in their company.

The owners and managers of an establishment or enterprise are likewise


responsible for damages caused by their employees in the service of the branches in
which the latter are employed or on the occasion of their functions.

Employers shall be liable for the damages caused by their employees and
household helpers acting within the scope of their assigned tasks, even though the
former are not engaged in any business or industry.

The State is responsible in like manner when it acts through a special agent; but
not when the damage has been caused by the official to whom the task done properly
pertains, in which case what is provided in article 2176 shall be applicable.
Lastly, teachers or heads of establishments of arts and trades shall be liable for
damages caused by their pupils and students or apprentices, so long as they remain
in their custody.

The responsibility treated of in this article shall cease when the persons herein
mentioned prove that they observed all the diligence of a good father of a family to
prevent damage.

b) When a domestic helper, while haggling for a lower price with a fish
vendor in the course of buying foodstuffs for her employer's family, slaps the fish
vendor, causing her to fall and sustain injuries. Explain. (2%)

SUGGESTED ANSWER:
Employer of the domestic helper who slapped a fish vendor. Under Article 2180,
par. 5 of the Civil Code, "employers shall be liable for the damages caused by their
employees and household helpers acting within the scope of their assigned tasks, even
though the former are not engaged in any business or industry."

Notes:
Under the Civil Code, employers are liable for the damage caused by their
household helpers acting within the scope of their assigned task.

c) A carpenter in a construction company accidentally hits the right foot of


his co-worker with a hammer. Explain. (2%)

SUGGESTED ANSWER:
The owner of the construction company. Article 2180, paragraph 4 states that
"the owners and managers of an establishment or enterprise are likewise responsible
for damages caused by their employees in the service of the branches in which the
latter are employed or on the occasion of their functions."

Notes:
Under the Civil Code, owner of an enterprise shall be held liable for the damage
caused by his employees on the occasion of his function.
d) A 15-year old high school student stabs his classmate who is his rival for
a girl while they were going out of the classroom after their last class. Explain.
(2%)

SUGGESTED ANSWER:
The school, teacher and administrator as they exercise special parental
authority. (Art. 2180, par. 7 in relation to Art. 218 and Art. 219 of the Family Code)

Notes:
The school, teacher and administrator are liable for the damage done by their
students while the students are in their custody.

e) What defense, if any, is available to them? (2%)

SUGGESTED ANSWER:
The defense that might be available to them is the observance of a good father of
the family to prevent the damage. (Last par., Art. 2180, Civil Code)

Notes:
Under the Civil Code, they can raise the defense that they observe diligence of a
good father of a family to prevent the damage.

Quasi-Delict; Acts contrary to morals (1996)


Rosa was leasing an apartment in the city. Because of the Rent Control Law, her
landlord could not increase the rental as much as he wanted to, nor terminate her
lease as long as she was paying her rent. In order to force her to leave the premises,
the landlord stopped making repairs on the apartment, and caused the water and
electricity services to be disconnected. The difficulty of living without electricity and
running water resulted in Rosa's suffering a nervous breakdown. She sued the
landlord for actual and moral damages. Will the action prosper? Explain.

SUGGESTED ANSWER:
Yes, based on quasi-delict under the human relations provisions of the New
Civil Code (Articles 19, 20 and 21) because the act committed by the lessor is
contrary to morals. Moral damages are recoverable under Article 2219 (10) in relation
to Article 21. Although the action is based on quasi-delict and not on contract, actual
damages may be recovered if the lessee is able to prove the losses and expenses she
suffered.
Notes:
Under the Civil Code, any person who causes damage, loss or injury to another
in a manner that is contrary to morals, good customs, or public policy shall pay for the
damage done.

ARTICLE 19. Every person must, in the exercise of his rights and in the
performance of his duties, act with justice, give everyone his due, and observe honesty
and good faith.

ARTICLE 20. Every person who, contrary to law, wilfully or negligently causes
damage to another, shall indemnify the latter for the same.

ARTICLE 21. Any person who wilfully causes loss or injury to another in a
manner that is contrary to morals, good customs or public policy shall compensate
the latter for the damage.

ALTERNATIVE ANSWERS:
a) Yes, based on breach of contract. The lessor has the obligation to undertake
repairs to make the apartment habitable and to maintain the lessee in the peaceful and
adequate enjoyment of the lease for the entire duration of the contract (Article 1654.
NCC). Since there was willful breach of contract by the lessor, the lessee is entitled to
moral damages under Article 2220, NCC. She is also entitled to actual damages, e. g.
loss of income, medical expenses, etc., which she can prove at the trial.

b) Yes, based on contract and/or on tort. The lessor willfully breached his
obligations under Article 1654. NCC, hence, he is liable for breach of contract. For
such breach, the lessee may recover moral damages under Art. 2220 of the NCC, and
actual damages that she may have suffered on account thereof. And since the conduct
of the lessor was contrary to morals, he may also be held liable for quasi-delict. The
lessee may recover moral damages under Article 2219 (10) in relation to Article 21,
and all actual damages which she may have suffered by reason of such conduct under
Articles 9, 20 and 21.

c) Yes, the action should prosper for both actual and moral damages. In fact,
even exemplary damages and attorney's fees can be claimed by Rosa, on the authority
of Magbanua vs. IAC (137 SCRA 328), considering that, as given, the lessor's
willful and illegal act of disconnecting the water and electric services resulted in
Rosa's suffering a nervous breakdown. Art. 20 NCC and Art. 21, NCC authorize the
award of damages for such willful and illegal conduct.
Notes:
Under the Civil Code, when the breach of contract is done with fraud or in
bad faith, the defendant shall be liable for moral damages.

Under the Civil Code, a lessor is obliged to make necessary repair of the thing
leased to keep it suitable for the use of the lessee, so if the lessor fails to comply with
the law, he is liable for moral damages under Article 21 of the Civil Code because
his failure to make necessary repair is contrary to law.

ARTICLE 1654. The lessor is obliged:

(1) To deliver the thing which is the object of the contract in such a condition as
to render it fit for the use intended;
(2) To make on the same during the lease all the necessary repairs in order to
keep it suitable for the use to which it has been devoted, unless there is a stipulation
to the contrary;
(3) To maintain the lessee in the peaceful and adequate enjoyment of the lease
for the entire duration of the contract.

ARTICLE 2220. Willful injury to property may be a legal ground for awarding
moral damages if the court should find that, under the circumstances, such damages
are justly due. The same rule applies to breaches of contract where the defendant
acted fraudulently or in bad faith.

ARTICLE 2219. Moral damages may be recovered in the following and


analogous cases:
(1) A criminal offense resulting in physical injuries;
(2) Quasi-delicts causing physical injuries;
(3) Seduction, abduction, rape, or other lascivious acts;
(4) Adultery or concubinage;
(5) Illegal or arbitrary detention or arrest;
(6) Illegal search;
(7) Libel, slander or any other form of defamation;
(8) Malicious prosecution;
(9) Acts mentioned in article 309;
(10) Acts and actions referred to in articles 21, 26, 27, 28, 29, 30, 32, 34, and 35.
The parents of the female seduced, abducted, raped, or abused, referred to in
No. 3 of this article, may also recover moral damages.

The spouse, descendants, ascendants, and brothers and sisters may bring the
action mentioned in No. 9 of this article, in the order named.

Notes:
Art. 309. Any person who shows disrespect to the dead, or wrongfully interferes
with a funeral shall be liable to the family of the deceased for moral or material
damages. (Reengineered)

Quasi-Delict; Mismanagement of Depositor’s Account (2006)


Tony bought a Ford Expedition from a car dealer in Muntinlupa City. As
payment, Tony issued a check drawn against his current account with Premium Bank.
Since he has a good reputation, the car dealer allowed him to immediately drive home
the vehicle merely on his assurance that his check is sufficiently funded. When the car
dealer deposited the check, it was dishonored on the ground of "Account Closed."
After an investigation, it was found that an employee of the bank misplaced Tony's
account ledger. Thus, the bank erroneously assumed that his account no longer exists.
Later it turned out that Tony's account has more than sufficient funds to cover the
check. The dealer however, immediately filed an action for recovery of possession of
the vehicle against Tony for which he was terribly humiliated and embarrassed.
Does Tony have a cause of action against Premium Bank? Explain. (5%)

SUGGESTED ANSWER:
Yes, Tony may file an action against Premium Bank for damages under Art.
2176. Even if there exists a contractual relationship between Tony and Premium
Bank, an action for quasi-delict may nonetheless prosper. The Supreme Court has
consistently ruled that the act that breaks the contract may also be a tort. There is a
fiduciary relationship between the bank and the depositor, imposing utmost diligence
in managing the accounts of the depositor. The dishonor of the check adversely
affected the credit standing of Tony, hence, he is entitled to damages (Singson v.
BPI, G.R. No. L-24932, June 27, 1968; American Express International, Inc. v. IAC,
G.R. No. 72383, November 9, 1988; Consolidated Bank and Trust v. CA, G.R. No. L-
70766 November 9,1998).

Notes:
ARTICLE 2176. Whoever by act or omission causes damage to another, there
being fault or negligence, is obliged to pay for the damage done. Such fault or
negligence, if there is no pre-existing contractual relation between the parties, is called
a quasi-delict and is governed by the provisions of this Chapter.

In Consolidated Bank and Trust v. CA, the Supreme Court ruled that an act that
breaks a contract may also be a tort, so an action for quasi-delict may prosper. In this
case, when the bank dishonors the check although it has sufficient funds, it causes
damage to the depositor; therefore, the bank may be held liable for damages under
Article 2176 although there is a contractual relation between the bank and the
depositor.

VICARIOUS LIABILITY

Vicarious Liability (1991)


Romano was bumped by a minivan owned by the Solomon School of Practical
Arts (SSPA). The minivan was driven by Peter, a student assistant whose
assignment was to clean the school passageways daily one hour before and one
hour after regular classes, in exchange for free tuition. Peter was able to drive the
school vehicle after persuading the regular driver, Paul, to turn over the wheel to him
(Peter). Romano suffered serious physical injuries. The accident happened at night
when only one headlight of the vehicle was functioning and Peter only had a student
driver's permit. As a consequence, Peter was convicted in the criminal case.
Thereafter, Romano sued for damages against Peter and SSPA.

a) Will the action for damages against Peter and SSPA prosper?
b) Will your answer be the same if, Paul, the regular driver, was impleaded
as party defendant for allowing Peter to drive the minivan without a regular
driver's license.
c) Is the exercise of due diligence in the selection and supervision of Peter
and Paul a material issue to be resolved in this case?

SUGGESTED ANSWER:
A. Yes. It will prosper (Art, 2180) because at the time he drove the vehicle, he
was not performing his assigned tasks as provided for by Art. 2180. With respect to
SSPA, it is not liable for the acts of Peter because the latter was not an employee as
held by Supreme Court in Filamer Christian Institute vs. CA. (190 SCRA 485).
Peter belongs to a special category of students who render service to the school in
exchange for free tuition fees.
B. I would maintain the same answer because the incident did not occur while
the employee was in the performance of his duty as such employee. The incident
occurred at night time, and in any case, there was no indication in the problem that he
was performing his duties as a driver.
C. In the case of Peter, if he were to be considered as employee, the exercise of
due diligence in the selection and supervision of peter would not be a material issue
since the conviction of Peter would result in a subsidiary liability where the defense
would not be available by the employer.
In the case of Paul, since the basis of subsidiary liability is the pater familias rule
under Art. 2180, the defense of selection and supervision of the employee would be a
valid defense.

Notes:
A. In Filamer Christian Institute vs. CA, the Supreme court held that a student
assistant who is assigned to clean a passageway in exchange for a free tuition fee is
not an employee of such school, so any damage caused by the student shall not make
the school liable.

B. Under the Civil Code, the school as an employer cannot be held solidarily
liable with its employee if the latter causes damage while not in the performance of
his assigned task.

C. The defense of due diligence in the selection and supervision of employee is


available only if the employer is held liable under Article 2180. Therefore, if an
employee causes damage while not in the performance of his assigned task or the one
who causes damage is not considered as an employee of the employer, the defense
mentioned above is not available.

In the case of Paul, since the basis of subsidiary liability is the pater familias rule
under Art. 2180, the defense of selection and supervision of the employee would be
a valid defense.

ARTICLE 2180. The obligation imposed by article 2176 is demandable not


only for one’s own acts or omissions, but also for those of persons for whom one is
responsible.

The father and, in case of his death or incapacity, the mother, are responsible
for the damages caused by the minor children who live in their company.
Guardians are liable for damages caused by the minors or incapacitated persons
who are under their authority and live in their company.

The owners and managers of an establishment or enterprise are likewise


responsible for damages caused by their employees in the service of the branches in
which the latter are employed or on the occasion of their functions.

Employers shall be liable for the damages caused by their employees and
household helpers acting within the scope of their assigned tasks, even though the
former are not engaged in any business or industry.

The State is responsible in like manner when it acts through a special agent; but
not when the damage has been caused by the official to whom the task done properly
pertains, in which case what is provided in article 2176 shall be applicable.

Lastly, teachers or heads of establishments of arts and trades shall be liable for
damages caused by their pupils and students or apprentices, so long as they remain in
their custody.

The responsibility treated of in this article shall cease when the persons herein
mentioned prove that they observed all the diligence of a good father of a family to
prevent damage.

ALTERNATIVE ANSWER:
C. In the case of Peter, if he were to be considered an employee, the exercise of
due diligence in the selection and supervision of Peter would not be a material issue
since the conviction of Peter would result in a subsidiary liability where the defense
would not be available by the employer.

In the case of Paul, since he was in the performance of his work at the time the
incident occurred, the school may be held subsidiarily liable not because of the
conviction of Peter, but because of the negligence of Paul under Art. 2180.

Notes:
ARTICLE 2194. The responsibility of two or more persons who are liable for
quasi-delict is solidary.

Vicarious Liability (2001)

After working overtime up to midnight, Alberto, an executive of an insurance


company drove a company vehicle to a favorite Videoke bar where he had some
drinks and sang some songs with friends to "unwind". At 2:00 a.m., he drove home,
but in doing so, he bumped a tricycle, resulting in the death of its driver. May the
insurance company be held liable for the negligent act of Alberto? Why?

SUGGESTED ANSWER:
The insurance company is not liable because when the accident occurred,
Alberto was not acting within the assigned tasks of his employment. It is true that
under Art. 2180 (par. 5), employers are liable for damages caused by their employees
who were acting within the scope of their assigned tasks. However, the mere fact that
Alberto was using a service vehicle of the employer at the time of the injurious
accident does not necessarily mean that he was operating the vehicle within the scope
of his employment. In Castilex Industrial Corp. v. Vasquez Jr (321 SCRA393
[1999]). the Supreme Court held that notwithstanding the fact that the employee did
some overtime work for the company, the former was, nevertheless, engaged in his
own affairs or carrying out a personal purpose when he went to a restaurant at 2:00
a.m. after coming out from work. The time of the accident (also 2:00 a. m.) was
outside normal working hours.
Notes:
In Castilex Industrial Corp. v. Vasquez Jr Employers, it was held that an
employer shall be liable for the damage caused by their employees and household
helpers only when the latter cause damage while they are in the performance of their
function.

Alberto was not in the performance of his function when he caused damage.
Therefore, his employer should not be held liable.

ALTERNATIVE ANSWER:
The insurance company is liable if Alberto was negligent in the operation of the
car and the car was assigned to him for the benefit of the insurance company, and
even though he was not within the scope of his assigned tasks when the accident
happened. In one case decided by the Supreme Court, where an executive of a
pharmaceutical company was given the use of a company car, and after office hours,
the executive made personal use of the car and met an accident, the employer was also
made liable under Art. 2180 of the Civil Code for the injury caused by the negligent
operation of the car by the executive, on the ground that the car which caused the
injury was assigned to the executive by the employer for the prestige of the company.
The insurance company was held liable even though the employee was not performing
within the scope of his assigned tasks when the accident happened [Valenzuela v.
CA, 253 SCRA 3O3 (1996)].
Vicarious Liability (2002)
Explain the concept of vicarious liability in quasi-delicts. (1%)
SUGGESTED ANSWER:
The doctrine of VICARIOUS LIABILITY is that which renders a person liable
for the negligence of others for whose acts or omission the law makes him responsible
on the theory that they are under his control and supervision.

Notes:
Under the doctrine of vicarious liability, a person will be held liable for the
negligence of another because the latter is under his control and supervision.

Article 2180 of the New Civil Code encompasses the concept of vicarious
liability which makes a person liable for the damage caused by another.

Vicarious Liability (2004)


OJ was employed as professional driver of MM Transit bus owned by Mr. BT.
In the course of his work, OJ hit a pedestrian who was seriously injured and later died
in the hospital as a result of the accident. The victim’s heirs sued the driver and the
owner of the bus for damages. Is there a presumption in this case that Mr. BT, the
owner, had been negligent? If so, is the presumption absolute or not? Explain.
(5%)

SUGGESTED ANSWER:
Yes, there is a presumption of negligence on the part of the employer. However,
such presumption is rebuttable. The liability of the employer shall cease when they
prove that they observed the diligence of a good father of a family to prevent damage
(Article 2180, Civil Code).

When the employee causes damage due to his own negligence while performing
his own duties, there arises the juris tantum presumption that the employer is
negligent, rebuttable only by proof of observance of the diligence of a good father of a
family (Metro Manila Transit v. CA, 223 SCRA 521 [1993]; Delsan Transport
Lines v, C&tA Construction, 412 SCRA 524 2003).
Likewise, if the driver is charged and convicted in a criminal case for criminal
negligence, BT is subsidiarily liable for the damages arising from the criminal act.

Notes:
In Metro Manila Transit v. CA, the Supreme Court held that it is legally
presumed that an employer is negligent when his employee causes damage while
performing his assigned task; however, this presumption is rebuttable; it is not
absolute.

Vicarious Liability; Independent Civil Action (2006)


Arturo sold his Pajero to Benjamin for P1 Million. Benjamin took the vehicle
but did not register the sale with the Land Transportation Office. He allowed his son
Carlos, a minor who did not have a driver's license, to drive the car to buy pan de sal
in a bakery. On the way, Carlos driving in a reckless manner, sideswiped Dennis, then
riding a bicycle. As a result, he suffered serious physical injuries. Dennis filed a
criminal complaint against Carlos for reckless imprudence resulting in serious
physical injuries.

1. Can Dennis file an independent civil action against Carlos and his father
Benjamin for damages based on quasi-delict? Explain. (2,5%)

SUGGESTED ANSWER:
Yes, Dennis can file an independent civil action against Carlos and his father
for damages based on quasi-delict there being an act or omission causing damage to
another without contractual obligation. Under Section 1 of Rule 111 of the 2000 Rules
on Criminal Procedure, what is deemed instituted with the criminal action is only the
action to recover civil liability arising from the act or omission punished by law. An
action based on quasi-delict is no longer deemed instituted and may be filed separately
[Section 3, Rule 111, Rules of Criminal Procedure].

Notes:
Under the Rules of Court, an action based on quasi-delict is an independent civil
action; it is not deemed instituted with the criminal action; what is deemed instituted
with the criminal action is a civil liability arising from criminal offense under Article
103 of the Revised Penal Code.

2. Assuming Dennis' action is tenable, can Benjamin raise the defense that
he is not liable because the vehicle is not registered in his name? Explain. (2.5%)

SUGGESTED ANSWER:
No, Benjamin cannot raise the defense that the vehicle is not registered in his
name. His liability, vicarious in character, is based on Article 2180 because he is the
father of a minor who caused damage due to negligence. While the suit will prosper
against the registered owner, it is the actual owner of the private vehicle who is
ultimately liable (See Duavit v. CA, G.R. No. L-29759, May 18, 1989). The purpose
of car registration is to reduce difficulty in identifying the party liable in case of
accidents (Villanueva v. Domingo, G.R. No. 144274, September 14, 2004).

Notes:
Under Article 2180, a father is liable for the damage caused by his minor child;
his liability is primary because his child is under his control and supervision.

Vicarious Liability; Public Utility (2000)


Silvestre leased a car from Avis-Rent-A-Car Co. at the Mactan International
Airport. No sooner had he driven the car outside the airport when, due to his
negligence, he bumped an FX taxi owned and driven by Victor, causing damage to
the latter in the amount of P100,000.00. Victor filed an action for damages against
both Silvestre and Avis, based on quasi-delict. Avis filed a motion to dismiss the
complaint against it on the ground of failure to state a cause of action. Resolve
the motion. (3%)

SUGGESTED ANSWER:
The motion to dismiss should be granted, AVIS is not the employer of Silvestre;
hence, there is no right of action against AVIS under Article 2180 of the Civil Code.
Not being the employer, AVIS has no duty to supervise Silvestre. Neither has AVIS
the duty to observe due diligence in the selection of its customers. Besides, it was
given in the problem that the cause of the accident was the negligence of Silvestre.
Notes:
Under the Public Service Law, the registered owner of the car involved in a
collision shall be held liable for the damage. Therefore, an enterprise engaged in the
business of renting out cars to its clients may be held liable for the damage caused by
the lessee.

However, if the cause of action is based on quasi-delict, the case should be


dismissed for lack of cause of action because the lessee is not an employee of the
lessor.

ALTERNATIVE ANSWER:
The motion should be denied. Under the Public Service Law, the registered
owner of a public utility is liable for the damages suffered by third persons through
the use of such public utility. Hence, the cause of action is based in law, the Public
Service Law.
INTELLECTUAL PROPERTY

IP is a category of property that includes intangible creations of the human


intellect; the most well-known types of IP are copyrights, patents, trademarks, and
trade secrets.

Intellectual Creation (2004)


Dr. ALX is a scientist honored for work related to the human genome project.
Among his pioneering efforts concern stem cell research for the cure of Alzheimer’s
disease. Under corporate sponsorship, he helped develop a microbe that ate and
digested oil spills in the sea. Now he leads a college team for cancer research in MSS
State. The team has experimented on a mouse whose body cells replicate and bear
cancerous tumor. Called “oncomouse”, it is a life-form useful for medical research
and it is a novel creation. Its body cells do not naturally occur in nature but are the
product of man’s intellect, industry and ingenuity. However, there is a doubt whether
local property laws and ethics would allow rights of exclusive ownership on any life-
form. Dr. ALX needs your advice:

(1) whether the reciprocity principle in private international law could be


applied in our jurisdiction; and
(2) whether there are legal and ethical reasons that could frustrate his claim
of exclusive ownership over the life-form called “oncomouse” in Manila? What
will be your advice to him? (5%)

SUGGESTED ANSWER:
(1) The reciprocity principle in private international law may be applied in our
jurisdiction. Section 3 of R.A. 8293, the Intellectual Property Code, provides for
reciprocity, as follows: "Any person who is a national, or who is domiciled, or has a
real and effective industrial establishment in a country which is a party to any
convention, treaty or agreement relating to intellectual property rights or the
repression of unfair competition, to which the Philippines is also a party, or extends
reciprocal rights to nationals of the Philippines by law, shall be entitled to benefits
to the extent necessary to give effect to any provision of such convention, treaty or
reciprocal law, in addition to the rights to which any owner of an intellectual property
right is otherwise entitled by this Act. (n)"
To illustrate: The Philippines may refrain from imposing a requirement of local
incorporation or establishment of a local domicile for the protection of industrial
property rights of foreign nationals (citizens of Canada, Switzerland, U.S.) if the
countries of said foreign nationals refrain from imposing said requirement on
Filipino citizens.

Notes:
Under the Intellectual Property Code, the principle of reciprocity means that
foreign nationals may be given a right similar to the right given by his country to
Filipinos where both counties are party to a treaty or convention pertaining to
intellectual property rights.

ALTERNATIVE ANSWER:
Reciprocity principle cannot be applied in our jurisdiction because the
Philippines is a party to the TRIPS agreement and the WTO. The principle involved is
the most-favored nation clause which is the principle of non-discrimination. The
protection afforded to intellectual property protection in the Philippines also applies to
other members of the WTO. Thus, it is not really reciprocity principle in private
international law that applies, but the most-favored nation clause under public
international law.

(2) There is no legal reason why "oncomouse" cannot be protected under the
law. Among those excluded from patent protection are "plant varieties or animal
breeds, or essentially biological process for the production of plants and animals"
(Section 22.4 Intellectual Property Code, R.A. No. 8293). The "oncomouse" in the
problem is not an essentially biological process for the production of animals. It is a
real invention because its body cells do not naturally occur in nature but are the
product of man's ingenuity, intellect and industry.

The breeding of oncomouse has novelty, inventive step and industrial


application. These are the three requisites of patentability. (Sec. 29, IPC)

There are no ethical reasons why Dr. ADX and his college team cannot be given
exclusive ownership over their invention. The use of such genetically modified
mouse, useful for cancer research, outweighs considerations for animal rights.
Notes:

To become patentable invention, the invention must be new, involves inventive


steps and it has industrial application.

SECTION 21. Patentable Inventions. - Any technical solution of a problem in


any field of human activity which is new, involves an inventive step and is
industrially applicable shall be patentable. It may be, or may relate to, a product, or
process, or an improvement of any of the foregoing.

There are no legal and ethical reasons that would frustrate Dr. ALX's claim of
exclusive ownership over "oncomouse". Animals are property capable of being
appropriated and owned'. In fact, one can own pet dogs or cats, or any other animal. If
wild animals are capable of being owned, with more reason animals technologically
enhanced or corrupted
by man's invention or industry are susceptible to exclusive ownership by the inventor.

ALTERNATIVE ANSWER:
The oncomouse is a higher life form which does not fall within the definition of
the term "invention". Neither may it fall within the ambit of the term "manufacture"
which usually implies a non-living mechanistic product. The oncomouse is better
regarded as a "discovery" which is the common patrimony of man.

You might also like